Clinical Reasoning Cases in Nursing

SEVENTH EDITION

Mariann Harding, PhD, RN, CNE Associate Professor, Department of Nursing, Kent State University at Tuscarawas, New Philadelphia, Ohio

Julie S. Snyder, MSN, RN-BC Lecturer, School of Nursing, Regent University, Virginia Beach, Virginia

Table of Contents

Cover image

Inside Front Cover

Brief Contents

Title page

Copyright

Contributors

Reviewers

Introduction

What Is Clinical Reasoning?

What Is New in This Edition

The “How To” of Case Studies

Acknowledgments

1: Perfusion

Case Study 1

Case Study 2

Case Study 3

Case Study 4

Case Study 5

Case Study 6

Case Study 7

Case Study 8

Case Study 9

Case Study 10

Case Study 11

Case Study 12

Case Study 13

Case Study 14

Case Study 15

Case Study 16

Case Study 17

Case Study 18

Case Study 19

Case Study 20

Case Study 21

Case Study 22

2: Gas Exchange

Case Study 23

Case Study 24

Case Study 25

Case Study 26

Case Study 27

Case Study 28

Case Study 29

Case Study 30

Case Study 31

Case Study 32

Case Study 33

Case Study 34

Case Study 35

Case Study 36

Case Study 37

Case Study 38

Case Study 39

Case Study 40

3: Mobility

Case Study 41

Case Study 42

Case Study 43

Case Study 44

Case Study 45

Case Study 46

Case Study 47

Case Study 48

Case Study 49

Case Study 50

Case Study 51

Case Study 52

Case Study 53

Case Study 54

4: Digestion

Case Study 55

Case Study 56

Case Study 57

Case Study 58

Case Study 59

Case Study 60

Case Study 61

Case Study 62

Case Study 63

Case Study 64

Case Study 65

Case Study 66

5: Urinary Elimination

Case Study 67

Case Study 68

Case Study 69

Case Study 70

Case Study 71

Case Study 72

Case Study 73

Case Study 74

Case Study 75

6: Intracranial Regulation

Case Study 76

Case Study 77

Case Study 78

Case Study 79

Case Study 80

Case Study 81

Case Study 82

Case Study 83

Case Study 84

Case Study 85

Case Study 86

Case Study 87

Case Study 88

7: Metabolism and Glucose Regulation

Case Study 89

Case Study 90

Case Study 91

Case Study 92

Case Study 93

Case Study 94

Case Study 95

Case Study 96

Case Study 97

Case Study 98

Case Study 99

Case Study 100

8: Immunity

Case Study 101

Case Study 102

Case Study 103

Case Study 104

Case Study 105

Case Study 106

9: Cellular Regulation

Case Study 107

Case Study 108

Case Study 109

Case Study 110

Case Study 111

Case Study 112

Case Study 113

Case Study 114

Case Study 115

10: Tissue Integrity

Case Study 116

Case Study 117

Case Study 118

Case Study 119

11: Cognition

Case Study 120

Case Study 121

Case Study 122

12: Infection and Inflammation

Case Study 123

Case Study 124

Case Study 125

Case Study 126

Case Study 127

Case Study 128

Case Study 129

Case Study 130

13: Developmental

Case Study 131

Case Study 132

Case Study 133

Case Study 134

Case Study 135

Case Study 136

14: Reproductive

Case Study 137

Case Study 138

Case Study 139

Case Study 140

Case Study 141

Case Study 142

Case Study 143

15: Mood, Stress, and Addiction

Case Study 144

Case Study 145

Case Study 146

Case Study 147

Case Study 148

Case Study 149

Case Study 150

Inside Front Cover

Brief Contents Chapter 1 Perfusion, 1 Chapter 2 Gas Exchange, 99 Chapter 3 Mobility, 187 Chapter 4 Digestion, 247 Chapter 5 Urinary Elimination, 301 Chapter 6 Intracranial Regulation, 339 Chapter 7 Metabolism and Glucose Regulation, 401 Chapter 8 Immunity, 455 Chapter 9 Cellular Regulation, 483

Chapter 10 Tissue Integrity, 527 Chapter 11 Cognition, 547 Chapter 12 Infection and Inflammation, 561 Chapter 13 Developmental, 593 Chapter 14 Reproductive, 615 Chapter 15 Mood, Stress, and Addiction, 645

Copyright

CLINICAL REASONING CASES IN NURSING, SEVENTH EDITION ISBN: 978-0-323-52736-1

Copyright © 2020 by Elsevier, Inc. All rights reserved.

No part of this publication may be reproduced or transmitted in any form or by any means, electronic or mechanical, including photocopying, recording, or any information storage and retrieval system, without permission in writing from the publisher. Details on how to seek permission, further information about the Publisher’s permissions policies and our arrangements with organizations such as the Copyright Clearance Center and the Copyright Licensing Agency, can be found at our website: www.elsevier.com/permissions.

This book and the individual contributions contained in it are protected under copyright by the Publisher (other than as may be noted herein).

Notice Practitioners and researchers must always rely on their own experience and knowledge in evaluating and using any information, methods, compounds or experiments described herein. Because of rapid advances in the medical sciences, in particular, independent verification of diagnoses and drug dosages should be made. To the fullest extent of the law, no responsibility is assumed by Elsevier, authors, editors or contributors for any injury and/or damage to persons or property as a matter of products liability, negligence or

http://www.elsevier.com/permissions
otherwise, or from any use or operation of any methods, products, instructions, or ideas contained in the material herein.

Previous editions copyrighted 2016, 2013, 2009, 2005, 2001, 1996.

Library of Congress Control Number: 2018954100

Executive Content Strategist: Lee Henderson Content Development Specialist: Laura Goodrich Publishing Services Manager: Julie Eddy Senior Project Manager: Tracey Schriefer Design Direction: Margaret Reid

Printed in the United States of America Last digit is the print number: 9 8 7 6 5 4 3 2 1

Contributors

Meghan Davis, MSN-Ed, CCRN, RN Registered Nurse, Virginia Beach, Virginia

Jatifha C. Felton, MSN-Ed, APRN, ACNCP-AG, CCRN Critical Care Nurse Practitioner, Chesapeake Regional Healthcare, Chesapeake, Virginia

Sherry D. Ferki, RN, MSN Adjunct Faculty, School of Nursing, Old Dominion University, Norfolk, Virginia

Joanna Van Sant, MSN, RN Clinical Nurse Manager–Oncology Unit, Sentara Northern Virginia Medical Center, Woodbridge, Virginia

Reviewers

Heidi Matarasso Bakerman, RN, BA Nursing, MscN Nursing Instructor, Nursing Vanier College, Montreal, Quebec, Canada

Beverly Banks, MSN, BSN, RN Full Time Faculty, Nursing, Alpena Community College, Alpena, Michigan

Mitzi L. Bass, MPH, MSN, RN Assistant Professor of Nursing, School of Nursing and Health Professions, Baltimore City Community College, Baltimore, Maryland

Michelle Bayard, BSN, RN Teacher, Faculty of Careers and Technology: Nursing Vanier College, Montreal, Quebec, Canada

Diana Lynne Burgess, MSN, RN Nursing Faculty—ADN Program, St. Petersburg College of Nursing, St. Petersburg, Florida

Lacey M. Campbell, MSN, RN Program Coordinator Accelerated LPN to RN Program, Texas County Technical College, Houston, Missouri

Diane Cohen, MSN, RN Professor—Nursing, MassBay Community College, Framingham, Massachusetts

Nicola Eynon-Brown, RN(EC), BNSc, MN, NP, CPNP- PC Professor, School of Baccalaureate Nursing, St. Lawrence College, Brockville, Ontario, Canada

Melissa Marie Fischer, MSN, RN ADN Nursing Faculty, Nursing, Blackhawk Technical College, Janesville, Wisconsin

Victoria A. Greenwood, MSN, MSEd, RN-BC Assistant Professor, Nursing, The Sage Colleges, Troy, New York

Rose A. Harding, MSN, RN Coordinator, Standardized Test

Evaluation Committee, JoAnne Gay Dishman School of Nursing, Lamar University, Beaumont, Texas

Antonea Jackson, PhD (c), MSN, RN CNE Clinical Assistant Professor, Nursing, Prairie View A&M University, Houston, Texas

Llynne C. Kiernan, DNP, MSN, RN-BC Assistant Professor of Nursing, Nursing, Norwich University, Northfield, Vermont

Tonie Metheny, MS, RN, CNE Clinical Instructor, Nursing, Fran and Earl Ziegler College of Nursing, University of Oklahoma Health Sciences Center, Oklahoma City, Oklahoma

Kathleen S. Murtaugh, MSN, RN, CNA Assistant Professor, Nursing, St. Elizabeth School of Nursing/University of Saint Francis, Lafayette, Indiana

Karen Noss, MSN, RN Professor, Nursing Department, Luzerne County Community College, Nanticoke, Pennsylvania

Alicia Powell, MSN, RNC Clinical Nurse Educator, Women’s Services, Chesapeake Regional Healthcare, Chesapeake, Virginia

Deborah J. Pumo, MS, RN, EdD Nursing Professor, Nursing Department, Illinois Valley Community College, Oglesby, Illinois

Sandra A. Ranck, MSN, RN Program Administrator, Auburn Practical Nursing Program, Auburn Career Center, Concord Township, Ohio

Cherie R. Rebar, PhD, MBA, RN, COI Professor of Nursing, Wittenberg University, Springfield, Ohio

Alicia N. Rose, ACNS-BC, PMHNP-BC, RN-BC, CSAC Psychiatric Nurse Practitioner, Meridian Psychotherapy, Virginia Beach, Virginia

Jane Tyerman, BA, RN, BScN, MScN, PhD Professor, Trent/Fleming School of Nursing, Trent University, Peterborough, Ontario, Canada

Introduction

To provide safe, quality care, nurses need to have well-developed clinical reasoning skills. As new graduates, you will make decisions and take actions of an increasingly sophisticated nature. You will encounter problems you have never seen or heard about during your classroom and clinical experiences. You will have to make complex decisions with little or no guidance and limited resources.

We want you to be exposed to as much as possible during your student days, but more importantly, we want you to learn to think. You cannot memorize your way out of any situation, but you can think your way out of any situation. We know that students often learn more and faster when they have the freedom to make mistakes. This book is designed to allow you to look at how to solve problems and find answers without the pressure of someone’s life hanging in the balance. We want you to do well. We want you to be the best. It is our wish for you to grow into confident, competent nursing professionals. We want you to be very, very good at what you do!

What Is Clinical Reasoning? Clinical reasoning is not memorizing lists of facts or the steps of procedures. Instead, clinical reasoning is an analytical process that can help you think about a patient care issue in an organized and efficient manner. Five steps are involved in clinical reasoning. Thinking about these steps may help you when you work through the questions in your cases. Here are the five steps with an explanation of what they mean.

1. Recognize and define the problem by asking the right questions:

Exactly what is it you need to know? What is the question asking?

2. Select the information or data necessary to solve the problem or answer the question: First you have to ask whether all the necessary information is there. If not, how and where can you get the additional information? What other resources are available? This is one of the most difficult steps. In real clinical experiences, you rarely have all of the information, so you have to learn where you can get necessary data. For instance, patient and family interviews, nursing charting, the patient medical chart, laboratory data on your computer, your observations, and your own physical assessment can help you identify important clues. Of course, information can rapidly become outdated. To make sure you are accessing the most current and accurate information, you will occasionally need to use the Internet to answer a question.

3. Recognize stated and unstated assumptions; that is, what do you think is or is not true? Sometimes answers or solutions seem obvious; just because something seems obvious does not mean it is correct. You may need to consider several possible answers or solutions. Consider all clues carefully and do not dismiss a possibility too quickly. Remember, “You never find an answer you don’t think of.”

4. Formulate and select relevant and/or potential decisions: Try to think of as many possibilities as you can. Consider the pros and cons of the consequences of making each decision. What is the best answer/solution? What could go wrong? This requires considering many different angles. In today’s health care settings, decision making often requires balancing the well- being needs of the patient, the preferences and concerns of the patient and caregiver, and financial limitations imposed by the reimbursement system. In making decisions, you need to take into account all relevant factors. Remember, you may need to explain why you rejected other options.

5. Draw a valid, informed conclusion: Consider all data; then

determine what is relevant and what makes the most sense. Only then should you draw your conclusion.

It may look as if this kind of reasoning comes naturally to your instructors and experienced nurses. You can be certain that even experienced nurses were once where you are now. The rapid and sound decision making that is essential to good nursing requires years of practice. The practice of good clinical reasoning leads to good thinking in clinical practice. This book will help you practice the important steps in making sound clinical judgments until the process starts to come naturally.

What Is New in This Edition The conceptual approach to nursing education is a new way to manage information and help students develop clinical reasoning skills. In this edition, we chose to reorganize the cases in each section by health-illness concepts. Within each section, you will see the basic principles of that concept applied in exemplars, or models of that concept, that cross care settings, the life span, and the health-illness continuum. For example, you may be enrolled in a course that focuses on gas exchange, including risk factors, physiologic mechanisms, assessment, and interventions to promote optimal oxygenation. Based on prevalence and incidence, exemplars such as pneumonia, influenza, and asthma, are used to show how to apply principles across ages and care settings. To ensure that there are cases that cover common exemplars you may see in class, we added over 20 new cases. Like the existing cases, each of these are adaptations of actual scenarios encountered in the clinical setting—there is no better way to learn than from real patients!

Because nurses play a vital role in improving the safety and quality of patient care, you need to learn interventions you will use to deliver safe care and enhance patient outcomes. To help you learn key principles, you will note questions marked with a

. These questions involve scenarios that typically include inherent

risks, such as medication administration, fall and pressure injury reduction protocols, and preventing health care–associated infections.

The “How To” of Case Studies When you begin each case, read through the whole story once, from start to finish, getting a general idea of what it is about. Write down things you have to look up. This will help you move through the case smoothly and get more out of it. How much you have to look up will depend on where you are in your program, what you know, and how much experience you already have. Preparing cases will become easier as you advance in your program.

Acknowledgments

We would like to express our appreciation to the editorial Elsevier staff, especially Laura Goodrich, Lee Henderson, and Tracey Schriefer for their professional support and contributions in guiding this text to publication. We extend a special thanks to our reviewers, who gave us helpful suggestions and insights as we developed this edition.

Mariann’s gratitude goes to those she loves most—her husband, Jeff, and her daughters, Kate and Sarah. She gives a special thanks to her students, colleagues, and patients; each inspire her passion for nursing and education. Lastly, Mariann praises God, who has graciously bestowed more blessings than could ever be imagined.

Julie thanks her husband, Jonathan, for his love, support, and patience during this project. She is grateful for the encouragement from daughter Emily, son-in-law Randy, and parents Willis and Jean Simmons. Julie appreciates the hard work of colleagues Sherry Ferki, Jatifha Felton, Meghan Davis, Joanna Van Sant, Alicia Rose, and Alicia Powell as contributors and reviewers for this edition. She is especially thankful to the students, whose eagerness to learn is an inspiration. Most importantly, Julie gives thanks to God, our source of hope and strength.

1

Perfusion

Case Study 1 Name_________________________________ Class/Group ______________________________ Date ___________

Scenario M.G., a “frequent flier,” is admitted to the emergency department (ED) with a diagnosis of heart failure (HF). She was discharged from the hospital 10 days ago and comes in today stating, “I just had to come to the hospital today because I can’t catch my breath and my legs are as big as tree trunks.” After further questioning, you learn she is strictly following the fluid and salt restriction ordered during her last hospital admission. She reports gaining 1 to 2 pounds (0.5 to 1 kg) every day since her discharge.

1. What error in discharge teaching most likely occurred? 2. An echocardiogram revealed that her ejection fraction (EF) is

30%, but it was 40% a month ago. What is EF, and what does the decreased number indicate?

Case Study Progress During the admission interview, the nurse makes a list of the medications M.G. took at home.

Chart View

Nursing Assessment: Medications Taken at Home

Enalapril (Vasotec) 5 mg PO bid

Pioglitazone (Actos) 45 mg PO every morning

Furosemide (Lasix) 40 mg/day PO

Potassium chloride (K-Dur) 20 mEq/day PO

3. Which of these medications may have contributed to M.G.’s HF? Explain.

4. How do angiotensin-converting enzyme (ACE) inhibitors, such as enalapril (Vasotec), work to reduce HF? Select all that apply.

a. Cause systemic vasodilation b. Increase cardiac contractility c. Reduce preload and afterload d. Prevent the conversion of angiotensin I to

angiotensin II e. Block sympathetic nervous system stimulation to

the heart f. Promote the excretion of sodium and water in the

renal tubules

Case Study Progress After reviewing M.G.’s medications, the cardiologist writes the following medication orders.

Chart View

Medication Orders

Enalapril (Vasotec) 5 mg PO bid

Carvedilol (Coreg) 3.125 mg PO twice daily

Metformin (Glucophage)

500 mg twice daily

Furosemide (Lasix) 80 mg intravenous push (IVP) now, then 40 mg/day IVP

Potassium chloride (K- Dur)

20 mEq/day PO

5. What is the rationale for changing the route of the furosemide (Lasix)?

6. You give furosemide (Lasix) 80 mg IVP. Identify at least 4 parameters you would use to monitor the effectiveness of this medication.

7. What lab tests should be ordered for M.G. related to the order for furosemide (Lasix)? Select all that apply.

a. Sodium level b. Potassium level c. Magnesium level d. Coagulation studies e. Serum glucose level f. Complete blood count

8. What is the reason for ordering the beta blocker carvedilol? a. Increase urine output b. Cause peripheral vasodilation c. Increase the contractility of the heart d. Reduce cardiac stimulation from catecholamines

9. You assess M.G. for conditions that may be a

contraindication to carvedilol. Which condition, if present, may cause serious problems if she takes this medication?

a. Angina b. Asthma c. Glaucoma d. Hypertension

Case Study Progress One day later, M.G. has shown only slight improvement, and digoxin (Lanoxin) 125 mcg PO daily is added to her orders.

10. What is the mechanism of action of digoxin? a. Causes systemic vasodilation b. Increases cardiac contractility and cardiac output c. Blocks sympathetic nervous system stimulation to

the heart d. Promotes the excretion of sodium and water in the

renal tubules 11. Which findings from M.G.’s assessment would indicate an

increased possibility of digoxin toxicity? Explain your answer. a. Digoxin level 1.6 ng/mL (2.05 mmol/L) b. Serum sodium level of 139 mEq/L (138 mmol/L) c. Apical heart rate of 64 d. Serum potassium level of 2.2 mEq/L (2.2 mmol/L)

12. When preparing to give the digoxin, you notice that it is

available in milligrams (mg) not micrograms (mcg). Convert 125 mcg to mg.

13. After 2 days, M.G.’s symptoms improve with intravenous diuretics and digoxin. She is placed back on oral furosemide (Lasix) once her weight loss is deemed adequate for achievement of a euvolemic state. What will determine whether the oral dose will be adequate for discharge to be considered?

14. M.G. is ready for discharge. According to the mnemonic MAWDS, what key management concepts should be taught to prevent relapse and another admission?

15. After the teaching session, the nurse asks M.G. to “teach back” one important concept of care at home. Which statement by M.G. indicates a need for further education? Explain your answer.

a. “I will not add salt when I am cooking.”

b. “I will use a weekly pill calendar box to remind me to take my medicine.”

c. “I will weigh myself daily and tell the doctor at my next visit if I am gaining weight.”

d. “I will try to take a short walk around the block with my husband three times a week.”

Case Study Outcome After M.G. has been at home for 2 days, the STOP Heart Failure Nurse Navigator calls to ask about her progress. M.G. reports that her weight has not increased since she has been home and she is breathing more easily.

Case Study 2 Name _________________________________ Class/Group _____________________________ Date _____________

Scenario M.P. is a 65-year-old African American woman who comes to the clinic for a follow-up visit. She was diagnosed with hypertension (HTN) 2 months ago and was given a prescription for a thiazide diuretic but stopped taking it 2 weeks ago because “it made me dizzy and I kept getting up during the night to empty my bladder.” During today’s clinic visit, she expresses fear because her mother died of a stroke (cerebrovascular accident [CVA]) at M.P.’s age, and M.P. is afraid she will suffer the same fate. She states, “I’ve never smoked and I don’t drink, but I am so afraid of this high blood pressure.” You review the data from her past clinic visits.

Chart View

Family History

Mother, died at age 65 years of CVA Father, died at age 67 years of myocardial infarction (MI) Sister, alive and well, age 62 years Brother, alive, age 70 years, has coronary artery disease (CAD),

HTN, type 2 diabetes mellitus (DM)

Patient Past History

Married for 45 years, 2 children, alive and well, 6 grandchildren Cholecystectomy, age 42 years Hysterectomy, age 48 years

Blood Pressure Assessments

January 2: 150/92 January 31: 156/94 (given prescription for hydrochlorothiazide

[HCTZ] 25 mg PO every morning) February 28: 140/90

1. According to the most recent guidelines from the Joint National Committee on Prevention, Detection, Evaluation, and Treatment of High Blood Pressure, M.P.’s blood pressure (BP) falls under which classification?

2. What could M.P. be doing that is causing her nocturia?

Case Study Progress During today’s visit, M.P.’s vital signs are as follows: BP: 162/102; P: 78; R: 16; T: 98.2°F (36.8 ° C). Her most recent basic metabolic panel (BMP) and fasting lipids are within normal limits. Her height is 5 ft, 4 in (163 cm), and she weighs 110 lb (50 kg). She tells you that she tries to go on walks but does not like to walk alone and so has done so only occasionally.

3. What risk factors does M.P. have that increase her risk for

cardiovascular disease?

Case Study Progress Because M.P.’s BP continues to be high, the provider decides to start another antihypertensive drug and recommends that she try again with the HCTZ, taken in the mornings.

4. According to the JNC 8 national guidelines, describe the drug therapy recommended for M.P. at this time.

5. M.P. goes on to ask whether there is anything else she should do to help with her HTN. She asks, “Do I need to lose weight?” Look up her height and weight for her age on a body mass index (BMI) chart. Is she considered overweight?

6. What nonpharmacologic lifestyle alteration measures might help M.P. control her BP? List 2 examples and explain.

Case Study Progress The provider decreases M.P.’s HCTZ dose to 12.5 mg PO daily and adds a prescription for benazepril (Lotensin) 5 mg daily. M.P. is instructed to return to the clinic in 1 week to have her blood work checked. She is instructed to monitor her BP at least twice a week and return for a medication management appointment in 1 month with her list of BP readings.

7. Why did the provider decrease the dose of the HCTZ? 8. You provide M.P. with education about the common side

effects of benazepril, which can include which of these? Select all that apply.

a. Cough b. Dizziness c. Headache d. Constipation e. Shortness of breath

9. It is sometimes difficult to remember whether one has

taken one’s medication. What techniques might you teach M.P. to help her remember to take her medicines each day? Name at least 2.

10. After the teaching session about her medicines, which statement by M.P. indicates a need for further instructions?

a. “I need to rise up slowly when I get out of bed or out of a chair.”

b. “I will leave the salt shaker off the table and not salt my food when I cook.”

c. “I will call if I feel very dizzy, weak, or short of breath while on this medicine.”

d. “It’s okay to skip a few doses if I am feeling bad as long as it’s just for a few days.”

11. Describe 3 priority problems that will guide M.P.’s nursing care.

Case Study Progress M.P. returns in 1 month for her medication management appointment. She tells you she is feeling fine and does not have any side effects from her new medication. Her BP, checked twice a week at the senior center, ranges from 132 to 136 systolic, and 78 to 82 diastolic.

12. When someone is taking HCTZ and an angiotensin-converting enzyme (ACE) inhibitor, such as benazepril, what lab test results would you expect to be monitored?

Chart View

Laboratory Test Results (Fasting)

Potassium 3.6 mEq/L (3.6 mmol/L)

Sodium 138 mEq/L (138 mmol/L)

Chloride 100 mEq/L (100 mmol/L)

CO2 28 mEq/L (28 mmol/L)

Glucose 112 mEq/L (6.2 mmol/L)

Creatinine 0.7 mg/dL (61.9 mcmol/L)

Blood urea nitrogen (BUN) 18 mg/dL (6.4 mmol/L)

Magnesium 1.9 mEq/L (0.95 mmol/L)

13. What lab test results, if any, are of concern at this time? 14. You take M.P.’s BP and get 138/88. She asks whether these BP

readings are okay. On what do you base your response? 15. List at least 3 important ways you might help M.P. maintain

her success.

Case Study Progress M.P. tells you she was recently at a luncheon with her garden club and that most of those women take BP pills different from the ones she does. She asks why their pills are different shapes and colors.

16. How can you explain the difference to M.P.? 17. During the visit, you ask M.P., “When was your last eye

examination?” She answers, “I’m not sure, probably about 2 years ago. What does that have to do with my blood pressure?” What is your response?

Case Study Outcome M.P. comes in for a routine follow-up visit 3 months later. She continues to do well on her daily BP drug regimen, with average BP readings of 130/78. She participates in group walking program for senior citizens at the local mall. She admits she has not done as well with decreasing her salt intake but says she is trying. She visited an ophthalmologist last week and had no problems except for a slight cataract in one eye.

Case Study 3 Name _________________________________ Class/Group _____________________________ Date _____________

Scenario A.M. is a 52-year-old woman who has gained over 75 lbs (32 kg) over the past 30 years, after the birth of her 3 children. She has a sedentary job that requires sitting at a desk for most hours of the work day. When she is at home, she stays inside because she is afraid to walk by herself in her neighborhood. She lives alone, but her children live in the same city. She has a history of hypertension and states that she does not take her medications regularly. She came to the clinic today stating that she thinks she might have a urinary tract infection. Her weight is 255 lbs (102.5 kg). She is 5 feet, 4 inches tall (162.5 cm) and has a waist circumference of 41 inches (104 cm). Her abdomen is large, nontender, soft, and round. Her blood pressure is 160/104. You review fasting labs results that were drawn a week ago.

Chart View

Laboratory Results

Glucose 170 mg/dL (9.4 mmol/L)

Total cholesterol 215 mg/dL (5.6 mmol/L)

Triglycerides 267 mg/dL (3.0 mmol/L)

HDL 60 mg/dL (1.56 mmol/L)

LDL 116 mg/dL (3.0 mmol/L)

HbA1C 5.9%

1. What is BMI? Calculate A.M.’s BMI and identify her classification based on the results.

2. Does A.M. have type 2 diabetes mellitus? Explain your answer.

Case Study Progress A.M.’s urinalysis is clear, and upon examination she is diagnosed with a vaginal yeast infection. The health care provider discusses A.M.’s condition with her and tells her that in addition to the yeast infection, she has metabolic syndrome and reviews some treatment goals with her. In addition, the HCP reinforces the need for A.M. to take her blood pressure medication regularly. A.M. is visibly upset and has many questions.

3. What is metabolic syndrome? 4. Review A.M.’s history and assessment. What criteria for

metabolic syndrome does A.M. have, if any? 5. What other lifestyle habits will you ask A.M. about during

your assessment? 6. What health problems may result if metabolic syndrome

remains untreated? Select all that apply. a. Stroke b. Diabetes c. Breast cancer d. Heart disease e. Renal disease

Case Study Progress A.M. is given the following prescriptions:

Metformin (Glucophage), 500 mg BID Atorvastatin (Lipitor), 10 mg PO at bedtime Lisinopril (Zestril) 5 mg PO, 1 tablet every morning Fluconazole (Diflucan) 150 mg tablet × 1 dose

7. Explain the purpose of each medication ordered.

8. Which are potential side effects of metformin? Select all that apply.

a. Nausea b. Diarrhea c. Dizziness d. Constipation e. Abdominal bloating

Case Study Progress You take the time to talk to A.M. about her concerns and provide health promotion teaching that includes increasing regular physical activity, weight reduction, and eating a diet low in saturated fats. A.M. tells you she is willing to make changes but that this is a lot of information to take in at this time.

9. She asks, “Why do I have to take a drug for diabetes if I don’t have diabetes?” What is the appropriate answer?

a. “Metformin will prevent you from ever developing diabetes.”

b. “Metformin provides the insulin your body is no longer making.”

c. “Metformin allows you to eat whatever you want and your glucose levels won’t increase.”

d. “Metformin helps your cells to be less resistant to insulin, and, as a result, your glucose levels will decrease.”

10. Explain the role of insulin resistance with metabolic syndrome and metformin’s effect on insulin resistance.

11. Is A.M. at greater risk for coronary artery disease? Explain your answer.

12. A.M. asks you, “Won’t all these pills help me? Why do I need to change how I eat and exercise?” Explain the role of reducing risk factors as part of the treatment for metabolic syndrome.

13. After visiting with the dietitian, you review what A.M. has

learned. You ask her to tell you what food choices would be good for a low-fat diet. Which answer reflects a need for further education?

a. “I will eat more fruits and vegetables.” b. “I will try to eat more chicken and fish.” c. “I can eat red meat as long as I don’t fry it.” d. “I will eat more whole grains, such as whole wheat

bread.”

Case Progress A.M. is referred to a registered dietitian for nutrition education and decides to join the local YMCA for exercise. You teach her how to monitor blood glucose levels at home. She has an appointment to return to the clinic in one month. However, A.M. does not return to the clinic for her appointment. When you call to follow up with her, she agrees to come in a week later. At that time, she tells you that she did not do well with the exercise because it “hurts too much.” She said she tried eating a low-fat diet but that it was difficult to stick to it. She did not check her blood glucose regularly, but told you that when she did check them, her fasting levels were in the “140s to 160s.” Her weight is now 250 lbs (113 kg). She tells you that she feels so discouraged and that she will “never get better.”

14. What resources do you suggest for A.M. at this time?

Case Study Outcome During the next year, A.M. continued to miss appointments, and her weight increased to 272 lbs (123 kg). She was eventually diagnosed with type 2 diabetes mellitus, and at her last visit she asked her HCP about having weight loss surgery.

Case Study 4 Name _________________________________ Class/Group

______________________________ Date ___________

Scenario You are working in the internal medicine clinic of a large teaching hospital. Today your first patient is 70-year-old J.M., a man who has been coming to the clinic for several years for management of coronary artery disease (CAD) and hypertension (HTN). A cardiac catheterization done a year ago showed 50% stenosis of the circumflex coronary artery. He has had episodes of dizziness for the past 6 months and orthostatic hypotension, shoulder discomfort, and decreased exercise tolerance for the past 2 months. On his last clinic visit 3 weeks ago, a chest x-ray (CXR) examination revealed cardiomegaly and a 12-lead electrocardiogram (ECG) showed sinus tachycardia with left bundle branch block. You review J.M.’s morning blood work and initial assessment.

Chart View

Laboratory Results Chemistry

Sodium 142 mEq/L (142 mmol/L)

Chloride 95 mEq/L (95 mmol/L)

Potassium 3.9 mEq/L (3.9 mmol/L)

Creatinine 0.8 mg/dL (70.7 mcmol/L)

Glucose 82 mg/dL (4.6 mmol/L)

BUN 19 mg/dL (6.8 mmol/L)

Complete Blood Count

WBC 5400/mm3 (5.4 x 109/L)

Hgb 11.5 g/dL (115 g/L)

Hct 37%

Platelets 229,000/mm3 (229 x 109/L)

Initial Assessment J.M. reports increased fatigue and shortness of breath, especially with activity, and “waking up gasping for breath” at night, for the past 2 days. Vital Signs

Temperature 97.9° F (36.6° C)

Blood pressure 142/83

Heart rate 105

Respiratory rate 18

1. As you review these results, which ones are of possible concern, and why?

2. Knowing his history and seeing his condition this morning, what further questions are you going to ask J.M. and his daughter?

Case Study Progress J.M. tells you he becomes exhausted and has shortness of breath climbing the stairs to his bedroom and must lie down and rest (“put my feet up”) at least an hour twice a day. He has been sleeping on 2 pillows for the past 2 weeks. He has not salted his food since the provider told him not to because of his high blood pressure, but he admits having had ham and a small bag of salted peanuts 3 days ago. He states that he stopped smoking 10 years ago. He denies having palpitations but has had a constant, irritating, nonproductive cough lately.

3. You think it’s likely that J.M. has heart failure (HF). From his history, what do you identify as probable causes for his HF?

4. You are now ready to do your physical assessment. For each potential assessment finding for HF, indicate whether the finding indicates left-sided HF (L) or right-sided HF (R).

1. Weakness 2. Jugular (neck) vein distention 3. Dependent edema (legs and sacrum) 4. Hacking cough, worse at night 5. Enlarged liver and spleen 6. Exertional dyspnea 7. Distended abdomen 8. Weight gain 9. S3/S4 gallop

10. Crackles and wheezes in lungs

Chart View

Medication Orders

Enalapril (Vasotec) 10 mg PO twice a day Furosemide (Lasix) 20 mg PO every morning Carvedilol (Coreg) 6.25 mg PO twice a day Digoxin (Lanoxin) 0.5 mg PO now, then 0.125 mg PO daily Potassium chloride (K-Dur) 10 mEq tablet PO once a day

Case Study Progress The provider confirms your suspicions and indicates that J.M. is experiencing symptoms of early left-sided heart failure. A two- dimensional (2D) echocardiogram is ordered. Medication orders are written.

5. For each medication listed, identify its class and describe its purpose in treating HF.

6. When you go to remove the medications from the

automated dispensing machine, you see that carvedilol (Coreg CR) is stocked. Will you give it to J.M.? Explain.

7. As you remove the digoxin tablet from the automated

medication dispensing machine, you note that the dose on the tablet label is 250 mcg. How many tablets would you give?

8. Based on the new medication orders, which blood test or tests should be monitored carefully? Explain your answer.

9. When you give J.M. his medications, he looks at the potassium tablet, wrinkles his nose, and tells you he “hates those horse pills.” He tells you a friend of his said he could eat bananas instead. He says he would rather eat a banana every day than take one of those pills. How will you respond?

10. The 2D echocardiogram shows that J.M.’s left ventricular ejection fraction (EF) is 49%. Explain what this test result means with regard to J.M.’s heart function.

Case Study Progress This is J.M.’s first episode of significant HF. Before he leaves the clinic, you want to teach him about lifestyle modifications he can make and monitoring techniques he can use to prevent or minimize future problems.

11. List 5 suggestions you might make and the rationale for each. 12. You tell J.M. that the combination of high-sodium foods he

had during the past several days might have contributed to his present episode of HF. He looks surprised. J.M. says, “But I didn’t add any salt to them!” To what health care professional could J.M. be referred to help him understand how to prevent future crises? State your rationale.

13. After visiting with the cardiac dietitian, you review potential food choices with J.M. Which foods are high in sodium and must be avoided? Select all that apply.

a. Canned soups

b. Cheddar cheese c. Processed meats d. Whole wheat bread e. Fat-free fruit yogurt

14. You also include teaching about digoxin toxicity. When teaching J.M. about the signs and symptoms of digoxin toxicity, which should be included? Select all that apply.

a. Diarrhea b. Visual changes c. Increased urine output d. Loss of appetite or nausea e. Dizziness when standing up

Case Study Outcome J.M.’s condition improves after 5 days of treatment, and he is discharged to home. He has a follow-up appointment with a cardiologist in 2 weeks. He is enrolled in the clinic’s STOP Heart Failure program, and a heart failure nurse navigator will contact him in a few days to check his progress.

Case Study 5 Name _________________________________ Class/Group ______________________________ Date ___________

Scenario It is midmorning on the cardiac unit where you work, and you are getting a new patient. G.P. is a 60-year-old retired businessman who is married and has 3 grown children. As you take his health history, he tells you that he began feeling changes in his chest about 10 days ago. He has hypertension (HTN) and a 3-year history of angina pectoris. During the past week, he has had frequent episodes of mid-chest discomfort. The chest pain responds to nitroglycerin (NTG), which he

has taken sublingually about 8 to 10 times over the past week. During the week, he has also experienced increased fatigue. He states, “I just feel crappy all the time.” A cardiac catheterization done several years ago revealed 50% stenosis of the right coronary artery and 50% stenosis of the left anterior descending coronary artery. He tells you that both his mother and his father had coronary artery disease (CAD). He is currently taking amlodipine (Norvasc), metoprolol (Lopressor), atorvastatin (Lipitor), and aspirin 81 mg/day. He is retired and says that he spends his days watching television, with some occasional yard work. He has gained 25 lb (11.3 kg) since retiring and admits that he is overweight.

1. What other information are you going to obtain about his episodes of chest pain?

2. What are common sites for radiation of ischemic cardiac pain? 3. There are several risk factors for coronary artery disease. For

each risk factor listed, mark whether it is “M” modifiable or “N” nonmodifiable.

a. ___ Age b. ___ Stress c. ___ Gender d. ___ Obesity e. ___ Smoking f. ___ Hypertension g. ___ Hyperlipidemia h. ___ Diabetes mellitus i. ___ Physical inactivity j. ___ Ethnic background k. ___ Excessive alcohol use l. ___ Family history of CAD

4. Based on the history you have so far, circle the modifiable and nonmodifiable risk factors in Question 3 that apply to G.P.

5. Although he has had a prescription for sublingual nitroglycerin (SL NTG) for a long time, you want to be certain he is using it correctly. Which actions are correct when taking SL NTG for

chest pain? Select all that apply. a. Call 911 immediately. b. Stop the activity and lie or sit down. c. Chew the tablet slowly then swallow. d. Place the NTG tablet under the tongue. e. Call 911 if the pain is not relieved after taking 1 SL

tablet. f. Call 911 if the pain is not relieved after taking 3 SL

tablets, 5 minutes apart. 6. You review the use and storage of SL NTG with G.P. Which

statement by G.P. indicates a need for further education? Explain your answer.

a. “I carry the tablets with me at all times.” b. “I will keep the pills in their original brown bottle.” c. “I will not store other pills in the nitroglycerin

bottle.” d. “I will discard any open bottle of nitroglycerin after

a year.”

Case Study Progress When you first admit G.P., you place him on telemetry and observe his cardiac rhythm.

7. Identify the rhythm:

(From Ignatavicius DD, Workman ML. Medical-Surgical Nursing, ed. 6, St. Louis, MO: Saunders; 2010.)

8. Explain the primary complication that could occur if this heart rhythm were not treated.

9. Review G.P.’s history. What conditions may have contributed to the development of this dysrhythmia?

10. You review G.P.’s lab test results and note that all of them are within normal range, including troponin and creatinine phosphokinase levels. His potassium level is 4.7 mEq/L (4.7 mmol/L). Given this and his current dysrhythmia, what is the likely cause of the symptoms he has been experiencing this past week?

Case Study Progress Within the hour, G.P. converts with intravenous diltiazem (Cardizem) to sick sinus syndrome with long sinus pauses that cause lightheadedness and hypotension.

11. What risks does the new rhythm pose for G.P.? Explain the reasons for your answers.

Case Study Progress Because G.P.’s dysrhythmia is causing unacceptable symptoms, he is taken to surgery and a permanent DDDR pacemaker is placed and set at a rate of 70.

12. What does the code DDDR mean? 13. What is the purpose of DDDR pacing? 14. The pacemaker insertion surgery places G.P. at risk for several

serious complications. List 3 potential problems you would monitor for as you care for him.

15. G.P. will need some education regarding his new pacemaker. What information will you give him before he leaves the hospital?

16. G.P. and his wife tell you they have heard that people with pacemakers can have their hearts stop because of microwave

ovens and cell phones. Where can you help them find more information?

Case Study Progress After discharge, G.P. is referred to a cardiac rehabilitation center to start an exercise program. He will be exercise tested, and an individualized exercise prescription will be developed for him, based on the results of the exercise test.

17. What information will be obtained from a graded exercise (stress) test?

18. What is included in an exercise prescription?

Case Study Outcome G.P. returns in 1 month for a pacemaker check. He reports that he and his wife go for a walk at least 3 times a week at the mall, and he is hoping to start volunteering soon. He has lost 8 lbs (3.6 kg).

Case Study 6 Name _________________________________ Class/Group ______________________________ Date ___________

Scenario S.P. is a 68-year-old retired painter who is experiencing right leg calf pain. The pain began approximately 2 years ago but has become significantly worse in the past 4 months. The pain is precipitated by exercise and is relieved with rest. Two years ago, S.P. could walk 2 city blocks before having to stop because of leg pain. Today, he can barely walk across the yard. S.P. has smoked 2 to 3 packs of cigarettes per day (PPD) for the past 45 years. He has a history of coronary artery disease (CAD), hypertension (HTN), peripheral artery disease (PAD), and osteoarthritis. Surgical history includes quadruple

coronary artery bypass graft 3 years ago. He has had no further symptoms of cardiopulmonary disease since that time, even though he has not been compliant with the exercise regimen his cardiologist prescribed, continues to eat anything he wants, and continues to smoke 2 to 3 PPD. Other surgical history includes open reduction and internal fixation of a right femoral fracture 20 years ago.

S.P. is in the clinic today for a routine semiannual follow-up appointment with his primary care provider. As you take his vital signs, he tells you that in addition to the calf pain, he is experiencing right hip pain that gets worse with exercise, the pain does not go away promptly with rest, some days are worse than others, and his condition is not affected by a resting position.

Chart View

General Assessment

Weight 261 lb (118.4 kg)

Height 5 ft, 10 in (178 cm)

BP 163/91

Pulse 82

Respiratory rate 16

Temperature 98.4° F (36.9° C)

Laboratory Testing (Fasting)

Cholesterol 239 mg/dL (6.2 mmol/L)

Triglycerides 150 mg/dL (1.69 mmol/L)

HDL 28 mg/dL (0.73 mmol/L)

LDL 181 mg/dL (4.69 mmol/L)

Current Medications

Ramipril (Altace) 10 mg daily

Metoprolol (Lopressor) 25 mg twice a day

Aspirin 81 mg daily

Atorvastatin (Lipitor) 20 mg daily

1. What are the likely sources of his calf pain and hip pain? 2. S.P. has several risk factors for PAD. From his history, list 2

risk factors, and explain the reason they are risk factors. 3. You decide to look at S.P.’s lower extremities. What signs do

you expect to find with PAD? Select all that apply. a. Ankle edema b. Thick, brittle nails c. Cool or cold extremity d. Thin, shiny, and taut skin e. Brown discoloration of the skin f. Decreased or absent pedal pulses

4. You ask further questions about the clinical manifestations of PAD. Which of these would you expect S.P. to have, given the diagnosis of PAD? Select all that apply.

a. Paresthesia b. Elevation pallor c. Dependent rubor d. Rest pain at night e. Pruritus of the lower legs f. Constant, dull ache in his calf or thigh

5. What is the purpose of the daily aspirin listed in S.P.’s current medication?

Case Study Progress S.P.’s primary care provider has seen him and wants you to schedule him for an ankle-brachial index (ABI) test to determine the presence of arterial blood flow obstruction. You confirm the time and date of the procedure and then call S.P. at home.

6. What will you tell S.P. to do to prepare for the tests?

Case Study Progress S.P.’s ABI results showed 0.43 right (R) leg and 0.59 left (L) leg. His primary care provider discusses these results with him and decides to wait 2 months to see whether his symptoms improve with drug changes and risk factor modification before deciding about surgical intervention. S.P. receives a prescription for clopidogrel (Plavix) 75 mg daily and is told to discontinue the daily aspirin. In addition, S.P. receives a consultation for physical therapy.

7. What do these ABI results indicate? 8. You counsel S.P. on risk factor modification. What would you

address, and why? 9. You provide teaching on proper care of his feet and lower

extremities, then use “teach-back” to assess S.P.’s learning. Which statements by S.P. indicate a need for further instruction? Select all that apply.

a. “I can go barefoot in the house, but not outside.” b. “I will wear shoes that are roomy and protective.” c. “I will avoid exposing my feet to extremes of heat

and cold.” d. “I will soak my feet in water once a day to make

sure they are clean.” e. “I will put lotion on my feet and lower legs, but not

in between the toes.” 10. How will the physical therapy help? 11. In addition to risk factor modification, what other measures to

improve tissue perfusion or prevent skin damage should you recommend to S.P.?

12. S.P. tells you his neighbor told him to keep his legs elevated higher than his heart and asks for compression stockings to keep swelling down in his legs. How should you respond?

13. S.P. has been on aspirin therapy but now will be taking

clopidogrel instead. What is the most important aspect of patient teaching that you will emphasize with this drug?

Case Study Outcome S.P. asks for nicotine patches to assist with smoking cessation and makes an appointment for a physical therapy evaluation and a nutritional assessment. He assures you he does not want to lose his leg and will be more careful in the future.

Case Study 7 Name _________________________________ Class/Group ______________________________ Date ___________

Scenario You are the nurse working in an anticoagulation clinic. One of your patients is K.N., who has a long-standing history of an irregular heartbeat, known as atrial fibrillation or A-fib, for which he takes the oral anticoagulant warfarin (Coumadin). Recently K.N. had his mitral heart valve replaced with a mechanical valve.

1. How does atrial fibrillation differ from a normal heart rhythm? 2. What is the purpose of the warfarin (Coumadin) in K.N.’s case?

Case Study Progress K.N. calls your anticoagulation clinic to report a nosebleed that is hard to stop. You ask him to come into the office to check his coagulation levels. The lab technician draws a PT/INR test.

3. What is a PT/INR test, and what are the expected levels for K.N.? What is the purpose of the INR?

4. When you get the results, his INR is critical at 7.2. What is

the danger of this INR level?

Case Study Progress The health care provider does a brief focused history and physical examination, orders additional lab tests, and determines there are no signs of bleeding other than the nosebleed, which has stopped. The provider discovers that K.N. recently started to take daily doses of an over-the-counter proton pump inhibitor (PPI), omeprazole (Prilosec OTC), for heartburn.

5. What happened when K.N. began taking the PPI? 6. What should K.N. have done to prevent this problem? 7. The provider gives K.N. a low dose of vitamin K orally, asks

him to hold his warfarin dose that evening, and asks him to come back tomorrow for another prothrombin time (PT) and INR blood draw. Why is K.N. instructed to take the vitamin K?

8. You want to make certain K.N. knows what “hold the next dose” means. What should you tell him?

9. K.N. asks you why his PT/INR has to be checked so soon. How will you respond?

Case Study Progress K.N.’s INR the next day is 3.7, and the health care provider makes no further medication changes. K.N. is instructed to return again in 7 days to have another PT/INR drawn.

10. Why should the INR be checked again so soon instead of the usual monthly follow-up?

11. K.N. grumbles about all of the lab tests but agrees to follow through. You provide patient education to K.N. and start with reviewing the signs and symptoms (S/S) of bleeding. What are potential S/S of bleeding that should be taught to K.N.? Select all that apply.

a. Insomnia b. Black, tarry stool

c. New onset of dizziness d. Stool that is pale in color e. New joint pain or swelling f. Unexplained abdominal pain

12. Identify 2 other patient education needs you will stress at this time.

13. K.N. tells you that he has had a lot of pain in his knee and wants to take ibuprofen (Advil) because it is an over-the- counter product. How do you reply to his request?

14. Four months later, K.N. informs you that he is going to have a knee replacement next month. What will you do with this information?

Case Study Progress You know that sometimes the only needed action is to stop the warfarin (Coumadin) several days before the surgery. Other times, the provider initiates “bridging therapy,” or stops the warfarin and provides anticoagulation protection by initiating low-molecular- weight heparin. After reviewing all of his anticoagulation information, the provider decides that K.N. will need to stop the warfarin (Coumadin) 1 week before the surgery and in its place be started on enoxaparin (Lovenox) therapy.

15. Compare the duration of action of warfarin (Coumadin) and enoxaparin (Lovenox) and explain the reason the provider switched to enoxaparin at this time.

Case Study Progress K.N. is in the office and ready for his first enoxaparin (Lovenox) injection.

16. Which nursing interventions are appropriate when administering enoxaparin? Select all that apply.

a. Massage the area after the injection has been given. b. Hold extra pressure over the site after the injection.

c. Monitor activated partial thromboplastin time (aPTT) levels.

d. The preferred site of injection is the lateral abdominal fatty tissue.

e. Administer via intramuscular (IM) injection into the deltoid muscle.

Case Study Progress K.N. undergoes knee surgery without complications. Just before his discharge, his physician reviews the instructions and gives him a new prescription for warfarin (Coumadin). K.N. tells his doctor, “I saw this commercial for a new blood thinner called Xarelto. I’d like to take that instead because I wouldn’t need to have all this blood work done.”

17. How do you expect the physician to respond?

Case Study Outcome K.N. is discharged to a rehabilitation facility, where he makes a quick recovery from the knee replacement surgery. He does not experience any thrombotic events or bleeding episodes during his recovery.

Case Study 8 Name _________________________________ Class/Group ______________________________ Date ___________

Scenario You are assigned to care for L.J., a 70-year-old retired bus driver who has just been admitted to your medical floor with right leg deep vein thrombosis (DVT). L.J. has a 48–pack-year smoking history, although he states he quit 2 years ago. He has had pneumonia several times and frequent episodes of atrial flutter or fibrillation. He has had 2 previous episodes of DVT and was diagnosed with rheumatoid arthritis 3 years

ago. Two months ago he began experiencing shortness of breath on exertion and noticed increasing swelling of his right lower leg that became progressively worse until it extended up to his groin. His wife brought him to the hospital when the pain in his leg became increasingly severe. After a Doppler study showed a probable thrombus of the external iliac vein extending distally to the lower leg, he was admitted for bed rest and to initiate heparin therapy. His basic metabolic panel was normal; other lab results were as follows.

Chart View

Laboratory Testing

PT 12.4 seconds

INR 1.11

aPTT 25 seconds

Hgb 13.3 g/dL (133 g/L)

Hct 38.9%

Cholesterol 206 mg/dL (5.34 mmol/L)

1. List 6 risk factors for DVT. 2. Identify at least 5 risk factors from L.J.’s history. 3. Something is missing from the scenario. Based on his history,

L.J. should have been taking an important medication. What is it, and why should he be taking it?

4. Keeping in mind L.J.’s health history and admitting diagnosis, outline the most important assessments you will make during your physical examination.

5. What is the most serious complication of DVT? 6. List at least 8 assessment findings you should monitor closely

for in the development of the complication identified in Question 5.

7. You review the literature for DVT and see the abbreviation VTE. What does VTE mean?

Case Study Progress Your assessment of L.J. reveals bibasilar crackles with moist cough, normal heart sounds, BP 138/88, pulse 104, 4 + pitting edema of right lower extremity, mild erythema of right foot and calf, and severe right calf pain. He is awake, alert, and oriented but a little restless. His Spo2 is 92% on room air. He denies chest pain but does have shortness of breath with exertion. He states he is anxious about missing his grandson’s wedding. He denies any voiding problems.

8. Your institution uses electronic charting. Based on the assessment noted previously, which of the following systems would you mark as “abnormal” as you document your findings? For abnormal findings provide a brief narrative note.

◻ Neurologic: ◻ Respiratory: ◻ Cardiovascular: ◻ Genitourinary: ◻ Skin: ◻ Psychosocial: ◻ Pain:

Case Study Progress L.J. is placed on 72-hour bed rest with bathroom privileges and given acetaminophen (Tylenol) for pain. The physician writes orders for enoxaparin (Lovenox) injections.

9. L.J. asks, “Why do I have to get these shots? Why can’t I just get a Coumadin pill to thin my blood?” What would be your response?” Explain your answer.

a. “Your physician prefers the injections over the pills.”

b. “The enoxaparin will work to dissolve the blood

clot in your leg.” c. “It would take the Coumadin pills several days to

become effective.” d. “Good idea! I will call and ask your physician to

switch medications.” 10. The order for the enoxaparin reads: Enoxaparin 70 mg

every 12 hours subQ. L.J. is 5 ft, 6 in tall and weighs 156 lb. Is this dose appropriate?

11. What special techniques do you use when giving the subcutaneous injection of enoxaparin? Select all that apply.

a. Rotate injection sites. b. Give the injection near the umbilicus. c. Massage the injection site gently after the injection

is given. d. After inserting the needle, do not aspirate before

giving the injection. e. Expel the bubble from the prefilled syringe before

giving the injection. 12. True or False? Enoxaparin dosage is directed by monitoring

aPTT levels. Explain your answer. 13. L.J. asks you how long it will take for the Lovenox injections to

dissolve his blood clot. What is your response to him? 14. After providing teaching about anticoagulant therapy,

you ask L.J. to teach back to you what he has learned. Which statements indicate a need for further education? Select all that apply.

a. “I will not blow my nose really hard.” b. “I will brush my teeth gently with a soft

toothbrush.” c. “I will take aspirin or ibuprofen if I have a

headache.” d. “I will shave very carefully with my disposable

razor.” e. “I will put lotion on my skin to keep it from getting

to dry.” f. “I will purchase and wear a medical alert necklace

for blood thinners.” g. “I will get help right away if I notice bleeding in my

stools or urine or if I have a bad headache or stomach pain.”

15. You identify pain as a key issue in the care of L.J. List 4 interventions you will choose for L.J. to address his pain.

16. What pertinent lab values and measurements would you expect the physician to order and the results of which you will monitor? Explain the reason for each test.

17. You evaluate L.J.’s electrocardiogram (ECG) strip. Name this rhythm, and explain what consequences it could have for L.J.

(Modified from Lilley LL, Rainforth Collins S, Harrington S, et al: Pharmacology and the Nursing Process, ed. 8, St. Louis, MO: Mosby; 2017.)

Case Study Progress A week has passed. L.J. responded to heparin therapy and was bridged to oral warfarin therapy. His heart dysrhythmia converted to sinus rhythm after he started taking cardiac medications, and he is being discharged to home with home care follow-up. “Good,” he says, “just in time to fly out west for my grandson’s wedding. His wife, who has come to pick him up, rolls her eyes and looks at the ceiling.

18. Although you are surprised at his comment, you realize he is serious about going to the wedding. What are you going to tell

him? 19. What discharge instructions about activity will you give L.J.?

Case Study Outcome L.J. listens to you, and his wife is quite relieved. They were able to watch the wedding ceremony via a live-stream connection, and he watches the recording daily and points out his favorite parts to the home care nurse every time she visits.

Case Study 9 Name _________________________________ Class/Group ______________________________ Date ___________

Scenario You are working at the local cardiac rehabilitation center, and R.M. is walking around the track. He summons you and asks if you could help him understand his recent lab report. He admits to being confused by the overwhelming data on the test and does not understand how the results relate to his recent heart attack and need for a stent. You take a moment to locate his lab reports and review his history. The findings are as follows.

R.M. is an active 61-year-old married man who works full time for the postal service. He spends most of his day in a mail truck, and admits he does not eat a “perfect diet.” He enjoys 2 or 3 beers every night, uses stick margarine, eats red meat 2 or 3 times per week, and is a self-professed “sweet eater.” He has tried to quit smoking and is down to 1 pack per day. Cardiac history includes a recent inferior myocardial infarction (MI) and a heart catheterization revealing three- vessel disease: in the left anterior descending (LAD) coronary artery, a proximal 60% lesion; in the right coronary artery (RCA), proximal 100% occlusion with thrombus; and a circumflex artery with 40% to 60% diffuse dilated lesions. A stent was deployed to the RCA and

reduced the lesion to 0% residual stenosis. He has had no need for sublingual nitroglycerin (NTG). He was discharged on enteric-coated aspirin 325 mg daily, clopidogrel (Plavix) 75 mg daily, atorvastatin (Lipitor) 10 mg at bedtime, and ramipril (Altace) 10 mg/day. Six weeks after his MI and stent placement, he had a fasting advanced lipid profile with other blood work.

Chart View

Six-Week Postprocedure Laboratory Work (Fasting)

Total cholesterol 188 mg/dL (4.87 mmol/L)

HDL 34 mg/dL (0.88 mmol/L)

LDL 98 mg/dL (2.54 mmol/L)

Triglycerides 176 mg/dL (1.99 mmol/L)

Homocysteine 18 mmol/L

C-reactive protein (CRP) 8 mg/dL (80 mg/L)

FBG 99 mg/dL (5.5 mmol/L)

TSH 1.04 mU/L

1. When you start to discuss R.M.’s lab values with him, he is pleased about his results. “My cholesterol level is below 200!— and my ‘bad cholesterol’ is good! That’s good news, right?” What would you say to him?

2. Which lab test is considered the “good cholesterol,” and why? 3. Discuss the significance of R.M.’s CRP level. 4. Discuss the significance of the homocysteine test and R.M.’s

results. 5. What else in R.M.’s history might be contributing to his

elevated homocysteine levels? 6. Identify R.M.’s health-related problems. List the problem that

is potentially life-threatening first.

Case Study Progress

7. R.M.’s physician adds niacin, a vitamin preparation (folic acid, vitamin B6, and vitamin B12 [Foltx]) daily with food, and omega-3 fatty acids to his list of medications. How do these medications affect lipids? R.M. states, “But I already take Lipitor. What do all these medications do?” How do you answer him?

8. You are teaching R.M. about the side effects of niacin. Which effects will you include in your teaching? Select all that apply.

a. Pruritus b. Dizziness c. Headache d. Flushed skin e. Gastrointestinal distress

9. R.M. tells you that he really does not want to “put up with” the side effects of the niacin. Is there an alternative to niacin?

10. You review his other medications, including atorvastatin (Lipitor). Which statement by R.M. indicates a need for further teaching about atorvastatin?

a. “I will take this drug at night.” b. “I will try to exercise more each week.” c. “I like to take my medicines with grapefruit juice.” d. “I will call my doctor right away if I experience

muscle pain.”

Case Study Progress You enter R.M.’s room and hear the physician say, “There are many options for changing your LDL and triglyceride levels. You need to continue modifying your diet and exercise to enhance your medication regimen. And stop smoking!” The physician asks R.M. whether he has any questions, and he responds, “No.”

11. After the physician leaves the room, R.M. tells you he really did not understand what the physician said. Explain the need for lifestyle changes to R.M.

12. You review the DASH diet with R.M. and his wife. Which food choices would follow the DASH diet? Select all that apply.

a. Fish and chips b. Fresh fruit salad c. Apples fried in butter and brown sugar d. Fat-free yogurt with tablespoon of almonds e. Grilled chicken sandwich on whole wheat bun

13. R.M. tells you that he knows exercise will help him to lose weight, which is good, but he does not understand how exercise helps his cholesterol levels. How do you answer him?

14. Of all of R.M.’s behaviors, which one is the most

significant in promoting cardiac disease and why? 15. Develop a comprehensive teaching plan directed toward

helping R.M. with addressing this behavior.

Case Study Outcome R.M. tells you that he is determined to stop smoking. At his next checkup 3 months later, he proudly shows off his 15 lb (6.8 kg) weight loss and tells you that he has not had a cigarette for 10 weeks.

Case Study 10 Name _________________________________ Class/Group ______________________________ Date ___________

Scenario The wife of C.W., a 70-year-old man, brought him to the emergency department (ED) at 0430. She told the ED triage nurse that he had diarrhea for the past 2 days and that last night he had a lot of “dark

red” diarrhea. When he became very dizzy, disoriented, and weak this morning, she decided to bring him to the hospital. C.W.’s vital signs (VS) in the ED were 70/− (systolic blood pressure [SBP] 70, diastolic blood pressure [DBP] inaudible), pulse rate 110, respirations 22, oral temperature 99.1° F (37.3° C). A 16-gauge IV catheter was inserted and a lactated Ringer’s infusion was started. The triage nurse learned C.W. has had idiopathic dilated cardiomyopathy for several years. The onset was insidious, but the cardiomyopathy is now severe. His last cardiac catheterization showed an ejection fraction of 13%. He has frequent problems with heart failure (HF) because of the cardiomyopathy. Two years ago, he had a cardiac arrest that was attributed to hypokalemia. He has a long history of hypertension and arthritis. He had atrial fibrillation in the past, but it has been under control recently. Fifteen years ago he had a peptic ulcer.

Endoscopy showed a 25- × 15-mm duodenal ulcer with adherent clot. The ulcer was cauterized, and C.W. was admitted to the medical intensive care unit (MICU) for treatment of his volume deficit. You are his admitting nurse. As you are making him comfortable, Mrs. W. gives you a paper sack filled with the bottles of medications he has been taking: enalapril (Vasotec) 5 mg PO bid, warfarin (Coumadin) 5 mg/day PO, digoxin (Lanoxin) 0.125 mg/day PO, potassium chloride 20 mEq PO bid, and diclofenac (Voltaren) 50 mg PO tid. As you connect him to the cardiac monitor, you note he is in sinus tachycardia. Doing a quick assessment, you find a pale man who is sleepy but arousable and slightly disoriented. He states he is still dizzy and feels weak and anxious overall. His BP is 98/52, pulse is 118, and respiratory rate 26. You hear S3 and S4 heart sounds and a grade II/VI systolic murmur. Peripheral pulses are all 2 +, and trace pedal edema is present. Capillary refill is slightly prolonged. Lungs are clear. Bowel sounds are present, mid-epigastric tenderness is noted, and the liver margin is 4 cm below the costal margin. Has not yet voided since admission. Rates his pain level as “2.” A Swan-Ganz pulmonary artery catheter and a peripheral arterial line are inserted.

1. What may have precipitated C.W.’s gastrointestinal (GI)

bleeding? 2. From his history and assessment, identify 5 signs and

symptoms of GI bleeding and loss of blood volume, and explain the pathophysiology for each one listed.

3. What is the most serious potential complication of C.W.’s bleeding?

4. Your institution uses electronic charting. Based on the assessment just described, which of the following systems would you mark as “abnormal” as you document your findings? Mark abnormal findings with an X and provide a brief narrative note.

☐ Neurologic: ☐ Respiratory: ☐ Cardiovascular: ☐ GI: ☐ Genitourinary: ☐ Musculoskeletal: ☐ Skin: ☐ Psychosocial: ☐ Pain:

5. What intervention is required to assess his renal function? 6. Calculate C.W’s mean arterial pressure (MAP) and explain

why this measure is important.

Case Study Progress As soon as you get a chance, you review C.W.’s admission lab results.

Chart View

Laboratory Results

Sodium 138 mEq/L (138 mmol/L)

Potassium 6.9 mEq/L (6.9 mmol/L)

BUN 90 mg/dL (32.1 mmol/L)

Creatinine 2.1 mg/dL (185.6 mcmol/L)

WBC 16,000/mm3 (16 x 109/L)

Hgb 8.4 g/dL (84 g/L)

Hct 25%

PT 23.4 seconds

INR 4.8

7. After examination of the lab results, do you have any concerns with C.W.’s electrolyte levels? Explain your answer.

8. In view of the previous lab results, what diagnostic test will be performed and why?

9. Evaluate this electrocardiogram (ECG) strip and note the effect of any electrolyte imbalances.

10. Why do you think the BUN and creatinine are elevated? 11. What do the low Hgb and Hct levels indicate about the

rapidity of C.W.’s blood loss? 12. What is the explanation for the prolonged PT/INR? 13. What will be your response to the prolonged PT/INR? Select

all that apply. a. Hold the warfarin dose. b. Avoid injections as much as possible. c. Obtain an order for aspirin if needed for pain.

d. Monitor C.W. for signs and symptoms of bleeding. e. Prepare to administer a STAT dose of protamine

sulfate. 14. What safety precautions should you initiate in light of his

prolonged PT and INR? 15. How do you explain the elevated WBC count?

Case Study Progress C.W. receives a total of 4 units of packed red blood cells (PRBCs), 5 units of fresh frozen plasma (FFP), and several liters of crystalloids to keep his mean BP above 60. On the second day in the MICU, his total fluid intake is 8.498 L and output is 3.66 L. His hemodynamic parameters after fluid resuscitation are pulmonary capillary wedge pressure (PCWP) 30 mm Hg and cardiac output (CO) 4.5 L/min.

16. Calculate his fluid balance and identify whether it is positive or negative.

17. Why will you want to monitor his fluid status very carefully? 18. List at least 6 things you will monitor to assess C.W.’s fluid

balance. 19. Explain the purpose of the FFP for C.W.

Case Study Progress Mrs. W. has been with her husband since he arrived at the emergency department and is worried about his condition and his care.

20. List 5 things you might do to make her more comfortable while her husband is in the MICU.

Case Study Outcome After the transfusions, C.W.’s BP is 110/78, his pulse is 94, and his respirations are 18. His urine output is now 60 mL/hour. He seems more alert and asks you, “What happened?” He is transferred to a step-down unit the next day and is eventually transferred to a

rehabilitation facility for a week of physical therapy before returning home.

Case Study 11 Name _________________________________ Class/Group ______________________________ Date ___________

Scenario J.F. is a 50-year-old married woman with a genetic autoimmune deficiency; she has had recurrent infective endocarditis. The most recent episodes were a Staphylococcus aureus infection of the mitral valve 16 months ago and a Streptococcus viridans infection of the aortic valve 1 month ago. During the latter hospitalization, an echocardiogram showed moderate aortic stenosis, moderate aortic insufficiency, chronic valvular vegetations, and moderate left atrial enlargement. Two years ago, J.F. received an 18-month course of parenteral nutrition (PN) for malnutrition caused by idiopathic, relentless nausea and vomiting (N/V). She has had coronary artery disease (CAD) for several years and 2 years ago had an acute anterior wall myocardial infarction (MI). In addition, she has a history of chronic joint pain.

Now, after having been home for only a week, J.F. has been readmitted to your floor with infective endocarditis (IE), N/V, and renal failure. Since yesterday, she has been vomiting and retching constantly. She also

Chart View

Admission Orders

STAT blood cultures (aerobic and anaerobic) × 2, 30 minutes apart

STAT CMP & CBC Begin PN at 85 mL/hr Piperacillin sodium/tazobactam sodium (Zosyn) 2.25 g q6hr Vancomycin (Vancocin), renal dosing per pharmacy, IVPB

q12hr Furosemide (Lasix) 80 mg PO daily Amlodipine (Norvasc) 5 mg PO daily Potassium chloride (K-Dur) 40 mEq PO daily Metoprolol (Lopressor) 25 mg PO bid Ondansetron 4 mg IV every 6 hours PRN Transesophageal echocardiogram ASAP

Admission Assessment

Oriented × 3 to person, place, and time, but drowsy Grade II/VI holosystolic murmur and a grade III/VI diastolic

murmur Lungs clear bilaterally Abdomen soft with slight left upper quadrant tenderness Multiple petechiae on skin of arms, legs, and chest; splinter

hemorrhages under the fingernails; hematuria noted in voided urine

Blood pressure 152/48 (supine) and 100/40 (sitting)

Pulse rate 116

Respiratory rate 22

Temperature 100.2° F (37.9° C)

has had chills, fever, fatigue, joint pain, and headache. As you go through the admission process with her, you note that she wears glasses and has dentures. Intravenous (IV) access is obtained with a double-lumen peripherally inserted central catheter (PICC) line. Other orders and your assessment are shown in the box.

1. Which of these statements about IE are true? Select all that apply.

a. IE may affect the heart valves. b. IE is an inflammation of the pericardial sac. c. IE is an infection of the innermost layer of the heart. d. Cardiac tamponade is a common complication of

IE. e. Heart failure, sepsis, and dysrhythmias may occur

with IE. 2. What is the significance of the orthostatic hypotension and

tachycardia? 3. What is the significance of the abdominal tenderness,

hematuria, joint pain, and petechiae? 4. What are splinter hemorrhages, and how are they related to IE? 5. Mark the area on the accompanying diagram where you would

place the stethoscope to auscultate an aortic valve murmur.

(From Jarvis C. Physical Examination and Health Assessment. 6th ed. St. Louis, MO: Saunders; 2012.)

6. As you monitor J.F. throughout the day, what other signs and symptoms of embolization will you watch for?

7. Explain the diagnostic criteria for infectious endocarditis.

Case Study Progress The next day, you review J.F.’s lab test results.

Chart View

Laboratory Test Results

Na 138 mEq/L (138 mmol/L)

K 3.9 mEq/L (3.9 mmol/L)

Cl 103 mEq/L (103 mmol/L)

BUN 85 mg/dL (30.3 mmol/L)

Creatinine 3.9 mg/dL (345 mcmol/L)

Glucose 165 mg/dL (9.2 mmol/L)

WBC 6700/mm3 (6.7 x 109/L)

Hct 27%

Hgb 9.0 g/dL (90 g/L)

8. Identify the values that are not within normal ranges and explain the reason for each abnormality.

9. You note that a new intern writes an order for “Fasting blood glucose levels daily.” Is this order appropriate for J.F.? Explain.

10. What is the greatest risk for J.F. during the process of

rehydration, and what would you monitor to detect its development?

Case Study Progress You were aware that as soon as J.F. became stable, she would be going home on parenteral nutrition (PN) and IV antibiotics. As part of the discharge preparations, you contact the home care agency that will be providing her care

11. List 5 important questions in assessing her home health care needs.

Case Study Progress Fortunately, J.F. has a supportive husband and 2 daughters who live nearby who can function as caregivers when J.F. is discharged. They,

along with J.F., will need teaching about endocarditis. Although J.F. has been ill for several years, you discover that she and her family have received little education about the disease. You prepare a teaching plan for the family.

12. Develop a teaching plan for J.F. and her family. 13. J.F.’s daughter tells you that she thinks her mother will have to

stay in bed until the infection is cured. How will you respond, and what measures can be implemented to prevent problems related to decreased mobility?

14. After you have taught J.F. about oral hygiene, which statement by J.F. reflects a need for further education?

a. “I will use a soft toothbrush to brush my teeth.” b. “I will use a water irrigation device to clean my

teeth and gums.” c. “I will rinse my mouth thoroughly with water after

brushing my teeth.” d. “I will remove my dentures after every meal and

clean them thoroughly before replacing them.”

Case Study Progress Your hospital discharge planner facilitates J.F.’s transition to home care. During the initial home visit, the home health nurse evaluates J.F.’s IV site for implementation of the IV therapy program. The nurse interviews the family members to determine their willingness to be caregivers and their level of understanding and enlists the patient’s and family’s assistance to identify goals.

15. The home health nurse also writes short- and long-term goals for J.F. and her family. Identify 2 short-term and 3 long-term goals.

Case Study Outcome Mr. F. and his 2 daughters learned to administer J.F.’s antibiotic and 8- month treatment of PN. J.F.’s endocarditis eventually resolves with no

worsening of her cardiac condition.

Case Study 12 Name _________________________________ Class/Group ______________________________ Date ___________

Scenario Your patient, 58-year-old K.Z., has a significant cardiac history. He has long-standing coronary artery disease (CAD) with occasional episodes of heart failure (HF). One year ago, he had an apical myocardial infarction (MI). In addition, he has chronic anemia, hypertension, chronic renal insufficiency, and a recently diagnosed 4- cm suprarenal abdominal aortic aneurysm. Because of his severe CAD, he had to retire from his job as a railroad engineer about 6 months ago. This morning, he is being admitted to your telemetry unit for a same-day cardiac catheterization. As you take his health history, you note that his wife died a year ago (at about the same time he had his MI) and he does not have any children. He is a current cigarette smoker with a 50-pack-year smoking history. His vital signs are 158/94, 88, 20, and 97.2° F (36.2° C). As you talk with him, you realize he has only a minimal understanding of the catheterization procedure.

1. Before he leaves for the cath lab, you briefly teach him the important things he needs to know before having the procedure. List 5 priority topics you will address.

2. Look at his past history. What other factors are present that could contribute to his risk for cardiac ischemia?

Case Study Progress Several hours later, K.Z. returns from his catheterization. The catheterization report shows 90% occlusion of the proximal left anterior descending (LAD) coronary artery, 90% occlusion of the

distal LAD, 70% to 80% occlusion of the distal right coronary artery (RCA), an old apical infarct, and an ejection fraction (EF) of 37%. About an hour after the procedure is finished, you perform a brief physical assessment and note a grade III/VI systolic ejection murmur at the cardiac apex, crackles bilaterally in the lung bases, and trace pitting edema of his feet and ankles. Except for the soft systolic murmur, these findings were not present before the catheterization.

3. Using the following diagram, identify the superior vena cava, the aorta, and the left and right ventricles. Identify the main coronary arteries and circle the areas of the LAD and RCA that have significant occlusion, as identified in the previous report. Lightly shade the area of the heart where K.Z. had the earlier infarct.

(Modified from Fuller JK. Surgical Technology: Principles and Practice. 5th ed. St. Louis, MO: Saunders; 2010.)

4. What is your evaluation of the catheterization results? 5. Explain the significance of having an EF of 37%. 6. What problem(s) do(es) the changes in assessment findings

suggest to you? What led you to your conclusion? 7. List 5 actions you should take as a result of your evaluation of

the assessment and state your rationale for each. 8. You decide to notify the physician of K.Z.’s condition. Using

SBAR (Situation, Background, Assessment, Recommendation), what information would you provide to the physician when you call?

Case Study Progress After assessing K.Z., the physician admits him with a diagnosis of CAD and HF and plans coronary artery bypass graft (CABG) surgery. Results of significant lab tests performed at this time are Hct 25.3%, Hgb 8.8 g/dL (88 g/L), BUN 33 mg/dL (11.8 mmol/L), and creatinine 3.1 mg/dL (274 mcmol/L). K.Z. is given furosemide (Lasix) and 2 units of packed red blood cells (PRBCs).

9. Review K.Z.’s health history. Can you identify a probable explanation for his chronic renal insufficiency and anemia?

10. Why is he receiving 2 units of PRBCs? What is the purpose of the furosemide?

Case Study Progress Two days later, after his condition is stabilized, K.Z. is taken to surgery for a three-vessel coronary artery bypass graft (CABG × 3 V). When he arrives in the surgical intensive care unit, he has a Swan- Ganz catheter in place for hemodynamic monitoring and is intubated. He is put on a ventilator at Fio2 0.70 and positive end-expiratory pressure (PEEP) at 5 cm H2O. His latest Hgb is 10.3 mg/dL (103 g/L). You review his first hemodynamic readings and arterial blood gases.

Chart View

Hemodynamic Readings

Pulmonary artery pressure (PAP) 38/23 mm Hg

Central venous pressure (CVP) 14 mm Hg

Pulmonary artery wedge pressure (PAWP) 18 mm Hg

Cardiac index (CI) 1.88 L/min/mm2

Arterial Blood Gases

pH 7.37

Paco2 46 mm Hg

Pao2 61 mm Hg

Sao2 85%

11. Why are ABG values necessary in K.Z.’s case? Explain why it would be inappropriate to use pulse oximetry to assess his O2 saturation status.

12. What is your interpretation of his ABG values on 70% oxygen? 13. What is your evaluation of K.Z.’s hemodynamic status, based

on the results displayed? 14. Do you think the hemodynamic values reported previously

reflect poor left ventricular function or fluid overload? Defend your answer.

15. K.Z. is receiving continuous IV infusions of norepinephrine (Levophed) and dobutamine. Why is K.Z. receiving these medications?

16. What assessment findings would indicate that these drugs are having a therapeutic effect?

17. What are the major side effects of norepinephrine and dobutamine, and what do you monitor while these drugs are infusing?

18. K.Z. states that he is feeling more “skipping beats,” even while lying quietly in the bed. What will you do next?

a. Stop the dobutamine immediately. b. Assess his vital signs and cardiac rhythm. c. This is a normal side effect; no interventions are

needed. d. Titrate the dobutamine to a higher dose to reduce

the palpitations.

Case Study Progress After 3 days in the SICU, K.Z.’s condition was stable and he was returned to your telemetry floor. Now, 5 days later, he is ready to go home and you are preparing him for discharge.

19. List at least 4 general areas related to his CABG surgery in which he should receive instruction before he goes home.

Case Study Outcome K.Z. decided to sell his home and move to a seniors’ apartment complex. He completed the cardiac rehabilitation program and became a volunteer to support others who have had heart surgery. He has not had a cigarette since his surgery.

Case Study 13 Name _________________________________ Class/Group ______________________________ Date ___________

Scenario R.K. is an 85-year-old woman who lives with her husband, who is 87. Two nights before her admission to your cardiac unit, she awoke with heavy substernal pressure accompanied by epigastric distress. The pain was reduced somewhat when she rolled onto her side but did not

completely subside for about 6 hours. The next night, she experienced the same chest pressure. The following morning, R.K.’s husband took her to the physician, and she was subsequently hospitalized to rule out myocardial infarction (MI). Lab specimens were drawn in the emergency department. She was given 325 mg chewable, non–enteric- coated aspirin, and an IV line was started. She was placed on oxygen (O2) at 2 L via nasal cannula.

You obtain the following information from your history and physical examination: R.K. has no history of smoking or alcohol use, and she has been in good general health, with the exception of osteoarthritis of her hands and knees and some osteoarthritis of the spine. Her only medications are simvastatin (Zocor), ibuprofen as needed for bone and joint pain, and “herbs.” Her admission vital signs (VS) are blood pressure 132/84, pulse 88, respirations 18, and oral temperature 99° F (37.2° C). Her weight is 114 lbs (51.7 kg) and height is 5 ft, 4 in (163 cm). Moderate edema of both ankles is present; capillary refill is brisk, and peripheral pulses are 1 +. You hear a soft systolic murmur. She denies any discomfort at present. You place her on telemetry, which shows the rhythm in the following figure.

1. Identify her cardiac rhythm.

(Modified from Huszar R. Basic Dysrhythmias: Interpretation and Management—Revised Reprint. ed. 3, St.

Louis, MO: Mosby; 2007.)

2. Give at least 2 reasons for inserting an IV line. 3. Explain the purpose of the aspirin tablet. Why is “non–enteric-

coated” aspirin specified? What would be a contraindication to administering aspirin?

4. What additional history and physical information should you obtain related to her admitting diagnosis? Name at least 4 for each.

5. List 7 lab or diagnostic tests you would expect to be performed; suggest what each might contribute.

6. What other sources, in addition to cardiac ischemia, might be responsible for her chest and abdominal discomfort?

7. Define the concept of differential diagnosis and explain how the concept applies to R.K.’s symptoms.

Case Study Progress After some rest, R.K.’s chest pain has subsided, and she tells you she feels much better now. You review her lab results.

Chart View

Laboratory Results

12-Lead ECG: Light left-axis deviation, normal sinus rhythm, with no ventricular ectopy

Cardiac troponin T is less than 0.01 ng/mL (0.01 mcg/L) (at admission) and same result 4 hours after admission

Cardiac troponin T is less than 0.03 ng/mL (0.03 mcg/L) (at admission) and the same 4 hours after admission

Serial CPK tests are 30 units/L at admission, 32 units/L 4 hours after admission

Copeptin 5 pmol/L at admission, 5.1 pmol/L 4 hours after admission

d-Dimer test result less than 250 ng/mL (250 mcg/L)

8. On the basis of the information presented so far, do you believe she had an MI? What is your rationale?

9. Do you think she may have a pulmonary embolus? 10. While you care for R.K., you carefully observe her. Identify 2

possible complications of coronary artery disease (CAD) and the signs and symptoms associated with each.

11. R.K. rings her call bell. When you arrive, she has her hand placed over her heart and tells you she is “having that heavy feeling again.” She is not diaphoretic or nauseated, but states she is short of breath. Use the PQRST Assessment of Angina to assess her episode of chest pain. What questions would you ask for each factor?

12. What else do you assess, and what priority actions does the nurse need to take right now?

Case Study Progress During the episode of chest pain, R.K.’s vital signs were as follows: BP 140/92, P 110, R 20. The rhythm strip shows sinus tachycardia, and she was very anxious. Her chest discomfort subsided in 3 minutes after 1 nitroglycerin (NTG) dose, and she is resting quietly with O2 per nasal cannula at 2 L/min. R.K.’s physician is making rounds.

13. Using SBAR (Situation, Background, Assessment, Recommendation), how would you communicate this episode to R.K.’s physician?

Case Study Progress R.K.’s husband is upset. He tells you they have been married for 62 years and he does not know what he would do without his wife. One way to help people deal with their anxieties is to help them focus on concrete issues.

14. What information would be useful to get from him? What

other health care professional might be able to help with some of these issues?

Case Study Progress The cardiologist diagnosed R.K. with angina associated with coronary artery disease. She has had no further episodes of chest pain and is discharged to home the next day. She is to see a cardiologist this week and set up an appointment for outpatient testing. As you present the discharge instructions, you review the proper technique for taking sublingual NTG for chest pain.

15. Using the teach-back method, you ask R.K. what to do if she experiences chest pain. Which statement by R.K. indicates that further teaching is needed? Explain your answer.

a. “If I have chest pain, I will place 1 nitroglycerin tablet under my tongue.”

b. “At the first sign of chest discomfort, I will stop what I’m doing and sit down.”

c. “If the chest pain does not stop or ease up, I can take another tablet in 5 minutes.”

d. “My husband will need to call 911 if the chest pain does not stop after 3 nitroglycerin tablets.”

16. R.K. tells you that she hates the headache that happens after she takes a nitroglycerin tablet. What can you suggest to her for this problem?

17. What essential safety point will you emphasize when

discussing sublingual nitroglycerin with R.K.?

Case Study Outcome R.K.’s outpatient testing showed coronary artery disease, and the cardiologist recommended medical treatment at this time. She has not experienced an increased number of episodes of angina.

Case Study 14

Name _________________________________ Class/Group ______________________________ Date ___________

Scenario The time is 1900. You are working in a small, rural hospital. It has been snowing heavily all day, and the medical helicopters at the large regional medical center, 4 hours away by car (in good weather), have been grounded by the weather until morning. The roads are barely passable. W.R., a 48-year-old plumber with a 36-pack-year smoking history, is admitted to your floor with a diagnosis of rule out myocardial infarction (R/O MI). He has significant male-pattern obesity (“beer belly,” large waist circumference) and a barrel chest and reports a dietary history of high-fat food. His wife brought him to the emergency department after he complained of unrelieved “indigestion.” His admission vital signs (VS) were BP 202/124, pulse 106, respirations 18, and oral temperature 98.2° F (36.8° C). W.R. was put on oxygen (O2) by nasal cannula (NC) titrated to maintain Spo2 over 92% and started on an IV nitroglycerin (NTG) infusion. He was given aspirin 325 mg to chew and swallow and was admitted to Dr. A.’s service. There are plans to transfer him by helicopter to the regional medical center for a cardiac catheterization in the morning when the weather clears. Meanwhile, you have to deal with limited lab and pharmacy resources. The minute W.R. comes through the door of your unit, he announces he’s “just fine” in a loud and angry voice and demands a cigarette. He also says he has no time to fool around with hospitals.

1. What is the first priority in his care? 2. Are these VS typical for a man of his age? If not, which one(s)

concern(s) you? Explain why or why not. 3. Identify 5 priority problems associated with the care of a

patient such as W.R. 4. Which lab tests might be ordered to investigate W.R.’s

condition? If the order is appropriate, place an A in the space

provided. If inappropriate, mark with an I. Provide rationales for your decisions.

_____1. Complete blood count (CBC) _____2. Electroencephalogram (EEG) in the morning _____3. Basal metabolic panel (BMP) _____4. Prothrombin time (PT) and partial

thromboplastin time (PTT) _____5. Bilirubin _____6. Urinalysis (UA) _____7. STAT 12-lead electrocardiogram (ECG) and

repeat in the morning _____8. Type and crossmatch for 2 units of packed red

blood cells (PRBCs) _____9. Chest x-ray on admission and in the morning

5. What significant lab tests are missing from the previous list? 6. How are you going to respond to W.R.’s angry demands for a

cigarette? He also requests something for his “heartburn.” How will you respond?

7. Mrs. R. asks you, “If he can’t smoke, why can’t you give him one of those nicotine patches or some nicotine gum?” How will you respond?

8. Are there any alternatives to help him with his nicotine cravings? Would they be helpful now?

Case Study Progress At 2000, you phone Dr. A.’s partner, who is on call. She prescribes morphine sulfate 4 mg STAT IV push (IVP), then 2 to 4 mg IVP q1hr prn for pain (burning, pressure, and angina).

9. Explain 2 reasons for this order. 10. What special precautions should you follow when

administering morphine sulfate via IVP? 11. The pharmacy supplies morphine for injection in vials of

5 mg/mL only. For the first dose, you will be giving 4 mg of

morphine. How many milliliters will you give for this dose? Mark the syringe with your answer.

(From Gray Morris D. Calculate with Confidence. 5th ed. St. Louis, MO: Mosby; 2010.)

12. What will you do with the rest of the morphine in the vial? a. Discard it b. Save it for the next dose c. Return it to the pharmacy d. Discard it with a second witness

13. Angina is not always experienced as “pain” as many people understand pain. How would you describe symptoms you want him to warn you about? Why is this important?

14. What safety measures or instructions would you give

W.R. before you leave his room? 15. Mrs. R. is unable to leave the hospital because of the bad

weather. She approaches you and asks, “Did my husband have a heart attack? I’m really scared. His father died of one when he was 51.” How are you going to respond to her question?

Case Study Progress When you come into W.R.’s room at 2230 hours to answer his call light, you see he is holding his left arm and complaining about aching in his left shoulder and arm.

16. What information will you gather? What questions will you

ask him?

Case Study Progress You titrate the NTG drip up, assess whether he is using the oxygen cannula, and assess his vital signs. In addition, you administer a dose of morphine, but his pain is not relieved. Based on your assessment findings, you decide to call the physician.

17. Using SBAR (Situation, Background, Assessment, Recommendation), what information would you provide to the physician when you call?

18. W.R.’s chest pain subsides after the dose of morphine and he settles down for the night. You monitor him closely and watch for side effects of the NTG infusion. Side effects of NTG include which of these? Select all that apply.

a. Headache b. Tachycardia c. Constipation d. Postural hypotension e. Decreased respirations

Case Study Progress In the morning, W.R. is transferred by helicopter to the medical center, and a cardiac catheterization is performed. It is determined that W.R. has coronary artery disease (CAD) but has not had an MI. The cardiologist suggests it would be best to treat him medically for now.

19. What does it mean to treat him “medically”? What other approaches might be used to treat CAD?

20. A new order for atenolol (Tenormin) is added to his medication list. Which is/are a rationale for starting a beta blocker at this time? Select all that apply.

a. Reduction of myocardial stimulation b. Increased force of cardiac contractions c. Decreased myocardial oxygen demand

d. Prolonged sinoatrial (SA) node recovery e. Increased conduction through the atrioventricular

(AV) node

Case Study Outcome The physician orders follow-up counseling regarding risk factor modification, especially smoking cessation, hypertension management, weight loss, and lipid (cholesterol) management. W.R. is discharged with a referral for a follow-up visit to his local internist in 1 week.

Case Study 15 Name _________________________________ Class/Group ______________________________ Date ___________

Scenario You are just getting caught up with your work when you receive the following phone call: “Hi, this is Deb in the emergency department (ED). We’re sending you M.M., a 63-year-old Hispanic woman with a past medical history of coronary artery disease (CAD). Her daughter reports that her mom has become increasingly weak over the past couple of weeks and has been unable to do her housework. Apparently, she has had swelling in her ankles and feet by late afternoon so much that she could not wear her shoes and has nocturnal diuresis × 4. Her daughter brought her in because she has had heaviness in her chest off and on over the past few days but denies any discomfort at this time. She says that the chest heaviness is not related to activity and has become increasingly more frequent over the past few days, sometimes lasting up to 10 minutes at a time. The daughter took her to see her family physician, who immediately sent her here. Vital signs are 146/92, 96, 24, 99° F (37.2° C). She has an IV of D5W at 50 mL/hr in her right forearm. Her lab results are as

follows: Na 134 mEq/L (134 mmol/L), K 3.5 mEq/L (3.5 mmol/L), Cl 103 mEq/L (103 mmol/L), HCO3 23 mEq/L (23 mmol/L), BUN 13 mg/dL (4.6 mmol/L), creatinine 1.3 mg/dL (115 mmol/L), glucose 153 mg/dL (8.5 mmol/L), WBC 8300/mm3 (8.3 x 109/L), Hct 33.9%, Hgb 11.7 g/dL (117 g/L), platelets 162,000/mm3 (162 x 109/L), PT/INR/PTT, and urinalysis are pending. She has had her chest x-ray and ECG, and her orders have been written.”

1. What additional information do you need from the emergency department nurse?

2. How are you going to prepare for this patient? 3. M.M. arrives by wheelchair. As she transfers to the bed, what

observations will you make? Why? 4. With the interpreter phone, M.M. tells you that she feels very

tired. Is this symptom significant? Explain your answer. 5. Based on M.M.’s history, you suspect that she is experiencing

angina. Which type of angina do you think she has? Explain your answer.

6. Given the previous information, you expect orders for M.M. Carefully review each to determine whether it is appropriate or inappropriate as written. If the order is appropriate, mark it as A; if the order is inappropriate, mark it as I and change the order to make it appropriate. Provide any other orders that might be appropriate for M.M.

_____1. VS once per shift _____2. Serum magnesium (Mg) STAT _____3. Up ad lib _____4. 10 g sodium (Na), low-fat diet _____5. Change IV to a saline lock _____6. Cardiac enzymes on admission and q8hr ×

24 hr, then daily every morning _____7. CBC, BMP, and fasting lipid profile in

morning _____8. Schedule for abdominal CT scan for morning _____9. Heparin 10,000 units subQ q8hr

_____10. Docusate sodium (Colace) 100 mg PO daily _____11. Ampicillin 250 mg IV piggyback q6hr _____12. Furosemide (Lasix) 200 mg IV push STAT _____13. Nitroglycerin (NTG) 0.4 mg 1 SL q4hr prn for

chest pain _____14. Schedule echocardiogram

7. Which interventions are appropriate for administering subcutaneous heparin? Select all that apply.

a. Massage the area after the injection. b. Rotate injection sites with each dose. c. Do not aspirate the syringe before injecting the

heparin. d. Give the injection at least 2 inches (5 cm) away from

the umbilicus. e. Monitor activated partial thromboplastin time

(aPTT) levels daily.

Case Study Progress Shortly after admission, M.M.’s call light comes on. When you respond to M.M.’s call light, you observe she is talking rapidly in Spanish and pointing to the bathroom. Her speech pattern indicates she is short of breath; she is having trouble completing a sentence without taking a labored breath. You help her use a bedpan and note that her skin feels clammy. While sitting on the bedpan, she vomits.

8. On a scale of 0 to 10 (0 being no problem, 10 being a code-level emergency), how would you rate this situation, and why?

9. Identify at least 4 actions you should take next and state your rationale.

10. M.M.’s physician calls your unit to find out what is happening. Using SBAR, what information would you need to convey at this time?

11. The hospital’s staff physician is coming to the floor immediately to evaluate the patient. In the meantime, she orders furosemide (Lasix) 40 mg IV push STAT. You have only

20 mg in stock. Should you give the 20 mg now, and then give the additional 20 mg when it comes up from the pharmacy? Explain your answer.

12. M.M. continues to experience vomiting and diaphoresis that are unrelieved by medication and comfort measures. A STAT 12-lead ECG reveals ischemic changes, and she is transferred to the coronary care unit. As you give the report to the receiving registered nurse, what lab value is the most important to report, and why?

13. You are observing while a new nurse prepares to administer IV potassium to M.M. Which technique is correct? Explain why the other answers are incorrect.

a. Give the IV potassium by slow IV push. b. Administer the IV potassium by gravity drip. c. Add potassium to a hanging IV bag as needed. d. The rate of IV administration should not exceed

10 mEq/hr (10 mmol/L).

Case Study Progress A case manager has been asked to evaluate M.M.’s home to see whether she can be discharged to her own home or will need to stay in a long-term care facility.

14. Identify at least 8 things that the case manager would assess. 15. M.M.’s nutritional intake over the past few weeks has been

poor. What are some of the nutritional needs that should be met? What would you recommend to help her with this?

Case Study Progress Because the case manager determined that M.M. lived in an apartment with poor access, M.M. elects to stay with her daughter and 5 grandchildren in their small home. A home care nurse comes 3 times a week to check on her. M.M. is easily fatigued, and the children are quite lively. School is out for the summer.

16. Suggest some ways for M.M.’s daughter to ensure that her mother is not overwhelmed and does not become exhausted in this situation.

Case Study Outcome M.M. stays with her daughter for 2 weeks, but the active children are too much for her and she moves back to her apartment. Her daughter checks on her there daily and brings her meals.

Case Study 16 Name _________________________________ Class/Group ______________________________ Date ___________

Scenario You are in the middle of your shift in the coronary care unit (CCU) of a large urban medical center. Your new admission, C.B., a 47-year-old woman, was just flown to your institution from a small, rural community more than 100 miles away. She had a STEMI (ST-segment- elevation myocardial infarction) last evening. Her current vital signs (VS) are 100/60, 86, 14. After you make C.B. comfortable, you receive this report from the flight nurse: “C.B. is a full-time homemaker with 4 children. She has had episodes of “chest tightness” with exertion for the past year, but this is her first known myocardial infarction (MI). She has a history of hyperlipidemia and has smoked 1 pack of cigarettes daily for 30 years. Surgical history consists of total abdominal hysterectomy 10 years ago after the birth of her last child. She has no other known medical problems. Yesterday at 8 p.m. she began to have severe substernal chest pain that referred into her neck and down both arms. She rated the pain as 9 or 10 on a scale of 0 to 10. She thought it was severe indigestion and began taking Maalox with no relief. Her husband then took her to the local emergency department, where a 12-lead electrocardiogram (ECG) showed

hyperacute ST elevation in the inferior leads II, III, aVF, and V5 to V6. Before any interventions could be started, she went into ventricular fibrillation (V-fib). CPR was started and when the code team arrived, she was successfully defibrillated after 2 shocks. She then was started on nitroglycerin (NTG), heparin, and amiodarone drips. She was given IV metoprolol (5 mg every 2 minutes for a total of 3 doses) and aspirin 325 mg to chew and swallow. This morning her systolic pressure dropped into the 80s, and she was placed on a low-dose norepinephrine drip and urgently flown to your institution for coronary angiography and possible percutaneous transluminal coronary angioplasty. Currently, she has amiodarone infusing at 1 mg/min, heparin at 18 units/kg/minute, and norepinephrine at 0.5 mcg/kg/min. The NTG has been stopped because of low blood pressure. Lab work that was done yesterday showed Na 145 mEq/L (145 mmol/L), K 3.6 mEq/L (3.6 mmol/L), HCO3 19 mEq/L (19 mmol/L), BUN 9 mg/dL (3.2 mmol/L), creatinine 0.8 mg/dL (70 mcmol/L), WBC 14,500/mm3 (14.5 x 109/L), Hct 44.3%, and Hgb 14.5 g/dL (145 g/L).”

1. Because the 12-lead ECG can tell you the location of the infarction, evaluate the leads that showed ST elevation. What areas of C.B.’s heart have been damaged?

2. Given the diagnosis of acute ST-segment–elevation myocardial infarction (STEMI), what other lab results are you going to review?

3. For each of the characteristics listed below, specify whether they are associated with a STEMI or an NSTEMI (non-ST- segment–elevation MI).

____a. Caused by a nonocclusive thrombus. ____b. Caused by an occlusive thrombus. ____c. An emergency situation; the artery must be

opened within 90 minutes of presentation. ____d. Patients usually undergo catheterization within

12 to 72 hours of presentation. ____e. A 12-lead ECG will show ST segment elevation.

____f. The patient will not need thrombolytic therapy. ____g. Percutaneous coronary intervention (PCI) is the

first-line treatment. 4. Indicate the expected outcome for C.B. associated with each

medication she is receiving. For each drug listed, state the purpose.

a. IV heparin b. IV amiodarone c. IV metoprolol d. Aspirin, chewed and swallowed e. IV norepinephrine

Chart View

Laboratory Test Results Creatine Phosphokinase (CPK) Levels

On ED admission 95 units/L

4 hours 1931 units/L

8 hours 4175 units/L

Creatine Kinase–Myocardial Bound (CK-MB) Isoenzymes

On ED admission 5%

4 hours 79%

8 hours 216%

Troponin T

On ED admission 11 ng/mL (11 mcg/L)

6 hours 30 ng/mL (30 mcg/L)

Troponin I

On ED admission 3.9 ng/mL (3.9 mcg/L)

6 hours 9 ng/mL (9 mcg/L)

LDL 160 mg/dL (4.1 mmol/L)

PT 11.9 sec

INR 1.02

aPTT (before heparin) 26.9 sec

Mg 2.2 mg/dL (0.9 mmol/L)

K 3.3 mEq/L (3.3 mmol/L)

5. You review the lab work on her chart. For each lab value listed previously, interpret the result and evaluate the meaning for C.B.

6. List the 2 primary complications C.B. is at risk for at this time and the assessments that are needed to identify these risks.

7. You note that C.B.’s Spo2 on oxygen at 6 L/min by nasal cannula is 92%. How do you interpret this result?

8. What can be done to promote her oxygenation at this time? 9. An hour after her admission, you are preparing C.B. for her

coronary intervention. Evaluate her readiness for teaching and her learning needs. What would you tell her?

Case Study Progress The following day, you care for C.B. again. She is now on oral metoprolol, amiodarone, aspirin, and clopidogrel (Plavix). The norepinephrine and heparin have been discontinued. VS are stable.

10. Which lab test result should you check before beginning the clopidogrel therapy?

a. aPTT b. PT/INR c. Potassium

d. Platelet count

Case Study Progress As you work with C.B., you notice that she is extremely anxious. You had observed some anxiety yesterday, which you had attributed to the strange CCU environment, pain, and anticipation of the stenting procedure. The postprocedure test results showed that the stent was performing appropriately. You wonder what is wrong. She tells you that her heart attack occurred right in the middle of a move with her family from her rural community to an even smaller and unfamiliar town some 500 miles away in a neighboring state. She is dreading the move. Her husband “becomes angry easily and starts lashing out” toward her and the children. She is afraid to move to a community where she will have no friends and family to support her.

11. How can you help your patient? Evaluate the situation and describe possible interventions.

Case Study Outcome C.B. agrees to speak with a social worker, and you set up the meeting before she is discharged. As a result, C.B. decides to postpone the move and stay with the children at her sister’s home while she recuperates and seeks counseling at a women’s support shelter. She tells you she will keep her appointment with the internist in 2 weeks.

Case Study 17 Name _________________________________ Class/Group ______________________________ Date ___________

Scenario A.H. is a 70-year-old retired construction worker who has experienced lumbosacral pain, nausea, and upset stomach for the past 6 months.

He has a history of heart failure, high cholesterol, hypertension (HTN), sleep apnea, and depression. His chronic medical problems have been managed over the years with benazepril (Lotensin) 5 mg/day, fluoxetine (Prozac) 40 mg/day, furosemide (Lasix) 20 mg/day, potassium chloride (KCl) 20 mEq (20 mmol) bid, and atorvastatin 40 mg each evening.

A.H. has just been admitted to the hospital for surgical repair of a 6.2-cm abdominal aortic aneurysm (AAA) that is now causing him constant pain. On arrival to your floor, his vital signs (VS) are 109/81, 61, 16, and 98.3° F (36.8° C). When you perform your assessment, you find that his apical heart rhythm is regular, and his peripheral pulses are all 2 +. His lungs are clear, and he is awake, alert, and oriented. There are no abnormal physical findings; however, he has not had a bowel movement for 3 days. His electrolytes, blood chemistries, and clotting studies are within normal range, except his hematocrit is 30.1%, and hemoglobin is 9 g/dL (90 g/L).

1. A.H. has several common risk factors for AAA in his health history. Name and explain 3 factors.

2. How is testing used to diagnose an AAA? 3. A.H.’s aneurysm has the shape as in the accompanying

illustration. What type of aneurysm is this? a. Aortic dissection b. False aortic aneurysm c. Saccular aortic aneurysm d. Fusiform aortic aneurysm

(From Lewis SL, Dirksen SR, Heitkemper MM, et al. Medical-Surgical Nursing: Assessment and Management of Clinical Problems. 8th ed. St. Louis, MO: Mosby; 2011.)

Case Study Progress While A.H. awaits his surgery, it is important that you monitor him carefully for decreased tissue perfusion.

4. Name 5 things you would assess for, stating your rationale for each.

5. What is the most serious, life-threatening complication of AAA and why?

6. What single problem mentioned at the beginning of this case study is a risk for this complication? Why?

7. During your assessment, you note a pulsation in A.H.’s upper abdomen, slightly left of the midline, between the umbilicus and the xiphoid process. True or False: You must palpate this mass as part of your physical assessment. Explain your answer.

8. What common complications may occur after a AAA repair?

Case Study Progress A.H.’s aneurysm resection is successful, and he maintains normal leg movement and sensation. However, for the first 2 postoperative days

he is delirious and needs one-to-one nursing care in the intensive care unit. After he becomes coherent and oriented again, he is transferred back to your floor.

9. What assessments do you need to make specific to his postoperative care?

10. List 6 problems that are high priorities in A.H.’s postoperative care.

11. When you perform A.H.’s abdominal assessment, you do not hear any bowel sounds when auscultating his abdomen. What should you do?

12. Which interventions would you implement after an abdominal aortic aneurysm repair? Select all that apply.

a. Keep the head of the bed elevated at 60 degrees. b. Assess peripheral pulses of both lower extremities. c. Change dressings as ordered with aseptic

technique. d. Use the bed’s knee gatch to allow for knee flexion

during bed rest. e. Keep firm pressure on the abdominal incision

during coughing exercises.

Case Study Progress A.H.’s recovery is uneventful. While preparing him for discharge, you talk to him about health promotion and lifestyle change issues that are pertinent to his health problems.

13. Name 4 health-related issues you might discuss with him and what you would teach in each area.

14. A.H. will be receiving follow-up visits from the home health care nurse to change his dressing and evaluate his incision. What can you discuss with A.H. before discharge that will help him understand what the nurse will be doing?

15. Which statement by A.H. indicates a need for further education?

a. “I will report any fever greater than 100 ° F.” b. “I will avoid heavy lifting for 3 more weeks.” c. “I will look for color changes in my feet and lower

legs.” d. “I will call the surgeon if I notice redness or

swelling at the incision.”

Case Study Outcome After a few more days in the hospital receiving physical therapy, A.H. could ambulate on his own. He was discharged from the hospital after a 9-day stay. Since discharge, he has not experienced any further problems and has not needed to return.

Case Study 18 Name _________________________________ Class/Group ______________________________ Date ___________

Scenario Three-week-old J.T. and her parents arrive at the cardiac cath lab for her cardiac catheterization. She was born at term with Down syndrome, and her pediatrician is concerned because of lack of weight gain and poor feeding. You are getting her and her parents prepared for the procedure.

1. As you obtain her history and take vital signs, which statements or findings would be concerning and suggestive of heart failure (HF)? Select all that apply and explain your answers.

a. Peripheral pulses + 3 b. Heart rate: 195 at rest c. Rectal temperature: 36.6° C (97.8° F) d. “J. gets damp and sweaty when she feeds.”

e. “J. takes 30 to 40 minutes to take 2 to 3 ounces (60 to 90 mL) of formula.”

f. “J. seems to have fewer wet diapers than when we brought her home from the hospital.”

2. You are preparing J.T.’s parents for the procedure. Describe the points you would address in your teaching.

Case Study Progress After the catheterization, J.T. returns to the unit in a crib. The orders shown in the chart have been written.

Chart View

Physician’s Orders

Daily weights: Current weight 8.8 lbs (4 kg) Strict intake and output O2 per nasal cannula as needed to maintain O2 saturation

greater than 93% VS every 15 minutes × 4, then every 1 hour × 4, then every 4

hours Digoxin (Lanoxin) 70 mcg PO now, then 35 mcg PO every 6

hours for 2 doses Furosemide (Lasix) 4 mg PO now, then 4 mg PO every 12 hours

3. What will you document in your postprocedure assessment? Include rationale.

4. You are reviewing J.T.’s medications. J.T’s mother states the following rationale for starting J.T. on digoxin (Lanoxin): “I need to give this to J. to decrease her blood pressure so her heart doesn’t have to work so hard.” Is this true or false? Explain your answer.

5. You have a student nurse working with you, and the student asks why the first ordered doses are high. What would be a

possible explanation for this? 6. The student nurse asks whether there are any precautions to

observe when giving digoxin to a neonate. Describe medication safety precautions that should be observed when giving this medication.

7. Which of these are potential signs of digoxin toxicity in an

infant? Select all that apply. a. Vomiting b. Bradycardia c. Tachycardia d. Decreased blood pressure e. Lack of interest in feeding

Case Study Progress You administer the ordered medication and proceed with your assessment.

8. Which of these are possible complications to monitor for after a cardiac catheterization? Select all that apply.

a. Hematoma b. Vasospasm c. Hemorrhage d. Hyperglycemia e. Transient dysrhythmia f. Decreased pulse in unaffected leg

9. You are preparing to administer J.T.’s furosemide. Your

drug reference gives the following therapeutic dose for edema in a neonate: 1 mg/kg per dose every 12 to 24 hours. Is the ordered dose of 4 mg a safe dose for J.T?

Case Study Progress You note the following serum metabolic panel results for J.T.

Chart View

Laboratory Results

Glucose 85 mg/dL (4.7 mmol/L)

Calcium 9.1 mg/dL (2.28 mmol/L)

Sodium 142 mEq/L (142 mmol/L)

Potassium 3.3 mEq/L (3.3 mmol/L)

Chloride 101 mEq/L (101 mmol/L)

10. Which lab finding would concern you, and why? 11. The cardiologist tells J.T.’s parents that J.T. has a ventricular

septal defect (VSD). On the diagram, circle the area affected by this defect.

(Modified from Harkreader H, Hogan MA. Fundamentals of Nursing: Caring and Clinical Judgment. 2nd ed. St. Louis, MO: Saunders; 2004.)

12. True or False? This defect would create decreased pulmonary flow. Explain your answer.

Case Study Progress The cardiologist consults with the family, and it is decided that J.T. will be discharged to home the following day with medications and close monitoring. J.T. will return in several months for surgical repair of the VSD.

13. You begin your discharge teaching. Describe the information you will include in teaching.

Case Study Outcome J.T. undergoes surgical repair of the VSD 3 months later, and her recovery goes well. She continues to grow without further cardiac issues.

Case Study 19 Name _________________________________ Class/Group ______________________________ Date ___________

Scenario J.E. is a 34-year-old Filipino man who presents to the emergency department reporting shortness of breath, dizziness, and a rapid fluttering in his chest. He states that he occasionally has had the same symptoms but they usually pass if he sits or lies down to rest. But this time resting has not helped the symptoms to subside. J.E. has a past medical history of an appendectomy and rheumatic fever as a child. His current vital signs are BP 164/78, P 160, R 28, T 98.1° F (36.7° C). His oxygen saturation is 94% on room air.

1. What further assessment and testing are needed at this time? Based on his vital signs and symptoms, what might the results be?

2. The echocardiogram reveals mitral valve stenosis. Describe mitral valve stenosis.

3. What typical heart sounds are heard with mitral stenosis, and where is the best place to listen?

4. What from J.E.’s past medical history could be the cause of mitral stenosis, given his young age? Explain your answer.

5. Locate the mitral valve on this diagram of the heart.

(From Solomon EP. Introduction to Human Anatomy and Physiology. 4th ed. Philadelphia, PA: Saunders; 2016.)

Case Study Progress J.E. is referred to a cardiothoracic surgeon to decide the best course of action for treating his mitral valve stenosis. At this time, he is still experiencing atrial fibrillation, with a heart rate in the 160s.

6. Which drugs do you expect to be started at this time? Explain your rationale.

a. Amiodarone infusion and a heparin infusion b. IV push metoprolol and enoxaparin (Lovenox)

subcutaneous injections c. A one-time oral dose of diltiazem (Cardizem) and

subcutaneous heparin d. Lidocaine bolus, followed by an infusion, and

subcutaneous heparin injections

Case Study Progress The cardiothoracic surgeon determines that J.E. must have valve replacement surgery because of the severity of the mitral valve stenosis. The surgeon discusses valve surgery with J.E. and answers questions.

7. There are 2 types of valves used for valve replacement. Identify the types and discuss the advantages and disadvantages of each type.

8. Discuss why the surgeon might pick one valve over the other for J.E.

Case Study Progress J.E. had successful mitral valve replacement surgery and is now recovering in the cardiac surgery intensive care unit. He is on a ventilator and has a chest tube but is awake and able to follow commands.

9. List 5 things the nurse will assess and monitor during the first 12 hours postoperatively.

Case Study Progress Twenty-four hours after J.E.’s surgery, he has been successfully extubated, and is transferred to the cardiac surgery step-down unit.

10. What teaching will you provide to prevent respiratory complications?

11. You are teaching J.E. about care of the sternal incision. Which statement by J.E. indicates a need for further education?

a. “I will call my doctor if I develop a fever.” b. “I will report any bleeding that occurs from the

incision.” c. “I will wash the incision with soap and water when

I get home.” d. “If the incision looks red or has drainage, I will call

my doctor right away.”

Case Study Progress J.E. is eager to go home. He has been started on warfarin (Coumadin) therapy and knows that he will need to be on anticoagulation for life.

12. What education about anticoagulation do you need to provide?

13. Which lab result reflects an appropriate goal for a patient who is on warfarin after heart valve surgery? Explain your answer.

a. INR 3.2 b. PT 12.5 seconds c. aPTT 40 seconds d. Bleeding time 9 minutes

14. J.E. has many questions about returning to work, driving, and exercise. What will you tell him?

15. True or False? There is no need for J.E. to inform health care providers about his valvular heart surgery because he has a mechanical valve. If false, correct the statement.

16. J.E. asks you, “What is a good way to let someone know that I have an artificial heart valve? Do I have to carry paperwork

with me at all times?” How will you answer him?

Case Study Outcome J.E. returns to work after 6 weeks and becomes an active member of the local American Heart Association Mended Hearts community support group.

Case Study 20 Name _________________________________ Class/Group ______________________________ Date ___________

Scenario You are the 1900- 0700 charge nurse on the intermediate cardiac care unit in a large hospital. One of the patients, R.J., is being cared for by a new graduate nurse under your supervision. R.J. was admitted at 1300 after an auto accident in which he sustained a chest contusion and fractures of the fourth and fifth ribs on his left side. At about 2000 hours, his wife runs up to you at the nurses’ station and says, “I think my husband just had a heart attack. Come quick!” She follows you into his room, where you find him still in bed. He is breathing and is cyanotic from the neck up. His pulse is rapid but very weak.

1. What will your first action be? 2. What immediate care will you provide to R.J.? 3. Given R.J.’s admitting diagnosis, what differential diagnoses

do you consider? 4. What immediate information do you need to obtain from the

nurse who is caring for R.J.? 5. Suddenly you remember R.J.’s wife, who is anxiously hovering

over you in the room. What are you going to do?

Case Study Progress

The code team arrives. R.J.’s trauma surgeon is making rounds on your unit when the code is called, and he runs into the room. R.J. is intubated, and the normal saline lock is changed to an IV of lactated Ringer’s solution at “wide open.” The trauma surgeon recognizes Beck’s triad associated with cardiac tamponade and calls for a cardiac needle and syringe. He inserts the needle below the xiphoid process and aspirates 75 mL of unclotted blood.

6. What is Beck’s triad? 7. Describe cardiac tamponade. 8. What is the most likely reason R.J. developed cardiac

tamponade? 9. Explain why the surgeon performed a pericardiocentesis.

10. What is the significance of the surgeon aspirating unclotted blood?

11. The surgeon orders IV dopamine to “begin at 4 mcg/kg/min and titrate to maintain a systolic BP over 100.” What is the reason for this order?

12. The stock dopamine solution contains 320 mg dopamine

in 100 mL of 5% dextrose. R.J. weighs 240 lb. How many micrograms should R.J. receive per minute? How many total milligrams of dopamine should R.J. initially receive per hour? (Round to the hundredth.) At how many milliliters per hour would you set the infusion pump? (Round to the tenth.)

13. Describe how you titrate a dopamine infusion.

14. Since R.J. underwent an emergency pericardiocentesis, which nursing interventions should you include in his immediate postprocedural care? Select all that apply.

a. Maintain continuous ECG monitoring b. Closely assess for further cardiac tamponade c. Obtain blood cultures at 2 sites and send to the lab d. Be prepared for an emergency thoracotomy if

tamponade recurs e. Observe for complications such as bleeding and

cardiac dysrhythmias 15. Name 4 assessment findings that would show R.J. is

responding to the immediate actions.

Case Study Progress R.J. is being transferred to the intensive care unit (ICU) for observation.

16. Using the SBAR framework, describe the report you will give the ICU nurse.

17. As the team prepares R.J.’s transfer, you go find R.J.’s wife to thank her for alerting you to the emergency so promptly and to tell her what has happened. Briefly, and in lay terms, how would you explain what happened to her husband?

18. As you both get up to leave, Mrs. J. suddenly turns pale and says she feels very dizzy. What should you do?

Case Study Outcome Once in the ICU, R.J. underwent placement of a central venous catheter and an emergency echocardiogram. After finding about 50 mL of additional fluid in the pericardial sac, the decision was made to take R.J. to the operating room. A thoracotomy was done with repair of a right atrium laceration. He made an uneventful recovery and was discharged home on postoperative day 5.

Case Study 21 Name _________________________________ Class/Group ______________________________ Date ___________

Scenario D.V. is a 34-year-old woman who had a ruptured appendix 8 days ago with subsequent peritonitis. Plans are in progress to discharge her to

home care later this afternoon, with a left peripherally inserted central catheter (PICC) for IV antibiotic therapy. As you are doing your full assessment on D.V., you notice a large ecchymotic area over the right upper arm. You ask her whether she recalls any trauma to that area. She tells you, “You nurses have taken my blood pressure so many times it bruised.”

1. Do you accept D.V.’s explanation? Why or why not? 2. In examining D.V. further, you find a fine, nonraised, dark red

rash over her trunk (petechiae). What other questions would you ask D.V.?

Case Study Progress D.V. had not noticed the petechiae before you pointed it out. She says the rash does not itch or cause pain and that she has never had one like it before. She denies any other bleeding.

3. What other information would you want to gather?

Case Study Progress Her vital signs are within normal limits except for a temperature of 99.8° F (37.7° C). The abdominal wound is not discolored or draining; however, her abdomen is tender to light palpation. The rash is confined to the trunk. There is slight oozing of serosanguineous fluid around the PICC insertion site. She has no other signs of bleeding. You decide to call the provider with your findings.

4. Using SBAR, what information will you relay to the provider? 5. The provider orders blood to be drawn for coagulation studies

and a CBC with differential. What tests would you expect to see performed in coagulation studies?

6. You draw D.V.’s blood and initiate her next ordered dose of IV antibiotic. She asks you, “What is going on?” How would you respond?

Case Study Progress An hour later, just as you are about to go to into the room to discontinue the IV antibiotic infusion, D.V. turns on her light and asks you to come to the room “right now.” She tells you that she went to the bathroom and urinated blood and shows you a tissue in which she has some bloody-appearing sputum. You perform a focused assessment and find that there is some bloody drainage from the blood draw site an hour earlier and more petechiae on her trunk. Her vital signs are unchanged.

Chart View

Laboratory Test Values

PT 19 seconds

aPTT 96 seconds

INR 1.8

d-Dimer 4.8 mcg/mL (4.8 ng/mL)

Fibrinogen 56 mg/dL (0.56 g/L)

WBC 12,500/mm3 (12.5 x 109/L)

Platelet count 46,000/mm3 (46 x 109/L)

7. Interpret D.V.’s lab values.

Case Study Progress The elevated WBC count is consistent with her diagnosis of peritonitis or other infection.

8. D.V. is diagnosed with disseminated intravascular coagulation (DIC). What is DIC?

9. What is the most likely cause of DIC in D.V.’s case?

a. Acute liver failure b. Presence of an undetected pregnancy c. Development of toxic shock syndrome d. Presence of infection in the abdominal cavity

10. Are D.V.’s presenting signs and symptoms consistent with DIC? Explain.

11. What are the goals of care for D.V.? 12. What complications are associated with DIC?

Case Study Progress You notify the provider of D.V.’s assessment and lab results. The provider writes for D.V. to be transferred immediately to the intensive care unit (ICU); however, no beds are available in the unit. The nursing supervisor informs you that it will be 2 to 3 hours before a bed will be available.

13. What medical interventions do you expect for D.V. and why? 14. List 3 nursing actions and the rationale for each that you need

to implement until she is transferred. 15. Describe 4 precautions you should begin to reduce the

risk for further bleeding. 16. Which activities can you delegate to the UAP who is assisting

you with D.V.’s care? Select all that apply. a. Recording intake and output every hour b. Evaluating foot sensation and peripheral pulses c. Assisting D.V. with repositioning every 2 hours d. Having D.V. sign a blood transfusion consent form e. Regulating IV fluid administration as prescribed

the by provider 17. D.V. appears anxious about all that is going on. How you

would offer her support?

Case Study Outcome Therapy for DIC is started with O2, fluids, and IV heparin. She is

transferred to the ICU in guarded condition. Her infection, however, quickly evolves into sepsis and she develops acute respiratory distress syndrome requiring ventilatory support. After 6 days, D.V.’s respiratory status improves, and she is extubated. She is discharged to home care with IV antibiotic therapy 19 days after her initial admission.

Case Study 22 Name _________________________________ Class/Group ______________________________ Date ___________

Scenario A 57-year-old man named A.T. has been admitted to the telemetry unit for continuous cardiac monitoring. He has a past medical history of nonischemic cardiomyopathy, hypertension, chronic heart failure (HF) with an ejection fraction (EF) of 10%, implantable cardioverter- defibrillator (ICD) placement 1 month ago, alcohol abuse, and a past smoker (quit 3 years ago). He was admitted after reporting chest pain and his ICD firing 3 times at home, before admission. During his morning assessment, A.T. suddenly reports chest pain and feels like he was just kicked in the chest. There is no witnessed loss of consciousness.

1. The nurse suspects that what just occurred? 2. What is an ICD? 3. How does the ICD get power to work? 4. What cardiac rhythm would the nurse expect to see during the

review of his continuous telemetry strip? 5. What conditions are indications for placement of an ICD?

Case Study Progress The nurse receives a call from the telemetry monitor technician, who

reports a 5-second episode of this dysrhythmia:

(From Wesley K. Huszar’s Basic Dysrhythmias. 4th ed. St. Louis, MO: Mosby; 2011.)

6. Identify this dysrhythmia 7. The nurse will assess and monitor for what other signs or

symptoms?

Case Study Progress The nurse calls the cardiologist to report the change in A.T.’s condition and receives orders for lab work.

8. What lab studies does the nurse expect to be ordered?

Case Study Progress The nurse notes that the results from A.T.’s blood work are ready for review.

9. Considering A.T.’s condition, what lab results would require further management? Select all that apply.

a. BTNP 435 pg/mL b. Calcium 9.4 mg/dL (2.35 mmol/L) c. Sodium 130 mEq/L (130 mmol/L) d. Glucose 107 mg/dL (5.9 mmol/L) e. Potassium 3.0 mEq/L (3.0 mmol/L) f. Hemoglobin 12.6 g/dL (126 g/L) g. Magnesium 1.3 mEq/L (0.5 mmol/L)

Case Study Progress The nurse is asked to perform a medication reconciliation. The nurse makes a list of all of A.T.’s medications, which are listed in the following chart.

Chart View

Home Medications

Mexiletine hydrochloride 150 mg PO every 12 hours

Carvedilol 25 mg PO twice daily

Furosemide 80 mg PO twice daily at 8 a.m. and 2 p.m.

Lisinopril 10 mg PO once daily

Spironolactone 25 mg PO once daily

Isosorbide mononitrate 20 mg PO once daily

Aspirin EC 81 mg PO once daily

Rosuvastatin 20 mg PO every evening

10. For each drug listed, state its classification and specific use for A.T.

11. Which of A.T.’s medications would have a possible effect on his potassium level?

12. The hospitalist following A.T. ordered magnetic

resonance imaging (MRI) because A.T. reported pain over his right middle quadrant. The patient escort has arrived to take him to the MRI room. Will you intervene? Explain your answer.

Case Study Progress A.T. reports to the nurse that he feels overwhelmed at home managing his heart failure. He is saddened that he cannot play in his softball league but instead walks 1 mile every day in his neighborhood. He is proud that has quit smoking. He tells the nurse that he is “terrified” whenever his ICD “kicks him down.” He also tells you that he doesn’t really know what to do if his ICD goes off.

13. The nurse provides instructions about the ICD. Which statement by A.T. indicates a need for further instruction?

a. “I will wear a medical alert bracelet at all times.” b. “I will carry a list of my current medications with

me.” c. “If my ICD goes off, I need to call my doctor right

away.” d. “If my ICD goes off more than once, or if I feel sick,

I will drive to the Emergency Room.”

Case Study Outcome A.T. spends 3 days on the telemetry unit. His continuous telemetry monitoring now shows a normal sinus rhythm with a heart rate of 82. His BP is 116/72. A 2D Echo was performed that showed an EF of 25%. His ICD was interrogated without adverse reports. A CT scan of the abdomen showed no abnormalities. His most recent potassium level is 3.5 mEq/L (3.5 mmol/L). The cardiologist has agreed to discharge A.T. with a follow-up appointment 1 week from discharge. A.T. joins an ICD support group that meets monthly and enjoys talking with others who have the same concerns.

2

Gas Exchange

Case Study 23 Name _________________________________ Class/Group _____________________________ Date _____________

Scenario You are a public health nurse working at a county immunization and tuberculosis (TB) clinic. B.A. is a 51-year-old woman who wants to obtain a food handler’s license and is required to show proof of a negative tuberculosis skin test (TST) result before being hired. She came to your clinic 2 days ago for the first step of the TST. She has returned to have you evaluate her reaction.

1. What is TB and what microorganism causes it? 2. What is the route of transmission for TB? 3. The Centers for Disease Control and Prevention (CDC)

recommends screening people at high risk for TB. List 5 populations at high risk for developing active disease.

4. Describe the 2 methods of TB screening. 5. How do you determine whether a TST result is positive or

negative?

6. Interpret B.A.’s TST. 7. What other information do you need to obtain from B.A.? 8. You inform B.A. of the test result. She asks you what the result

means. How will you respond?

Case study progress B.A. is a natural-born American and has no risk factors for TB infection by the CDC guidelines. She has a 6-year history of type 2 diabetes mellitus that is well controlled with metformin (Glucophage). She admits that her mother had TB when she was a child but says she herself has never tested positive before. She is angry at the thought that she might have TB and says, “I feel just fine, and I don’t think anything else is necessary.”

9. What needs to be done to figure out whether B.A. has an active TB infection?

Case study progress The provider orders a chest x-ray examination and informs B.A. that the image is clear, showing no signs of active TB infection. He tells her that she has class 2 TB, or a latent TB infection (LTBI), and that he will report her condition to the local public health department.

10. What is a LTBI? 11. What parameters are used to decide whether a person is

treated for LTBI? 12. Is B.A. a candidate for LTBI treatment? State your rationale. 13. Outline the current CDC guidelines for the treatment of LTBI.

Case study progress The provider decides to place B.A. on a 12-dose, once-weekly regimen of isoniazid and rifapentine as directly observed therapy (DOT).

14. How will you describe LTBI and DOT to B.A? 15. The medications used to treat LTBI have different side effects.

Match the possible side effects to the tests used to assess for their presence:

_____A. Peripheral neuropathy 1. Audiogram

_____B. Hepatotoxicity 2. CBC

_____C. Fever and bleeding problems

3. BUN and creatinine

_____D. Nephrotoxicity 4. AST and ALT

_____E. Hyperuricemia 5. Monofilament testing

_____F. Optic neuritis 6. Red-green discrimination and visual acuity

_____G. Hearing neuritis 7. Uric acid

16. What other information does B.A. need to receive before leaving the clinic?

Case study outcome B.A. is hired under the condition that she complies with LTBI therapy and will immediately report any signs and symptoms of active disease

to the clinic. She reports weekly for her medications and finishes her 12 weeks of therapy without experiencing any significant effects.

Case Study 24 Name _________________________________ Class/Group _____________________________ Date _____________

Scenario It is 1130 and M.N., age 65, is being admitted to your surgical floor after having undergone an open cholecystectomy for acute cholecystitis. She has a nasogastric tube to continuous low wall suction, one peripheral IV line, and a large abdominal dressing. Her orders are as follows.

Chart View

Physician’s Orders

Clear liquid diet; progress low-fat diet as tolerated D5 ½NS with 40 mEq KCl at 125 mL/hr Turn, cough, and deep breathe q2hr Incentive spirometer q2hr while awake Oxygen per protocol to maintain Spo2 at 95% Dangle in a.m. Morphine sulfate 10 mg IM q4hr prn for pain Piperacillin/tazobactam (Zosyn) 3.375 grams IV q6hr CXR in a.m.

1. Are these orders appropriate for M.N.? State your rationale.

Case study progress

At 1530, the UAP reports the following:

Chart View

Vital Signs

BP 148/82

Heart rate 118

Respiratory rate 24

Temperature 101° F (38.3° C)

Spo2 92%

2. Based solely on her vital signs, what could be happening with M.N., and why?

3. You go to assess M.N. What do you need to include in your assessment at this time?

Case study progress Your assessment of M.N. finds her with decreased breath sounds and crackles in the right base posteriorly. Her right middle and lower lobes percuss slightly dull. She splints her right side when attempting to take a deep breath. Her skin is pale, warm, and dry. She does not have a productive cough, chest pain, or any anxiety.

4. What complication do you suspect M.N. is experiencing? State your rationale.

5. Why is M.N. at risk for developing this complication? 6. What is your nursing priority at this time? 7. Describe 6 interventions you will perform over the next few

hours based on this priority. 8. To promote optimal oxygenation with M.N., which actions

could you delegate to the UAP? Select all that apply.

a. Auscultating M.N.’s lung sounds b. Encouraging M.N. to splint the incision c. Assisting M.N. in getting up to the chair d. Reminding M.N. to cough and deep breathe e. Instructing M.N. on the use of incentive spirometry f. Taking M.N.’s temperature and reporting elevations

9. Identify 3 outcomes that you expect for M.N. as a result of your interventions.

Case study progress At 1830, the UAP reports the following:

Chart View

Vital Signs

BP 136/72

Heart rate 104

Respiratory rate 24

Temperature 100.6° F (38.1° C)

Spo2 93%

10. Has M.N.’s status improved or not? Defend your response. 11. You need to call the surgeon regarding M.N.’s status. Using

SBAR, what would you report to the surgeon? 12. The surgeon orders a stat CXR. Afterward, the radiology

department calls with a report, confirming that M.N. has atelectasis. Should this diagnosis change your plan of care for M.N.?

13. If M.N. had pneumonia, what changes might the surgeon have made to her plan of care?

14. M.N.’s sister questions you, saying, “I don’t understand. She

came in here with a bad gallbladder. What has happened to her lungs?” How would you respond?

Case study outcome M.N. stayed 2 extra days in the hospital, receiving IV antibiotics and continued care focusing on improving her respiratory status. Her atelectasis resolved, and she experienced no other complications from the cholecystectomy. She went home with her sister, and both thanked the staff for their care efforts by sending flowers.

Case Study 25 Name _________________________________ Class/Group _____________________________ Date _____________

Scenario S.R. is a 59-year-old man who comes to the clinic because his wife complains “my snoring is difficult to live with.”

1. As the clinic nurse, what routine information would you want to obtain from S.R.?

Case study progress After interviewing S.R., you note the following: S.R. is under considerable stress. He owns his own business. The stress of overseeing his employees, meeting deadlines, and carrying out negotiations has led to poor sleep habits. He sleeps 3 to 4 hours per night. He keeps himself going by drinking 2 quarts (2 liters) of coffee and smoking three to four packs of cigarettes per day. He has gained 50 pounds (22.5 kg) over the 2 years, leading to a current weight of 250 pounds (114 kg). He tells you he has difficulty staying awake, wakes up with headaches on most mornings, and has midmorning somnolence. He says he is depressed and irritable most of the time

and reports difficulty concentrating and learning new things. He has been involved in three auto accidents in the past year.

S.R.’s vital signs are 164/90, 92, 18, and SpO2 90% on room air. His examination findings are normal, except for a few bruises over the right side of the rib cage. You inquire about the bruises, and S.R. reports that his wife jabs him with her elbow several times every night. In her own defense, the wife states, “Well, he stops breathing and I get worried, so I jab him to make him start breathing again. If I don’t jab him, I find myself listening for his next breath and I can’t go to sleep.” You suspect sleep apnea.

2. Name the 2 main types of apnea and explain the pathology of each.

3. Based on your findings, which type of sleep apnea do you think S.R. has?

4. Name at least 5 signs or symptoms of this type of sleep apnea. Put a star next to those S.R. is experiencing.

5. How does the provider use diagnostic testing to diagnose sleep apnea?

Case study progress The primary care provider examines S.R. and documents a long soft palate, recessed mandible, and medium-sized tonsils. S.R. undergoes an overnight home screening study, which shows 143 episodes of desaturation ranging from 68% to 76% with episodes of apnea. He is tentatively diagnosed with obstructive sleep apnea (OSA), and a full sleep study is ordered.

6. S.R. and his wife ask about a full sleep study. How would you explain a nocturnal polysomnogram to them?

7. S.R. and his wife ask why they need to be concerned about OSA. You tell them that treating OSA is necessary to prevent which common complications? Select all that apply.

a. Stroke b. Hypotension

c. Cardiac dysrhythmias d. Right-sided heart failure e. Early onset of chronic obstructive pulmonary

disease (COPD) 8. The provider asks you to teach S.R. about lifestyle changes that

he could make immediately to help with his situation. Describe 4 priority topics you would discuss with S.R.

Case study progress The polysomnogram confirms S.R.’s diagnosis of OSA. At his 6-week follow-up visit, he reports he has lost 8 pounds (3.6 kg), but there has been little improvement in his symptoms. He says that he fell asleep while driving to work and wrecked his car. He wants to discuss further treatment options.

9. What are the treatment options for OSA? Describe 3.

Case study progress S.R. and the provider decide to begin S.R. on continuous positive airway pressure (CPAP). The provider writes a prescription for CPAP.

10. List 3 education topics you need to address with S.R. so

he can safely self-manage CPAP therapy. 11. Which member of the interprofessional team will likely be

involved in S.R.’s care and how? 12. S.R. calls 2 weeks later with complaints of dry nasal

membranes, nosebleeds, and sores behind his ears. What instructions would you give S.R.?

13. Outline how you would document the phone call with S.R.

Case study outcome S.R. returns to the clinic 2 months after starting CPAP therapy. He reports that he is a “happier, healthier man.” He says he now sleeps 7 hours a night, is not having to drink coffee just to get through the day,

and his work performance has dramatically improved. S.R. tells you, “It is just remarkable, I never thought that sleep could be such a problem.”

Case Study 26 Name _________________________________ Class/Group _____________________________ Date _____________

Scenario G.C. is a 78-year-old widow who comes to the outpatient clinic saying that over the past 2 to 3 months she has felt increasingly tired, despite sleeping well at night. Over the past few years, she describes eating less and less meat because of her financial situation and has trouble preparing a meal “just for me.” G.C. relates how she relies on her late husband’s Social Security income for all her expenses. She struggles financially to buy medicines for the treatment of hypertension and arthritis. Her vital signs are 136/76, 16, 80. She denies any dyspnea or palpitations. The nurse practitioner orders blood work. G.C.’s chemistry panel findings are all within normal limits, and a stool fecal occult blood test result is negative. Her other results are shown in the chart.

Chart View

Laboratory Test Results

WBC 7600/mm3 (7.6 x 109/L)

Hematocrit (Hct) 27.3%

Hemoglobin (Hgb) 8.3 mg/dL (83 g/L)

Platelets 151,000/mm3 (151 x 109/L)

Red Blood Cell (RBC) Indices

Mean corpuscular volume (MCV) 65 fL (65 mm3)

Mean corpuscular hemoglobin (MCH) 31.6 pg

MCH concentration (MCHC) 35.1%

Iron, total 30 mcg/dL (5 mcmol/L)

Iron-binding capacity 422 mcg/dL (75.5 mcmol/L)

Ferritin 8 mg/dL (18 pmol/L)

Vitamin B12 414 pg/mL (305 pmol/L)

Folate 22 ng/mL (50 nmol/L)

1. Which lab values are normal, and which are abnormal? 2. Explain the significance of each abnormal result. 3. Based on these results and her history, what condition does

G.C. have? 4. Who is at risk for this condition? 5. What other signs and symptoms of this condition do you assess

for in G.C.? 6. Which question would best help you determine the impact of

fatigue on her activities of daily living? a. “Are you upset about feeling more tired?” b. “Do you sleep more now than you used to?” c. “How far can you walk until you get short of

breath?” d. “Have you been able to do what you would like to

do?” 7. Discuss the treatment options for her condition. 8. When would parenteral iron therapy be indicated? 9. The nurse practitioner starts G.C. on ferrous sulfate 325 mg

orally once per day. What teaching does G.C. need about this medication?

10. As you are evaluating your teaching, you determine that more instruction is needed if G.C. says:

a. “My stools will likely turn a tarry, black color soon.”

b. “I can take the iron and my calcium supplements at the same time.”

c. “I will increase my fluid and fiber intake as long as I am on the iron.”

d. “Taking the tablets when I eat my meals will help my stomach not be upset.”

11. Discuss some ideas that might help her with her meal planning.

12. You teach G.C. about foods she should include in her diet. You determine that she understands your teaching if she says she will increase her intake of which foods?

a. Whole-wheat pastas and skim milk b. Lean cuts of poultry, pork, and fish c. Beans and dark green, leafy vegetables d. Cooked cereals, such as oats, and bananas

13. What community resources may benefit G.C.? 14. What parameters could you use to determine whether G.C.’s

condition is improving? 15. Using a SOAP format, write a sample documentation entry for

this encounter.

Case study outcome G.C. returns to the clinic in 3 months. She says she is feeling “a bit” less tired. Although her blood work shows some improvement, anemia is still present. G.C. and you discuss ways to promote better eating. She tells you she did not follow up on any of your suggestions about seeking community resources. She now agrees to speak with the social worker and explore available resources.

Case Study 27

Name _________________________________ Class/Group _____________________________ Date _____________

Scenario V.M. is a 29-year-old African American married man who has sickle cell disease (SCD) marked by frequent episodes of severe pain. His anemia has been managed with multiple transfusions. Six months ago, he started showing signs of chronic renal failure. His regular medications are acetaminophen, hydroxyurea (Hydrea), and folic acid. In the hematology clinic this morning, V.M.’s hemoglobin (Hgb) measured 6.7 g/dL (67 g/L). He received 2 units of packed red blood cells (PRBCs) over 3 hours and then went home. He developed dyspnea and shortness of breath about 1 hour later, and his wife called 911. The emergency medical system crew started O2 at 8 L per nasal cannula and transported V.M. to the emergency department (ED).

1. What is SCD? 2. Evaluate each of the following statements about SCD. Enter

“T” for true or “F” for false. Discuss why the false statements are incorrect.

_____1. Only African Americans get SCD. _____2. There is currently no cure for SCD. _____3. Those with SCD should not receive childhood

vaccinations. _____4. SCD cannot be diagnosed before an infant is 3

to 4 months old. _____5. An ophthalmologist should perform an eye

examination every 1 to 2 years. _____6. Those with SCD should fly only on airplanes

with pressurized cabins. 3. Which statement is true about the inheritance pattern of SCD?

a. If V.M.’s wife has sickle cell trait, each child will either have SCD or be a carrier.

b. If V.M.’s wife does not have sickle cell trait, each child has a 50% risk for having SCD.

c. If V.M.’s wife has sickle cell trait, each child can either have SCD or be normal.

d. If V.M. has children, each child will automatically have SCD regardless of his wife’s status.

4. V.M.’s Hgb measured 6.7 g/dL (67 g/L). Why is anemia common in patients with SCD?

5. Why is it difficult to crossmatch blood to transfuse V.M.? 6. What role does hydroxyurea (Hydrea) play in managing V.M.’s

SCD?

Case study progress When V.M. arrives at the ED, you perform a quick assessment and note a grade III systolic murmur and crackles in V.M.’s bases bilaterally. Vital signs are BP 176/102, P 94, R 28, T 97.8° F (36.6° C), and Spo2 78%. Peripheral pulses are equal and 3 +. He is alert and oriented; he tells you he is nauseated. His abdomen is nontender, with hypoactive bowel sounds in × 4 quadrants. Acting according to the standing orders for your institution, you start an intravenous (IV) line, obtain arterial blood gas (ABG) values, and draw blood for complete blood count with differential and basal metabolic panel.

7. The ED resident asks him whether he is in pain and whether he needs pain medication. V.M. answers “no” to both questions. Why did the ED resident ask these questions?

8. V.M.’s ABGs on 8 L O2 by simple face mask show Pao2 74 mm Hg. Is V.M. being adequately oxygenated?

9. Are your assessment findings consistent with fluid overload or deficit? What findings led you to your conclusion?

10. Your institution uses electronic charting. Based on the assessment described, document your findings by providing a brief narrative note.

□ Neurologic □ Respiratory

□ Cardiovascular □ Gastrointestinal □ Psychosocial

Chart View

Laboratory Test Values

Sodium 137 mEq/L (137 mmol/L)

Potassium 4.9 mEq/L (4.9 mmol/L)

Chloride 110 mEq/L (110 mmol/L)

CO2 16 mEq/L (16 mmol/L)

BUN 27 mg/dL (9.6 mmol/L)

Creatinine 2.7 mg/dL (239 mcmol/L)

WBC 4300/mm3 (4.3 x 109/L)

Hgb 7.8 g/dL (78 g/L)

Hct 20.9%

Platelets 208,000/mm3 (208 x 109/L)

11. Interpret V.M.’s lab results. 12. V.M. complains of being short of breath. Do you believe the

low Hgb level is responsible for his complaints? 13. What action will you expect the ED resident to take next and

why?

Case study progress The ED resident prescribes furosemide (Lasix) 40 mg IV push now, methylprednisolone (Solu-Medrol) 75 mg IV push now, and ceftriaxone 1 g IV piggyback after the furosemide (Lasix).

14. Indicate the expected outcome for V.M. associated with each medication he is receiving.

15. The methylprednisolone 75 mg IV is supplied as a

125 mg/2 mL solution. Shade in the dose to be given on the syringe.

(From Gray Morris D. Calculate with Confidence. 5th ed. St. Louis, MO: Mosby; 2010.)

16. Identify 3 outcomes that you expect for V.M. as a result of your interventions.

Case study progress V.M. voids 1900 mL within 2 hours of giving the furosemide. As V.M.’s dyspnea is relieved, he shakes the ED resident’s hand and thanks him for asking about the presence of pain and the need for pain medication. V.M. states, “One of my biggest fears is that I’ll come in here in crisis and the doctor won’t treat my pain aggressively enough. I don’t want to be labeled as a drug seeker or an emergency room abuser.”

17. Why would V.M. be concerned about obtaining adequate pain control in the ED?

18. What issues will you discuss with V.M. before discharge?

Case study outcome V.M. is discharged from the ED after being observed for 6 hours. He is

instructed to follow up with his primary care provider about the episode and discuss with the hematology clinic staff longer transfusion times for any future transfusions.

Case Study 28 Name _________________________________ Class/Group _____________________________ Date _____________

Scenario B.T., a 31-year-old man who lives in a small mountain town in Colorado, is highly allergic to dust and pollen and has a history of mild asthma. B.T.’s wife drove him to the emergency department when his wheezing was unresponsive to his fluticasone/salmeterol (Advair) inhaler, he was unable to lie down, and he began to use accessory muscles to breathe. B.T. is immediately started on 4 L O2 by nasal cannula and IV D5W at 75 mL/hr. A set of arterial blood gases (ABGs) is sent to the lab. B.T. appears anxious and says that he is short of breath.

Chart View

Vital Signs

BP 152/84

Pulse rate 124

Respiratory rate 42

Temperature 100.4° F (38.4° C)

1. Are B.T.’s vital signs acceptable? Give your rationale. 2. What is the rationale for immediately starting B.T. on O2?

3. Keeping in mind B.T.’s health history and presenting complaint, what are the critical areas you need to evaluate during your physical assessment?

Chart View

Arterial Blood Gases

pH 7.31

Paco2 48 mm Hg

HCO3 26 mEq/L

Pao2 55 mm Hg

4. Interpret B.T.’s ABG results.

Chart View

Medication Orders

Albuterol 2.5 mg plus ipratropium 0.5 mg nebulizer treatment STAT, then q4hr

Methylprednisolone 100 mg IV now Albuterol inhaler 2 puffs q4hr as needed

5. What is the rationale for the albuterol 2.5 mg plus ipratropium 0.5 mg nebulizer treatment STAT then q4hr?

6. Why is B.T. receiving methylprednisolone?

Case study progress You assess B.T. and find that he has diminished lung sounds with inspiratory and expiratory wheezes in all lung fields with a nonproductive cough and accessory muscle use. His skin is pale,

warm, and dry. The ECG shows sinus tachycardia without ectopy. He is alert and oriented × 4 spheres. He appears anxious and is sitting upright, leaning over the bedside table, and continuing to complain of shortness of breath.

7. What is your primary nursing goal right now? 8. Describe 6 actions you must implement based on this priority. 9. You will need to monitor B.T. closely for the next few hours.

What is the most serious complication to anticipate? 10. Name 4 signs and symptoms of this complication you will

assess for in B.T. 11. What are your responsibilities while administering aerosol

therapy?

Case study progress After several hours of rehydration and aerosol treatments, B.T.’s wheezing and dyspnea resolve, and he is able to expectorate his secretions. The provider discusses B.T.’s asthma management with him; B.T. says he has had several asthma attacks over the last few weeks. The provider discharges B.T. with prescriptions for fluticasone/salmeterol (Advair HFA 230/21) 1 inhalation every morning and evening, albuterol (Proair) metered-dose inhaler (MDI) 2 puffs q6hr as needed using a spacer, montelukast (Singulair) 10 mg/day each evening, and prednisone (5 mg tablets) 40 mg/day for 3 days, then taper by reducing the dose by 5 mg/day until discontinued. He gives him a prescription for a peak flow meter (PFM) and instructs B.T. to call the pulmonary clinic for follow-up with a pulmonary specialist.

12. How is montelukast (Singulair) different from other asthma medications?

13. Patients need to follow a specific sequence of

administration when using multiple inhalers. Describe the instructions you will give B.T. about the correct sequence he

should follow to safely use his inhalers. 14. Explain the drug class and expected outcomes associated with

the drugs contained in fluticasone/salmeterol (Advair HFA). 15. B.T. states he had taken his Advair that morning, then again

when he started to feel short of breath. He says, “It did not help,” and wants to know why he has to stay on it. Is fluticasone/salmeterol (Advair) appropriate for use during an acute asthma attack? Explain.

16. Based on this information, what specific issue do you need to address in discharge teaching with B.T.?

17. You review with B.T the possible side effects he may experience, including hoarseness, dry mouth, fungal infection in the oral cavity, and coughing. What actions can you teach him to prevent or diminish the incidence of these effects? Select all that apply.

a. Decrease his fluid intake. b. Use a spacer with the inhaler. c. Use the inhaler and Advair only as prescribed. d. Rinse his mouth with water immediately after

taking the Advair. e. Clean the spacer in the dishwasher on “hot cycle

with heated dry” daily. 18. What is the reason for the tapering doses of prednisone? 19. What is a PFM? Outline the instructions B.T. needs to perform

the PFM maneuver. 20. B.T. asks why he has to use the PFM. Explain the purpose of

the peak expiratory flow rate (PEFR) measurement, what an asthma action plan is, and the role the PEFR plays in an asthma action plan.

21. You set up an asthma action plan for B.T. What will you teach him to do if his PEFR value falls into the yellow or red zone?

22. You would recognize the need for additional teaching if B.T. makes which statements? Select all that apply and correct the incorrect answers.

a. “I will keep a diary of all of my PEFR measures.”

b. “I will place a plastic cover on our mattress and my pillows.”

c. “I will use the albuterol inhaler 30 minutes before exercising.”

d. “My wife needs to know what to do in case I have a serious attack.”

e. “The bed linens need to be washed in cold water to reduce dust mites.”

f. “If the PEFR is in the yellow zone, I will use rescue drugs and go to the hospital.”

23. B.T. states that he would like to read more about asthma on the Internet. List 3 credible websites to which you could direct him.

Case study outcome At the next follow-up visit, B.T.’s peak flow is 94% of the predicted. His symptoms have subsided, and he has not been experiencing any problems. He says he has been reading more about asthma and has a complete peak flow diary and asthma symptom journal on his smartphone.

Case Study 29 Name _________________________________ Class/Group _____________________________ Date _____________

Scenario L.S. is a 7-year-old who has been brought to the emergency department (ED) by his mother. She immediately tells you he has a history of ED visits for his asthma. He uses an inhaler when he wheezes, but it ran out a month ago. She is a single parent and has 2 other children at home with a babysitter. Your assessment finds L.S. alert, oriented, and extremely anxious. His color is pale, and his nail

beds are dusky and cool to the touch; other findings are heart rate 136, respirations regular and even, rate 26, oral temperature T 37.3°C (99.1°F), Spo2 89%, breath sounds decreased in lower lobes bilaterally and congested with inspiratory and expiratory wheezes, prolonged expirations, and a productive cough. As you ask L.S.’s mother questions, you note that L.S.’s respiratory rate is increasing; he is sitting on the side of the bed, leaning slightly forward, and having difficulty breathing. You are concerned that he is experiencing status asthmaticus.

1. You check the orders and need to decide which interventions are the priority at this time. Select all that apply and explain the rationale.

a. Have L.S. lie flat. b. Have L.S. perform incentive spirometry. c. Administer oxygen via face mask to keep his Spo2

above 90%. d. Administer albuterol (Proventil) and ipratropium

bromide (Atrovent) via hand-held nebulizer (HHN) STAT.

e. Reassess in 20 minutes, and if no improvement, administer salmeterol (Serevent Diskus) via dry- powder inhaler (DPI).

f. Reassess in 20 minutes, and if no improvement, administer albuterol (Proventil) and ipratropium (Atrovent) via hand-held nebulizer again.

g. Start IV normal saline (NS) at 15 mL/hr and administer methylprednisolone 2 mg/kg IV STAT × 1 dose.

2. Explain what the nurse will assess before, during, and after the nebulizer treatment with albuterol.

Case study progress You give L.S. the albuterol and Atrovent twice. His O2 saturation does not improve and remains at 88% with oxygen at 6 L/min via face

mask. He says he “does not feel any better.” He is retracting and his respiration rate remains 34. You have started his IV infusion and administered the methylprednisolone. L.S.’s mother is pacing and tells you she is very upset and worried. You overhead page the attending ED resident to assess, and you notify the patient-family advocate. The ED resident, Dr. S., arrives within 2 minutes to assess L.S. and to speak to L.S.’s mother. New orders are pending.

3. Chart your actions and the patient’s response using the SBAR (Situation, Background, Assessment, and Recommendation) format.

Case study progress L.S. is admitted to the pediatric intensive care unit (PICU) for close monitoring. His condition improves, and 24 hours later he is transferred to the floor. Asthma teaching is ordered. You assess Ms. S.’s understanding of asthma and her understanding of the disorder.

4. Which statement by Ms. S. would indicate a need for further teaching? Explain your answer.

a. “If he takes medications for a while, he will outgrow his asthma.”

b. “Part of his treatment should be avoiding things that irritate his lungs.”

c. “If I recognize early warning signs, he might be able to take medicine and not go to the ED.”

d. “He should go to the doctor regularly to make sure his asthma is being treated correctly.”

5. You are educating L.S. and his mother on possible asthma triggers in their environment. They live in public housing in an apartment without air conditioning. Which statements indicate possible asthma triggers? Select all that apply.

a. “We have a pet fish.” b. “L. collects stuffed animals.” c. “There are hardwood floors.”

d. “Our visitors smoke outside.” e. “The building has copper pipes.” f. “There are dark stains in our bathroom.” g. “We had to get the housing authority to treat for

bugs.” h. “He coughs when we have cold nights after a warm

day.” 6. Discuss strategies to avoid the triggers you identified in the

previous question.

Case study progress The following day, L.S. gets the discharge orders shown in the chart.

Chart View

Discharge Orders

Discharge to home Follow up with primary care provider in 3 days for evaluation Albuterol (Proventil HFA) MDI: 2 puffs with spacer every 4

hours prn Prednisolone (Prelone) 1 mg/kg PO every day for 5 days (L.S.

weighs 23 kg.) Fluticasone (Flovent HFA) MDI, 44 mcg/inhalation: 2 puffs

with spacer twice a day Montelukast (Singulair) 5 mg every evening PO Provide peak flow meter and teaching Regular diet

7. Ms. S. asks why she will use the spacer with the medicine L.S. inhales. Explain the purpose of using a spacer with the metered-dose inhaler (MDI).

8. Place the steps of using the MDI with the spacer in the correct order (1 = first step, 6 = last step)

a. ___ Depress the top of the inhaler to release medication, and breathe in slowly for 3 to 5 seconds, holding the breath for 5 to 10 seconds at the end of inspiration.

b. ___ Shake the inhaler well, 10 to 15 times, and attach to the spacer.

c. ___ Wait 1 to 2 minutes between puffs if more than 1 puff of the quick-relief medication is ordered.

d. ___ Remove and exhale slowly through the nose. e. ___ At the end of expiration, place mouthpiece into

the mouth, forming an airtight seal. f. ___ Tilt the head back and exhale completely.

9. During your medication teaching session with Ms. S. and L.S., you ask Ms. S. to teach back what she has learned about taking 2 different inhalers. Ms. S. makes this statement: “So, if he has to take both inhalers at the same time, he should take the Flovent first, then the albuterol. Right?” Is this statement true or false? Explain your answer.

10. Ms. S. then asks, “How long should we wait between giving the two inhalers if they are both due at the same time? Can we just give them one after the other?” What is your response?

11. As you continue your medication teaching, you explain the difference between long-term controllers and quick relief medications. Place a C beside the controller medication(s) and an R beside the quick relief medication(s).

____a. Albuterol ____b. Prelone ____c. Flovent ____d. Singulair

12. After L.S. takes a dose of the inhaled corticosteroid Flovent, what is the most important action he should do next? Explain your answer.

a. Hold his breath for 45 seconds. b. Rinse out his mouth with water. c. Repeat the dose in 5 minutes if he feels short of

breath. d. Check his PFM reading for an improvement of

function. 13. Ms. S. comes back from the pharmacy with the Prelone

and asks you to show her how much to give. Prelone is dispensed as 15 mg/5 mL. You give her a 10-mL oral dosage syringe. How much will she draw up for this dose? (Round to tenths.)

14. During the teaching session, you give L.S. a peak flow meter (PFM) and provide teaching for him and Ms. S. But L.S. looks puzzled and asks you, “Is this another medicine I have to take?” How would you explain the purpose of a peak flow meter to L.S.?

15. L.S. tells you that he loves to play basketball and football and asks you whether he can still do these activities. How will you respond?

16. Discuss the points to include in your discharge teaching regarding prevention of acute asthmatic episodes and symptom management.

17. List 3 Internet sites to which you can refer them for further information.

Case study outcome L.S. is discharged to home and has a follow-up appointment scheduled in 2 weeks. His mother has arranged for swimming lessons, and he plans to try out for his school’s swim team.

Case Study 30 Name _________________________________ Class/Group _____________________________ Date _____________

Scenario H.K.’s sister has brought her 71-year-old brother to the primary care clinic because he has had a fever for 2 days. She says he has shaking chills and a productive cough and he cannot lie down to sleep because “he can’t stop coughing.” After H.K. is examined, he is diagnosed with community-acquired pneumonia (CAP) and admitted to your floor at 1130. The resident is busy and asks you to complete your routine admission assessment and call her with your findings.

1. Name 4 priority areas to include in your assessment.

Case study progress Your assessment findings are as follows: H.K.’s vital signs (VS) are 154/82, 105, 32, 103° F (39.4° C), Spo2 84% on room air. You auscultate decreased breath sounds and coarse crackles in the left lower lobe anteriorly and posteriorly. His nail beds are dusky on fingers and toes. He has cough productive of rust-colored sputum and complains of pain in the left side of his chest when he coughs. He is a lifetime nonsmoker. His medical history includes coronary artery disease and myocardial infarction with a stent. He is currently on metoprolol, amlodipine, lisinopril, and furosemide; for his type 2 diabetes mellitus, he is taking metformin and glipizide. He has never gotten a pneumococcal or flu vaccination. He does report getting “hives” when he took “an antibiotic pill” a few years ago but does not remember the name of the antibiotic.

2. Which assessment findings are significant? Give your rationale.

Chart View

Admission Orders

Consistent carbohydrate diet VS q2hr

IV of D5 ½NS at 125 mL/hr Ceftriaxone 1gram IV every 12 hours Albuterol 2.5 mg/ipratropium 250 mcg nebulizer treatment

STAT, then q4hr Titrate O2 to maintain Spo2 over 90% Obtain sputum for C&S STAT blood cultures & sensitivity Blood glucose ac and hs with sliding scale regular insulin per

protocol #2 CBC with differential and basic metabolic panel CXR now and in the morning Continue home medications

3. You obtain orders from the resident. Outline a plan of what you need to do in the next 2 to 3 hours.

4. Is D5 ½NS an appropriate IV fluid for H.K.? State your rationale.

5. What is the rationale for ordering O2 to maintain Spo2 over 90%?

6. What is a C&S test, and what role will blood and sputum cultures play in H.K.’s care?

7. What would you expect the CXR results to reveal? 8. You need to follow a specific protocol when obtaining

peripheral blood cultures. Place in order the steps you will perform.

_____1. Select venipuncture site. Cleanse and allow to dry.

_____2. Inject 10 mL of blood into the aerobic bottle. _____3. Perform venipuncture and collect 20 mL of

venous blood. _____4. Verify patient’s identity and perform hand

hygiene. _____5. Attach identification to specimens and send to

lab within 30 minutes.

_____6. Inject 10 mL of blood into the anaerobic bottle. 9. The pharmacy sends the ceftriaxone in 100 mL 0.9% NaCl

with instructions to infuse over 40 minutes. At how many milliliters per hour will you regulate the IV infusion pump?

10. How will you ensure H.K.’s home medication list is

accurate?

Case study progress The next morning you are again assigned to care for H.K. Your assessment findings are as follows: VS 154/82, 92, 26, 100° F (37.8° C), Spo2 94% on 2 L O2 per nasal cannula. He appears to be in no apparent distress and denies any dyspnea. You auscultate decreased breath sounds and coarse crackles in the left lower lobe anteriorly. His skin is pale, warm, and dry. He has a cough productive of yellow-colored sputum and complains of pain in the left side of his chest when he coughs.

11. Is H.K. recovering as expected? Explain your rationale. 12. Based on your evaluation of H.K., write an outcome to achieve

by the end of your shift, then list 6 priority interventions you will perform toward achieving this goal.

13. By the end of your shift, which assessment findings would best indicate that H.K. is responding to therapy?

a. Cough productive of yellow sputum; lung sounds clear; Spo2 96% on room air

b. Complaints of dyspnea; respiratory rate of 26 on 2 L O2; clear lung sounds

c. Cough productive of white sputum; temperature 100.0° F (37.8° C); Spo2 98% on 2 L O2

d. Coarse crackles in posterior lower lobes; respiratory rate 22; no complaints of chills

Case study progress

After continuing the plan of care for 2 more days, H.K. is recovering from his pneumonia and preparing for discharge.

14. You know that H.K. is at increased risk for contracting another CAP infection. Describe 4 strategies for preventing CAP infections you will include in H.K.’s discharge teaching plan.

15. H.K. confides in you, “You know, my wife died a year ago, and I live alone now. I’ve been thinking … this pneumonia stuff has been a little scary.” How will you respond?

16. What are some community resources from which H.K. may benefit?

Case study outcome H.K. is discharged in the company of his sister. His plan is to stay with her for a few days. He thanks you for your care and taking the time to listen and talk with him. He says his sister and he talked, and they intend to join the local senior center after he is feeling better.

Case Study 31 Name _________________________________ Class/Group _____________________________ Date _____________

Scenario J.R., a 13-year-old with cystic fibrosis (CF), is being seen in the outpatient clinic for a biannual evaluation. J.R. lives at home with his parents and 7-year-old sister, C.R., who also has CF. J.R. reports that he “doesn’t feel good,” explaining that he has missed the last week of school, doesn’t have any energy, is coughing more, and is having “a hard time breathing.”

1. Discuss additional data that should be obtained from J.R. and his parents.

2. CF is a multisystem disorder. Describe the condition and its physiologic effect on the following systems: respiratory system, gastrointestinal system, reproductive system, skin, and electrolyte balance. Focus on factors that place J.R. at risk for developing respiratory infections.

Case study progress J.R. is admitted to the hospital for a suspected respiratory tract infection and CF exacerbation. His mother helps get him settled into the room. Your assessment includes the following vital sign 115/76, 85, 28, oral T 38.8°C, and Spo2 88% on room air. J.R. weighs 30 kg. His color is pale and skin is dry, with bluish-tinged nail beds and clubbing; capillary refill is 3 seconds. Respiratory effort is labored, with coarse productive cough and rhonchi noted throughout. He states his pain is 3 to 4 on a scale of 10, with coughing. His thorax has a barrel-chest appearance and appears thin with decreased muscle mass. His last void and bowel movement were this morning, with no problems. J.R. is anxious and answers questions in short phrases.

3. Your institution uses electronic charting. Based on the assessment just described, which of the following systems would you mark as abnormal as you document your findings? Mark abnormal findings with an “X” and provide a brief narrative note.

□ Neurologic: □ Respiratory: □ Cardiovascular: □ Gastrointestinal: □ Genitourinary: □ Musculoskeletal: □ Skin: □ Psychosocial: □ Pain:

4. What are the common microorganisms that cause respiratory infections in children with CF?

Chart View

Medication Orders

Ceftazidime (Fortaz) 2 g IV q8hr Gentamicin 100 mg IV q8hr Vancomycin (Vancocin) 450 mg IV q8hr

5. You review the drugs that have been ordered to treat J.R.’s suspected infection. You are orienting a new nurse. Which statement would you question as you review the ordered medications with her?

a. “I have assessed for possible allergies or hypersensitivities.”

b. “I will need to monitor serum levels for some of these medications.”

c. “I will verify dosage and medication compatibility with the pharmacy.”

d. “I will obtain a blood glucose level before administering the medications.”

6. Using a nursing drug reference, find the safe dosage

ranges and calculate the dosage for the prescribed antibiotics. Are the prescribed doses within the safe ranges? Explain your answers and show all work.

7. You are reviewing the physician orders for respiratory care. State whether you would expect to perform each of the following interventions and give your rationale.

a. Administer aerosolized albuterol (2.5 mg/3 mL) b. Administer chest physiotherapy (CPT) before

administering the albuterol c. Monitor continuous pulse oximetry d. Administer aerosolized dornase alfa (Pulmozyme)

after administration of bronchodilator e. Administer nebulized normal saline (NS)

f. Administer tobramycin (TOBI) via jet nebulizer g. Limit fluid intake

8. Methods of providing CPT are changing. Traditionally, someone other than the patient had to administer the chest percussion or vibration to help loosen mucus secretions. Newer technology, such as high-frequency chest wall oscillation (the Vest) and the positive expiratory pressure (PEP) devices (the Flutter mucus clearance device) are available. Explain the benefits of these new devices.

9. J.R.’s weight is below the 5th percentile. He has been on a high- calorie, high-protein diet with unrestricted fat at home. J.R. reports that he hasn’t been hungry and really hasn’t been eating much. Describe the link between malnutrition and CF.

10. Which of the following actions can be delegated to the unlicensed assistive personnel (UAP)? Explain your answer.

a. Charting daily weights and intake and output b. Increasing O2 during an episode of desaturation c. Instructing the parents on correct administration of

NS nebulizers d. Administering pancreatic enzymes from the home

supply with each snack 11. Which strategy is appropriate to manage GI dysfunction that

patients with CF often experience? Select all that apply. a. Restrict fat intake b. Encourage a high-protein diet c. Encourage snacks between meals d. Administer fat-soluble vitamins daily e. Hold proton pump inhibitor or H2 blocker f. Breastfeeding is contraindicated in infants with CF g. Administer pancreatic enzymes with meals and

snacks 12. What clinical sign assists in determining the effective dosage

of pancreatic enzymes? 13. Which GI comorbidities might you see in a patient with CF?

Select all that apply.

a. Constipation b. Celiac disease c. Rectal prolapse d. Meconium ileus e. Chronic vomiting

Case study progress J.R. will be spending 14 to 21 days in the hospital for treatment of his pulmonary infection.

14. List interventions that would foster development while J.R. is hospitalized.

15. J.R. asks you, “Will I be able to have children when I grow up?” While keeping his age in mind, which of these would be the best response? Explain your answer.

a. “You should discuss this with your parents. I will let them know you asked.”

b. “Most males have a significant chance of being sterile, and you won’t need to consider use of contraception.”

c. “CF does not affect the male reproductive system; however, it does affect the female reproductive tract.”

d. “Although nearly 95% of males are sterile, you should discuss this with your physician and family.”

Case study progress J.R.’s condition improves with antibiotic therapy, and he is being discharged to home.

16. As you provide your discharge teaching, discuss health promotion points to reinforce with J.R. and his parents.

Case study outcome

J.R. is discharged to home, with a PEP device, and enjoys the fact that he is able to apply it himself.

Case Study 32 Name _________________________________ Class/Group _____________________________ Date _____________

Scenario C.E., a 73-year-old married man and retired railroad engineer, visited his physician complaining, “Whenever I try to do anything, I get so out of breath I can’t go on. I think I’m just getting older, but my wife told me I had to come see you about it.” He has a history of hyperlipidemia and hypertension, which are managed with lisinopril, furosemide, atorvastatin, and metoprolol, and has a 30-pack-year smoking history. He had greenish-yellow sputum, an oral temperature of 100.4 ° F (38.4 ° C), and an Spo2 of 83%. He was admitted to the local hospital with a new diagnosis of COPD exacerbation. After a 5-day stay during which the diagnosis of COPD was confirmed, he was discharged on continuous oxygen (O2) therapy at a 2 L flow rate with new prescriptions for fluticasone/salmeterol (Advair Diskus) and albuterol (ProAir). You are the case manager at the home agency assigned to C.E.

1. What is the rationale for C.E.’s O2 being at a 2 L flow rate? 2. What criteria need to be met for Medicare to pay for home O2

therapy? 3. Which member of the interprofessional team would be

involved in C.E.’s care and how? 4. How would you prepare for the initial home visit? 5. What would you address with C.E. and his wife at the first

visit? 6. What manifestations of COPD would you expect C.E. to have?

(Select all that apply.) a. Elevated temperature b. Complaints of fatigue c. Wheezing on auscultation d. Chronic, intermittent cough e. Prolonged expiratory phase f. Decreased anterior-posterior diameter

7. What patient problems would you to address as you develop C.E.’s plan of care?

8. What ongoing monitoring do you need to do at each visit with C.E.?

9. What are 2 specific questions you can ask C.E. to determine if he may be experiencing any complications?

Case study progress Your first visit with C.E. and his wife is uneventful. You perform his baseline assessment, provide teaching, and establish a monitoring plan. The next time you visit, C.E. complains of sores behind his ears. He explains, “That long oxygen tubing seems to take on a life of its own. It twists around and gets caught under doors, chairs, everything. It darn near rips the ears off my head.”

10. What can you tell him that could help? 11. You auscultate C.E.’s breath sounds and detect the odor

of menthol rub. When you question C.E. about it, he tells you that he started to apply it in and around his nose to prevent his nose from becoming dry and sore. What specific teaching do you need to reinforce with C.E. and his wife?

12. After you have finished, his wife seems upset and tells

you that C.E. is still “smoking a couple of cigarettes” a day. How do you handle this situation?

Case study progress

At your next visit 2 weeks later, C.E. tells you that he has not smoked since your previous visit. He is upset, though, over an episode a few days ago. He says he walked to the kitchen for a snack and became short of breath. Per your instructions, C.E. removed the nasal cannula, tested the flow against his check, and felt no O2 flowing from the catheter. He said he could not yell for help and was too short of breath to return to the living room to check his O2 tank. He bent forward with his elbows on the countertop and struggled to breathe. He became more frightened with each passing second, and his breathing became increasingly more difficult. Several minutes later, C.E.’s wife found him and reconnected his O2 tubing. C.E. sat at the table for 20 minutes before he could walk back to the living room.

13. Why did C.E. assume the peculiar position at the countertop? 14. C.E.’s wife states that since the incident, C.E. “doesn’t want her

out of his sight.” She asks you to “talk some sense into him.” She goes on to say that since then “All he does is sit in a chair all day. He won’t even get up to get himself a glass of water. I’ve got a bad hip, and this is all very hard on me.” What will you do to help C.E. and his wife cope with his condition?

15. What referrals could you consider at this time and why?

Case study progress The next few visits are uneventful. C.E. has continued to not smoke and is doing better with managing episodes of dyspnea. At your next visit, you greet C.E., immediately note that he sounds congested, and comment that he sounds like he has a cold. He replies, “Oh, our great- grandchildren were over to visit several days ago and they all had snotty noses.”

16. What is your immediate concern and why? 17. What assessment do you need to perform? 18. Which assessment finding would require your immediate

intervention? a. C.E.’s sputum is yellowish

b. The pulse oximeter reading is 93% c. His oral temperature is 102.0° F (38.9° C) d. He is dyspneic when walking to the bathroom.

19. What information would you want to review with C.E. and his wife about the signs and symptoms of infection and when to seek treatment?

20. What basic hygiene measures would you include in a teaching plan for C.E. and his wife to prevent his developing an infection? Select all that apply.

a. Avoid enclosed, public areas at all times. b. Get pneumonia and flu vaccines every year. c. Avoid people with cold and flu infections and

screen visitors. d. Practice good hand washing technique, and wash

hands often. e. Use the dishwasher to wash eating utensils, glasses,

and plates. f. Use antibacterial wipes daily to clean frequently

touched surfaces. 21. C.E.’s wife says she would like to read more about COPD on

the Internet. List 2 credible resources to which you could direct her.

Case study outcome You continue to visit C.E. and his wife and monitor his progress over the next several months. C.E. experiences an episode of pneumonia and experiences acute respiratory failure and sepsis, which leave him with residual neurologic deficits. He is discharged to a long-term care facility.

Case Study 33 Name _________________________________ Class/Group _____________________________ Date _____________

Scenario D.Z., a 68-year-old man, is admitted at 1600 to a medical floor with an acute exacerbation of chronic obstructive pulmonary disease (COPD). His other medical history includes hypertension and type 2 diabetes. He has had pneumonia yearly for the past 3 years and has been a two- pack-a-day smoker for 38 years. His current medications include enalapril (Vasotec), hydrochlorothiazide, metformin (Glucophage), and fluticasone/salmeterol (Advair). He appears a cachectic man who is having difficulty breathing at rest. D.Z. seems irritable and anxious; he complains of sleeping poorly and states that lately he feels tired most of the time. He reports cough productive of thick yellow-green sputum. You auscultate decreased breath sounds, expiratory wheezes, and coarse crackles in both lower lobes anteriorly and posteriorly. His vital signs (VS) are 162/84, 124, 36, 102° F (38.9° C), and Spo2 88%.

Chart View

Admission Orders

Diet as tolerated Out of bed with one-person assistance Oxygen (O2) to maintain Spo2 of 90% IV of D5W at 50 mL/hr ECG monitoring ABGs in a.m. CBC with differential now Basic metabolic panel now Chest x-ray daily Sputum culture Albuterol 2.5 mg plus ipratropium 250 mcg nebulizer treatment

STAT

1. Are D.Z.’s VS and Spo2 acceptable? If not, explain why.

2. Describe a plan for implementing these orders. 3. What is the primary nursing goal at this time? 4. Based on this priority, name 3 independent nursing actions you

would implement and why. 5. Identify 3 expected outcomes for D.Z. as a result of your

interventions.

Chart View

Medication Administration Record

Methylprednisolone (Solu-Medrol) 125 mg IV push every 8 hours

Azithromycin (Zithromax) 500 mg IVPB q24hr Fluticasone/salmeterol (Advair) 100/50 mcg 2 puffs twice daily Heparin 4000 units subcutaneous every 12 hours Enalapril (Vasotec) 10 mg PO daily Albuterol 2.5 mg/ipratropium 250 mcg nebulizer treatment

every 6 hours Metformin (Glucophage) 500 mg PO twice daily Nicotine patch 24 mg for 24 hours, daily

6. Indicate the expected outcome associated with each medication D.Z. is receiving.

7. Because D.Z. is on azithromycin, what interventions need to be included in his plan of care? Select all that apply.

a. Place D.Z. on intake and output. b. Administer the medication over 30 minutes. c. Request a hearing test before initiating therapy. d. Monitor IV site for inflammation or extravasation. e. Assess liver function study results and bilirubin

levels. f. Carefully dilute the medication in the proper

amount of solution.

8. D.Z is ordered heparin 4000 units subcutaneous q12hr.

The following vial is available. How many milliliters will D.Z. receive? Shade in the dose on the syringe.

9. What common side effects of bronchodilator therapy do you need to assess for?

10. When you apply the nicotine patch, you take the opportunity to ask D.Z. if he would like information on smoking cessation. He tells you he is interested because he has to “stop while he is in here.” How can you support his continuing cessation after discharge?

11. You deliver D.Z.’s dietary tray, and he comments on how hungry he is. As you leave the room, he is rapidly consuming the mashed potatoes. When you pick up the tray, you notice that he has not touched anything else. When you question him, he states, “I don’t understand it. I can be so hungry, but

when I start to eat, I have trouble breathing and I have to stop.” Describe 4 interventions that might improve his caloric intake.

12. What other interprofessional team members could you involve in D.Z.’s care and how?

13. After speaking with D.Z. about his diet and reviewing his

medications, you are now concerned about his glycemic control. Hospital policy allows you to obtain as-needed blood glucose levels for patient with diabetes, so you direct the UAP to obtain D.Z.’s blood glucose level before the next meal. What is your responsibility in delegating this task to the UAP?

14. The UAP reports that D.Z.’s blood glucose level is 366 mg/dL. What action do you need to take and why?

Case study progress The next morning, D.Z. is sitting in the bedside chair and appears to be experiencing less difficulty breathing. He states his cough remains productive of yellow-green sputum, although it is “easier to cough up” than it was the previous day. You auscultate decreased breath sounds and a few coarse crackles in both lower lobes posteriorly. His VS are 150/78, 94, 24, 99.7° F (37.6° C). His Spo2 is 92% with O2 on at L per nasal cannula.

Chart View

Arterial Blood Gases

pH 7.33

Paco2 58 mm Hg

HCO3 32 mEq/L (32 mmol/L)

Pao2 65 mm Hg

Sao2 92%

15. Interpret D.Z.’s ABG values. 16. Has D.Z.’s status improved or not? Defend your response. 17. What interventions would you include in your plan of care for

D.Z. today?

Case study progress D.Z.’s wife approaches you in the hallway and says, “I don’t know what to do. My husband used to be so active before he retired 6 months ago. Since then he’s lost 35 pounds (16 kg). He is going downhill so fast that it scares me. He wants me in the room with him all the time, but if I try to talk with him, he snarls and does things to irritate me. I have to keep working. We don’t want to drain all of our savings, and I have to be able to support myself when he’s gone. Sometimes I go to work just to get away from the house and his constant demands. He calls me several times a day asking me to come home, but I can’t go home. I just don’t know what to do.”

18. How would you respond to her statement? 19. What topics do you need to address with D.Z. to reduce his

risk for readmission?

Case study outcome D.Z. is discharged after 6 days with instructions to follow up with the primary care provider in 1 week. At the appointment, D.Z. reports he has been able to maintain his smoking cessation and is feeling less dyspnea. The provider continues the respiratory medication regimen and arranges for D.Z. to begin pulmonary rehabilitation therapy the next week.

Case Study 34 Name _________________________________ Class/Group _____________________________ Date _____________

Scenario E.M., a 5-month-old girl, is brought to the emergency department (ED) with respiratory distress, hypoxia, and fever. Her parents state that she has had mild cold symptoms for a few days. She has breastfed poorly over the last few days, with a decreased number of wet diapers. You take her vital signs and complete an initial physical assessment and history.

Chart View Vital Signs

Blood pressure 130/72

Respiratory rate 83

Heart rate 188

Temperature 38.6°C (101.5°F)

Spo2 88% on room air

Weight 17.6 lbs (8 kg)

Initial Assessment

Neur o l o g ic

Alert, fussy; consoles briefly; anterior fontanel soft and slightly depressed

Card i o v a

Tachycardia; capillary refill less than 3 seconds

s c u la r

Resp ir at o r y

Upper airway congestion; coarse cough; tachypnea, transient bilateral wheezing; coarse rhonchi and slightly decreased breath sounds at bases; mild intercostal retractions; mild nasal flaring

Gast r o i n te st i n al

Positive bowel sounds; last bowel movement yesterday

Geni t o u ri n a r y

Decreased urine output (per history); no urine output in last 4 hours

Skin No rashes; slightly flushed

Othe r

Mucous membranes “sticky”; decreased tearing

Emergency Department Orders

Acetaminophen (Tylenol) 80 mg PO for fever × 1 dose Start IV and administer normal saline (NS) bolus 20 mL/kg IV

bolus over 30 minutes Oxygen to keep saturation greater than 92%

1. Review the standing ED orders. Prioritize your interventions and give rationales.

2. Based on E.M.’s vital signs and assessment, what diagnostic tests would you anticipate?

3. Calculate how much normal saline E.M. will receive as a

bolus.

Case study progress E.M. begins coughing and has copious nasal secretions. You provide nasopharyngeal suctioning and obtain a large amount of thick secretions. Deep suctioning is used as infrequently as possible because it can be counterproductive, causing increased edema and secretions, and increased agitation of infant. She is allowed to recover and is reassessed. The respiratory rate and retractions have not changed significantly. Her breath sounds are less coarse but are diminished in the bases. The Spo2 is now 92% to 93% on 1.5 L oxygen. After E.M. settles, her mother asks whether she can feed her because she has not eaten much for the past few days. You tell her that with a current respiratory rate higher than 65, she should not be fed.

4. What is the rationale for holding feedings? 5. When E.M.’s respiratory rate decreases, what teaching would

you provide the parents concerning feeding?

Chart View

Medication Administration Record

Normal saline drops to nares q3hr with bulb suctioning Acetaminophen (Tylenol) 80 mg PO q4hr prn for fever greater

than 38.5°C (101.3°F) Oxygen via nasal cannula (N/C) adjusted up to 2 L/min to

maintain Spo2 greater than 90%. Amoxicillin (Amoxil) 45 mg/kg/day PO tid × 7 days

6. You are reviewing the medication administration record. Which order(s) would you question? Explain.

Case study progress E.M.’s mother calls you to the room because her baby is “not right.” You note E.M.’s respiratory rate is 23, and the intercostal retractions and nasal flaring have increased. She also has moderate subcostal retractions. The Spo2 is 89% on 1.5 L/min of O2. She is pale and listless and does not cry with stimulation. The oxygen is increased to 2 L/min with no change in status after 5 minutes.

7. Why is the respiratory rate significantly lower even though other signs of respiratory distress have increased?

Case study progress You are concerned and call the rapid response team. You check her Spo2 again with results of 88%. The senior resident orders a portable chest x-ray (CXR) examination and capillary blood gas (CBG). The CXR is consistent with bronchiolitis with atelectasis.

Chart View

Capillary Blood Gas

pH 7.31

Paco2 72 mm Hg

HCO3 29 mEq/L (29 mmol/L)

Pao2 57 mm Hg

8. Interpret E.M.’s CBG results.

Case study progress E.M. is transferred to the pediatric intensive care unit (PICU) and placed on a heated high-flow nasal cannula, a noninvasive method that offers many of the same benefits as continuous positive airway pressure (CPAP), at 5 L/min. You know that patients with bronchiolitis generally have progressively increased symptoms from days 3 to 5. This is day 3 for E.M., and therefore she may stay in the PICU for several days until she starts improving and can be transferred to the general care floor. You explain this to the parents, who are very distressed.

9. What resources might you seek for E.M.’s parents during this unanticipated change in status?

10. You are considering measures to help E.M.’s parents deal with the anxiety and fear created by their infant’s hospitalization and transfer to PICU. For each measure listed, explain why each answer is appropriate or not.

a. The presence of the parents helps calm the infant. b. Parents need to put the needs of E.M. ahead of their

own. c. Parental anxiety cannot be recognized by a 5-

month-old infant. d. Provide detailed technical information about her

care in the PICU to help the parents cope with the situation better.

Case study progress After 2 days in the PICU, E.M. is transferred back to your unit. You note that she is taking increased oral fluids and requiring less suctioning. Her Spo2 is 96% to 98% on room air. She is still having some mucus production and coughing. As you are preparing the parents for discharge, they want to know how they can prevent this in the future. They ask whether there is a “shot” E.M. can get to avoid getting this again.

11. How would you address their concerns? 12. E.M.’s parents ask you for instructions about the treatment of

cold symptoms if E.M. develops them again. Which answer is correct? Explain your answer.

a. “If a fever is present, you can treat the fever with baby aspirin.”

b. “Saline nose drops and bulb suctioning can be done before feedings.”

c. “Over-the-counter cough suppressants may be safely administered at night.”

d. “You do not need to worry if she is not drinking; intake should improve in a day or so.”

13. What other specific teaching relating to fluid balance and normal signs and symptoms on discharge should be done before E.M. is discharged? E.M. now weighs 17 lbs (7.8 kg) and is drinking from a 6-oz bottle.

Case study outcome E.M. is discharged to home with her parents. The parents are instructed follow up with her pediatrician within the next few days and to call with any concerns.

Case Study 35 Name _________________________________ Class/Group

_____________________________ Date _____________

Scenario You are the trauma nurse working in the emergency department (ED) of a busy tertiary care facility. You receive a call from the paramedics that they are en route with the victim of gunshot wounds to the chest and abdomen. Perpetrators shot the 32-year-old man during a convenience store robbery. The victim was a customer; the clerk died at the scene. They started 2 16-gauge IV lines with lactated Ringer’s solution and oxygen by nonrebreather mask at 15 L/min. The patient has a sucking chest wound on the left and a wound in the right upper quadrant of the abdomen. Vital signs in the field are 90/46, 140, and 42. The paramedics state they are having difficulty ventilating the patient, who is diaphoretic, very pale, and lethargic. The estimated time of arrival is 4 minutes.

1. To help determine the extent of the patient’s injuries, the most important question the nurse needs to ask the paramedics is:

a. “How long ago was the patient shot?” b. “Do you have the weapon that was used?” c. “What was the reason this incident occurred?” d. “Where are the assumed entry and exit wounds?”

2. Describe a sucking chest wound. 3. What are the classic assessment findings associated with a

sucking chest wound? 4. Explain why the paramedics are having difficulty ventilating

the patient. 5. List at least 6 things you will do to prepare for the patient’s

arrival. 6. Describe the priority roles of the nurse in a trauma situation. 7. Besides the lead provider, who usually responds to a trauma

code and what are their roles? 8. What are your basic responsibilities regarding gathering

forensic evidence?

Case study progress On arrival, the patient, B.W., is cyanotic and in severe respiratory distress. His VS are 70/30, 160, and 42. When he is transferred to the trauma stretcher, you notice that there is an occlusive dressing over the open chest wound. It is taped down on all sides.

9. Is taping the occlusive dressing on all sides appropriate? Explain.

10. Which assessment findings would indicate B.W. may have a tension pneumothorax? Select all that apply.

a. Bilateral jugular vein distention b. Tracheal deviation toward the affected side c. Decreased breath sounds on the opposite side d. Tracheal deviation toward the unaffected side e. Decreased heart rate and decreased respirations

Chart View

Arterial Blood Gases (100% O2)

pH 7.25

Paco2 92 mm Hg

Pao2 32 mm Hg

HCO3 26 mEq/L (26 mmol/L)

Spo2 53%

11. Interpret B.W.’s ABG results. 12. Based on B.W.’s vital signs and condition, what are the priority

interventions?

Case study progress The surgeon determines B.W. has a tension pneumothorax and

performs a needle decompression followed by insertion of a size 32-Fr chest tube in the sixth intercostal space, midaxillary line. Treatments aimed toward stabilizing B.W.’s respiratory and volume statuses are started, and the anesthesiologist prepares to intubate B.W.

13. What are your responsibilities regarding endotracheal intubation?

14. How is endotracheal tube placement confirmed after

intubation? 15. After B.W. is successfully intubated, describe the actions that

you will perform as part of the secondary survey. 16. Would you expect his lung to reexpand immediately after the

chest tube insertion and starting suction? Explain. 17. When evaluating the effectiveness of chest tube therapy, you

would assess for: a. Increased drainage in the collection chamber b. Continuous bubbling in the water seal chamber c. A fall in the water seal chamber with inspiration d. Symmetrical breath sounds and chest expansion

18. You note the following ecchymotic area on B.W.’s abdomen along with distention and rigidity. What might this signify?

19. You report your findings to the physician, who orders a focused abdominal sonography for trauma (FAST). Why is this procedure appropriate for B.W.?

Case study progress The FAST is positive. B.W. is sent for a CT of the abdomen, which shows a large liver laceration. The surgeon needs to take B.W. to the operating room (OR) for an exploratory laparotomy with repair of the liver laceration.

20. Because B.W. is not able to give consent for surgery, how

will consent be obtained? 21. When you return from transporting the patient to the OR,

B.W.’s wife is in the ED, upset and frightened. The social worker has been called to another emergency. What information do you need to obtain from B.W.’s wife?

22. How would you support her?

Case study outcome Surgeons repaired B.W.’s wounds. He spent 3 days in the surgical intensive care unit, where he required continued fluid and respiratory support, and 14 days later he went home. After discharge, B.W. spent 3 months receiving physical therapy and counseling for victims of violent crime.

Case Study 36 Name _________________________________ Class/Group _____________________________ Date _____________

Scenario A.B., a 68-year-old man, is admitted to your medical floor with a diagnosis of pleural effusion. He relates having shortness of breath; pain in his chest; weakness; and a dry, irritating cough. His vital signs (VS) are 142/82, 118, respirations 38 and labored and shallow, 102.1° F (38.9° C), and Spo2 85% on room air. Chest x-ray examination shows a large pleural effusion and pulmonary infiltrates in the right lower lobe consistent with pneumonia.

1. Given his diagnosis, are A.B.’s admission VS expected? Explain.

2. How does the underlying pathophysiology relate to A.B.’s presenting signs and symptoms?

Case study progress The provider performs a thoracentesis and drains 1500 mL of fluid. A specimen for culture and sensitivity (C&S) is sent to the lab, and A.B. is started on ceftriaxone 500 mg IV every 12 hours.

3. What is a thoracentesis? 4. The order for the ceftriaxone reads to infuse the 100 mL

0.9% NaCl solution over 30 minutes. An IV pump will not be available for a few hours, so you decide to run the infusion by gravity. You have IV tubing that supplies 20 gtt/mL. At how many drops per minute will you regulate the infusion?

5. What interventions will you implement to help A.B.’s clear pulmonary secretions?

Case study progress The pleural C&S results show a large amount of Klebsiella organism growth that is not sensitive to ceftriaxone.

6. What action will you take next?

7. Because fluid continues to collect in the pleural space, the physician decides to insert a pleural chest tube under nonemergent conditions. What are your responsibilities as A.B.’s nurse regarding this procedure?

8. Part of your responsibilities after the chest tube has been inserted is to assess for fluctuation in the water-seal chamber and bubbling in the suction-control chamber. Label the areas on the chest drainage system that you would be monitoring.

(Modified from Linton AL. Introduction to Medical-Surgical Nursing, 4th ed. St. Louis, MO: Saunders; 2007.)

9. What interventions will you implement afterward to maintain A.B.’s chest tube system?

10. Evaluate each of the following statements about chest tube drainage systems. Enter T for true or F for false. State why false statements are incorrect.

_____1. The height of the water in the suction control mechanism limits the amount of suction transmitted to the pleural cavity.

_____2. A suction pressure of + 20 cm H2O is usually recommended for adults.

_____3. Bubbling in the water-seal chamber usually

means that air is leaking from the lungs, the tubing, or the insertion site.

_____4. The rise and fall of the water level with the patient’s respirations reflect normal pressure changes in the pleural cavity with respirations.

_____5. Water cannot evaporate from the system. _____6. To declot the drainage tubing, put lotion on

your hands, compress the tubing, and vigorously strip long segments of the tubing before releasing.

_____7. You lower the bed on top of the drainage system and break it. You immediately clamp the chest tube, leaving it clamped until you can reestablish the drainage system.

_____8. The collection chamber is full, so you need to connect a new drainage system. Briefly clamp the chest tube while you disconnect the old system and connect the new.

11. While A.B. has a chest drainage system, what instructions do you need to give to the UAP who is working with A.B.?

12. It is now the end of your shift and A.B.’s condition has stabilized. Using the SBAR framework, describe the bedside change-of-shift report you will give the oncoming nurse.

Case study progress The next day you are again assigned to care for A.B. At the beginning of the shift, you assess A.B. and find that his condition is stable. His lung sounds remain diminished in the right lower lobe and his Spo2 is 95% on oxygen at 2 L per nasal cannula. The chest drainage system is attached to suction at 20 mm Hg; there is still an air leak present. His morning chest x-ray examination showed some residual pleural effusion. Four hours into your shift, he pages you through the call system and tells you he feels “short of breath.” You immediately go to his room. A.B. is sitting in the chair.

13. Describe the priority assessment you must perform at this

time. 14. You find the chest tube has become disconnected from the

drainage system and is now contaminated. What do you need to do?

15. After the chest drainage system has been reestablished, A.B.’s dyspnea resolves, and you need to document what happened. Write an example of a documentation entry describing this event.

Case study progress The remainder of A.B.’s admission is uneventful. After 6 days of aggressive antibiotic and pulmonary therapy, the chest tube is removed and A.B. is ready for discharge.

16. What type of discharge instructions do you need to give to A.B.?

Case study outcome A week later, A.B. and his wife send a cookie tray to the floor, along with a note praising the staff for the quality care A.B. received. They also nominate you for one of your facility’s nursing excellence awards, saying that you “provided amazing care” and were the type of nurse “everyone wants by their side.”

Case Study 37 Name _________________________________ Class/Group _____________________________ Date _____________

Scenario S.K., a 51-year-old roofer, was admitted to the hospital 3 days ago after falling 15 feet from a roof. He sustained bilateral fractured wrists and an open fracture of the left tibia and fibula. He was taken to

surgery for open reduction and internal fixation (ORIF) of all his fractures. He is recovering on your orthopedic unit. You have orders to begin getting him out of bed and into the chair today. When you enter the room to get S.K. into the chair, you note he is agitated and dyspneic. He says to you, “My chest hurts really badly. I can’t breathe.” You auscultate S.K.’s breath sounds and find they are diminished in the left lower lobe. S.K. is diaphoretic and has circumoral cyanosis. His VS are 142/82, 118 and irregular, respirations 38 and labored and shallow, 99.8° F (37.7° C), and Spo2 85% on O2 at 2 L nasal cannula

1. Identify 5 possible reasons for S.K.’s symptoms. 2. List in order of priority 3 actions you should take next. 3. Using SBAR, what information will you give to the provider?

Case study progress The attending orders the following: STAT arterial blood gases (ABGs), chest x-ray (CXR) examination, 12-lead electrocardiogram, and a helical (spiral) computed tomography (CT) scan of the lungs.

Chart View

Arterial Blood Gases

pH 7.49

Paco2 30.6 mm Hg

Pao2 52 mm Hg

HCO3 24.2 mmol/L

Sao2 83%

4. Interpret S.K.’s ABG results. 5. Based on the ABGs and your assessment findings, what

complication do you think S.K. is experiencing? 6. Why is S.K. at risk for developing this complication? 7. What are the typical manifestations of this complication?

Underline those that S.K. has. 8. How is this complication best prevented from occurring? 9. The resident writes the following orders for S.K. Review each

order. Mark with an A if the order is appropriate; mark with an I if the order is inappropriate. Correct all inappropriate orders and give rationales for your decisions.

_____1. Albuterol (Proventil) metered-dose inhaler, 2 puffs q6hr

_____2. Heparin 20,000 units IV now, then 25,000 units in 250 mL/D5 W to run at 18 units/kg per hour

_____3. Prothrombin time/international normalized ratio/partial thromboplastin time (PT/INR/PTT) q6hr; call house officer with results

_____4. Increase O2 to 3 L by nasal cannula _____5. Patient-controlled analgesia (PCA) pump with

morphine sulfate: Loading dose 4 mg; dose 2 mg; lock-out time 15 minutes; maximum 4-h dose 30 mg

_____6. Streptokinase 250,000 international units IV over 30 minutes, then 100,000 international units/hr for 24 hr

_____7. Warfarin (Coumadin) 7.5 mg/day PO × 2 days _____8. Complete blood count daily

10. S.K. weighs 224 lb. Calculate the starting infusion rate in

milliliters per hour for the heparin. (Round to the nearest tenth.)

11. S.K. asks why he is starting heparin. Your best response is: a. “It will stop any blood clots from going to your

lungs.” b. “The heparin will dissolve any other blood clots

you have.” c. “Heparin will prevent any new blood clots from

developing.” d. “The heparin will thin your blood so you will be

able to breathe better.”

Case study progress All the orders are corrected. S.K.’s helical CT scan confirms the diagnosis of pulmonary embolism (PE) in the left lower lobe. Two hours later, repeat ABGs provide the values shown in the chart.

Chart View

Arterial Blood Gases

pH 7.46

Paco2 33 mm Hg

Pao2 82 mm Hg

HCO3 24.1 mEq/L

Sao2 90%

12. What do these ABGs indicate? 13. The attending orders furosemide 20 mg IV push now. What is

the expected outcome associated with giving furosemide to S.K.?

14. What is your primary nursing goal at this time? 15. List 4 independent nursing interventions you would

implement for S.K. that support this goal and the rationale for each.

16. Because S.K. is receiving heparin therapy, he has the

potential for bleeding. What interventions will be part of his plan of care to reduce this risk? Select all that apply.

a. Assess VS every 4 hours.

b. Use a central line to obtain blood specimens. c. Apply direct pressure to any venipuncture site for 5

minutes. d. Do not give any intramuscular medications unless

absolutely necessary. e. At least once a shift, check stool, urine, sputum, and

vomitus for occult blood. 17. To ensure S.K.’s safety, what instructions would you give

to the UAP assisting with his care? Select all that apply. a. Use an electric razor when shaving S.K. b. Immediately report any signs of bleeding. c. Inflate the BP cuff only as high as needed to obtain

a reading. d. Position S.K. with the head of his bed elevated, on

his left side. e. Use a sponge-toothed applicator when helping S.K.

with oral care.

Chart View

Laboratory Test Values

PT 12.1 seconds

PTT 54 seconds

INR 1.4

Chart View

Weight-Based Heparin Protocol

1. Begin IV infusion of heparin at 18 units/kg per hour using 25,000 units heparin in 250 mL D5 W. Obtain coagulation panel every 6 hours.

2. Adjust IV heparin based on activated partial thromboplastin time results:

• ↓ 49 s, bolus with 80 units/kg and ↑ rate by 4 units/kg per hour

• 50 to 59 s, bolus with 40 units/kg and ↑ rate by 2 units/kg per hour

• 60 to 90 s, no change • ↑ 91 to 109 s, bolus with 80 units/kg and ↓ rate by 2

units/kg per hour • ↑ 110 s, stop infusion for 1 hour, then resume and ↓

rate by 3 units/kg per hour

18. Coagulation times are rechecked after S.K. has been on heparin therapy for 6 hours. What action do you need to take?

19. The heparin is supplied in a multiple-dose vial. How

many milliliters will you administer? (Round to the hundredth.)

Case study progress S.K. is watched closely for the next several days for the onset of pulmonary edema. Anticoagulant therapy, O2, pulse oximetry, daily CXR studies and ABG analysis, and pain management are continued.

20. On postoperative day 7, S.K. suddenly becomes very angry and throws the physical therapist out of his room. He yells, “I’m sick and tired of having everyone tell me what to do.” How are you going to deal with this situation?

Case study outcome Ten days after the occurrence of the PE, S.K. was discharged to a long- term acute care facility for assistance with strengthening and ambulation.

Case Study 38 Name _________________________________ Class/Group _____________________________ Date _____________

Scenario The intensive care unit (ICU) nurse calls to give you the following report: “D.S. is a 66-year-old man who is postop day 3 after a right lower lobe lobectomy. He had quit smoking 12 years ago and exercises regularly. He went to see his physician with complaints of increasing exertional dyspnea; CXR and imaging showed a mass in the right lung without any evidence of metastases. The pathology report came back today and confirming stage IB non–small cell lung cancer. The surgeon said there were clean margins. He has no neurologic deficits. His vital signs run 120/70, 110, about 30, with a fever around 100.2° F. His heart tones are clear, all peripheral pulses are palpable, and he is receiving IV D5 ½ NS at 50 mL/hr in his PICC line. He has a right midaxillary chest tube; there is no air leak, and it is draining small amounts of serosanguineous fluid. He has pain at the insertion site, but the site looks good, and the dressing is dry and intact. He is on 4 L oxygen by nasal cannula. He has not had any pain medication today, nor has he slept since surgery. He’ll be there in about 20 minutes.”

1. What additional information would you ask the nurse to provide?

2. Smoking accounts for 60% of lung cancers. True or false? 3. D.S. presented with exertional dyspnea. What are other

common manifestations of lung cancer? 4. Describe the type of surgery D.S. underwent and its role in

lung cancer treatment. 5. What are the implications of D.S.’s cancer being a stage IB and

the surgeon reporting clean margins? 6. What is the health care team’s primary goal for D.S. at this

time? 7. Outline interventions you will include in his plan of care to

help achieve this goal and give a rationale for each.

Case study progress D.S. is transported by wheelchair past the nurses’ station to a room at the end of the hall. You enter his room for the first time to find him sitting on the edge of the bed with his left leg in bed and his right foot on the floor. As you introduce yourself, you note that he keeps rubbing his left hand over the right side of his chest. His respiratory rate is 30 with an Spo2 of 90%. His respirations are slightly labored, and he appears dyspneic.

8. What issues or problems can you already identify? 9. Describe 3 things you need to do right now for D.S.

Case study progress When asking D.S. about his reluctance to take pain medication, he replies, “I have a nephew who rolled his Jeep and busted himself up real bad. He got hooked on those drugs, and I don’t want any part of them.”

10. How would you respond to D.S.’s statement? 11. Which nonpharmacologic methods of pain relief may help

D.S.? Select all that apply.

a. Hypnosis b. Distraction c. Positioning d. Biofeedback e. Acupuncture

12. You administer morphine sulfate 4 mg IV and tell D.S. that you will return in 30 minutes; 15 minutes later, he turns on his call light. When you enter the room, D.S. says, “I think I’m going to throw up.” What are the next 3 things you would do?

13. D.S. states, “I started to feel sick a couple minutes ago. It just kept getting worse until I knew I was going to throw up.” What do you think caused the sudden onset of nausea?

14. Should you give D.S. another dose of morphine? What are your options?

15. You decide to call the surgeon to report the reaction. Using SBAR, what would you report?

16. The surgeon changes D.S.’s pain medication to fentanyl

and orders ondansetron (Zofran) 4 mg IV every 6 hours as needed for nausea. You use a floor-stocked multidose vial to administer the first dose. How many milliliters will you give?

(From Macklin D, Chernecky C, Infortuna MH. Math for Clinical Practice. 2nd ed. St. Louis, MO: Mosby; 2011.)

17. D.S. is having difficulty using his right arm. Given the type of surgery he underwent, is this expected and why?

18. D.S.’s pain and nausea are under control an hour later. The physical therapist arrives to perform passive exercises on D.S.’s right arm. The exercises are done to:

a. Lower the risk for developing lymphedema b. Reduce spasticity of the shoulder and chest muscles c. Assist the patient in learning to reuse the affected

arm d. Preserve muscle strength and reduce the effects of

disuse

Case study progress The next afternoon, the nurse giving you D.S.’s report says that he has been driving her crazy all morning. She tells you that he has been irritable and demanding. She says his assessment and VS are “okay” except for his Spo2, which has been around 85%. The morning CXR showed infiltrates in the left lung.

19. What is your immediate concern?

Case study progress You enter D.S.’s room to assess him and to tell him you are going to be his nurse again today. He is sitting on the side of the bed with his arms hunched up on the overbed table. You note that his head bobs up and his mouth opens, like a fish taking in water, every time he inhales. He says, “I just can’t [breath] seem to [breath] get enough [breath] air.”

20. Name 6 possible problems that D.S. could have that would account for his behavior.

21. What actions will you take next?

Case study progress D.S.’s respiratory rate is 46, with an Spo2 of 66%. You auscultate slight air movement over the large airways and no breath sounds distal to

the third intercostal space. The chest drainage system is intact. His gown is in his lap, he is diaphoretic, you note intercostal retractions with inspiration, and all muscles of the upper torso are engaged in respiration.

22. What will you do next? 23. The rapid response team stabilizes D.S., and you go with

him during transfer to the ICU. Why do you do this, and what information would you give to the ICU nurse?

Case study outcome After stabilizing D.S. in the ICU, the surgeon returns to your floor and is upset that he was not notified earlier about the change in D.S.’s condition. The next day you approach the nurse who gave you report, who is a new graduate, and explain how to better identify patients in early stages of respiratory difficulty. She thanks you for helping her learn from her mistake. D.S. experiences no further complications and completely recovers from the lobectomy after 2 months. He receives 6 months of external beam radiation therapy to the chest. His examination at 5 years shows no recurrence.

Case Study 39 Name _________________________________ Class/Group _____________________________ Date _____________

Scenario P.R., a 66-year-old woman who has no history of respiratory disease, is being admitted to your intensive care unit (ICU) from the emergency department (ED) with a diagnosis of pneumonia and acute respiratory failure (ARF). The ED nurse tells you that P.R. was stuporous and cyanotic on her arrival to the ED. Her initial vital signs

(VS) were 90/68, 134, 38, 101° F (38.3° C) an Spo2 of 53%. She was endotracheally intubated orally, was placed on mechanical ventilation, and has equal breath sounds. Her ventilator settings are synchronized intermittent mandatory ventilation of 12/min, tidal volume (VT) 700 mL, Fio2 0.50, and positive end-expiratory pressure (PEEP) 5 cm H2O. The nurse tells you P.R. had a chest x-ray (CXR) examination that confirmed the diagnosis of pneumonia before being intubated, but she needs an another CXR examination STAT.

1. Describe the pathophysiology of ARF. 2. How does the underlying pathophysiology relate to P.R.’s

presenting signs and symptoms? 3. Describe each of P.R.’s ventilator settings and the rationale for

each. 4. Why does P.R. need a second CXR examination?

Chart View

Arterial Blood Gases

pH 7.28

Paco2 62 mm Hg

HCO3 26 mEq/L

Pao2 48 mm Hg

Spo2 56%

5. The ABG results from the sample drawn in the ED before intubation are sent to you. Interpret P.R.’s ABG results.

Case study progress The ICU attending assesses P.R. and after reviewing the ED orders,

writes the following orders in addition to the mechanical ventilation protocol.

Chart View

Physician’s Orders

Blood and sputum cultures STAT ABGs via arterial line every 6 hours NGT to intermittent, low suction Insert indwelling urinary catheter Enoxaparin (Lovenox) 40 mg subcutaneous q24hr at 1700 Apply anti-embolism and intermittent pneumatic compression

stockings Pantoprazole (Protonix) IV 40 mg daily Lorazepam 1 mg IV every 4 hours Ceftriaxone 1gram IV q8hr D5 ½ NS at 125 mL/hr

6. What is the rationale for each order? 7. What is your priority nursing goal at this time? 8. Describe the interventions you will perform over the next 2

hours based on this priority and the orders you need to implement.

9. P.R. is not heavily sedated and seems anxious about all that is going on. Describe how you can help her.

Chart View

Arterial Blood Gases

pH 7.30

Paco2 52 mm Hg

22 mEq/L

HCO3

Pao2 70 mm Hg

Spo2 88%

10. ABGs are redrawn after P.R. has been on mechanical ventilation for 2 hours. What ventilator setting changes do you expect based on your interpretation? Select all that apply and explain your rationale.

a. Decreasing the Fio2 to 0.40 b. Increasing the VT to 850 mL c. Increasing the PEEP to 10 cm d. Increasing the rate on the ventilator to 16/min e. Changing to continuous mandatory ventilation

11. Evaluate each statement about caring for P.R. or a similar patient receiving mechanical ventilation with an endotracheal tube (ETT). Enter T for true or F for false. State why the false statements are incorrect.

_____1. Give muscle-paralyzing agents to keep P.R. from “fighting the vent.”

_____2. Check ventilator settings at the beginning of each shift and then hourly.

_____3. When suctioning the ETT, each pass should not exceed 15 seconds.

_____4. Assign experienced UAP to take vital signs every 2 to 4 hours.

_____5. Perform a respiratory assessment once per shift.

_____6. Empty excess water that collects in the ventilation tubing back into the humidifier.

_____7. Keep a resuscitation bag at the bedside. _____8. Monitor the cuff pressure of the ETT every 8

hours. _____9. Silence ventilator alarms when in the room to

maintain a quiet environment. _____10. Change all ventilator tubings every 12 hours.

12. What are 3 evidence-based practices you will need to

implement to prevent ventilator-assisted pneumonia? 13. You hear the high-pressure alarm sounding on the mechanical

ventilator and see that P.R.’s Sao2 is 80%. What are potential causes of this problem?

14. You decide that P.R. needs to be suctioned. Place in order

the steps for safely performing in-line or closed suctioning. _____1. Hyperoxygenate the patient. _____2. Use 5 to 10 mL of saline to rinse the catheter

clear of secretions. _____3. Insert catheter until you meet resistance or the

patient coughs. _____4. Assess patient’s status and document

procedure. _____5. Put on clean gloves and face shield and attach

suction. _____6. Apply suction as you withdraw the catheter,

not exceeding 10 seconds. _____7. Reassess patient status and suction again as

needed.

Case study progress As P.R.’s primary nurse, you are responsible for her nursing care plan. Although the primary concern is her respiratory status, you are concerned about hydration, nutrition, oral hygiene, and skin integrity and decide to address each of these areas in P.R.’s plan of care.

15. Discuss 5 indicators you can use to assess her fluid status. 16. Write a nutrition-related outcome for P.R. 17. Describe 4 interventions that could assist in meeting P.R.’s

nutrition goals. 18. You plan to assess P.R.’s skin every 4 hours. Name 4 other

strategies that will help meet the expected outcome of maintaining skin integrity?

Case study progress Over the next few days, P.R. progressively regains adequate respiratory functioning.

19. What factors would be considered in deciding when P.R. is ready to be weaned from mechanical ventilation?

20. What are your responsibilities during the weaning process? 21. Which assessment finding during the weaning process would

indicate P.R. should be placed back on the ventilator? a. Sao2 of 94% b. Heart rate 92 c. Temperature 99.3° F (37.4° C) d. Respiratory rate 34

Case study outcome P.R. is easily weaned from the ventilator, and her respiratory function continued to improve. Unfortunately, she developed a urinary tract infection, experienced acute confusion for a few days, and had difficulty ambulating. She stayed 7 more days after being weaned before being discharged home.

Case Study 40 Name _________________________________ Class/Group _____________________________ Date ____________

Scenario G.S., a 56-year-old woman, was involved in a motor vehicle accident; a car drifted left of the center line and struck her head-on, pinning her

behind the steering wheel. She was intubated immediately after extrication and flown to your trauma center. Her injuries were extensive: bilateral flail chest, right hemothorax and pneumothorax, fractured spleen, multiple small liver lacerations, open fractures of both legs, and a cardiac contusion. She was taken to the operating room for repair of her injuries. There she received 36 units of packed red blood cells, 20 units of platelets, 12 units of fresh frozen plasma, and 18 L of lactated Ringer’s solution. G.S. was admitted to the ICU postoperatively, where she developed acute respiratory distress syndrome (ARDS).

1. What is ARDS? 2. What are the risk factors for developing ARDS? Which does

G.S. have? 3. With her extensive injuries, how was ARDS diagnosed? 4. Describe the collaborative care patients generally receive in the

ICU for ARDS.

Case study progress After a 3-week stay in ICU, G.S. is being transferred to your unit. The ICU nurse gives you the following report: “She is awake, alert, and oriented to person and place. Both legs remain casted from hip to toe. She can wiggle her toes on both feet. Heart tones are clear, last vital signs were 138/90, 88, 26, 99.3° F (37.4° C); bilateral radial pulses 3 +. All the surgical incisions are healed. She has bilateral chest tubes to water suction with closed drainage, both dressings are dry and intact. She has a duodenal feeding tube, a Foley catheter to down drain, and a left double-lumen PICC line. Urine output is good; urine is clear yellow. Her morning labs are still pending.”

5. What additional information do you need from the ICU nurse? 6. Describe the phase of ARDS G.S. is in. 7. What are the long-term complications of ARDS you need to

monitor for in G.S.?

Case study progress You complete your assessment of G.S. You note she is dyspneic and has fine crackles throughout all lung fields posteriorly and in both lower lobes anteriorly, and coarse crackles over the large airways. She has O2 on at 2 L per nasal cannula and her Spo2 is 94%.

8. What is the significance of the fine and coarse crackles? 9. The nurse from the previous shift charted the following

statement: “Fine and coarse crackles that clear with vigorous coughing.” Based on your knowledge of pathophysiology, determine the accuracy of this statement.

10. It is time to administer scheduled furosemide (Lasix) 60 mg intravenous push. What effect, if any, should furosemide have on G.S.’s breath sounds?

11. What action do you need to take before giving the

furosemide?

Chart View

Laboratory Results

Sodium 129 mmol/L

Potassium 3.0 mmol/L

Chloride 92 mmol/L

HCO3 26 mEq/L

BUN 37 mg/dL

Creatinine 2 mg/dL

Glucose 128 mg/dL

Calcium 7.1 mg/dL

12. Which lab values concern you, and why? 13. Given G.S.’s lab values, what action do you need to take

and why?

Case study progress The provider wants you to administer the furosemide and prescribes the following.

Chart View

Physician’s Orders

STAT magnesium (Mg) level Potassium chloride (KCl) 40 mEq IVPB Calcium gluconate 2 g in 100 mL NS IVPB over 3 hours

14. Why did the provider order a magnesium level? 15. G.S. has 1 available port to use on the PICC line. Outline a plan

for giving the potassium chloride and calcium gluconate. 16. What interventions do you need to perform to safely

administer IV potassium chloride? Select all that apply. a. Administer the infusion using an IV pump. b. Place G.S. on continuous electrocardiogram (ECG)

monitoring. c. Assess the patency of the PICC line before initiating

the infusion. d. Administer the potassium infusion over a period of

at least 2 hours. e. Invert the potassium-containing IV bag several

times before and during the infusion. 17. You go to prepare G.S.’s furosemide dose and find only

one 20-mg vial in the medication-dispensing system. The floor

stock is empty. The pharmacist tells you that it will be at least an hour before he can send the drug to you. What are your options?

18. While you are giving the furosemide, G.S. says, “This is so weird. A couple times this morning, I felt like my heart flipped upside down in my chest, but now I feel like there’s a bird flopping around in there.” What are 2 actions you should take next?

19. G.S.’s pulse is 66 and irregular. Her blood pressure is 92/70 and respirations are 26. She admits to being “a little lightheaded” but denies having pain or nausea. Your co- worker connects G.S. to the code cart monitor for a “quick look.” Interpret what you see.

(From Aehlert B. ECGs Made Easy. 4th ed. St. Louis, MO: Mosby/JEMS; 2011.)

20. Why is G.S. likely experiencing this dysrhythmia? 21. What will your next actions be?

Chart View

Arterial Blood Gases on 6 L O2 by Nasal Cannula

pH 7.30

Paco2 59 mm Hg

Pao2 82 mm Hg

HCO3 36 mEq/L

Spo2 91%

22. You increase her O2 to 6 L, and the provider orders a STAT set of ABGs. How would you interpret G.S.’s ABGs?

23. What are your nursing priorities at this time? 24. Describe 5 interventions you will perform over the next few

hours based on these priorities. 25. You notice that G.S. looks frightened and is lying stiff as a

board. How would you respond to this situation?

Case study outcome G.S.’s pulmonary status does not improve after the furosemide, and she continues to have frequent ventricular dysrhythmias despite receiving the electrolytes. The provider transfers G.S. back to the ICU, where she is found to have a pulmonary embolus. Unfortunately, 1 week later she throws another embolus and all attempts at resuscitation fail.

3

Mobility

Case Study 41 Name _________________________________ Class/Group _____________________________ Date _____________

Scenario M.S., a 74-year-old woman, comes to your clinic for a complete physical examination. She has not been to a provider for 11 years because “I don’t like doctors.” Her only complaint today is “pain in my upper back.” She describes the pain as sharp and knifelike. The pain began approximately 3 weeks ago when she was getting out of bed in the morning and hasn’t changed at all. M.S. rates her pain as 6 on a 0- to 10-point pain scale and says the pain decreases to 3 or 4 after taking “a couple of ibuprofen.” She denies recent falls or trauma.

M.S. admits she needs to quit smoking and start exercising but states, “I don’t have the energy to exercise and, besides, I’ve always been thin.” She has smoked one to two packs of cigarettes per day since she was 17 years old. Her last blood work was 11 years ago, and she cannot remember the results. She went through menopause at the age of 47 and has never taken hormone replacement therapy. The physical examination findings are unremarkable other than moderate tenderness to deep palpation over the spinous process at T7. There are no masses or tenderness to the tissue surrounding the tender spot. No visible masses, skin changes, or erythema are noted. Her neurologic findings are intact, and no muscle wasting is noted.

1. An x-ray examination of the thoracic spine reveals a collapsed

vertebra at T7 and bone density changes in the spine. What could this result indicate?

2. The physician suspects osteoporosis. List 7 risk factors associated with osteoporosis.

3. Place a star or asterisk next to those risk factors specific to M.S.

Case study progress M.S. has never had an osteoporosis screening. She confides that her mother and grandmother were diagnosed with osteoporosis when they were in their early 50s.

4. What diagnostic test is most commonly used to diagnose osteoporosis?

5. M.S.’s diagnostic test revealed a bone density T-score of −2.7. How will this be interpreted?

6. M.S. receives a prescription for alendronate (Fosamax) 70 mg/wk. Which instructions are appropriate as you provide patient teaching to M.S. about this drug? Select all that apply.

a. “You can eat your breakfast along with this medication.”

b. “Take the medication with 8 ounces (236 mL) of water immediately on arising.”

c. “You can take this medication with your morning coffee or orange juice.”

d. “You need to sit or stand upright for at least 30 minutes after taking the medication.”

e. “If you experience any severe abdominal pain, vomiting, or jaw pain, notify your doctor immediately.”

7. M.S. asks whether she needs to take a calcium supplement. How do you answer her?

Case study progress M.S. asks you about foods that contain calcium. “I’d rather eat than take all these pills,” she states. You review food sources of calcium

with her.

8. Which foods are considered good sources of dietary calcium? Select all that apply.

a. Banana b. Chicken salad c. 8 ounces (226 grams) of yogurt d. 1 cup (236 mL) of cooked spinach e. Baked potato with margarine

Case study progress After reviewing her new prescription, you continue your teaching session about osteoporosis with M.S.

9. What nonpharmacologic interventions will you teach M.S. to prevent further bone loss?

10. M.S. begins to cry and says, “I cannot possibly stop smoking, change my diet, and exercise all at the same time.” You encourage M.S. to start working on one problem at a time. How should it be decided which problem M.S. should attempt first?

Case study progress One month later, M.S. comes in to the clinic and tells you that she stopped the Fosamax a week ago because it upset her stomach too much. The physician decides to start her on denosumab (Prolia), but asks M.S. to see her dentist first.

11. M.S. asks you why she needs to see a dentist before starting the new drug. Which problem may occur during therapy with denosumab?

a. Gingivitis b. Tooth loss c. Tooth decay (cavities) d. Osteonecrosis of the jaw

12. How is denosumab given?

Case study outcome M.S. receives her first injection of denosumab 1 month later and has no significant adverse effects. At her 3-month check-up, she tells you that she finally stopped smoking and was excited to note a 5-pound (2.25 kg) weight loss.

Case Study 42 Name _________________________________ Class/Group _____________________________ Date _____________

Scenario J.C. is a 41-year-old man who comes to the emergency department with acute low back pain. He states that he did some heavy lifting yesterday, went to bed with a mild backache, and awoke this morning with terrible back pain, which he rates as a 10 on a scale of 1 to 10. He admits to having had a similar episode of back pain years ago “after I lifted something heavy at work.” J.C. has a medical history of peptic ulcer disease related to nonsteroidal antiinflammatory drug (NSAID) use. He is 6 feet tall (183 cm), weighs 265 pounds (120 kg), and has a prominent “potbelly.”

1. What questions would be appropriate to ask J.C. in evaluating the extent of his back pain and injury?

2. What observable characteristic does J.C. have that makes him highly susceptible to low back injury?

3. J.C. took piroxicam (Feldene) 20 mg until he developed his duodenal ulcer. What is the relationship between the two? What signs and symptoms would you expect if an ulcer developed?

Case study progress

All serious medical conditions are ruled out, and J.C. is diagnosed with lumbar strain. The nurse practitioner (NP) orders a physical therapy consultation to develop a home stretching and back- strengthening exercise program and a dietary consultation for weight reduction. The NP gives J.C. prescriptions for cyclobenzaprine (Flexeril) 10 mg tid for 3 days only, and celecoxib (Celebrex) 100 mg/day for 3 months. He receives the following instructions: heat applications to the lower back for 20 to 30 minutes four times a day (using moist heat from heat packs or hot towels), no twisting or unnecessary bending, and no lifting more than 10 pounds (4.5 kg). J.C. is instructed to rest his back for 1 or 2 days, getting up only now and then to move around to relieve muscle spasms in his back and strengthen his back muscles. He is given a written excuse to stay off work for 7 days and, when he returns to work, specifying the limitation of lifting no more than 10 pounds (4.5 kg) for 3 months. He is instructed to contact his primary care provider if the pain gets worse.

4. J.C. looks at the prescription for cyclobenzaprine and states, “I’m glad you didn’t give me that Valium. They gave me Valium last time and that stuff knocked me out.” How would you respond to J.C.?

5. Why do you think he was prescribed cyclobenzaprine instead of diazepam (Valium)?

6. J.C. states, “Well, I’m glad I’ll still be able to take my sleeping pill.” True or False? Explain.

Case study progress J.C. asks, “What is Celebrex? I hope it won’t do what that Feldene did to me years ago.”

7. Why do you think it was prescribed for J.C., considering his gastrointestinal (GI) history?

8. It has been over 5 years since his last episode of GI bleeding. Are there any other conditions you need to assess for before

J.C. begins to take the celecoxib? Explain. 9. Why would the NP prescribe an NSAID rather than

acetaminophen for J.C.’s pain? 10. A physical therapist teaches J.C. maintenance exercises he can

do on his own to promote back health. Describe 3 common exercises that would be included.

11. In addition to learning exercises, J.C. needs to learn

proper body mechanics for lifting and performing day-to-day activities. Which statement by J.C. indicates a need for further instruction?

a. “I will bend my knees when I lean forward.” b. “When lifting heavy boxes, I will bend at the

waist.” c. “I will not lift anything above the level of my

elbows.” d. “I will avoid standing in one position for a long

period of time.” 12. J.C. tells you that when his back hurts, he has trouble getting

comfortable when he goes to bed at night. What sleeping position would help him?

a. Prone position b. Supine with legs out straight. c. Side-lying with knees and hips flexed. d. Position of the body during sleep does not affect

back pain.

Case study outcome J.C. takes a week off from work to rest and uses warm compresses twice a day on his back. He returns to “light duty” at work for a period of 3 months.

Case Study 43 Name _________________________________ Class/Group _____________________________ Date _____________

Scenario D.M., a 25-year-old man, hops into the emergency department (ED) with complaints of right ankle pain. He states that he was playing basketball and stepped on another player’s foot, inverting his ankle. You note swelling over the lateral malleolus down to the area of the fourth and fifth metatarsals, and pedal pulses are 3 + bilaterally. His vital signs (VS) are 124/76, 82, 18. He has no allergies and takes no medication. He states he has had no prior surgeries or medical problems.

1. In assessing D.M.’s injured ankle, what should you evaluate? 2. What will initial management of the ankle involve to prevent

further swelling and injury? 3. You note significant swelling over the fourth and fifth

metatarsals. How would you further evaluate this finding? 4. Which assessment finding would indicate that D.M. has a

fracture instead of a sprain? Select all that apply. a. Bruising b. Slight swelling c. Inability to bear weight d. Increased pain with movement e. Deformity or abnormal position of the ankle

Case study progress X-ray results are negative for fracture, and a second-degree sprain is diagnosed. The physician orders immobilization with an elastic bandage and an air stirrup brace, with instructions for crutches. The physician tells D.M. not to bear weight on his ankle for 2 days, then to use only partial weight bearing until the ankle heals.

5. Describe the technique for applying an elastic wrap. Give the rationale.

6. An hour after you apply the elastic wrap, D.M. states that his “toes are tingling” and his foot feels numb. What is the best

action at this time? 7. When instructing D.M. to use crutches, D.M. states that he

“likes it better” when the crutches rest under his arms while he walks with the crutches. Is this correct? Explain.

8. You instruct D.M. on using the three-point gait with the crutches. Which would be the correct first step for the three- point gait? Explain your answer.

a. Step first with the affected leg. b. Step first with the unaffected leg. c. Step first with both crutches and the affected leg. d. Step first with the affected leg and the crutch

opposite of the affected leg. 9. You are to instruct D.M. on application of cold, activity, and

care of the ankle. What would be appropriate instructions in these areas?

10. D.M. is given a prescription for ibuprofen tablets, 400 mg, every 6 hours as needed for pain. Explain the purpose of this drug.

11. You provide instructions about the use of ibuprofen. Using the Teach-Back method, you ask D.M. to teach you what he has learned. Which statement by D.M. indicates a need for more teaching?

a. “I will take these pain pills with food or a snack.” b. “I will not take aspirin while taking this medicine.” c. “It would not hurt if I have a few beers while taking

this.” d. “I will take no more than 2 pills every 6 hours if

needed for pain.”

Case study progress Six days later, D.M. hobbles into the ED and boldly informs you that he “did it again, only this time it was touch football.” He states that the pain pills worked so well, he thought it would be okay. You detect the odor of beer on his breath.

12. What are you going to do? 13. D.M.’s blood alcohol concentration (BAC) result is 0.06 mg%.

Interpret this result. Does this level reflect legal intoxication? 14. Are there any concerns about his alcohol consumption and his

prescription medicine? Explain. 15. You remove his sock and find a large hematoma forming on

the lateral aspect of an already swollen ankle. The ankle shows the color of a bruise that is several days old. You inquire about D.M.’s pain perception. He states, “It doesn’t feel too bad now, as long as I don’t walk on it, but I sure saw stars when it popped.” What is the significance of his statement?

Case study outcome D.M.’s ankle is fractured. His ankle is splinted for now with plans for a cast in a few days, after the initial swelling subsides. A referral to an orthopedic surgeon is obtained. You discuss his use of the pain medication and his drinking. He tells you that he did realize it could be harmful. He is now upset that it happened because he will not be playing any sports for some time.

Case Study 44 Name _________________________________ Class/Group _____________________________ Date _____________

Scenario W.S., a 75-year-old man, was just admitted to an orthopedic surgery unit after undergoing right knee arthroplasty surgery. He is groggy but awake and states he is not in pain at this time. His right knee has a surgical dressing that is dry and intact. He has knee-high compression stockings on and is attached to sequential compression devices.

1. What is arthroplasty surgery of the knee? 2. What factors contribute to the decision to have knee

arthroplasty surgery? 3. What immediate assessments will you perform?

Chart View

Post-Op Orders

Ice pack to operative site for 30 minutes every 4 hr Cefazolin (Ancef) 1 gram IV q8hr × 2 doses Acetaminophen (OFIRMEV) 1000 mg IV q6hr × 2, then

acetaminophen 1000 mg PO q6hr Ketorolac (Toradol) 15 mg IV q8hr × 3 doses, then start

celecoxib (Celebrex) 100 mg PO q12hr Morphine 4 mg IV q4hr prn for moderate to severe pain Enoxaparin (Lovenox) 30 mg q12hr SubQ for 10 days Advance diet as tolerated Incentive spirometer Knee-high compression stockings SCDs Begin PT/OT Case Management referral

4. Which interventions are part of pain management? Explain the purpose for each one.

5. What measures are ordered to prevent deep vein thrombosis (DVT)? Explain the function of each one.

6. Describe how you will assess for a DVT. 7. What are other possible postoperative complications of knee

arthroplasty? 8. The UAP is assisting W.S. with positioning in the bed. Which

position would you intervene to correct? a. The knee is hyperextended. b. The knee is in a neutral position. c. The knee is not rotated internally or externally.

d. One pillow is placed under the lower calf and foot to encourage slight knee joint extension.

Case Study Progress W.S. tells you that his friend had knee surgery that was “ruined” by an infection, and he is worried about that happening to him.

9. What signs and symptoms would alert the nurse to a possible infection?

10. What is the most important intervention that can be done

to prevent infection? 11. W.S. comments that the compression stockings are so difficult

to put on, and asks you if they can be left on all the time. What would you teach W.S. about compression stockings? Select all that apply.

a. “These stockings can be worn 24/7.” b. “The stockings should be hand washed and air

dried daily.” c. “They should be applied before you get out of bed

in the morning.” d. “These stockings are used to strengthen the

muscles in your lower legs.” e. “We need to take them off daily to check the

condition of the skin on your legs and feet.” 12. Correct the incorrect statements in the previous question.

Case study progress The next morning, the physical therapist works with W.S. on getting out of bed and walking with a walker. W.S. tolerates the session well but is glad to get back into the bed after all the activity.

13. After his first PT session, you notice a small amount of drainage on W.S.’s knee dressing. Which is the best action at this time?

a. Remove the dressing b. Do nothing and continue to monitor c. Replace the dressing with a fresh dressing. d. Draw a circle around the drainage area, and

date/time it with your initials

Case study progress The next morning during your assessment, W.S. tells you that he wants to improve his walking, but he is afraid that the PT will “hurt too much.”

14. When is the best time for W.S. to receive his pain medication? Explain your answer.

15. What types of exercises can W.S. perform in between PT sessions?

16. W.S. tells the physical therapist that he cannot wait to take long walks again when he gets home. Which answer by the physical therapist is correct?

a. “Yes, that will be great exercise for you.” b. “That should be fine after a month of therapy.” c. “You will not be able to take long walks again now

that you’ve had this surgery.” d. “It takes time to reach your maximum strength and

endurance, from 6 months to a year.”

Case study outcome W.S. refused to go to a rehab facility and was discharged to his home with home health visits from a physical therapist and a nurse. He performed all the prescribed exercises, and his wife and daughter helped with his care. Within 3 months he was driving again and taking short walks in his neighborhood.

Case Study 45 Name _________________________________ Class/Group

_____________________________ Date _____________

Scenario H.K. is a 26-year-old man who tried to light a cigarette while driving and lost control of his truck. The truck flipped and landed on the passenger side. H.K. was transported to the emergency department with a deformed, edematous right lower leg and a deep laceration wound approximately 2 inches (5 cm) long over the deformity. Blood continues to ooze from the wound.

1. What further assessment will you make of the leg injury and what precautions will you take in making this assessment?

2. What is the most appropriate method for controlling bleeding at this wound site?

3. What is the best way to immobilize the leg injury before surgery?

4. From the information given, it is clear that H.K. is a smoker. List at least 3 issues related to his smoking that can complicate his care and recovery. What interventions could be instituted to counter these complications?

Case study progress H.K. is taken to surgery for open reduction and internal fixation (ORIF) of the tibia and fibula fractures. He returns with a full-leg fiberglass cast with windows over the areas of surgery.

5. Describe the assessment of a patient with a long leg cast involving trauma and surgery.

6. In assessing H.K.’s cast on the third day postoperatively, you notice a strong foul odor. Drainage on the cast is extending, and H.K. is complaining of pain more often and seems considerably more uncomfortable. Vital signs are 123/78, 102, 18, 102.2° F (39° C). What is your analysis of these findings, and what action is needed?

Case study progress H.K. returns to surgery. The wound over H.K.’s fracture site has become necrotic with purulent drainage. The wound is debrided and cultured and a posterior splint is applied. H.K. returns to his room with orders for wet-to-moist dressing changes. The physician suspects osteomyelitis and orders nafcillin (Unipen) and ciprofloxacin (Cipro). Contact precautions are implemented.

7. Why are 2 antibiotics ordered? 8. H.K. asks you about the isolation precautions. “Does this mean

I have something bad?” What is your best answer? a. “These are precautions that we use for every patient

who has surgery.” b. “These are precautions we are taking to help your

infection get better.” c. “This is an extremely serious infection; these

precautions will keep the infection from getting worse.”

d. “These precautions prevent the spread of the infection to other patients and to health care personnel.”

9. As you continue to assess H.K. over the following days, what evidence will you look for that antibiotics are effectively treating the infection?

10. Develop a teaching plan concerning the care of his cast. 11. After the teaching session, you use the Teach-Back method to

assess H.K.’s learning. You ask H.K.: “We have reviewed a lot of information on how to care for your cast. To make sure that I explained it well to you, please tell me three things you will do.” Which statements by H.K. indicates a good understanding of cast care? Select all that apply.

a. “I will not get the cast wet.” b. “I will keep plastic over the cast to protect it.” c. “I will call my doctor if I notice a bad odor or see

drainage.”

d. “I will keep my leg elevated above the level of my heart for the first 2 days.”

e. “I will use a long file if it gets itchy inside my cast to scratch my skin gently.”

12. What nutritional needs does H.K. have and why?

Case study progress To ensure pain management, H.K. is given hydromorphone (Dilaudid) 2 mg IV push every 4 hours prn pain. You prepare to give him the first dose.

13. Name this drug’s therapeutic category. 14. What signs and symptoms would you see if he were to have a

toxic or overdose reaction to the Dilaudid? Select all that apply.

a. Nausea b. Pruritus c. Dilated pupils d. Respiratory depression e. Central nervous system (CNS) depression

15. What is the first thing you will need to do if you note a toxic or overdose reaction to the Dilaudid?

16. What is the antidote for toxic opioid reactions? How is it administered?

Case study progress H.K. has no further complications with his leg wound and responds well to physical therapy. The discharge planner meets with him to discuss his posthospital care.

17. What issues would the discharge planner need to address with H.K.?

Case study outcome H.K. was discharged to his apartment, where he could stay with

financial help from his parents. He continued on oral antibiotics for another month. Friends drove him to physical therapy on their way to classes at the university and took him back on their way home. He managed well and went back to work while still in his cast.

Case Study 46 Name _________________________________ Class/Group _____________________________ Date _____________

Scenario M.M., a 76-year-old retired schoolteacher, is postoperative day 2 after an open reduction and internal fixation (ORIF) for a fracture of his right femur. He has been on bed rest since surgery. At 0800, his vital signs (VS) are 132/84, 80 with regular rhythm, 18 unlabored, and 99° F (37.2° C), and Spo2 97% on room air. He is awake, alert, and oriented with no adventitious heart sounds. Breath sounds are clear but diminished in the bases bilaterally. Bowel sounds are present, and he is taking sips of clear liquids. He is receiving an intravenous (IV) infusion of D5 ½ NS at 75 mL/hr in his left hand, and orders are to change it to a saline lock this morning if he is able to maintain adequate oral fluid intake. He has orders for oxygen (O2) to maintain Spo2 over 92%, but he has been refusing to wear the nasal cannula. Pain is controlled with morphine sulfate 4 mg IV as needed every 4 hours, and he has ondansetron (Zofran) 4 mg IV every 4 hours as needed for nausea. He is receiving enoxaparin (Lovenox) 30 mg subcutaneously once daily, and taking docusate sodium (Colace) PO once daily.

At 1830 you answer M.M.’s call light and find him lying in bed breathing rapidly and rubbing the right side of his chest. He is complaining of right-sided chest pain and appears to be restless.

1. What will you do?

Case study progress You check his VS, with these results: BP 98/60; P 120; R 24. You note he is restless and slightly confused. The pulse oximeter reads 86%, so you start him on 6 L O2 by nasal cannula (NC). You identify faint crackles in the posterior bases bilaterally. The heart monitor on lead II shows nonspecific T wave changes.

2. Using situation, background, assessment, recommendation (SBAR), what information, based on the findings, would you provide to the physician when you call?

3. The physician asks you to transfer M.M. to the ICU and orders blood coagulation studies, arterial blood gases (ABGs) on room air, continuous pulse oximetry, STAT chest x-ray (CXR), and STAT 12-lead ECG. What information will the physician gain from each of these?

4. Why would the physician order ABGs on room air as opposed to with supplemental O2?

Case study progress You evaluate the room air ABG results.

Chart View

Arterial Blood Gases

pH 7.55

Paco2 24 mm Hg

HCO3 24 mEq/L (24 mmol/L)

Pao2 56 mm Hg

Sao2 86% (room air)

5. What is your interpretation of the ABGs, and what do you think the physician will order next, and why?

Case study progress The CXR image shows a small right infiltrate. The physician suspects an embolism and orders a STAT spiral CT scan of the lungs. The interpretation of the results reads “strongly suggestive of a pulmonary embolus (PE).”

6. What are the most likely sources of the embolus? 7. For each characteristic in the following list, note whether it is a

characteristic of a fat embolus (F), a blood clot embolus (BC) in the lungs, or both (B).

a. Altered mental status b. Decreased Sao2 c. Petechiae d. Chest pain e. Crackles f. Increased respirations and pulse

Case study progress The physician orders heparin using the Heparin Protocol listed below. M.M. weighs 76 kg.

Chart View

Heparin Protocol

Initial bolus: Give 80 units/kg. Initial infusion: 18 units/kg/hr.

Every 6 hours, check aPTT. Follow titration instructions:

a. PTT less than 55 seconds Give bolus of 80 units/kg and increase infusion rate by

4 units/kg/hr b. 55 to 69 seconds

Give 40 units/kg bolus and increase infusion rate by 2 units/kg/hr

c. 70 to 105 seconds NO change in rate

d. 106 to 140 seconds Decrease infusion rate by 2 units/kg/hr from previous

rate e. Greater than 140 seconds

i. Hold heparin infusion for 1 hour, then reduce infusion by 3 units/kg/hr

ii. Notify MD

8. How many units will M.M. receive for this bolus?

9. What is the rate (milliliters per hour) for the initial

infusion (round to the whole number)? The heparin solution is 25,000 units in 500 mL 0.45% NS.

10. After 6 hours, the lab calls you with a critical PTT result

of 155 seconds. Per the protocol, what action will you take? 11. The physician is considering administering an antidote to the

heparin. Which generic drug is considered an antidote to heparin therapy?

a. Atropine b. Vitamin K c. Protamine sulfate d. Potassium chloride

Case study progress The physician decides not to administer an antidote and orders the heparin to be held, with a second aPTT drawn in 2 hours. M.M. is monitored closely. After 2 hours, the aPTT is 95 seconds.

12. What is your evaluation of this aPTT level? 13. The physician resumes the heparin at a lower rate, and there

are no further issues with the lab results. The next day, the physician’s orders read, “Warfarin (Coumadin) 2.5 mg PO, PT/INR in a.m.; D/C heparin.” Do you have any concerns about these orders? Explain your answer.

14. List 3 priority problems related to the care of M.M. in his current situation.

15. Several days later you hear M.M. asking his son to bring in a “decent razor” because he is tired of having beard stubble. How would you address this issue?

16. Before M.M. goes home, the physician switches the oral anticoagulant to rivaroxaban (Xarelto), after the heparin drip is discontinued. You inform him that he will be on this medication for several months to prevent another occurrence of VTE. Which statements will you include in the teaching? Select all that apply.

a. “Take the Xarelto at the same time every day.” b. “There are no dietary restrictions while on this

drug.” c. “Watch for bleeding from your gums, nose, and

bowels.” d. “You will need to have blood work done on a

regular basis while on this drug.” e. “It’s a good idea to wear a medical alert bracelet or

necklace while on this drug.”

Case study progress M.M.’s condition has improved, and he has had several physical therapy sessions. He is not permitted to bear weight on his affected leg, but the physical therapist has encouraged him to exercise.

17. What types of exercises are recommended after this type of surgery?

Case study outcome M.M. is discharged to a rehabilitation facility to help him to regain strength after his surgery and illness. He is thankful that the surgery is over and hopeful to get back to his retirement activities soon.

Case Study 47 Name _________________________________ Class/Group _____________________________ Date _____________

Scenario J.F., a 67-year-old woman, was involved in an auto accident and flown by emergency helicopter to your facility. She sustained a ruptured spleen, fractured pelvis, and compound fractures of the left femur. On admission (5 days ago) she underwent a splenectomy. Her pelvis was stabilized with an external fixation device 3 days ago, and yesterday her left femur was stabilized using balanced suspension with skeletal traction. She has a Thomas splint with a Pearson attachment on her left leg. She has 20 pounds (9 kg) of skeletal traction and 5 pounds (2.25 kg) applied to the balanced suspension. Her left femur is elevated off the bed at approximately 45 degrees. The lower leg is parallel to the bed and lies in a sling that the nurse adjusts on the frame, and the foot hangs freely. This morning, J.F. was transferred to your orthopedic unit for specialized care. You are the nurse assigned to care for her on the night shift.

1. You enter J.F.’s room for the first time. What aspects of the traction will you inspect?

2. What are characteristics of skeletal traction? Select all that apply.

a. Traction weights are usually 5 to 10 pounds (2.25 to 4.5 kg).

b. Traction weights range from 5 to 45 pounds (2.25 to 20 kg).

c. Used for short-term treatment, 48 to 72 hours. d. The primary goal is to align injured bones and

joints. e. Tape, boots, or splints are attached directly to the

skin. f. The primary goal is to reduce muscle spasms in the

injured extremity. g. Traction is applied using pins or wires that are

surgically inserted into the bone. 3. When inspecting the skeletal pin sites, you note that the skin is

reddened for an inch (2.5 cm) around the pin on both the medial and lateral left leg. What does this finding indicate, and what action will you take?

Case study progress You perform a neurovascular assessment and note the following findings: Left foot pale, temperature slightly cooler than right foot. Right foot color pink. Capillary refill less than 3 seconds on both sides. Edema + 1 on left foot and lower leg; no edema on right leg. Dorsalis pedis palpable and equal on both feet. Sensation equal on both sides. Able to dorsiflex feet and rotate ankles freely. Rates pain in left femur as a 5 out of 10.

4. Your institution uses electronic charting. Based on the assessment just described, which of the following systems would you mark as abnormal as you document your neurovascular assessment? Mark abnormal findings with an “X” and provide a brief narrative note.

☐ Skin color: ☐ Skin temperature: ☐ Capillary refill: ☐ Edema: ☐ Peripheral pulses: ☐ Sensation: ☐ Motor function:

☐ Pain: 5. What other key points of the assessment will you document in

the patient’s record? 6. An hour later, you check on J.F. and find her body in the lower

75% of the bed and her left upper leg at an exaggerated angle (more than 45 degrees). The knot at the end of the bed is caught in the pulley, and the 20-pound (9 kg) weight is dangling just above the floor. What are you going to do?

7. When you lift J.F., you notice that her sheets are wet. You decide to change J.F.’s linen. How would you accomplish this task?

8. J.F. tells you that she feels like she needs to have a bowel movement, but it is too painful to sit on the bedpan. How would you respond?

9. J.F. expels a few small, hard, round pieces of stool. What could be done to promote normal elimination?

Case study progress You ask J.F. whether she is ready for her bath, and she responds positively. You let her bathe the parts she can reach and engage her in a conversation as you attend to the rest of her body. While performing perineal care, you notice that the folds of skin around her perineal area are reddened and excoriated.

10. What is the likely cause of the problem and what needs to be done to encourage healing?

11. You ask J.F. what she is doing to exercise while she is confined to the bed. She looks surprised and states that she isn’t doing anything. What activities can J.F. engage in while on bed rest?

12. You realize that maintaining skin integrity is a challenge in J.F.’s case. What measures will you take to prevent skin breakdown?

Case study progress Although J.F. is recovering nicely, she is becoming increasingly

withdrawn. You enter her room and find her crying. She tells you that she is all alone here, that she misses her family terribly. You know that her son is flying into town tomorrow but will only be able to stay a few days.

13. What can be done so that J.F. benefits from her family support system?

Case Study Outcome J.F. remains in the hospital for several weeks, then is discharged to a rehabilitation center. She talks to her grandchildren every other night via social media on her new electronic device. She is looking forward to an upcoming visit with them.

Case Study 48 Name _________________________________ Class/Group _____________________________ Date _____________

Scenario K.B. is a 16-year-old girl who fell while skiing. She briefly lost consciousness but is now alert and oriented. She was transported down the hill by the ski patrol after being stabilized and then was flown to the hospital. She has a fractured right femur and humerus. She will be admitted to your unit after an open reduction and internal fixation (ORIF) of the femur fracture and casting of her leg and arm.

1. You are taking the report from the postanesthesia care unit (PACU) nurse. K.B. is awake and taking ice chips. What information will you document on the admission form?

2. How will you use this information in planning your immediate assessment and care of K.B. on admission?

Chart View

Physician’s Orders

Vital signs per routine for postop patient Continuous pulse oximetry Neurologic checks every 4 hours Turn, cough, and deep breathe and incentive spirometer every

2 hours while awake Heat pack and elevate right lower extremity and right upper

extremity Neurovascular checks every 1 hour NPO IV fluids NS at 100 mL/hr Morphine sulfate 5 mg IV every 4 to 6 hours prn

3. For each order listed in the chart, state whether the order is appropriate or not and state why.

Case study progress K.B. is settled into her room and begins to complain of pain (7/10) in her leg and arm. She weighs 65 kg. You note that the ordered dose of morphine sulfate was given 2 hours ago. Your drug reference states that the appropriate dose is 0.05 to 0.1 mg/kg every 2 to 4 hours.

4. Is this dose safe and therapeutic for your patient?

5. The morphine for injection comes in a concentration of

2 mg/mL. How much will you draw up and have a second RN double-check? Indicate the amount on the syringe.

Case study progress After K.B. has been on the unit for 6 hours, you identify the following changes in her assessment data: K.B. is difficult to arouse, but when awake she is able to identify who and where she is; PERRLA 1 + with slower reaction time than earlier; color pale, pink; skin cool and clammy; heart rate 126, respiratory rate 28, temperature (oral) 37.2 ° C (99° F); Spo2 90%. The findings of the neurovascular checks of the affected extremities are unchanged.

6. What will your immediate nursing interventions include? 7. K.B.’s Glasgow Coma Scale score begins to decline from 15 to

11. What are possible reasons for changes in her neurologic status?

8. What would you document about this incident?

Case study progress K.B. is transferred to the pediatric intensive care unit (PICU) and treated for changes in her neurologic status. The next day her surgeon determines that her condition is stable and has her transferred back out to the pediatric unit. It is now 36 hours after surgery. K.B. suddenly begins to complain of extreme pain in her lower right leg. She had pain medication 2 hours ago and rates her pain as 10/10.

9. Which of these findings are early signs of compartment syndrome? Select all that apply.

a. Edema b. Paresthesia c. Macular rash

d. Increased pain e. Diminished pedal pulse f. Capillary refill less than 2 seconds

10. You page the orthopedic surgeon. Use SBAR (situation, background, assessment, recommendation) to address patient status.

11. K.B.’s cast is split, and her foot pulses are restored. K.B. and her parents are extremely anxious. What education and support will be provided to K.B. and her parents?

Case study progress K.B.’s status continues to improve. Physical and occupational therapists work with her on transfers and performing activities of daily living. She has many questions about how she will be able to go to school and resume her normal routine.

12. Recognizing K.B.’s developmental and cognitive stage, which of the following statements best supports your approach to discharge teaching?

a. Adolescents are capable of thinking in concrete terms only.

b. Family acceptance is more important than peer acceptance.

c. Adolescents can anticipate future implications of current decisions.

d. Adolescents are preoccupied with the immediate situation rather than future events.

13. A new cast was applied several days later. Safety is an

important issue for K.B. Describe what issues need to be addressed in discharge teaching in reference to safety?

14. The multidisciplinary team is made aware of K.B.’s progress in discharge rounds. Discuss how the following disciplines will be incorporated into her follow-up care after discharge: Discharge planning, PT, OT, dietitian, and hospital educator.

Case study outcome K.B. is discharged to home on postoperative day 5 with homebound schooling ordered, PT and OT ordered, and follow-up with orthopedics in 2 weeks.

Case Study 49 Name _________________________________ Class/Group _____________________________ Date _____________

Scenario You are working in the emergency department when M.C., an 82- year-old widow, arrives by ambulance. Because M.C. had not answered her phone since noon yesterday, her daughter went to her home to check on her. She found M.C. lying on the kitchen floor, incontinent of urine and stool, and stating she had pain in her right hip. Her daughter reports a medical history of hypertension, angina, and osteoporosis. M.C. takes propranolol (Inderal), denosumab (Prolia), and hydrochlorothiazide, and uses a nitroglycerin patch. M.C.’s daughter reports that her mother is normally very alert and lives independently. On examination, you see an elderly woman, approximately 100 pounds (45 kg), holding her right thigh. You note shortening of the right leg with external rotation and a large amount of swelling at the proximal thigh and right hip. M.C. is oriented to person only and is confused about place and time, but she is able to say that her “leg hurts so bad.” M.C.’s vital signs (VS) are 90/65, 120, 24, 97.5° F (36.4° C), Spo2 89%. She is profoundly dehydrated. Preliminary diagnosis is a fracture of the right hip.

1. Considering her medical history and that she has been without her medications for at least 24 hours, explain her current VS.

2. Based on her history and your initial assessment, what 3 priority interventions would you expect to be initiated?

3. M.C.’s daughter states, “Mother is always so clear and alert. I

have never seen her act so confused. What’s wrong with her?” What are 3 possible causes for M.C.’s disorientation that should be considered and evaluated?

Case study progress X-ray films confirm the diagnosis of intertrochanteric femoral fracture. Knowing that M.C. is going to be admitted, you draw admission labs and call for the orthopedic consultation.

4. What lab and diagnostic studies will be ordered to evaluate M.C.’s condition, and what critical information will each give you?

5. You insert an indwelling catheter and take careful note of the amount and appearance of M.C.’s urine. Why?

6. What are the 5 Ps that should guide the assessment of M.C.’s right leg before and after surgery?

7. In evaluating M.C.’s pulses, you find her posterior tibial pulse and dorsalis pedis pulse to be weaker on her right foot than on her left. What could be a possible cause of this finding?

8. You carefully monitor M.C.’s right extremity for compartment syndrome. Which are characteristics of compartment syndrome? Select all that apply.

a. Pallor of the extremity b. Warmth of the extremity c. Petechiae over the extremity d. Numbness and tingling of the extremity e. Pain on passive stretch of the muscle traveling

through the compartment 9. In planning further care for M.C., list 4 potential complications

for which M.C. should be monitored and the reason for each. 10. M.C. keeps asking about “peaches.” No one seems to be

paying attention. You ask her what she means. She says Peaches is her little dog, and she’s worried about who is taking care of it. How will you answer?

Case study progress M.C. is placed in Buck’s traction and sent to the orthopedic unit until an open reduction and internal fixation (ORIF) can be scheduled. Oxycodone-acetaminophen (Percocet 2.5/325) q4hr prn is ordered for severe pain and acetaminophen (Tylenol) 650 mg q4hr prn, for mild or moderate pain. She is placed on enoxaparin (Lovenox) 30 mg subQ bid. M.C.’s cardiovascular, pulmonary, and renal status is closely monitored.

11. As you assess the traction, you check the setup and M.C.’s comfort. Which are characteristics of Buck’s traction? Select all that apply. Explain your answers.

a. Weights need to be freely hanging at all times. b. The weights can be lifted manually as needed for

comfort. c. Weights used for Buck’s traction are limited to 5 to

10 pounds (2.25 to 4.5 kg). d. Pin site care is an essential part of nursing

management for Buck’s traction. e. A Velcro boot is used to immobilize the affected leg

and connect to the weights. 12. What assessment is the priority when a patient is in Buck’s

traction? 13. Percocet can cause constipation. What will you do to prevent

constipation? 14. Between her admission at 1500 and the next day, she

receives four doses of Percocet and one dose of acetaminophen (Tylenol). At 1300, she develops a fever of 101° F (38.3° C), and the physician writes an order to give acetaminophen (Tylenol), 650 mg PO every 4 hours for temperature over 100.5° F (38.1° C). Is there a concern with this order?

Case study outcome After an uneventful postoperative course, M.C. is transferred to a

skilled-care facility for physical and occupational therapy rehabilitation.

Case Study 50 Name _________________________________ Class/Group _____________________________ Date _____________

Scenario E.B., a 69-year-old man with type 1 diabetes mellitus (DM), is admitted to a large regional medical center complaining of severe pain in his right foot and lower leg. The right foot and lower leg are cool and without pulses (absent by Doppler). An arteriogram demonstrates severe atherosclerosis of the right popliteal artery with complete obstruction of blood flow. Despite attempts at percutaneous catheter- directed thrombolytic therapy with alteplase (tissue plasminogen activator [tPA]) over 48 hours and surgical thrombectomy, the foot and lower leg become necrotic. Finally, the decision is made to perform an above-the-knee amputation (AKA) on E.B.’s right leg. E.B. is recently widowed and has a son and daughter who live nearby. In preparation for E.B.’s surgery, the surgeons wish to spare as much viable tissue as possible. Hence, an order is written for E.B. to undergo 5 days of hyperbaric therapy for 20 minutes bid.

1. What is the purpose of hyperbaric therapy?

Case study progress As you prepare E.B. for surgery, he is quiet and withdrawn. He follows instructions quietly and slowly without asking questions. His son and daughter are at his bedside, and they also are very quiet. Finally, E.B. tells his family, “I don’t want to go like your mother did. She lingered on and had so much pain. I don’t want them to bring me back.”

2. You look at his chart and find no advance directives. What is your responsibility?

3. What is your assessment of E.B.’s behavior at this time? 4. What are some appropriate interventions and responses to

E.B.’s anticipatory grief?

Case study progress E.B. returns from surgery with the right residual limb dressed with gauze and an elastic wrap. The dressing is dry and intact, without drainage. He is drowsy and has the following vital signs: 142/80, 96, 14, 97.9° F (36.6° C), Spo2 92%. He is receiving a maintenance IV infusion of D5 NS at 125 mL/hr in his right forearm.

5. The surgeon has written to keep E.B.’s residual limb elevated on pillows for 48 hours; after that, have him lie in a prone position for 30 minutes, 3 to 4 times a day. In teaching E.B. about his care, how will you explain the rationale for these orders?

6. In reviewing E.B.’s medical history, what factors do you notice that might affect the condition of his residual limb and ultimate rehabilitation potential?

Case study progress You have just returned from a 2-day workshop on guidelines for the care of surgical patients with type 1 DM. You notice that E.B.’s daily fasting blood glucose has been running between 130 and 180 mg/dL. You recognize that patients with blood glucose values even slightly above normal levels experience impaired wound healing. However, you also recognize that the risks related to hypoglycemia, such as falls, might outweigh the risk for impaired healing from elevated glucose levels.

7. Identify 4 interventions that would facilitate timely healing of E.B.’s residual limb.

8. What should the postoperative assessment of E.B.’s residual

limb dressing include? 9. You are reviewing the plan of care for E.B. Which care

activities can be safely delegated to the UAP? Select all that apply.

a. Checking E.B.’s vital signs b. Assessing E.B.’s IV insertion site c. Rewrapping the residual limb bandage d. Assisting E.B. with repositioning in the bed e. Asking E.B. to report his level of pain on a scale of 1

to 10 10. On the evening of the first postoperative day, E.B. becomes

more alert and begins to complain of pain. He states, “My right leg is really hurting; how can it hurt so bad if it’s gone?” What is your best response?

a. “That is a side effect of the medication.” b. “Are you able to rate that pain on a scale of 1 to

10?” c. “Don’t worry, that sensation will go away in a few

days.” d. “You can’t be feeling that because your leg was

amputated.” 11. What is causing E.B.’s pain? 12. A compression bandage was applied to E.B.’s residual limb

after surgery. The next day, it is removed so that the surgical site can be assessed. Which actions by the nurse are correct? Select all that apply.

a. Two elastic wrap bandages may be required. b. Use a figure-eight style of wrapping the residual

limb. c. Remove and reapply the bandage several times

daily. d. Apply the bandages very tightly to provide

support and reduce edema. e. E.B. must wear the bandage at all times, even

during physical therapy.

Case study progress The case manager is contacted for discharge planning. E.B. will be discharged to an extended care facility for strength training. Once he receives his prosthesis, he will receive balance training. After that, he will be discharged to his daughter’s home. A physical therapy and occupational therapy home evaluation are ordered. You have provided several sessions of teaching to E.B. and his daughter about care of the residual limb.

13. After your education sessions, you use the Teach-Back method to assess his understanding. Which statement by E.B. reflects a need for further instruction?

a. “I will apply an alcohol-based lotion to my limb twice a day.”

b. “I will perform range-of-motion exercises to all my joints daily.”

c. “I will inspect my limb daily for signs of skin irritation and infection, such as redness, drainage, and odor.”

d. “I will wash my limb every night with warm water and a bacteriostatic soap and dry it gently, allowing it to air dry for 20 minutes.”

14. Before E.B. goes home, you request the home health

agency to perform a home safety evaluation. How would you describe this evaluation to E.B.’s daughter?

15. What instructions should be given to E.B.’s daughter

concerning home safety? 16. What other discharge instructions should be given? Provide at

least three.

Case study outcome E.B. makes a smooth transition from the hospital to the rehabilitation facility and then to the daughter’s home. He was never able to adapt

to independent living, so he eventually moved into his daughter’s home.

Case Study 51 Name _________________________________ Class/Group _____________________________ Date _____________

Scenario T.M. is a 3-year-old boy with cerebral palsy (CP) who has been admitted to your unit. He is scheduled for surgery tomorrow morning for a femoral osteotomy and tendon lengthening to stabilize hip joints and to help reduce spasticity. You are orienting the parents to the unit and have a nursing student assisting you.

1. After getting the family settled, you return to the nurses’ station and the nursing student asks you to explain what CP is and what might have caused it in this patient. How would you answer the student’s question?

2. The nursing student asks what the family might have noticed that would indicate CP in T.M. when he was a baby. Which findings will you include in your discussion with the student? Select all that apply, and state your rationale.

a. Leg scissoring b. Increased irritability c. Head lag at 5 months d. Use of pincer grasp at 9 months e. Ability to sit unassisted at 7 months f. Right hand preference at 12 months g. Positive Moro (startle) reflex at 2 months

Case study progress You and the nursing student finish a health history with the family and determine that T.M. has impaired vision corrected with glasses, a

speech impairment, a seizure disorder, and has had poor weight gain and feeding issues since birth. He has a skin-level feeding device (Mic- Key button) and receives supplemental tube feedings in addition to oral intake. He is not able to ambulate without braces and wears ankle-foot orthotics (AFOs). He receives physical, occupational, and speech therapy on an outpatient basis. T.M. is verbal and able to answer questions with simple phrases and responds to commands. He weighs 12 kg. The surgeon orders the following.

Chart View

Admission Orders

Baclofen (Lioresal) 5 mg every 8 hours PO Diazepam (Valium) 2 mg twice a day PO Lamotrigine (Lamictal) 60 mg twice a day PO Diet as tolerated; NPO for solids and hold tube feedings at

midnight; clear fluids until 4 a.m. Place IV on admission and begin D5 ½NS at 10 mL/hr, increase

rate to 45 mL/hr at midnight VS every 4 hours

3. Explain the rationale for each order. 4. Calculate T.M.’s maintenance fluid requirements. Do the IV

fluids ordered meet this requirement? Show your work. 5. You ask T.M.’s mother about his history of seizures. She states

that he has not had seizures since his medication doses were adjusted several months ago. With this knowledge, which admission orders would you question?

Case study progress T.M. returns to your unit the next afternoon from the postanesthesia care unit (PACU). He is in a bilateral long leg hip spica cast, has an indwelling catheter, and has a family-controlled patient-controlled

analgesia (PCA) device for pain control. You assess T.M. and chart the following findings.

Chart View

Postoperative Assessment

Neur ol o gi c

Awake and alert, verbalizes, and responds to commands. Periods of agitation and restlessness.

Respi ra to ry

RR 25. Bilateral breath sounds, equal, clear, good air exchange. O2 saturation 98% on room air.

Cardi o v as cu la r (C V)

IV line to right forearm infusing D5 ½NS at 45 mL/hr. IV site is clean and dry, no signs of irritation or infiltration. HR 85. Temperature 36.8° C (98.2° F) (axillary).

Gastr oi nt es ti n al ( G

Positive bowel sounds; taking sips of juice PO; Mic-Key button to abdomen (left upper quadrant) clamped. Mic-Key site clean, no signs of breakdown.

I)

Genit o ur in ar y ( G U )

8 French Foley catheter intact and secured, draining yellow clear urine to collection bag. Diaper to spica cast opening.

Neur o m us cu la r

Bilateral long leg hip spica casts with hip abductor bar intact. Toes warm to touch; able to move, unable to palpate pedal pulses due to cast. Cap refill less than 2 seconds.

Pain T.M. occasionally whines and frowns but is comforted by parents. PCA is Y- connected to IV and infusing morphine at 0.01 mg/kg/hr continuous and 0.015 mg/kg per dose PCA with 15-minute lockout.

Psyc h os oc ia l

Parents at bedside active in care.

6. What are your top 5 priorities while providing nursing care to T.M. postoperatively? Outline how you will you address each.

7. Score T.M. on the FLACC pain scale. Why is the FLACC scale an appropriate tool?

Each of the five categories—(F) Face, (L) Legs, (A) Activity, (C) Cry, (C) Consolability—is scored from 0 to 2, which results in a total score between 0 and 10.

8. Which nonpharmacologic interventions would be age appropriate for T.M.? Select all that apply.

a. Use guided imagery b. Read a favorite book c. Offer a favorite DVD or video d. Use bubbles to “blow the hurt away” e. Teach T.M. about pain and its relationship to the

procedure f. Encourage “positive self-talk,” such as, “I will feel

better when the cast is off.”

Case Study Progress T.M.’s condition is stable throughout the day, and the surgeon writes the following orders:

“Patient can PO ad lib. Resume home schedule of 520 mL PediaSure via G-tube from 10 p.m. to 6 a.m. Remove indwelling urinary catheter.”

9. Place the steps for setting up a feeding with a Mic-Key button in the correct order.

a. _____ Clean the skin-level device with alcohol b. _____ Wash hands and apply gloves c. _____ Place the correct amount of PediaSure in the

feeding bag with overfill for priming d. _____ Insert extension set with secure lock, feeding

and medication ports in to Mic-Key

e. _____ Start the feeding f. _____ Attach the feeding bag to the pump and start

“auto prime” g. _____ Attach feeding bag tubing to extension set

feeding port 10. Calculate the hourly rate at which you will set the pump. 11. What nursing interventions and teaching will you include for

T.M. and his family as you discontinue the indwelling urinary catheter?

Case study progress T.M.’s condition continues to improve, and you provide discharge education. T.M.’s mother asks how she will care for his cast when he gets home. You discuss with her the care of a synthetic spica cast.

12. Which statements by T.M.’s mother would indicate that further education is needed? Select all that apply.

a. “I need to keep his feet elevated when I get him home.”

b. “If he has an itch, it is okay to use a knitting needle to scratch under the cast.”

c. “Because T. uses pull-up diapers, I will need to use plastic tape to protect the cast opening.”

d. “It is okay if I give him a tub bath. Because it is a synthetic cast, I can dry it with a blow dryer.”

e. “I need to check the toes on his feet several times a day for the first week or so to see if they are warm and he is able to wiggle them.”

13. What other information should you include in your discharge teaching?

14. The UAP, who is also a nursing student, offers to review the hospital “home medication” form with Mrs. M. True or False? This is an appropriate delegation of tasks. Explain your answer.

Case study outcome T.M.’s parents verbalize understanding of the discharge instructions. T.M. is discharged to home with a wheelchair and a home-health follow-up. He will return to the orthopedic surgeon in 2 weeks.

Case Study 52 Name _________________________________ Class/Group _____________________________ Date _____________

Scenario You are working evenings on an orthopedic floor. One of your patients, J.O., is a 25-year-old Hispanic man who was a new admission on day shift. He was involved in a motor vehicle accident during a high-speed police chase on the previous night. His admitting diagnosis is status post (S/P) open reduction internal fixation (ORIF) of the right femur, multiple rib fractures, sternal bruises, and multiple abrasions. He speaks some English. He is under arrest for drug trafficking. He has one wrist shackled to the bed and a guard stationed inside his room. He says people are “coming to get him” because they think he is working with a rival group. Hospital security is aware of the situation.

Your initial assessment reveals stable vital signs (VS) of 116/78, 84, 16, 98.6° F (37° C). His only complaint is pain, for which he has a patient-controlled analgesia (PCA) pump. Lungs are clear to auscultation. His abdomen is soft and nontender. He is receiving IV D5 LR through the proximal port of a left subclavian triple-lumen catheter at 75 mL/hr; the remaining two ports are locked. The right femur is connected to 10 pounds (4.5 kg) of skeletal traction. The dressing is dry and intact over the incision site.

1. What complications is J.O. at risk for because he has rib fractures and sternal bruising?

2. Outline the assessment you need to perform to detect these

complications. 3. Which findings would you expect when performing his

respiratory assessment? a. Continuous pain, with slow, shallow respirations b. Pain on inspiration, with deep, rapid respirations c. Pain on expiration, with slow, shallow respirations d. Pain on inspiration, with shallow, rapid

respirations 4. J.O. has an antiembolism stocking ordered for his left leg. What

is the rationale for putting a stocking only on this leg? 5. What other measures would be instituted as part of

venous thromboembolism (VTE) prevention? 6. J.O. has an indwelling catheter inserted to drain his urine.

What would you assess for in relation to the indwelling catheter?

7. J.O. has not had a cigarette since the accident and is irritable. Is J.O. a good candidate for nicotine replacement therapy? Give your rationale.

8. Because of the threat made on J.O.’s life and his vulnerable situation, what precautions should the nursing unit take to protect him?

Case study progress The nurse in the emergency department phones to tell you that J.O.’s immunization status could not be determined when he arrived, but he did not receive a tetanus immunization. When you ask J.O. the date of his last tetanus shot, you find out that he was born and raised in Mexico and immigrated to the United States 5 years ago. He does not know whether he has ever had a tetanus shot. You inform the provider, and she orders diphtheria/tetanus toxoid 0.5 mL IM and tetanus immune globulin (HyperTET) 250 units deep IM.

9. Why is J.O. getting two injections? 10. When you give J.O. the tetanus injections, you find J.O. in the

position shown in the illustration. Are any of these findings of concern to you? If so, how would you fix it?

11. While assessing the leg distal to the fractured femur, you find his toes are slightly cool to the touch. Why does this concern you?

12. What assessment do you need to complete? 13. J.O.’s assessment is negative for any further signs and

symptoms of an evolving complication. What do you think

your best course of action is?

Case study progress At 2100, J.O.’s guard summons you to his room. J.O. is pale, slightly confused, and reporting chest pain and dyspnea. Vital signs are 90/60, 120, 28, 100.0° F (37.8° C), and Spo2 of 84%. His pulse is weak, and there are petechiae on his chest.

14. What do you expect is occurring and why? 15. Explain the pathophysiology of this complication and the

reason J.O. is at risk for experiencing this complication. 16. List the priority actions you should take next and the reason

for each.

Chart View

Arterial Blood Gases on 2 L Nasal Cannula

pH 7.32

Paco2 53 mm Hg

HCO3 22 mEq/L (22 mmol/L)

Pao2 84 mm Hg

17. Interpret J.O.’s ABG results.

Case study progress The provider comes and examines J.O. She writes the following orders, then leaves, saying she will be back in 1 hour to check on J.O.

Chart View

Physician’s Orders

Oxygen (O2) to maintain Spo2 of 90% Increase IV D5 LR to 125 mL/hr ECG monitoring Repeat ABGs in 1 hour CBC with differential and serum lipase now STAT chest x-ray (CXR) Methylprednisolone (Solu-Medrol) 12 mg IV push now Furosemide (Lasix) 60 mg IV push now Digoxin 0.25 mg IV push now

18. Describe a plan for implementing these orders in order of priority.

Chart View

Arterial Blood Gases on 10 L Face Mask

pH 7.29

Paco2 56 mm Hg

HCO3 22 mEq/L (22 mmol/L)

Pao2 74 mm Hg

19. J.O. is placed on O2 at 10 L via face mask. ABGs are redrawn after 1 hour. Interpret the results.

20. What intervention do you expect based on your interpretation of these values?

Case study progress The provider returns to reexamine J.O. Because J.O.’s status is deteriorating despite receiving O2 and the IV medications, the provider decides to transfer J.O. to the ICU for ventilator support.

21. You go with J.O. on the transfer to the ICU. Outline the report you would give the ICU nurse.

Case study outcome J.O. recovers for several weeks in the hospital before being transferred to jail to await trial. Shortly before his trial date, he is found stabbed to death in his cell. Although there is an investigation, the murder weapon is never found, and no one is ever charged in his death.

Case Study 53 Name _________________________________ Class/Group _____________________________ Date _____________

Scenario D.V., a 32-year-old man, is being admitted to the medical floor from the neurology clinic with symptoms of multiple sclerosis (MS). D.V. has experienced increasing urinary frequency and urgency over the past 2 months. D.V. recently had two brief episodes of eye “fuzziness” associated with diplopia and flashes of brightness. He has noticed ascending numbness and weakness of the right arm with the inability to hold objects over the past few days. Now he reports rapidly progressing weakness in his legs along with blurred, patchy vision.

1. MS is associated with scattered, patchy demyelination of the sheath around nerves in the brain and spinal cord. What does myelin do? What is demyelination?

2. MS is characterized by remissions and exacerbations. What happens to the myelin during each of these phases?

3. Is D.V. too young to get MS? What is the cause of MS? 4. Outline the subjective and objective assessment data associated

with MS. Place a star next to those D.V. has. 5. The neurologist orders an MRI scan of the brain and spine.

What role does this test play in diagnosing MS?

6. How will the neurologist determine whether D.V. has MS? 7. What are the overall goals of collaborative care for a patient

with MS? 8. D.V. asks, “If this turns out to be MS, how will I be treated?”

How would you respond? 9. D.V. is started on interferon beta-1a (Avonex) 30 mcg

intramuscularly each week. What teaching do you need to provide regarding interferon therapy?

10. The neurologist also orders 500 mg methylprednisolone

(Solu-Medrol) IV daily. What do you need to do to administer this medication safely? Select all that apply.

a. Begin the infusion before 0900 each day b. Reconstitute with 8 mL of benzyl alcohol c. Use the solution within 60 minutes of reconstitution d. Administer a total dose of 8 mL of reconstituted

solution e. Deliver the dose over 30 minutes using IV pulse

administration

11. Because D.V. is experiencing urinary frequency and urgency, the neurologist orders oxybutynin. In addition to medication teaching, what will you teach him to do to assist in controlling urinary symptoms?

12. When planning D.V.’s care, what goal is the most appropriate goal for the clinical problem of activity intolerance related to muscle weakness?

a. “D.V. is free of trauma related to muscle weakness.”

b. “D.V. will maintain muscle strength in his arms and legs.”

c. “D.V. can identify three factors that aggravate muscle weakness.”

d. “D.V. will participate in daily activities as desired without fatigue.”

13. As part of your teaching plan, you want D.V. to be aware of situations or factors that are known to exacerbate symptoms. List 4.

Case study progress After testing is complete, D.V. is diagnosed with MS. He confides in you that he has been depressed since his parents’ divorce and the onset of these symptoms. He tells you that he knows his girlfriend has been unfaithful, but he is afraid of being alone. He is afraid if he tells her about his MS diagnosis, she will leave him.

14. What are you going to do with this information? 15. You are concerned with D.V.’s psychological status,

particularly the negative feelings he expresses regarding himself and the concerns he has voiced. Write a nursing outcome addressing this issue and identify independent nursing actions you would implement.

16. In view of his personal history and current diagnosis, what 2 critical psychosocial issues require monitoring in his follow-up visits?

17. What community resources might D.V. find helpful?

Case study outcome D.V. takes advantage of his time with a psychiatric nurse specialist,

joins a local MS support group, and tells his girlfriend to move out. He later marries a woman from the support group.

Case Study 54 Name _________________________________ Class/Group _____________________________ Date ___________

Scenario C.B. is a single, self-employed 54-year-old man. Three weeks ago, he saw his primary care provider because of symptoms of fatigue, myalgia, fever, and chills, which were accompanied by a hacking cough. He was diagnosed with viral influenza. Today he has weakness, numbness, and tingling of both lower extremities, which is progressing into his upper body. His brother brought him to the emergency department after recognizing the seriousness of his illness. The attending physician immediately suspects Guillain-Barré syndrome (GBS).

1. Describe the cause of GBS. 2. What factors in C.B.’s history support a diagnosis of GBS? 3. Describe the clinical manifestations of GBS. 4. How is GBS diagnosed, and what tests would you expect to be

performed? 5. What is your immediate concern for C.B. and why?

6. What assessment findings would tell you this is occurring? 7. Which set of arterial blood gases would be consistent with the

presence of this complication? a. pH 7.50, Pco2 52 mm Hg b. pH 7.35, Pco2 40 mm Hg c. pH 7.25, Pco2 60 mm Hg d. pH 7.51, Pco2 31 mm Hg

8. Which assessment finding, if present, would require your

immediate intervention? a. Diaphoresis b. Slurred speech c. Facial flushing d. Urinary retention

Case study progress Shortly after arrival, C.B. becomes completely paralyzed and requires endotracheal intubation and mechanical ventilation. He is transferred to the neurologic ICU for further support.

9. What are the goals of medical management in GBS? 10. Describe the collaborative care patients generally receive in the

ICU for GBS. 11. What are the overall goals of nursing care for C.B. at this time? 12. The attending physician states C.B. will begin either IV

immune globulin or plasmapheresis. What is plasmapheresis? 13. You are concerned about the possibility of disuse syndrome

related to C.B.’s paralysis. Describe an outcome of nursing care for C.B., and 4 independent nursing interventions you would implement to meet that outcome.

14. How would performing passive ROM exercises benefit C.B. from a cardiovascular perspective?

a. ROM will help prevent C.B. from developing contractures.

b. Keeping his muscles active will assist in maintaining muscle tone.

c. ROM is an effective way to help prevent VTE. d. The stretching motion accompanying ROM will

decrease C.B.’s pain. 15. Name 5 interventions you would implement to meet the

expected outcome of maintaining skin integrity. 16. What is foot drop? How can it be prevented? 17. How would C.B.’s nutritional needs be met? 18. What evaluative parameters would you use to determine

whether C.B.’s nutritional needs were being met? 19. What interventions can you implement to help decrease C.B.’s

fear and anxiety? Select all that apply. a. Speak calmly to him when providing care. b. Administer continuous intravenous sedation. c. Limit visitors only to immediate family members. d. Continually reassure him that his needs are being

met. e. Use a communication system that allows C.B. to let

his needs be known. 20. C.B.’s brother asks how long C.B. will be paralyzed. How

should you respond?

Case study outcome After staying in the ICU for 8 days, C.B. is transferred to a special ventilator unit of an extended care facility because he shows no signs of improvement in respiratory muscle strength. After several weeks, he progressively regains neurologic function and is weaned from the ventilator. He is able to go home with his brother 5 months after the initial admission.

4

Digestion

Case Study 55 Name_________________________________ Class/Group ______________________________ Date ___________

Scenario T.H., a 57-year-old stockbroker, has come to the gastroenterologist for treatment of recurrent mild to severe cramping in his abdomen and blood-streaked stool. You are the RN doing his initial workup. Your findings include a mildly obese man who demonstrates moderate guarding of his abdomen with both direct and rebound tenderness, especially in the left lower quadrant (LLQ). His vital signs are 168/98, 110, 24, 100.4° F (38° C); he is slightly diaphoretic. T.H. reports that he has periodic constipation. He has had previous episodes of abdominal cramping, but this time the pain is getting worse.

Past medical history reveals that T.H. has a “sedentary job with lots of emotional moments,” he has smoked a pack of cigarettes a day for 30 years, and he had “two or three mixed drinks in the evening” until 2 months ago. He states, “I haven’t had anything to drink in 2 months.” He denies having regular exercise: “just no time.” His diet consists mostly of “white bread, meat, potatoes, and ice cream with fruit and nuts over it.” He denies having a history of cardiac or pulmonary problems and has no personal history of cancer, although his father and older brother died of colon cancer. He takes no medications and denies the use of any other drugs or herbal products.

1. Identify 4 general health risk problems that T.H. exhibits.

2. Identify a key factor in his family history that might have profound implications for his health and present state of mind.

3. Identify 3 key findings on his physical examination and indicate their significance.

Case study progress The physician ordered a complete blood count (CBC), complete metabolic profile, and a CT scan of the abdomen with contrast. Based on the radiology and lab results, physical examination findings, and history, the physician diagnoses T.H. as having acute diverticulitis and discusses an outpatient treatment plan with him.

4. What is diverticulitis? What are the consequences of untreated diverticulitis?

5. While the patient is experiencing the severe crampy pain of acute diverticulitis, what interventions would you perform to help him feel more comfortable?

Case study progress T.H. is being sent home with prescriptions for metronidazole (Flagyl) 500 mg PO q6hr and ciprofloxacin (Cipro) 500 mg PO q12hr.

6. For each medication, state the drug class and the purpose for T.H.

7. Given his history, what questions must you ask T.H. before he takes the initial dose of metronidazole? State your rationale.

8. Which are symptoms of a disulfiram-type reaction? Select all that apply.

a. Headache b. Flushed skin c. Constipation d. Bradycardia e. Abdominal cramps f. Nausea and vomiting

9. When teaching T.H. about the metronidazole prescription,

which instructions need to be included? Select all that apply. a. Take the medication with or after meals. b. This medication might cause a metallic taste. c. If his urine turns reddish brown, notify the doctor

immediately. d. Avoid all alcohol-containing products while on this

medication. e. Take the medication exactly as scheduled, without

skipping doses. f. Some tingling or numbness might be felt in the

hands, which is an expected effect. 10. What specific information would you want to know

before T.H. starts the antibiotics? 11. What are the signs and symptoms of an allergic reaction? 12. T.H. asks you if he can take a laxative for his occasional

constipation. What is your answer? 13. T.H. asks you about his diet. “I’m confused. I was always told

that I needed to be eating a high-fiber diet, which is difficult for me. But the doctor just told me that I need to be on a low- fiber diet for now, so now I’m confused. Which is it supposed to be?” How will you answer his question?

Case study progress You discuss ways to manage while recovering. To help T.H. work through his dietary concerns, you obtain a referral to a registered dietitian.

14. What measures do you think the dietitian will discuss with T.H. to avoid recurrent diverticulitis once his acute symptoms have resolved? Name at least 5.

15. You discuss the need to avoid increased intraabdominal pressure. What are ways that he can avoid increasing intraabdominal pressure and why is this important?

Case study outcome T.H. returns for a checkup 14 days later; all signs and symptoms of diverticulitis are gone. He is working on his lifestyle changes, including smoking cessation, and reports he is walking 30 minutes every day.

Case Study 56 Name _________________________________ Class/Group _____________________________ Date _____________

Scenario J.V., a 56-year-old delivery truck driver, has been taken to the emergency department (ED) because he was experiencing chest pain. It started just after he had a quick lunch at a food truck. He told the paramedic that he often has chest pain but that it goes away when he “takes a swig of antacid,” but this time the pain did not stop. On arrival he was given another dose of antacid and sublingual nitroglycerin, and the chest pain stopped. The first set of cardiac enzymes and basic metabolic profile (BMP) were drawn, and a 12-lead ECG was done. He weighs 275 pounds (125 kg), is 5 ft, 5 inches (165 cm) tall, and tells the nurse he has been overweight all his life. He said he’s had the chest pains for about 2 years but did not go to get checked because they always went away when he took antacids and he was too busy with work to go to a doctor. He works late hours, “lives on coffee,” and grabs fast food when he has time to eat. He smokes 1.5 to 2 packs of cigarettes a day, has a beer every evening once he is home, and usually finishes a 6 pack on the weekends. Vital signs: T 98.9° F (37.2° C), P 110, R 14, BP 148/98. The test results are listed below.

Chart View

Test Results

1. Review the test results and explain any abnormalities. 2. Six hours later, the second set of cardiac enzymes was normal.

Do you think J.V. has had a myocardial infarction (MI)? Defend your answer.

Case study progress After noting the ECG results and the normal second set of cardiac enzymes, the ED provider tells J.V. that the “chest pain” was more likely gastrointestinal (GI) in origin. J.V. was discharged from the ED with a referral to the hospital’s GI clinic with a possible diagnosis of GERD. One week later, at the GI clinic, he is examined by the GI nurse practitioner (NP). The NP tells J.V. that she thinks he has GERD, but the diagnosis will be confirmed by an upper endoscopic examination. The upper endoscopy is scheduled for 0700 on Tuesday of the following week.

3. What is GERD, and what causes it? 4. List the signs and symptoms of GERD. 5. Which of these are potential risk factors for GERD? Select all

that apply. Place an asterisk (*) by the ones that pertain to J.V. a. Obesity b. Alcohol use c. Caffeine use d. Cardiac history e. Cigarette smoking

6. J.V. has self-treated his pain with over-the-counter antacids for a few years. What concerns are there with this long-term self- treatment?

7. Describe the upper endoscopy procedure J.V. will undergo.

8. J.V. has several questions about the upper endoscopy. The nurse provides teaching about the procedure, required preparation, and what to expect afterward. Which statement by J.V. indicates a need for further teaching?

a. “I will be wide awake during the procedure.” b. “I will not eat or drink anything after midnight.” c. “I will not take any antacid the morning of this

test.” d. “This test will help the doctor figure out why I have

so much heartburn.”

Case study progress The upper endoscopy is performed successfully. The gastroenterologist tells J.V. and his daughter that the GERD diagnosis is confirmed, and that he has severe esophageal erosion but no visible ulcers. In addition, a gastric mucosal biopsy was sent for Helicobacter pylori. His postprocedure blood glucose level was 122 mg/dL.

9. What is H. pylori, and how is it treated? 10. H. pylori can be detected with other diagnostic testing. Which

tests are potential diagnostic tests for H. pylori? Select all that apply.

a. Urine test b. Stool culture c. Urea breath test d. Stool antigen test e. Serum or whole antibody tests

11. Which nursing actions are appropriate for delegation to the UAP after the procedure?

a. Taking J.V.’s vital signs. b. Assessing J.V.’s level of consciousness. c. Reviewing J.V.’s point-of-care glucose results. d. Providing postop instructions to J.V. and his

daughter.

Case study progress J.V. was started on omeprazole (Prilosec) 20 mg every morning. The following week, J.V. has a follow-up appointment with the GI nurse practitioner. It turns out that the testing for H. pylori is negative, but he is told that his symptoms are caused by GERD and he will need to make some lifestyle changes in addition to taking the omeprazole for 6 months.

12. What is the mechanism of action of omeprazole? a. It neutralizes gastric acidity. b. It coats the stomach and provides relief of

symptoms. c. It partially blocks histamine-2 receptors and

reduces gastric acid secretion. d. It inhibits the proton pump, resulting in

irreversible blocking of all gastric acid secretion. 13. The nurse has provided teaching regarding the omeprazole.

Which statement by J.V. indicates a correct understanding of therapy with a proton pump inhibitor (PPI) such as omeprazole?

a. “I will take it at night just before bedtime.” b. “I will take the capsule with breakfast every day.” c. “I will take it first thing in the morning, 30 to 60

minutes before breakfast.” d. “I will open the capsule and chew the contents

thoroughly to help it work better.” 14. J.V. asks the nurse, “Why do I have to take this fancy

medicine? The antacid made my heartburn go away. Isn’t that enough?” How will you answer J.V.?

15. List at least 6 lifestyle modifications that J.V. can make to reduce the symptoms of GERD.

Case study outcome After 4 months, J.V. lost almost 40 pounds and is determined to lose at least 40 more. He stopped smoking “cold turkey” and has reduced his

beer intake to once on the weekend. His daughter has helped him with his meal-planning, and he now brings his lunch and snacks with him instead of stopping for fast food. He is happy with his weight loss, how he is feeling, and the amount of money he has saved by not buying his lunches and the cigarettes.

Case Study 57 Name _________________________________ Class/Group _____________________________ Date _____________

Scenario M.R. is a 56-year-old general contractor who is admitted to your telemetry unit directly from his internist’s office with a diagnosis of chest pain. On report, you are informed that he has an intermittent 2- month history of chest tightness with substernal burning that radiates through to the mid-back off and on, in a stabbing fashion. Symptoms occur after a large meal, with heavy lifting at the construction site, and in the middle of the night when he awakens from sleep with coughing, shortness of breath, and a foul, bitter taste in his mouth. Recently he has developed nausea, without emesis, that is worse in the morning or after skipping meals. He reports having “heartburn” three or four times a day. When this happens, he takes a couple of Rolaids or Tums. He keeps a bottle at home, at the office, and in his truck. Vital signs (VS) at his physician’s office were BP 130/80 lying and 120/72 standing, 100, 20, 98.6° F (37° C), Spo2 92% on room air. A 12-lead ECG showed normal sinus rhythm with a rare premature ventricular contraction (PVC).

1. Outline the common causes of chest pain. 2. What mnemonic can you use to help you better evaluate his

pain? 3. What other history is important for you to obtain?

Case study progress M.R. indicates that usually the chest pain is relieved by his antacids, but this time they had no effect. A “GI cocktail” consisting of Mylanta and viscous lidocaine given at his physician’s office briefly helped decrease symptoms.

4. What tests can be done to determine the source of his problems?

Case study progress M.R. has smoked one pack of cigarettes a day for the past 35 years, drinks 2 or 3 beers on most nights, and has noticed a 20-pound (9 kg) weight gain over the past 10 years. He feels “so tired and old now.” M.R. has dark circles under his eyes and complains of constant daytime fatigue. His wife is even sleeping in another bedroom because he is snoring so loudly. He also reinjured his lower back a month ago at work, lifting a pile of boards, so his physician prescribed ibuprofen (Motrin) 400 mg twice a day for 4 weeks.

5. Which factors in M.R.’s life are likely contributing to his chest pain and nausea? Explain how.

Case study progress M.R. explains that 6 months ago his physician prescribed ranitidine (Zantac) 150 mg PO at bedtime for heartburn, and that it helped a little, but that it never really “did the job,” so he stopped taking it. Now he keeps a bottle of Tums or Rolaids in his truck and at his bedside “because I always seem to need them.”

6. Why do you think the ranitidine did not help M.R.?

Case study progress M.R.’s 12-lead ECG and the first set of cardiac enzymes were normal. The chest x-ray showed no abnormalities. Room air Spo2 is 94% and

breathing is unlabored. Other lab results are listed in the chart.

Chart View

Admission Laboratory Test Results

WBC 6000/mm3 (6 × 109/L)

Hgb 15.0 g/dL (150 g/L)

Hct 47%

Platelets 220,000/mm3 (220 × 109/L)

Na 140 mEq/L (140 mmol/L)

K 3.7 mEq/L (3.7 mmol/L)

BUN 18 mg/dL (6.4 mmol/L)

Creatinine 1.0 mg/dL (88 mcmol/L)

Lipase 20 units/L

Amylase 32 units/L

PT/INR 12.0 sec/1.0

Urea breath test positive

7. What is the significance of the urea breath test result? 8. Are there any concerns with the other lab results?

Case study progress Suddenly, M.R. begins to complain of nausea; as you hand him the emesis basin, he promptly vomits coffee-ground emesis with specks of bright red blood. VS remain stable.

9. What concerns do you have about the coffee-ground emesis? 10. How do you confirm that the emesis contains blood?

Case study progress You ask the charge nurse to contact the gastrointestinal (GI) consulting physician to explain the recent events while you stay with M.R. The gastroenterologist gives several orders and states he will be there in 30 minutes. The orders are as follows.

Chart View

Physician’s Orders

Nothing by mouth (NPO) status for emergent esophagogastroduodenoscopy (EGD)

Repeat CBC STAT O2 by nasal cannula; titrate O2 to maintain Spo2 over 94% Type and crossmatch (T&C) 2 units packed red blood cells

(PRBCs), and hold Start a pantoprazole (Protonix) drip at 8 mg/hr, preceded by a

40-mg bolus IV over 2 minutes Insert a Salem Sump nasogastric tube (NGT) and start a gastric

lavage with 100 mL normal saline (NS) Insert 2 large-bore IVs and start NS at 100 mL/hr

11. Place the previous orders in order of priority. 12. Explain the rationale for each of the preceding orders.

Case study progress The gastroenterologist performs the EGD and finds erosive esophagitis Los Angeles (LA) class B, a moderately sized hiatal hernia, diffuse erosive gastritis, and an ulcer in the antrum of the stomach that is oozing blood. The duodenal bulb shows a normal endoscopic appearance. During the EGD, the bleeding is stopped with cautery. Biopsy specimens of the gastric mucosa are obtained, and his bleeding ulcer is attributed to the NSAIDs (i.e., ibuprofen). He is kept NPO until the next morning to allow sufficient hemostasis of the cauterized

site. Clear liquids are allowed at breakfast. M.R. tolerates the liquid diet without any nausea and vomiting and is discharged to home the next day with the following instructions:

• Advance diet slowly, as tolerated, to mechanical soft. • Take pantoprazole 40 mg PO q am before breakfast • Amoxicillin 1 gram PO twice daily for 2 weeks • Clarithromycin extended-release 1 gram once daily for 2

weeks • Sucralfate 1 gram 4 times a day (before meals and bedtime) for

3 months • Make a follow-up appointment in 6 to 8 weeks with physician • Stop all aspirin and over-the-counter or herbal pain relief

medications, especially NSAIDs (ibuprofen, naproxen, and so on)

• Stop or limit alcohol intake and smoking

13. M.R. has prescriptions for 4 different drugs. You are providing teaching about the timing of the various drugs. Help M.R. map out a schedule for taking his new medications with regard to his meals. Explain your reasons for the times provided.

14. What is the purpose of the antibiotics? 15. After discussing lifestyle modifications for controlling acid

reflux with M.R., which statement by M.R. indicates a further need for teaching?

a. “I will stop smoking.” b. “I will stop drinking alcohol.” c. “I will avoid eating 2 to 3 hours before my

bedtime.” d. “I will wait 30 minutes before lying down or sitting

in my recliner after meals.”

Case study outcome M.R. goes home and vows to stop smoking, stop drinking alcohol, and

take better care of himself. He calls his primary care physician to schedule a sleep study because of his snoring and cuts back his working hours. The biopsies were positive for H. pylori bacteria, and his symptoms disappeared after 2 weeks of antibiotic therapy. He will remain on the pantoprazole for another 3 months.

Case Study 58 Name _________________________________ Class/Group _____________________________ Date _____________

Scenario C.G. is admitted to your medical-surgical unit from the emergency department. He is 65 years old and presented with indigestion, loss of appetite (especially to meat), and a 30-lb (13.6 kg) weight loss over the last 6 months. He also reports that he has been having feelings of fullness that were only relieved by belching and vomiting. Other symptoms reported are abdominal pain, nausea, vomiting, and dysphagia. He has a history of smoking (20 pack-years), gastric ulcers, asthma, and pernicious anemia. He reports regular alcohol use of 3 to 4 beers each day. He reports, “I haven’t had anything to drink in 2 weeks because of my stomach.” He reports a family history of COPD, hypertension, Crohn’s disease, and colon cancer. He denies any allergies or any previous hospitalizations.

1. Identify 4 health risk problems you have identified while reviewing C.G.’s admission information.

2. What are some possible medical diagnoses that will be considered, based on this initial assessment?

3. What diagnostic tests or exams would you anticipate being ordered? Select all that apply.

a. Chest x-ray b. Hemoglobin A1C c. Liver function tests

d. Renal function tests e. Complete blood count f. Thyroid function tests g. CT scan of the abdomen h. Comprehensive metabolic panel i. Upper GI endoscopy and barium studies

Case study progress After reviewing the test and pathology results, the physician has diagnosed C.G. with gastric cancer. Although he is upset by the news, C.G. states, “I am happy we caught it early and can cure it!”

4. How are the symptoms of gastric cancer related its disease progression?

Chart View

Laboratory Results

Hgb 7.3 g/dL (73 g/L)

Hct 27%

5. What is the significance of C.G.’s hemoglobin and hematocrit in relation to his diagnosis?

6. What condition in C.G.’s medical history could contribute to his anemia? Explain your rationale.

7. Which of these goals are appropriate pretreatment interventions for the patient with gastric cancer? Select all that apply.

a. Treatment of anemia b. Eliminating infection c. Toleration of high-residue diet d. Replacement of blood volume e. Correction of nutritional deficits

Case study progress After discussing his diagnosis, C.G. and his medical team have decided to perform surgical removal of the tumor. The treatment team spends several days improving his nutritional status by consulting nutrition services, encouraging several small meals per day and toleration of liquid supplements. He then undergoes a total gastrectomy and is admitted to the postsurgical unit. Postoperative care orders include nutrition consult, wound care consult, nasogastric tube (NGT) to low intermittent wall suction, and IV replacement of vitamins C, D, K, and B.

8. What is a total gastrectomy? 9. You note another postoperative order for cyanocobalamin, 100

mcg/day IM. What is this drug, and what is its purpose? 10. C.G. asks the nurse to explain why he needs to take all of the

dietary supplements if the cancer is now gone. What teaching will you provide?

11. Explain the rationale for the NGT setting of “low intermittent suction” for this patient.

12. Postoperatively the patient has an NGT in place. The nursing student assigned to your patient comes to you and reports that the NGT has not drained any secretions in the last 2 hours. What response will you provide to the nursing student?

Case study progress On postoperative day 3, C.G. is started on small amounts of clear liquids by mouth. He is tolerating the liquids without discomfort when suddenly, on postop day 7, C.G. begins to complain of severe abdominal pain and nausea. His VS are BP 96/68, P 117, R 24, T 102.1° F (38.9° C).

13. What condition do you anticipate based on C.G.’s symptoms? 14. What tests should the nurse expect to be ordered for

evaluation of C.G.’s condition? Select all that apply. a. CT abdomen

b. X-ray of the abdomen c. Upper GI endoscopy d. Upper GI series with gastrografin e. Complete blood count with differential

Case study progress The UAP reports that the patient’s VS are now BP 84/58, P 125, R 28, T 101.8° F (38.8° C), and Spo2 85%. The physician is notified, and you receive orders for oxygen, an IV fluid bolus, and broad-spectrum antibiotics. C.G. is transferred to the intensive care unit for management of a suspected anastomotic leak with development of a duodenal stump fistula.

15. Which of these conditions are an indication that the antibiotic treatment is unsuccessful in managing the leak and fistula? Select all that apply.

a. Hemodynamic instability b. Decreased abdominal pain c. Hyperactive bowel sounds d. Increased body temperature e. Pain during abdominal palpation

16. For each intervention for an anastomotic leak with fistula that is listed, answer “Yes” if it is an appropriate intervention or “No” if it is not. For the “No” answers, explain the reason they are incorrect.

a. Administration of IV fluids b. Clamping of the NGT tube c. Administration of IV antibiotics d. Acid suppression of the GI tract

17. The interventional radiologist is consulted to perform a percutaneous drainage procedure. What is the treatment goal of percutaneous drain placement?

Case study outcome C.G. was started on IV antibiotics, and a percutaneous drain was

placed. His condition continued to deteriorate, and he was ultimately found to have acute abdomen and peritonitis with worsening sepsis. He was taken to the OR for laparoscopy with peritoneal washing. Postoperatively, he remained intubated and continued to receive IV antibiotic and fluid therapy. On postop day 2 after the laparoscopy, C.G. was found to be in septic shock and was started on IV vasopressors and IV fluids. His condition continued to deteriorate and progressed to cardiac arrest. After resuscitation efforts, C.G. succumbed to his condition secondary to severe septic shock from the anastomotic leak and peritonitis.

Case Study 59 Name _________________________________ Class/Group _____________________________ Date _____________

Scenario You are on duty in the intermediate care unit and scheduled to take the next admission. The emergency department (ED) nurse calls to give you the following report: “This is Barb in the ED, and we have a 62-year-old man, K.L., with lower GI bleeding. He is a sandblaster with a 12-year history of silicosis. He is taking 40 mg of prednisone per day. During the night, he developed severe diarrhea. He was unable to get out of bed fast enough and had a large maroon-colored stool in the bed. His wife ‘freaked’ and called the paramedics. He is coming to you. His vital signs (VS) are stable—110/64, 110, 28, Spo2 93%—and he’s a little agitated. His temperature is 98.2° F (36.8° C). He has not had any stools since admission, but his rectal examination was guaiac positive and he is pale but not diaphoretic. We have him on 5 L O2/NC. We started a 16-gauge IV with LR at 125 mL/hr. He has an 18- gauge Salem Sump to continuous low suction; that drainage is guaiac negative. We have done a CBC with differential, chem panel, coagulation times, a T&C for 4 units, ABGs, and a UA. He’s all ready for you.”

1. How do you prepare for the patient’s arrival? 2. What are common causes of lower GI bleeding? 3. Given K.L.’s history, what do you think significantly

contributed to the GI bleeding? 4. What other signs and symptoms would you ask him about? 5. Compare and contrast among melena, hematochezia, and

occult blood in the stool.

Case study progress K.L. arrives on your unit. As you help him transfer from the ED stretcher to the bed, K.L. becomes very dyspneic and expels 800 mL of maroon stool.

6. What immediate complication concerns you most? 7. What are the first 3 actions you should take?

Case study progress K.L. reports that he is feeling nauseated. VS are 92/58, 116, 32, Spo2 93%. The provider orders an IV fluid bolus of 500 mL 0.9% normal saline and 2 units packed red blood cells (PRBCs) STAT.

8. What other interventions do you need to do? 9. What assessment indicators would you monitor in K.L.?

10. In caring for K.L., which activities can be safely delegated to UAP? Select all that apply.

a. Applying a pulse oximetry monitor b. Measuring his VS every 15 minutes c. Assessing K.L.’s peripheral circulation d. Monitoring K.L.’s hemoglobin and hematocrit

levels e. Emptying the Foley catheter collection bag each

hour f. Obtaining consent from K.L. for the blood

transfusions

Chart View

Arterial Blood Gases

pH 7.47

Paco2 33 mm Hg

Pao2 65 mm Hg

HCO3 23 mEq/L (23 mmol/L)

Spo2 91%

Complete Blood Count

WBCs 4300/mm3 (4.3 × 109/L)

RBCs 4.0 million/mm3 (4 × 1012/L)

Hemoglobin (Hgb) 7.8 g/dL (78 g/L)

Hematocrit (Hct) 23%

Platelets 208,000/mm3 (208 × 109/L)

11. K.L.’s ED lab results are sent to you. Interpret his ABGs. What do they tell you?

12. Discuss K.L.’s Hgb and Hct results. 13. You are preparing to administer the first of the 2 units of

PRBCs. Evaluate each of the following statements about the safe administration of blood. Enter T for true or F for false. Tell why the false statements are incorrect.

_____1. Prime the correct tubing and filter with normal saline.

_____2. Verify K.L.’s identification with the secretary in the endoscopy suite

_____3. Obtain baseline vital signs before starting the

transfusion _____4. Begin the transfusion at a rate of 125 mL/hr _____5. Take K.L.’s vital signs 30 minutes after starting

the transfusion _____6. Complete the transfusion within 6 hours of

receiving the unit

Case study progress The provider discusses K.L. with the gastroenterologist, who schedules K.L. for an immediate colonoscopy. You go with K.L. to the endoscopy suite and help the endoscopy nurse in giving him IV midazolam (Versed) and morphine sulfate during the procedure.

14. Why is K.L. receiving midazolam and morphine sulfate? 15. What is the priority nursing responsibility during the

procedure? a. Reorienting K.L. as needed b. Monitoring K.L.’s IV fluid intake c. Assessing K.L.’s VS and oxygen saturation d. Documenting K.L.’s response to the procedures

Case study progress During the colonoscopy, K.L. begins passing large amounts of bright red blood. He becomes paler and more diaphoretic and begins to have an altered level of consciousness.

16. Name 3 immediate interventions you need to do.

Case study progress The gastroenterologist finds the site of the bleeding and cauterizes the affected vessels. There is no further evidence of active bleeding. K.L. is transferred back to the unit and care continued with fluids and blood products to stabilize his condition. He received one dose of IV esomeprazole in endoscopy and is placed on twice-daily therapy.

17. Describe the ongoing assessment you need to obtain. 18. What assessment data would lead you to determine K.L.’s

condition was stabilizing? 19. Which finding would warrant your immediate intervention?

a. Palpable splenomegaly b. Hypoactive bowel sounds c. Complaints of abdominal cramping d. Orthostatic change in BP of 20

20. What is the expected outcome associated with giving esomeprazole?

21. Later, when he seems to be feeling better, K.L. tells you he is really embarrassed about the mess he made for you. How are you going to respond to him?

22. K.L. is being prepared for discharge. What do you need to address in your teaching with K.L.?

Case study outcome The gastroenterologist concludes that the GI hemorrhage was prednisone induced. Because the prednisone was being used to suppress the progression of silicosis, the provider will try to decrease his maintenance dose of prednisone while monitoring his respiratory status.

Case Study 60 Name _________________________________ Class/Group _____________________________ Date _____________

Scenario S.G. is a 6-month-old girl who is scheduled for sequential repair of her cleft lip (cheiloplasty) and palate (palatoplasty). She has recently been adopted from China and her medical history is unknown. S.G. is scheduled for her cleft lip repair, and Mrs. G. brings her to the same- day surgery unit the week before for her preoperative workup. As

you do her workup, you recognize that care of the child with clefting uses a multidisciplinary approach.

1. Discuss the health problems for which these children are at risk and who on the craniofacial team would address each issue.

2. List at least 4 reasons why would it be important to include a social worker and/or psychologist to the craniofacial team following this infant?

3. S.G. weighs 6.5 kg. Plot this finding on the Centers for Disease Control and Prevention (CDC) growth chart (see http://www.cdc.gov/growthcharts/data/who/GrChrt_Girls_24LW_9210.pdf Which statement best summarizes your findings?

a. S.G.’s weight falls below the 5th percentile. b. S.G.’s weight is between the 10th and the 25th

percentile. c. S.G.’s weight is between the 50th and the 75th

percentile. d. S.G.’s weight is above the 95th percentile.

4. What information from her health history is important to obtain from her mother to plan her perioperative care? Select all that apply. Explain your rationale.

a. Adoption status b. Immunization status c. Gross motor milestones d. Current method of feeding e. Parent’s employment status f. Current known health status

5. Choose the lab tests that you expect to be obtained preoperatively. Discuss the rationale for your choices.

a. CHEM-7 b. Urinalysis c. Stool sample for fat content d. Arterial blood gases (ABGs) e. Complete blood count with differential

http://www.cdc.gov/growthcharts/data/who/GrChrt_Girls_24LW_9210.pdf
Case study progress The lab test results and findings of S.G.’s preoperative workup are normal, and she is scheduled for her cheiloplasty.

6. What will you include in your preoperative teaching to S.G.’s parents?

7. Determine S.G.’s daily fluid maintenance requirements. How can her parents ensure this intake and determine adequate hydration status?

Case study progress S.G. returns to the unit after her cheiloplasty. The surgeon notes on the chart that surgical glue was used to close the incision. You note standing postoperative orders to clean with normal saline and apply antibiotic ointment three times per day.

8. True or False? You would call the surgeon to question this order. Explain your answer.

9. S.G. also has an order for acetaminophen 120 mg rectally

or PO every 4 to 6 hours as needed for pain. Is the ordered dose safe and therapeutic?

10. Pain management is essential for healing. Which instructions should be given to the parents before S.G. is discharged? Select all that apply. Discuss your rationale for each.

a. Administer pain medication with a teaspoon. b. You may add the pain medication to S.G.’s formula

to help her take it more easily. c. Use distraction like a music box in the crib to act as

an adjunct to pain medication. d. Give pain medication around the clock, every 4 to 6

hours for the first 24 to 48 hours after surgery. e. Provide diagrams to show parents how much pain

medication to give with an oral medication syringe. 11. S.G.’s parents are advised that S.G. will return 6 months later

for the palatoplasty. They are concerned about the delay between surgeries. What will your response be?

Case study progress S.G. returns to your unit 6 months later for her cleft palate repair (palatoplasty).

12. Which nursing interventions are appropriate as you plan her care? Select all that apply. Discuss your rationale for each.

a. Use elbow restraints as needed. b. Maintain strict intake and output. c. Administer pain medications as ordered. d. Oral suction with a plastic oral suction catheter as

needed. e. Clear fluids; advance as tolerated. Patient may use

a straw. f. Position patient side-lying or on abdomen

postoperatively. 13. S.G. has a normal recovery and is being discharged. When

giving her parents discharge instructions, what will you advise them concerning diet and signs and symptoms to report?

Case study outcome S.G. is discharged to home with instructions for follow-up with the surgeon and multidisciplinary team.

Case Study 61 Name _________________________________ Class/Group _____________________________ Date _____________

Scenario As a nurse on a gastrointestinal (GI) unit, you receive a call from an

affiliate outpatient clinic notifying you of a direct admission with an estimated time of arrival of 60 minutes. The clinic nurse gives you the following information: A.G. is an 82-year-old woman with a 3-day history of intermittent abdominal pain, abdominal bloating, and nausea and vomiting (N/V). A.G. moved from Italy to join her grandson and his family only 2 months ago, and she speaks very little English. All information was obtained through her grandson. Her medical history includes colectomy for colon cancer 6 years ago and ventral hernia repair 2 years ago. She has no history of coronary artery disease, diabetes mellitus, or pulmonary disease. She takes only ibuprofen (Motrin) occasionally for mild arthritis. Allergies include sulfa drugs and meperidine. A.G.’s tentative diagnosis is small bowel obstruction (SBO) secondary to adhesions. A.G. is being admitted to your floor for diagnostic workup. Her vital signs are stable, she is receiving an IV infusion of D5 ½NS with 20 mEq KCl at 100 mL/hr, and 2 L oxygen by nasal cannula.

1. Based on the nurse’s report, what signs of bowel obstruction does A.G. have?

2. Are there other signs and symptoms of a bowel obstruction that you should observe for while A.G. is in your care?

3. While A.G. is on the way, you secure the hospital’s interpreter service on the telephone. A.G. arrives on your unit with her grandson. You admit A.G. to her room and introduce yourself as her nurse. As her grandson introduces her, she pats your hand. You know that you need to complete a physical examination and take a history. What will you do first?

4. Before you begin your examination, the grandson, an attorney, tells you that elderly Italian women are extremely modest and might not answer questions completely. He says he wants to stay in the room during the examination. How will you proceed?

5. What key questions must you ask this patient while you have the use of an interpreter?

6. For each characteristic listed, specify whether it is a

characteristic of small bowel obstruction (SBO), large bowel obstruction (LBO), or both (B).

_____a. Persistent abdominal cramping _____b. Colicky, cramping pain at frequent intervals _____c. Upper or epigastric abdominal distention _____d. Distention in the lower abdomen _____e. Obstipation _____f. Ribbon-like stools _____g. Nausea and early, profuse vomiting, which may

contain fecal material _____h. Minimal or no vomiting _____i. Severe fluid and electrolyte imbalances

7. What is obstipation? 8. During your examination, you note that she does not have

muscle guarding or rebound tenderness on palpation. Is this important? Explain your answer.

9. A.G. vomits during your examination. Which indicates an obstruction that is more distal?

a. The emesis contains bile. b. There is a minimal amount of emesis. c. The emesis is foul-smelling and fecal. d. The emesis is mostly mucus and clear liquid.

Case study progress The physician orders the insertion of a Salem Sump nasogastric tube (NGT). You insert the NGT into A.G. and connect it to intermittent low wall suction.

10. How will you check for placement of the NGT? 11. List, in order, the structures through which the NGT must pass

as it is inserted. 12. A.G.’s grandson asks you, “What is that blue thing at the end

of the tube? Shouldn’t it be connected to something?” How do you answer?

13. Which are appropriate nursing actions for a patient with an

NGT? Select all that apply. a. Provide frequent oral care. b. Check the nares around the tube for signs of

irritation. c. Apply a petroleum-based lubricant on the skin

around the tube. d. Pull the tube tightly and pin to the patient’s gown

to prevent movement. e. Tape the tube to the nose so it does not pull on the

nares or cause ulceration. f. Obtain an order for a topical antiseptic spray (i.e.,

Chloraseptic) if she has a sore throat. 14. You note that A.G.’s NGT has not drained in the last 3 hours.

What can you do to facilitate drainage? 15. The NGT suddenly drains 575 mL; then drainage slows down

to about 250 mL over 2 hours. Is this an expected amount? 16. You enter A.G.’s room to initiate your shift assessment. A.G.’s

abdomen seems to be more distended than yesterday. How would you determine whether A.G.’s abdominal distention has changed?

Case study progress After 24 hours, A.G.’s symptoms are unrelieved. She reports continued nausea, cramps, and sometimes strong abdominal pain; her hand grips are weaker; and she seems to be increasingly lethargic. You look up her latest lab values and compare them with the admission data.

Chart View

Laboratory Test Results

Test Admission Hospital Day 3

Sodium 136 mEq/L (136 mmol/L) 130 mEq/L (130 mmol/L)

Potassium 3.7 mEq/L (3.7 mmol/L) 2.5 mEq/L (2.5 mmol/L)

Chloride 108 mEq/L (108 mmol/L) 97 mEq/L (97 mmol/L)

Carbon dioxide 25 mEq/L (25 mmol/L) 31 mEq/L (31 mmol/L)

BUN 19 mg/dL (6.8 mmol/L) 38 mg/dL (13.6 mmol/L)

Creatinine 1 mg/dL (88.4 mcmol/L) 2.2 mg/dL (194.5 mcmol/L)

Glucose 126 mg/dL (7.0 mmol/L) 65 mg/dL (3.6 mmol/L)

Albumin 3.0 g/dL (30 g/L) 3.1 g/dL (31 g/L)

Protein 6.8 g/dL (68 g/L) 4.9 g/dL (49 g/L)

17. Which lab value is of most concern to you? Why? Are there others that are also of concern? Explain.

18. What measures do you anticipate to correct in each of the imbalances described in Question 17?

Case study outcome In view of A.G.’s continued deterioration, the surgeon meets with the patient and her family and they agree to surgery. The surgeon releases an 18-inch (46 cm) section of proximal ileum that had been constricted by adhesions. Several areas look ischemic, so these are excised, and an end-to-end anastomosis is done. A.G. tolerates the procedure well. Her recovery is slow but steady. A.G. goes home in the care of her grandson and his wife on the seventh postoperative day. Discharge plans include a home health nurse, home health aide, in-home physical therapy, and dietitian consultation. The grandson is included in the plans.

Case Study 62 Name _________________________________ Class/Group _____________________________ Date _____________

Scenario C.W., a 36-year-old woman, was admitted several days ago with a diagnosis of recurrent inflammatory bowel disease (IBD) and possible small bowel obstruction (SBO). C.W. is married, and her husband and 11-year-old son are supportive, but she has no extended family in the state. She has had IBD for 15 years and has been taking mesalamine (Asacol) for 15 years and prednisone 40 mg/day for the past 5 years. She is very thin; at 5 feet 2 inches (157 cm), she weighs 86 lbs (39 kg) and has lost 40 lbs (18 kg) over the past 10 years. She averages 5 to 10 loose stools per day. C.W.’s life has gradually become dominated by her disease, with anorexia, lactase deficiency, profound fatigue, frequent nausea and diarrhea, frequent hospitalizations for dehydration, and recurring, crippling abdominal pain that often strikes unexpectedly. The pain is incapacitating and relieved only by a small dose of diazepam (Valium), oral electrolyte solution (Pedialyte), and total bed rest. She confides in you that sexual activity is difficult: “It always causes diarrhea, nausea, and lots of pain. It’s difficult for both of us.” She is so weak she cannot stand without help. You indicate complete bed rest on the nursing care plan.

1. Identify 6 priority problems for C.W. 2. Considering C.W.’s weakness, chronic diarrhea, and lower-

than-desired body weight, what nursing interventions need to be implemented to minimize skin breakdown? Name at least 6.

3. What is the mechanism of action of the mesalamine (Asacol) in relation to the IBD?

a. It increases bulk and moisture content in the stool. b. It relaxes the smooth muscle of the intestines, thus

reducing motility. c. It slows intestinal motility, prolonging transit time

of intestinal contents. d. It blocks prostaglandin production, thus

diminishing inflammation in the colon.

Case study progress C.W.’s condition deteriorates. On the third day after admission she experiences intractable abdominal pain and unrelenting nausea and vomiting. C.W. is taken to the operating room because of probable SBO and is readmitted to your unit from the postanesthesia care unit. During surgery, 38 inches (96 cm) of her small bowel was found to be severely stenosed, with 2 areas of visible perforation. Much of the remaining bowel is severely inflamed and friable. A total of 5 feet (152 cm) of distal ileum and 2 feet (61 cm) of colon have been removed, and a temporary ileostomy was established. She has a Jackson-Pratt (JP) drain to bulb suction in her right lower quadrant (RLQ), and her wound was packed and left open. She has 2 peripheral IV lines, a Salem Sump nasogastric tube (NGT), and a Foley catheter. Her vital signs (VS) are 112/72, 86, 24, 100.8° F (38.2° C) (tympanic). You attach her NGT to low-continuous wall suction per the postoperative orders.

4. You begin a thorough postoperative assessment of C.W.’s abdomen. What does your assessment include? List the steps in the order in which the assessment should be completed.

5. A nursing student enters C.W.’s room and auscultates her abdomen. She looks at you and excitedly announces that she hears good bowel sounds. You take the opportunity to teach her the proper method of auscultating bowel sounds on a patient who has NGT to low-continuous wall suction. How would you correct her error?

6. Four hours later, you measure the drainage from the JP tube. Look at the following figure and state how much drainage you obtained.

7. What else will you note about the drainage? 8. Describe the proper method for reestablishing suction on the JP

drain after you have emptied the bulb container. 9. C.W. asks you, “I know why I have the pouch. Why do I have

to have this other little tube?” How will you explain the purpose of the JP drain?

Case study progress It is 4 days after C.W.’s surgery. During the routine dressing change, you note a small pool of yellow-green drainage in the deepest part of the wound. You obtain an order for a wound culture.

10. How will you obtain a culture specimen from C.W.’s wound? 11. What information do you need to send to the lab with the

wound culture specimen? 12. You obtain a wound culture specimen, complete the dressing

change, obtain a full set of VS, note a temperature of 100.4° F (38° C), and assess increased tenderness in C.W.’s abdomen. What orders do you anticipate receiving once you notify the surgeon of C.W.’s condition?

13. As you assess C.W.’s stoma and drainage, what would indicate that they are healthy? Select all that apply.

a. The stoma will be level with the skin. b. The stoma will be in the shape of a donut. c. The drainage will be thick and dark brown. d. The skin around the stoma should be intact. e. The stoma will be a uniform medium cherry red. f. The stoma will be light pink, and an occasional dark

spot might appear. 14. Will any aspect of C.W.’s history significantly affect the wound

healing process? If so, how? 15. The surgeon tells you she will be there to examine C.W. As

you tell C.W. that her doctor is coming to talk to her, C.W. says that she feels something wet running down her side. You find some leakage of intestinal drainage onto the skin. What should you do?

Case study progress You change the ileostomy appliance before the surgeon arrives. C.W. is evaluated, and it is determined that she has developed peritonitis, and needs to return to surgery for exploratory laparotomy. The surgery revealed another area of perforated bowel and generalized peritonitis. Another 12 inches (30.5 cm) of ileum were resected. The peritoneal cavity was irrigated with normal saline (NS) and 3 drainage tubes were placed: a Jackson-Pratt (JP) drain to bulb suction, a rubber catheter to irrigate the wound bed with NS, and a sump drain to remove the irrigation fluid. The initial JP drain remains in place. A right subclavian triple-lumen catheter was also inserted.

16. A few hours later, C.W. is still experiencing pain from the peritonitis as well as from the surgical incision. Which position

may help to make her more comfortable? a. Lying on her left side b. Lying on her right side c. Supine with legs extended d. Head of bed slightly elevated, with knees flexed

17. C.W. has been on NPO status since the surgery. The surgeon orders total parenteral nutrition (TPN) at a rate of 50 mL/hr. What is the purpose of these orders?

18. The pharmacy delivers C.W.’s first bag of TPN. You have an order to stop the maintenance IV infusion after starting the TPN. What is the purpose of this order?

19. During the night shift the TPN solution bag becomes nearly empty, and the night nurse discovers that the next bag of TPN has not been prepared. The hospital pharmacy does not prepare TPN during the night shift. What does the nurse need to do next?

a. Hang a bag of D5W when the TPN is finished. b. Hang a bag of D10W when the TPN is finished. c. Convert the line to a saline lock until the TPN

solution is ready. d. Slow the TPN rate to 10 mL/hr until the next TPN

bag can be prepared.

Case study progress You discuss your concerns about C.W.’s nutritional status with C.W.’s surgeon. She agrees to request a consultation from a registered dietitian (RD). After gathering data and making several calculations, the RD makes recommendations to the surgeon. The TPN orders are adjusted, C.W. begins to gain weight slowly, and her wound shows signs of healing.

20. You discuss with the RD what specific digestive difficulties C.W. is likely to face. What problems might C.W. be prone to develop after having so much of her bowel removed?

21. The RD talks with C.W. about her dietary needs. You attend

the session so you will be able to reinforce the information. What basic information is the RD likely to discuss with C.W.?

Case study outcome C.W. is successful in her battle with peritonitis. Gradually tubes are removed as she grows stronger with TPN and time. C.W. learns how to change her ostomy appliance and is discharged to home. She attends an enterostomal support group on a regular basis.

Case Study 63 Name _________________________________ Class/Group _____________________________ Date _____________

Scenario Mr. and Mrs. B. arrive in the emergency department (ED) with their 6- week-old infant, S.B. As the triage nurse, you ask the couple why they have brought S.B. to the ED. Mrs. B. states, “My baby breastfed well for the first couple of weeks but has recently been throwing up all the time, sometimes a lot and really forcefully. He looks skinny and is hungry and fussy all the time.” You determine that the couple is homeless and has been living out of their car for the past month. S.B. has had no primary care since discharge after delivery.

1. What additional information will you need to obtain from Mr. and Mrs. B.?

Case study progress Your primary assessment of the infant reveals the following: S.B. is alert and fussy and consoles with a bottle of Pedialyte (per physician orders). His anterior fontanel is slightly depressed and posterior fontanel cannot be palpated. You auscultate regular breath sounds at a rate of 30 breaths/min. No adventitious sounds. Pulse oximetry is 98% on room air. Heart rate is 190 with regular rate and rhythm. Brachial

and pedal pulses are + 3 and equal. Abdomen is round and nontender to palpation. Positive bowel sounds. Diaper is dry. S.B. moves all extremities and there are no rashes noted. Rectal temperature is 37.2° C (98.9° F). There is a quarter-sized flat red area on occiput that “has been there since he was born” according to the mother. Slight “tenting” noted.

You transport S.B. to radiology, and he vomits a large amount of clear fluid. Patient returns to the room in his mother’s arms, awake and alert. The mother appears anxious and states, “I don’t know what’s wrong with my baby! Why can’t you people tell me anything?”

2. How do you respond to the mother? 3. Your institution uses electronic charting. Based on the

assessment described, which of the following systems would you mark as abnormal as you document your findings? Mark abnormal findings with an “X,” and provide a brief narrative note.

☐ Neurologic: ☐ Respiratory: ☐ Cardiovascular: ☐ Gastrointestinal: ☐ Genitourinary: ☐ Musculoskeletal: ☐ Skin: ☐ Psychosocial: ☐ Pain:

4. The emergency physician orders a complete blood count, complete metabolic profile, urinalysis, blood pH, and x-rays. The physician suspects dehydration and metabolic alkalosis secondary to hypertrophic pyloric stenosis. Which lab findings would you expect with metabolic alkalosis?

a. Na: 128 mEq/L (128 mmol/L), K: 2.6 mEq/L (2.6 mmol/L), Cl: 90 mEq/L (90 mmol/L), HCO3: 30 mEq/L (30 mmol/L)

b. Na: 130 mEq/L (130 mmol/L), K: 5.7 mEq/L (5.7

mmol/L), Cl: 94 mEq/L (94 mmol/L), HCO3: 22 mEq/L (22 mmol/L)

c. Na: 130 mEq/L (130 mmol/L), K: 3.9 mEq/L (3.9 mmol/L), Cl: 98 mEq/L (98 mmol/L), HCO3: 17 mEq/L (17 mmol/L)

d. Na: 148 mEq/L (148 mmol/L), K: 4.1 mEq/L (4.1 mmol/L), Cl: 108 mEq/L (108 mmol/L), HCO3: 13 mEq/L (13 mmol/L)

5. What is the underlying cause of S.B.’s diagnosis of metabolic alkalosis?

6. Which clinical manifestations might you find with metabolic alkalosis? Select all that apply.

a. Tetany b. Hyperthermia c. Increased respiratory rate d. Increased risk for seizures e. Neuromuscular irritability

7. What additional assessment findings might reflect the consequences of frequent prolonged vomiting in the infant?

Case study progress S.B. is diagnosed with hypertrophic pyloric stenosis, admitted to the pediatric unit, is NPO, and scheduled for surgery.

8. S.B.’s parents are concerned that their living situation contributed to S.B.’s diagnosis. How would you respond to their concerns?

9. Mr. and Mrs. B. have questions about the necessity of surgery and question what is going to be done next. What are your responsibilities as you respond to Mr. and Mrs. B.’s concerns?

Chart View

Preoperative Orders

Vital signs q4hr Strict intake and output (I&O) 30 mL Pedialyte q3hr PO Place IV and begin D5⅓NS at 50 mL/hr Nasogastric (NG) tube placed to low continuous wall suction Daily weights

10. Which preoperative orders would you question? 11. Which interventions can be delegated to unlicensed assistive

personnel (UAP)? Select all that apply. a. Assessing for NG tube placement every shift b. Reminding parents to save diapers to be weighed c. Teaching parents the rationale for NG tube

insertion d. Assisting parents in holding infant without

removing NG tube e. Obtaining VS every 4 hours and reporting any

abnormal findings to the RN 12. You note that your patient was hypokalemic and the

fluids you hung per orders do not include potassium. You contact the surgeon to clarify. You receive the following order: “Change IV to D5⅓NS with 20 mEq KCl at maintenance.” You obtain the new fluids and hang per orders. True or False: This is an appropriate nursing action. Explain your answer.

Case study progress S.B. returns to your unit after a pyloromyotomy. Mrs. B. is concerned about when she will be able to resume breastfeeding and what they need to do for their baby.

13. What postoperative teaching would you provide to them?

Case study outcome

S.B. progresses well and is tolerating normal breastfeeding within 48 hours with minimal vomiting. He is discharged with follow-up in 2 weeks with the parents’ new primary care provider. A social worker has helped Mr. and Mrs. B. obtain temporary housing and apply for available insurance and local resources.

Case Study 64 Name _________________________________ Class/Group _____________________________ Date _____________

Scenario You are a nurse on an inpatient psychiatric unit. J.M., a 23-year-old woman, was admitted to the psychiatric unit last night after assessment and treatment at a local hospital emergency department for “blacking out at school.” She has been given a preliminary diagnosis of anorexia nervosa. As you begin to assess her, you notice that she has very loose clothing, she is wrapped in a blanket, and her extremities are very thin. She tells you, “I don’t know why I’m here. They’re making a big deal about nothing.” She appears to be extremely thin and pale, with dry and brittle hair, which is very thin and patchy, and she constantly complains about being cold. As you ask questions about weight and nutrition, she becomes defensive and vague, but she does admit to losing “some” weight after an appendectomy 2 years ago. She tells you that she used to be fat, but after her surgery she did not feel like eating and everybody started commenting on how good she was beginning to look, so she just quit eating for a while. She informs you that she is eating lots now, even though everyone keeps “bugging me about my weight and how much I eat.” She eventually admits to a weight loss of “about 40 pounds (18 kg) and I’m still fat.”

1. Using Diagnostic and Statistical Manual of Mental Disorders, Fifth Edition (DSM-V) criteria, how is the diagnosis of anorexia

nervosa determined? 2. Identify 8 clinical signs or symptoms of anorexia nervosa. Place

a star or asterisk next to those that J.M. has. 3. What other disorders might occur along with anorexia

nervosa? Name at least 4. 4. How does bulimia nervosa differ from anorexia nervosa? 5. Name 5 behaviors that J.M. or any other patient with anorexia

may engage in other than self-starvation. 6. What common family dynamics are associated with anorexia

nervosa?

Case study progress You review her admission lab studies. An electrocardiogram (ECG) has also been ordered.

Chart View

Admission Laboratory Work

Sodium 135 mEq/L (135 mmol/L)

Potassium 3.4 mEq/L (3.4 mmol/L)

Chloride 99 mEq/L (99 mmol/L)

Magnesium 1.5 mEq/L (0.62 mmol/L)

Blood urea nitrogen 18 mg/dL (6.4 mmol/L)

Creatinine 1.0 mg/dL (88.4 mcmol/L)

Hemoglobin 11 g/dL (110 g/L)

Hematocrit 35%

7. Which lab results are of concern at this time? Explain your answers.

8. What clinical symptoms of anorexia nervosa, if present, should

have the highest priority? Explain your answers.

Case study progress J.M.’s ECG results show normal sinus rhythm with no ST segment or other changes. You meet with J.M. to formulate a plan of care.

9. Which of these are appropriate nutrition interventions for J.M.? Select all that apply.

a. Do not prompt her to eat. b. Discuss food choices with her. c. Allow her to eat alone during mealtimes. d. Have normal conversations during meals. e. Allow her to select her menu on her own. f. Weigh her each morning after voiding, with her

back to the scale. g. Restrict her from returning to her room or a

restroom without staff supervision after meals. 10. List at least 6 psychologic aspects of the plan of care for J.M.

Case study progress After 3 weeks, you are providing discharge teaching for J.M. You ask her whether she is ready to go home. J.M. states, “I’ll be so glad to get out of this place. I’m so fat and ugly. I need to lose 10 pounds (4.5 kg). I bet I can do it in just a couple of days. Otherwise, I don’t want to live anymore.”

11. What will you discuss with the provider before any further discharge teaching or plans?

12. You report J.M.’s statements to the provider. What actions do you expect the provider to take?

13. What medications may help J.M. with resolution of both her anorexia nervosa and her depression?

14. Which statements by J.M. would indicate successful treatment? Select all that apply.

a. “I just have to stay skinny to feel good.”

b. “When you say I look ‘healthy’ I feel fat.” c. “Lately I’ve been feeling a little better about

things.” d. “I am looking forward to going out with my

friends again.” e. “It’s up to me to take care of my body by eating

enough food.”

Case study outcome After 2 weeks, J.M. has gained 5 lbs (2.25 kg) and seems to be more willing to eat. She still expresses fears of “getting fat,” but she states that she is ready to go home and back to school. The provider arranges for J.M. to participate in an outpatient partial hospitalization program that specializes in eating disorders. J.M. expresses interest in meeting others with the same problems.

Case Study 65 Name _________________________________ Class/Group _____________________________ Date _____________

Scenario You are working on the postoperative surgical unit and admitted S.B., a 42-year-old woman with a history of morbid obesity, hypertension, sleep apnea, and diabetes mellitus. She underwent a laparoscopic Roux-en-Y procedure. For the past year, she had tried to lose weight with diet and exercise, but the progress was minimal. Her preoperative weight was 298 pounds (135 kg), and she is 5 feet, 5 inches (165 cm) tall.

1. What is BMI? Calculate S.B.’s BMI and specify the degree of obesity her BMI reflects.

2. What is the Roux-en-Y procedure? What are the intended outcomes of this surgery?

3. Using arrows, specify the direction of food passage and the direction of digestive juices after Roux-en-Y gastric bypass surgery.

4. In preparing the room for S.B.’s admission, what

equipment would you ensure is in place to eliminate embarrassment and promote patient safety? Explain your

rationale.

Case study progress S.B. is admitted to the bariatric unit and positioned in a bariatric bed. Postoperative orders include placing the nasogastric (NG) tube on low continuous suction. S.B.’s family asks, “How can she eat if this tube is in her nose?”

5. What education will you need to offer the patient and family about the need for the NG tube and delivery of nutrition?

6. How much volume is the gastric pouch able to hold after a Roux-en-Y surgery?

a. 10 to 15 mL b. 15 to 30 mL c. 30 to 50 mL d. 50 to 100 mL

Chart View

Post-Op Orders

1000 mL normal saline with thiamine 100 mg, folic acid 1 mg, and multivitamins 10 mL, once daily to infuse at 125 mL/hr

Famotidine (Pepcid) 20 mg IVP twice daily

Head of bed at 30 degrees

Ambulate every 4 hours

Out of bed to chair twice daily

Incentive spirometry

7. What is the purpose of the IV infusion? 8. What type of medication is famotidine, and what is its

purpose?

9. What is the purpose for raising the head of the bed 30 degrees?

Case study progress On postoperative day 2, S.B. is refusing to get out bed to ambulate or to transfer to the bedside recliner. She is requesting to lie in bed with the head of bed flat.

10. List at least 4 interventions you should encourage for the immobile patient with risk for postsurgical complications.

11. What are the risks associated with immobility and bed rest after surgery?

12. What education should you give to S.B. about the importance of using the incentive spirometer (IS)?

13. You are giving S.B. instructions on how to use the IS. Which statements are correct about the proper use of the IS? Select all that apply.

a. “Incentive spirometry should be done every hour.” b. “If able, sit in the chair or as far up in bed as

possible to use the IS.” c. “Place the mouthpiece in your mouth, but there

should not be an airtight seal.” d. “After each set of deep breaths using the IS, cough

to make sure your lungs are clear.” e. “Breathe in slowly and as deeply as possible, until

you see the piston reach the goal amount for volume.”

f. “After breathing in, hold the breath and the piston in place for 3 to 5 seconds and then release the breath.”

Case study progress On postoperative day 3, S.B. has been out of bed to the chair once and has ambulated in the hall twice. She has been using the IS based on your instructions and has been able to meet her volume goal. Suddenly, S.B. reports that she is not able to achieve the IS goals and is

having increased abdominal pain. The UAP takes vital signs and reports them as follows: BP 100/76, P 113, R 12, T 101.4° F (38.6° C).

14. What is the nurse’s priority assessment? 15. What information would the nurse document while assessing

the surgical incision? 16. Based on the patient’s vital signs, what does the nurse suspect? 17. The surgical incision showed no drainage or other abnormal

findings. Which tests would the nurse expect to be ordered? Select all that apply, and explain your answers.

a. Lactic acid b. Chest x-ray c. Urine culture d. Blood cultures e. Abdominal CT scan f. Liver function panel g. CBC with differential

Case study progress The lab results all come back within defined limits for the patients, but the cultures are pending. S.B. has been able to achieve adequate pain control and has been more mobile. Her temperature is down to 98.7° F (37.1° C). The NG tube was removed, and she is now taking a full liquid diet and advancing as tolerated. You overhear the patient state, “I have probably lost 15 pounds (6.8 kg) already because of the surgery. I can eat whatever I want now.”

18. How would you address this comment? What education should you offer the patient about gastric bypass and weight loss?

19. Once S.B. begins eating regularly, what complication should the nurse anticipate and assess for?

20. Which nutrient deficiencies are common with this type of surgery? Select all that apply.

a. Iron

b. Calcium c. Vitamin C d. Magnesium e. Cobalamin (B12)

Case study outcome The dietitian follows up with S.B. and discusses healthy eating practices and smart food choices. They develop a plan and goals for managing and maintaining S.B’s weight loss. S.B. was discharged home with home health care, home physical therapy, and nutritional services. After 8 months, S.B. successfully lost 88 lbs (40 kg) after gastric bypass surgery and is an active member of a local bariatric surgery support group.

Case Study 66 Name _________________________________ Class/Group _____________________________ Date _____________

Scenario You are a nurse working on a surgical unit and take the following report from the registered nurse in the emergency department. “We are sending you a patient with rule out small bowel obstruction and/or food blockage. Dr. N., the gastrointestinal specialist, is on his way in to see the patient. D.S. is a 68-year-old obese man who has had a sudden onset of severe abdominal cramping, distention, and nausea and vomiting; he denies passing of flatus or stool within the past 12 hours. Past medical history includes heart failure, hypertension, colon cancer, and ulcerative colitis. He underwent a total colectomy 6 years ago and had an enterocutaneous fistula 4 years ago. Lab samples have been drawn, and the results will be sent to your floor. We started an IV and placed a Salem Sump nasogastric tube (NGT). His vital signs are 143/76, 82, respirations 26 and slightly labored, and 101.1° F (38.4° C). He is on his way up.”

1. Given that D.S. previously had a total colectomy, would he have a colostomy or an ileostomy? Explain your answer.

2. What would you expect to see if D.S.’s ostomy has normal function?

Case study progress After D.S. is settled into his room, the NGT and IV line are functioning well, and he receives pain medication, you begin your admission assessment. His abdomen is extremely large, firm to touch, with multiple scars and an ileostomy pouching system in his right lower quadrant (RLQ).

3. Describe 3 common complications of an ileostomy. 4. D.S.’s past medical history of fistula, combined with the

probability of blockage or obstruction, places him at an increased risk for which problems?

5. As you assess the stoma, you look for signs that it is healthy. Which assessment findings are characteristics of a healthy stoma? Select all that apply.

a. The stoma is dry. b. The stoma is moist. c. The stoma is pale pink in color. d. The stoma is flat against the skin. e. The stoma is cherry red to dark pink in color.

6. What stoma changes would you report immediately to the physician? Name at least 4.

7. Why are transparent ostomy pouches recommended for postoperative patients or for patients who are hospitalized?

8. True or False: The stoma will present visual clues of D.S.’s bowel blockage or obstruction. Explain your answer.

Case study progress D.S. continues to complain of abdominal pain and cramping and becomes increasingly restless. You notice that the abdomen behind and around his stoma and pouch appears larger when compared with

the other side of his abdomen.

9. How would you assess for a possible peristomal hernia? 10. Why is a peristomal hernia a problem?

Case study progress You note the ostomy pouch has liquid brown effluent along the lateral edge of the wafer. You check to see that the pouch is properly attached to the wafer and discover that stool is indeed leaking from under the barrier. D.S. apologizes for not bringing any supplies with him, stating, “My ostomy nurse told me to always carry extra supplies for times like this.” Right now, D.S. cannot remember what size he needs. You note he is wearing a two-piece system with a plastic ring- flange that attaches to the pouch with a matching ring.

11. How will you determine the correct pouching size and system?

Case study progress You finish your general head-to-toe assessment and order the appropriate pouching products for D.S. You take clean towels, washcloths, and underpads into his room, along with a hamper for soiled linens. You gather scissors, Skin-Prep, and adhesive remover to assist with the pouching change.

12. As you return to his room, you review the steps for changing an ostomy pouch. What are the steps you will need to follow?

Case study progress You have gathered all needed supplies, and D.S. is as comfortable as possible. You begin the pouching change. Using the adhesive remover, with the push-pull method, you gently remove the wafer. As you lift the wafer, you note that the peristomal skin has severe erythema directly encircling the stoma. There is denudation (partial- thickness breakdown) at the medial stoma-skin edge.

13. How should the skin around the stoma look? 14. In general, there are 4 different causes of erythema or skin

breakdown. Identify 2. 15. After you discover the reddened skin, how will you proceed

with the ostomy care? 16. As you clean the site, D.S. tells you, “You need to scrub that

with strong soap! It’s dirty!” How do you reply to D.S.? 17. As you plan care for D.S. for the shift, which activities may be

performed by UAP? Select all that apply. a. Empty ostomy bag. b. Provide skin care around the ostomy. c. Assess and document stoma appearance. d. Measure liquid contents from ostomy bag. e. Teach ostomy care and skin care to the patient and

caregiver.

Case study progress The next day, D.S.’s vital signs return to normal and his abdomen is less distended. The ileostomy is steadily draining greenish-brown liquid stool. The NGT is removed, and D.S. is started on sips of clear liquids. When you go to check his ileostomy pouch, D.S. tells you, “I know I’ve had this a long time, but I still can’t stand to look at this thing. My wife usually helps me with it, and I hate that.”

18. What will you suggest for D.S. at this time?

Case study outcome D.S. is discharged from the hospital and has no further complications. He and his wife now attend the local ostomy support group.

5

Urinary Elimination

Case Study 67 Name _________________________________ Class/Group _____________________________ Date _____________

Scenario You are working in an extended care facility when M.Z.’s daughter brings her mother in for a week’s stay while she goes on a planned vacation. M.Z. is an 89-year-old widow with a 4-day history of nonlocalized abdominal discomfort, incontinence, new-onset mental confusion, and loose stools. Her most current vital signs are 118/60, 88, 18, 98.4° F (37.4° C). The medical director ordered a postvoid catheterization, which yielded 100 mL of cloudy urine that had a strong odor, and several lab tests on admission. Urine culture and sensitivity (C&S) results are pending; the other results are shown in the chart.

Chart View

Laboratory Test Results Complete metabolic panel (CMP): Within normal limits except for the following results:

BUN 25 mg/dL (8.9 mmol/L)

Sodium 131 mEq/L (131 mmol/L)

Potassium 3.2 mEq/L (3.2 mmol/L)

White blood cell count 11,000/mm3 (11 x 109/L)

Urinalysis

Appearance Cloudy

Odor Foul

pH 8.9

Protein Negative

Nitrites Positive

Crystals Negative

WBCs 6 per low-power field

RBCs 3

1. What condition do the lab reports point toward? 2. Which assessment findings are typical of an older adult with

the condition in Question 1? a. Fever b. Hematuria c. Bladder spasms d. Nonlocalized abdominal discomfort

3. Considering her history and lab results, what other condition is a possibility?

4. The medical director makes rounds and writes orders to start an IV of D5 NS at 75 mL/hr. Because M.Z. is unable to take oral medications, the medical director orders ciprofloxacin (Cipro) 400 mg q12hr IV piggyback (IVPB). Are the type of fluid and rate appropriate for M.Z.’s age and condition? Explain.

5. While the IVPB ciprofloxacin is being administered, which adverse effects might occur? Select all that apply.

a. Nausea b. Headache

c. Drowsiness d. Hypotension e. Restlessness f. Tendon rupture

6. You enter the room to start the IV infusion and find that the UAP had taken M.Z. to the bathroom for a bowel movement. M.Z. asks you to help her, and, as you open the door, you observe her wiping herself from back to front. What do you need to do at this time?

7. Later that day, M.Z. has difficulty voiding, and palpation of the bladder reveals distention. A bedside bladder scanner indicates at least 250 mL of urine in the bladder. A Foley catheter is ordered and inserted. Because M.Z. has been having diarrhea, what special instructions should you give the UAP assigned to give basic care to M.Z.?

Case study progress The next day, you are the nurse assigned to M.Z.’s care. You notice that the UAP emptying the gravity drain is not wearing personal protection devices. You observe that the drainage port of the drainage bag was contaminated during the process because the UAP allowed it to touch the floor.

8. What issues need to be considered in protecting M.Z.’s

safety? Describe your actions in working with the UAP. 9. As you assess M.Z., you notice that her catheter tubing is not

secured. Why does the tubing need to be secured? Where is the correct place for the catheter tubing?

Case study progress On the third day after M.Z.’s admission, the urinary C&S results are as follows: E. coli, more than 100,000 colonies, sensitive to ciprofloxacin, trimethoprim-sulfamethoxazole, and nitrofurantoin.

10. What changes, if any, will be made to the antibiotic therapy? 11. The UAP reports that M.Z.’s 8-hour intake is 520 mL and the

output is 140 mL. Identify 2 possible reasons that could account for the difference and explain how you would assess each.

Case study progress Further monitoring of M.Z.’s urine output reflects adequate output amounts. After a week, M.Z. has completed her antibiotic therapy. Her mental status has cleared, the Foley catheter has been discontinued, and she is voiding without difficulty. She is ready for discharge.

12. What instructions should you discuss with the daughter? 13. She needs to notify the primary care physician if her mother

develops which problems? Name at least 6.

Case study outcome The diarrhea subsides, and M.Z.’s urine is more clear and normal in appearance. She goes back to her daughter’s home after a week and is more alert.

Case Study 68 Name _________________________________ Class/Group _____________________________ Date _____________

Scenario You are working in the emergency department (ED) when M.B., a 72- year-old man, comes in and states he is unable to void. His initial vital signs (VS) are 168/92, 88, 20, 98.2° F (36.8° C).

1. Are M.B.’s VS appropriate for a man of his age? If not, offer a rationale for the abnormal readings.

2. Given M.B.’s statement, what would you expect to find during your initial assessment?

Case study progress While you are taking M.B.’s history, he tells you he is generally in good health and leads an active life. His current medications include finasteride (Proscar) 5 mg/day and vitamin supplements. He reports that he has been unable to void for 12 hours and is very uncomfortable. He asks you to help him.

3. Which of these statements best describes the therapeutic effect of the finasteride?

a. It reduces urinary flow. b. It strengthens the detrusor muscle. c. It reduces the size of the prostate gland. d. It causes relaxation of the urinary sphincter.

4. What do you need to know about the history of his use of the finasteride?

5. If you are going to administer the finasteride, what

precautions are necessary? 6. What are your priorities for this patient? 7. After examining M.B., the ED physician asks you to insert an

indwelling urethral Foley catheter. What will you include in M.B.’s teaching before placing the Foley?

8. You have just finished providing peri-care and are preparing to insert the Foley catheter. Put in order the steps to follow when inserting a Foley catheter, with 1 being the first step:

____a. Apply sterile gloves. ____b. Anchor the catheter to the patient’s inner thigh. ____c. Ensure that the drainage bag is secured to the

bed, below the level of the bladder. ____d. Position and drape the patient. ____e. Cleanse the urethral meatus, following the

proper procedure for a male patient.

____f. Open lubricant container, antiseptic container. Most catheter manufacturers no longer recommend testing the balloon.

____g. Inflate the balloon fully with the amount of fluid recommended for the catheter.

____h. Gently insert the catheter 7 to 9 inches (17 to 22.5 cm) or until urine flows into the catheter tubing, then insert catheter at least 1 inch further.

____i. Perform hand hygiene. ____j. Lubricate catheter. ____k. Open the catheterization kit and place the

underpad, if present, under the patient. 9. After two unsuccessful attempts to advance the catheter into

the bladder, you stop. What is your next intervention? Why? What could be causing this problem?

10. The ED physician successfully inserts the indwelling catheter with the use of the type of catheter illustrated in the accompanying figure. What type of catheter is this, what is its advantage in this situation, and how is it inserted?

11. As the physician begins to inflate the catheter balloon, M.B. winces in pain and states, “Ouch, you’re hurting me!” What happened, and what will the physician do?

12. You watch the urine drain into the bag and note that the amount is approaching 500 mL. What do you do at this time?

13. After the catheter is in place, the ED physician writes orders to discharge M.B. with instructions to see his primary care provider (PCP) on the following day. It is your responsibility to give discharge instructions. Outline your care plan.

14. The next day, M.B. is seen by his PCP, who changes M.B.’s

medication to alfuzosin (Uroxatral). The catheter will be discontinued 2 days later. What teaching is essential regarding this new medication?

a. Alfuzosin needs to be taken in the morning. b. M.B. needs to take each dose on an empty stomach. c. This medication might cause fainting when he first

starts taking it. d. M.B. can stop taking the alfuzosin once the urinary

symptoms subside. 15. You provide teaching on managing episodes of urinary

retention. You use the Teach-Back technique and ask M.B. to teach back the concept he has learned. Which statement by M.B. indicates a need for further teaching?

a. “A warm shower may help me urinate.” b. “I will drink small amounts of water throughout

the day.” c. “I will limit alcoholic drinks to 1 per day if I have

any.” d. “I will drink large amounts of water at mealtimes

to help me urinate.”

Case study outcome M.B. is discharged but continues to have difficulty emptying his bladder. He is referred to a urologist who eventually recommends surgery for an enlarged prostate gland.

Case Study 69 Name _________________________________ Class/Group _____________________________ Date _____________

Scenario N.H., an 89-year-old widow, recently experienced a left-sided cerebrovascular accident (CVA). She has right-sided weakness and

expressive aphasia with minimal swallowing difficulty. N.H. has a medical history of a minor left-sided CVA 2 1/2 years ago, chronic atrial flutter, and hypertension. She has lived with her daughter’s family in a rural town since her previous stroke. Since admission to an acute care facility 5 days ago, N.H. has gained some strength, has become oriented to person and place, and is anxious to begin her rehabilitation program. She is transferred for rehabilitation to your skilled nursing facility with the orders shown in the chart.

Chart View

Admission Orders

Hydrochlorothiazide 25 mg/day PO Digoxin 0.125 mg/day PO Warfarin (Coumadin) 5 mg/day PO Acetaminophen 325 mg q6hr PO prn for pain Zolpidem (Ambien) 5 mg PO at bedtime prn for sleep Diet: Mechanical soft, low sodium with ground meat Foley catheter to gravity drainage, and then begin bladder

training Referrals for speech therapy, occupational therapy, and

physical therapy to evaluate and treat swallowing, communication, and functional abilities

1. What lab orders would you anticipate as a result of this specific list of orders? With each response, describe your rationale.

Case study progress A week later, at the interdisciplinary care conference, you report that bladder training is progressing and recommend removing the catheter if N.H.’s mobility and communication abilities have progressed sufficiently. The group and N.H. agree that she is ready for the Foley catheter to be removed.

2. Identify 3 problems that N.H. is at risk for developing after catheter removal and describe specific interventions for each problem.

Case study progress Two days after the Foley catheter is removed, you observe that N.H.’s urine is cloudy and concentrated and has a strong odor, even though the volumes voided have been adequate.

3. What are your immediate actions?

Case study progress N.H. is started on sulfamethoxazole 800 mg/trimethoprim 160 mg (Bactrim DS) 1 tab PO bid × 10 days for a urinary tract infection (UTI). However, 2 days later, N.H. is in the bathroom and she is very upset. She has just voided; there is blood on the toilet, and the water is bright red with blood. You help the UAP clean N.H. and help her into bed.

4. Describe your assessment steps. 5. Identify at least 2 potential causes for N.H.’s hematuria. 6. Using SBAR, what information would you provide to the

physician when you call?

Case study progress N.H.’s physician changes her antibiotic to oral ciprofloxacin (Cipro) and holds the warfarin for 2 days. Two days later, N.H.’s UTI is responding to antibiotics and she has had no further bleeding in the urine. You want to prepare her and her daughter for eventual discharge.

7. You have provided teaching about preventing a recurrent UTI to N.H. and her daughter. You use the Teach-Back technique to confirm understanding. Which statement by N.H.’s daughter indicates an adequate understanding of the information provided?

a. “She needs to limit how much water she drinks each day.”

b. “When Mom uses the toilet she needs to wipe from back to front.”

c. “She needs to try to urinate every 2 to 3 hours and not delay it if she needs to go.”

d. “If she thinks she might be getting an infection, I will mention it at her next appointment.”

8. You discuss with N.H.’s daughter how certain foods and drinks may irritate the bladder and should be avoided. These foods include which of the following? Select all that apply.

a. Bananas b. Chocolate c. Spicy foods d. Citrus juices e. Caffeinated beverages

9. You talk with N.H.’s daughter about her understanding of caregiving responsibilities for her mother. What kind of questions do you ask to assess whether she is capable of taking care of her mother since her mother’s health has declined? List at least four.

Case study outcome N.H.’s right-sided weakness and expressive aphasia do not resolve. Her daughter takes N.H. home and, with the help of her sister, nieces, and a home health aide, they have adjusted well to living together.

Case Study 70 Name _________________________________ Class/Group _____________________________ Date _____________

Scenario S.M. is a 68-year-old man who is being seen at your clinic for routine

health maintenance and health promotion. He reports that he has been feeling well and has no specific complaints, except for some trouble “emptying my bladder.” Vital signs at this visit are 148/88, 82, 16, 96.9° F (36.1° C). He had a complete blood count and complete metabolic panel completed 1 week before his visit, and the results are listed in the chart.

Chart View

Laboratory Test Results

Sodium 140 mEq/L (140 mmol/L)

Potassium 4.2 mEq/L (4.2 mmol/L)

Chloride 100 mEq/L (100 mmol/L)

Bicarbonate 26 mEq/L (26 mmol/L)

BUN 19 mg/dL (6.8 mmol/L)

Creatinine 0.8 mg/dL (72 mcmol/L)

Glucose 94 mg/dL (5.2 mmol/L)

RBC 5.2 million/mm3 (5.2 x 1012/L)

WBC 7400/mm3 (7.4 x 109/L)

Hgb 15.2 g/dL (152 g/L)

Hct 46%

Platelets 348,000/mm3 (348 x 109/L)

Prostate-specific antigen (PSA) 4.23 ng/mL (4.23 mcg/L)

Urinalysis Within normal limits

1. What can you tell S.M. about his lab work? 2. What is the significance of the PSA result? 3. What other specific examination will S.M. need to have along

with the PSA test?

Case study progress While obtaining your nursing history, you record no family history of cancer or other genitourinary problems. S.M. reports frequency, urgency, and nocturia × 4; he has a weak stream and has to sit to void. These symptoms have been progressive over the past 6 months. He reports he was diagnosed with a “large prostate” a number of years ago. Last month, he began taking saw palmetto capsules but had to stop taking them because “they made me sick.”

4. Why did S.M. try taking the saw palmetto, and why do you think he stopped taking it?

5. S.M. is curious why his enlarged prostate would affect his urination. He is concerned that he has prostate cancer. What would you teach him?

Case study progress The primary care provider (PCP) performs a digital rectal examination (DRE) and asks for a post-void residual (PVR) urine test.

6. Which findings from the DRE indicate BPH? Select all that apply.

a. Prostate is firm. b. Prostate feels nodular. c. Prostate feels spongy. d. Prostate feels smooth. e. Prostate is symmetrically enlarged.

7. You use a bedside bladder scanner and document that S.M. voided 60 mL and his PVR is 120 mL. You report the PVR to the PCP. What is the significance of his PVR?

8. Which of these are considered risk factors for BPH? Select all that apply.

a. Aging b. Obesity

c. Smoking d. High cholesterol levels e. Lack of physical activity f. History of urinary tract infections

9. Commonly used medications for BPH are 5-alpha-reductase inhibitors, such as finasteride (Proscar), and alpha-blocking drugs, such as tamsulosin (Flomax). How do these drugs differ?

10. The PCP orders tamsulosin (Flomax) 0.4 mg/day PO. You enter S.M.’s room to teach him about this medication. What side effects will you tell S.M. about? Select all that apply.

a. Diarrhea b. Dizziness c. Headache d. Heartburn e. Dry mouth f. Orthostatic hypotension

11. S.M. asks, “Will this condition affect my relationship with my wife?” What should you tell him?

12. What would you expect S.M. to report if the medication was successful?

Case study progress S.M. returns in 6 months to report that his symptoms are worse than ever. He has tried several different medications, but medication management failed, and he is told that surgical intervention is necessary.

13. What surgical options are available to S.M.? Describe at least 3.

Case study outcome S.M. chose an outpatient procedure. He did well postoperatively and was discharged to home.

Case Study 71 Name _________________________________ Class/Group _____________________________ Date _____________

Scenario G.W., a 34-year-old African American man, presents with increasing right knee swelling. He states that the swelling has gotten worse over the past 2 weeks and on presentation is now having difficulty ambulating. He reports taking over-the-counter ibuprofen 200 mg tablets at least 4 to 8 tablets per day for nearly 1 year for persistent back and knee pain. He has not seen his primary care physician (PCP) in nearly 2 years. G.W. also complains of weakness, fatigue, decreased urine output, and joint pain and stiffness. He also tells you that when he does urinate, it looks “rusty.” His vital signs are as follows: BP 210/100, P 86, R 24, T 98.7° F (37.1° C).

1. What are the health risks associated with taking ibuprofen for an extended period?

2. What specific questions would you ask G.W. based on his reported symptoms?

Case study progress G.W. tells you that a few years ago he was diagnosed with high blood pressure, but he did not like the medication’s side effects, so he stopped taking it. He said that he was told that he had “kidney problems” but never kept the appointments to check his kidneys. After further assessment, the nurse finds that the abdomen appears firm, round, and distended with edema. He has + 2 edema on his ankles and shins bilaterally. He reports decreased urine output; on admission urine is dark and rust-colored. G.W. is alert and oriented to person, place, time and situation. He is lethargic but easily arousable and coherent. His blood work shows a BUN of 35 mg/dL (12.5 mmol/L), serum creatinine of 4.7 mg/dL (415 mcmol/L), albumin 1.2

g/dL (12 g/L), and H&H 7.1 g/dL (71 g/L) & 23.5%. The results of his urinalysis are listed here:

Chart View

Urinalysis

Appearance Clear

Color: Rust

Odor: Aromatic

pH 6.2

Protein 14 mg/dL

Glucose Negative

White blood cells 5

WBC casts Many

Red blood cells 10

RBC casts Many

3. The physician suspects glomerulonephritis. Which assessment findings and lab results support this diagnosis?

4. What risk factors, if any, does G.W. have for developing glomerulonephritis?

5. Differentiate acute and chronic glomerulonephritis. Which one does G.W. have? Defend your answer.

6. What diagnostic tests are used to confirm the diagnosis of glomerulonephritis?

7. G.W. asks you, “What is glomerulonephritis? Do I have a kidney infection?” Which answer is correct?

a. “No, you have had an allergic reaction to the ibuprofen.”

b. “Yes, glomerulonephritis is a chronic infection of

the kidneys.” c. “Yes, you had a bladder infection that led to a

kidney infection.” d. “No, glomerulonephritis is an inflammation of a

section of the kidneys.”

Case study progress The nephrologist is consulted and the results of a renal biopsy confirm the diagnosis of chronic glomerulonephritis. G.W. received a furosemide (Lasix) drip, and had a total urine output of 450 mL in the next 24 hours. G.W.’s BP has improved but remains elevated at 198/102. The nephrologist ordered lisinopril 5 mg PO once daily, IV methylprednisolone (Solu-Medrol) and cyclophosphamide 2 mg/kg PO daily.

8. How does lisinopril work to reduce blood pressure? a. Increases the heart rate. b. Increases preload and afterload. c. Causes systemic vasoconstriction. d. Prevents the conversion of angiotensin I to

angiotensin II. 9. What nursing considerations are important when giving

lisinopril? 10. What are the expected outcomes of furosemide (Lasix)

therapy? Select all that apply. a. Diuresis of excess fluid b. Reduced blood pressure c. Decreased BUN and creatinine levels d. Increased systemic vascular resistance e. Increased water, sodium, and potassium excretion

11. What do you need to monitor while G.W. is on a furosemide (Lasix) infusion?

12. Which findings would indicate potential adverse effects of a furosemide (Lasix) infusion? Select all that apply.

a. Tinnitus

b. Dizziness c. Weakness d. Dry mouth e. Increased blood pressure

13. Cyclophosphamide comes in 50 mg tablets. G.W. weighs

110 pounds (50 kg). How many 50 mg tablets will he receive for each daily dose of cyclophosphamide?

14. Discuss at least 3 nursing interventions that are important while the patient is on cyclophosphamide therapy.

Case study progress Orders for G.W. include fluid restriction and a “renal diet.” The dietitian visits G.W. to discuss the changes to his diet.

15. Which of these reflect a renal diet? Select all that apply. a. High protein diet b. Reduced salt intake c. Increased potassium intake d. Reduced phosphorus intake e. Taking calcium supplements

16. Discuss the rationale behind the fluid restriction and renal diet.

Case study outcome After 3 days, G.W.’s creatinine and BUN remained elevated with continued hypertension, edema, and decreased urine output. He was started on hemodialysis for management of renal function and the Solu-Medrol was changed to PO prednisone. He remained in the hospital for 3 weeks before being transferred to a rehabilitation facility.

Case Study 72 Name _________________________________ Class/Group

_____________________________ Date _____________

Scenario K.B. is a 32-year-old woman being admitted to the medical floor for fatigue and dehydration. While taking her history, you discover that she has diabetes mellitus (DM) and has been insulin dependent since the age of 8. She has undergone hemodialysis (HD) for the past 2 years because of end-stage renal disease (ESRD). Your initial assessment of K.B. reveals a pale, thin, slightly drowsy woman. Her skin is warm and dry to the touch with poor skin turgor, and her mucous membranes are dry. Her vital signs are 140/88, 116, 18, 99.9° F (37.7° C). She tells you she has been nauseated for 2 days so she has not been eating or drinking. She reports severe diarrhea. The following blood chemistry results are back.

Chart View

Laboratory Test Results

Sodium 145 mEq/L (145 mmol/L)

Potassium 6.0 mEq/L (6.0 mmol/L)

Chloride 93 mEq/L (93 mmol/L)

Bicarbonate 27 mEq/L (27 mmol/L)

BUN 48 mg/dL (17.1 mmol/L)

Creatinine 5.0 mg/dL (442 mcmol/L)

Glucose 238 mg/dL (13.2 mmol/L)

1. What aspects of your assessment support her admitting diagnosis of dehydration?

2. Explain any lab results that might be of concern.

3. Identify 2 possible causes for K.B.’s low-grade fever.

Case study progress The rest of K.B.’s physical assessment is within normal limits. You note that she has an arteriovenous (AV) fistula in her left arm.

4. What is an AV fistula? Why does K.B. have one? 5. What steps do you take to assess K.B.’s AV fistula, and what

physical findings are expected? Explain. 6. As you continue the assessment, you notice that the UAP

comes in to take K.B.’s BP. The UAP places the BP cuff on K.B.’s left arm. What, if anything, do you do?

Case study progress K.B.’s admission CBC yields the following results:

Chart View

Laboratory Test Results

WBC 7600/mm3 (7.6 x 109/L)

RBC 3.2 million/mm3 (3.2 x 1012/L)

Hgb 8.1 g/dL (81 g/L)

Hct 24.3%

Platelets 333,000/mm3 (330 x 109/L)

7. Are these values normal? If not, what are the abnormalities? 8. K.B.’s physician notes that she is anemic, which most likely is

the cause of her increasing fatigue. Why is K.B. anemic?

Case study progress K.B. is sent for an HD treatment. Over the next 24 hours, K.B.’s nausea

subsides, and she is able to eat normally. While you are helping her with her morning care, she confides in you that she does not understand her diet. “I just get blood drawn every week and meet with the dialysis dietitian every month—I just eat what she tells me to eat. It’s so hard!”

9. Because K.B. is on HD and has DM, who would be the best resource to help her with her nutrition questions?

10. What are her special nutritional needs? Name at least 4 specific components of the diet recommended for K.B.

11. Which of these statements about fluid intake for dialysis patients is correct? Select all that apply.

a. There are no fluid restrictions for dialysis patients. b. Dialysis patients need to restrict fluid intake to

water only. c. Fluids are restricted to 600 mL plus the amount

equal to the previous day’s urine output. d. A fluid is anything that is liquid at room

temperature, such as coffee, popsicles, and ice cream.

e. Hypertension may occur if large amounts of fluid are retained between dialysis treatments.

12. Patients in renal failure have the potential to develop comorbid conditions. Identify 5 potential problems, determine how you would assess the problem, then delineate nursing interventions and patient education strategies for each.

Case study progress The following day, K.B. is discharged feeling much better and with a good understanding of her dietary restrictions. Her iron stores have been evaluated and found to be low. Her physician has instructed her to resume her preadmission medications, with the addition of ferrous fumarate oral suspension 100 mg PO bid between meals with water, if tolerated (or with meals if GI distress occurs) and epoetin (Epogen) to be given 3 times a week intravenously with dialysis. She is also given

a prescription for Nephrocaps vitamin supplements to be taken daily.

13. Explain the purpose of the new medications for K.B. 14. You spend some time with K.B. to explain the new

medications. Using the Teach-Back technique, you ask K.B. to explain what she has learned. Which statement by K.B. reflects need for further teaching?

a. “The liquid iron will cause my bowel movements to turn black or dark green.”

b. “I should dilute the liquid iron and drink it with a straw so that it won’t stain my teeth.”

c. “I won’t need to take the iron supplements as long as I get the Epogen during dialysis.”

d. “Hopefully I will feel less tired all the time when these medicines start building up my red blood cells.”

15. K.B. asks, “Why do I need a prescription for vitamins? I can just take something on sale at the drugstore, right?” How do you respond?

16. The ferrous fumarate suspension comes in a bottle that is

labeled 100 mg/5 mL. Indicate on the measuring cup how much medication will be used for each dose.

17. K.B. asks you, “I can just use a teaspoon to measure this

medicine when I’m at home, right?” What is your response? 18. In monitoring K.B.’s response to the epoetin, what adverse

effects would you expect? Select all that apply. a. Diarrhea b. Headache c. Arthralgia d. Drowsiness e. Tachycardia f. Hypertension

19. Which vital sign will you monitor carefully while K.B. is on epoetin therapy? Explain your answer.

20. During the following weeks, which lab result is most

important to monitor while K.B. is on the epoetin? Explain.

Case study outcome K.B. is discharged to home and goes to the local dialysis center 3 times a week. She also keeps appointments with the registered dietitian and reports that she is feeling much better.

Case Study 73 Name _________________________________ Class/Group _____________________________ Date _____________

Scenario You are a registered nurse in the emergency department (ED). It is a hot summer day and S.R., a 25-year-old woman, comes to the ED with severe left flank and abdominal pain and nausea with vomiting. S.R. looks very tired, her skin is warm, and she is perspiring. She paces about the room doubled over and is clutching her abdomen. S.R. tells you the pain started early this morning and has been pretty steady for the past 6 hours. She gives a history of working outside as a landscaper and takes little time for water breaks. Her past medical history includes three kidney stone attacks, all occurring during late summer. Her abdomen is soft and without tenderness, but her left flank is extremely tender to touch. You place S.R. in one of the examination rooms and take the following vital signs: 188/98, 90, 20, 99° F (37.2° C). A voided urinalysis shows RBCs of 50 to 100 on voided specimen and WBCs of zero.

1. What could be the cause of the blood in her urine? How could you rule out some of these causes?

2. The physician orders an intravenous pyelogram (IVP). What questions do you need to ask S.R. before the test is conducted?

What blood test results do you need to check before she has an IVP?

3. S.R. states she had an allergic reaction during her last IVP and was instructed, “Don’t let anyone give you dye for any testing.” The physician cancels the IVP. What alternative test will be conducted?

Case study progress The noncontrast CT scan shows a left 2-mm ureteral vesicle junction stone.

4. What are the most common types of stones? Select all that apply.

a. Cystine b. Struvite c. Uric acid d. Calcium oxalate e. Calcium phosphate

5. What is the most likely cause of S.R.’s stone? 6. What is a possible complication if S.R.’s stone is not removed?

a. Trabeculation b. Hydronephrosis c. Nephrosclerosis d. Nephrotic syndrome

7. Identify 2 methods of treating a patient with a ureteral vesicle junction stone.

Case study progress S.R. was discharged with instructions to strain all urine and return if she experienced pain unrelieved by the pain medication or increased nausea and vomiting.

8. What specific instructions will you give S.R. about straining her urine, fluid intake, medications, and activity?

Case study progress S.R. returns to the ED in 6 hours with pain unrelieved by the pain medication and increased blood in her urine. She is being held in the ED until she can be transported to surgery.

9. What is the immediate plan of care for S.R.?

Case study progress A 2-mm calculus was removed by basket extraction. Pathologic examination reported the stone to be calcium oxalate.

10. If S.R. continues to form calcium oxalate stones, what recommendations would the physician make for S.R.?

11. Because S.R.’s stone has been reported as calcium oxalate, she is referred to a registered dietitian for guidance on a diet that will prevent further development of stones. Which statements are true regarding recommendations for S.R.’s diet? Select all that apply.

a. Decrease fat intake. b. Decrease sodium intake. c. Drink at least 3 to 4 liters of water each day. d. Avoid eating organ meats, bacon, citrus fruits, and

red wine. e. Avoid chocolate, spinach, tomatoes, dried fruits,

beans, sardines, and nuts. 12. After an education session, you use the Teach-Back technique

to assess S.R.’s understanding of her new dietary guidelines. Which statement by S.R. indicates a need for further teaching?

a. “I will avoid coffee, cocoa, and chocolate.” b. “I will not add salt to my food when cooking.” c. “I will drink at least 3 liters of water each day.” d. “I will increase my intake of milk and cheese

daily.”

Case study outcome

S.R. recovers from this most recent episode and continues to follow the protocol for fluid intake and dietary measures. One year later, she has yet to report a recurrence of stones.

Case Study 74 Name _________________________________ Class/Group _____________________________ Date _____________

Scenario You are working on a telemetry unit and have just received a transfer from the ICU. The 70-year-old male patient, T.A., is postoperative day 2 after three-vessel coronary bypass graft surgery. He has a history of hypertension, hyperlipidemia, and type 2 diabetes mellitus requiring insulin for the past 6 months to control glucose levels. The ICU nurse tells you that there were complications during surgery and he received 3 units of blood to treat hypotension. Since surgery, T.A. has experienced intermittent atrial fibrillation that is under control with amiodarone and metoprolol. The nurse voices concern his urine output seems to be decreasing.

1. Four hours after his admission to your floor, you note that T.A. has had a total urine output of 75 mL of dark amber urine. Why are you concerned?

2. You check the urinary catheter and tubing for obstructions and find none. What other assessments do you need to gather?

Case study progress You notify the surgeon of the decreased urine output. The surgeon orders a stat electrolyte panel and asks you to call with the results.

Chart View

Laboratory Test Results

Potassium 5.8 mEq/L (5.8 mmol/L)

Sodium 132 mEq/L (132 mmol/L)

Glucose 224 mEq/L (12.4 mmol/L)

BUN 86 mg/dL (30.7 mmol/L)

Creatinine 4.4 mg/dL (389 mcmol/L)

3. Interpret T.A.’s lab results. 4. What actions do you need to take because of the serum

potassium level?

Case study progress

Chart View

Medication Administration Record

Dopamine IV infusion at 2 mcg/kg/min Furosemide 80 mg IV push daily Sodium polystyrene sulfonate (Kayexalate) 1 gram PO twice

daily Sevelamer hydrochloride (Renagel) 800 mg PO with meals

5. The surgeon writes new orders. Identify the expected outcome associated with each medication he will be receiving.

6. T.A. weighs 164 lbs. The pharmacy-supplied IV bag reads

“dopamine 400 mg/250 mL.” Calculate the hourly rate for the dopamine infusion. Round to the tenth.

Case study progress The surgeon determines that T.A. is in the oliguric phase of acute

kidney injury (AKI). T.A. is sent to radiology for placement of a dialysis catheter.

7. What is the likely reason T.A. developed AKI? 8. The RIFLE criteria delineate the three stages of AKI based on:

a. Glomerular filtration rate (GFR) b. Serum creatinine and urine output c. Urine osmolality and specific gravity d. Blood pressure and BUN/creatinine ratio

9. You decide to assess T.A. for indications of AKI. What do these include?

10. What are your priority nursing problems right now? 11. The dialysis catheter is inserted into T.A.’s left subclavian

vein. You are preparing to give the IV furosemide and find that his only other IV access, a peripheral line, is the site of the dopamine infusion. What are your options?

12. T.A. asks if he is going to be on dialysis for the rest of his life. How would you respond?

13. T.A. is placed on a fluid restriction and a renal diet. T.A. asks how much he is going to be able to drink. What is your reply?

14. Briefly describe a renal diet. 15. What referral may be needed and why? 16. What are some interventions you can use to help T.A. be more

comfortable while on a fluid restriction? 17. As you plan your care of T.A. for the rest of the shift, identify

which aspects of his care you can delegate to the UAP. Select all that apply.

a. Measure vital signs every 2 hours b. Assist him with oral hygiene as needed c. Obtain T.A.’s glucose level before dinner d. Monitor T.A.’s lung sounds every 4 hours e. Obtain and record an accurate daily weight f. Evaluate T.A.’s I/O trends for the past 48 hours

18. You note that T.A.’s postoperative blood glucose levels range from 62 to 387 mg/dL (3.4 to 21.6 mmol/L). He comments,

“That’s funny, you’re giving me almost twice the amount of insulin that I give myself at home. I don’t understand why it’s not working.” How should you respond?

19. In addition to ongoing assessment, describe nursing

interventions to place in T.A.’s plan of care that are part of patient safety initiatives aimed at minimizing his risk for a VTE developing.

Case study progress The next morning, T.A. is scheduled for his first dialysis treatment at 0800.

20. What type of assessment data do you need to gather before his dialysis treatment?

21. Doses of IV amiodarone, metoprolol, and furosemide are scheduled for 0800. What should you do?

a. Give all three medications 1 hour before dialysis b. Hold all three medications and notify the surgeon c. Give the amiodarone and hold the metoprolol and

furosemide d. Hold all three medications and give them

immediately upon return 22. T.A. is off the unit 4 hours for therapy. When he returns, what

assessments do you need to make? 23. Shortly after his return, T.A. tells you he has a headache and

severe nausea. He is restless and slightly confused, and his BP is 180/102. What is the significance of these findings?

24. You page the surgeon. What will you do while waiting for a return call?

Case study outcome T.A. recovers from the episode of disequilibrium syndrome. During the rest of his hospitalization, he continues to have trouble maintaining fluid and electrolyte balance between his dialysis

treatments. He spends two weeks in a rehabilitation center before being discharged on dialysis and with home health. He eventually regains kidney function.

Case Study 75 Name _________________________________ Class/Group _____________________________ Date _____________

Scenario W.V. is a 51-year-old man who lives with his wife and two teenage sons. W.V. developed chronic kidney disease 15 years ago after using a drug for migraine headaches that was later shown to cause severe nephrotoxicity. W.V. underwent hemodialysis for 5 years before receiving a cadaveric transplant, or a cadaver kidney. He recovered without complications, and his serum lab values returned to normal. He started immunosuppression therapy with prednisone and tacrolimus before his discharge to home.

1. Why are kidney transplants done? 2. What histocompatibility studies are usually done before a

transplant? Why are they important? 3. By what criteria was W.V. considered a suitable candidate for a

kidney transplant? 4. True or False? During transplant surgery, the old kidney is

surgically removed and the new one is sewn in its place. Defend your response.

5. Why is W.V. receiving prednisone and tacrolimus? 6. Name 4 ways in which W.V. might have difficulty adjusting

after an organ transplant. 7. If W.V.’s kidney is producing enough urine and he is feeling

well, why is ongoing lab monitoring necessary? 8. Why should W.V. be concerned about infection? 9. Which statement would show W.V. needs further teaching

regarding posttransplant care and immunosuppressant therapy?

a. “I should wash my hands often.” b. “I will need to have regular lab testing.” c. “I will call the doctor if I urinate less frequently.” d. “It will be nice to go to all of my grandkids’

activities now.”

Case study progress Today, W.V. reports to his provider for a 12-week follow-up. W.V. has gained 5 pounds since his last appointment 2 weeks ago.

Chart View

Vital Signs

Blood pressure (BP) 148/82

Pulse rate 88

Respiratory rate 24

Temperature 99.2° F (37.3° C)

Laboratory Test Results

Sodium 148 mmol/L (148 mmol/L)

Potassium 4.0 mmol/L (4.0 mmol/L)

Glucose 198 mg/dL (11.0 mmol/L)

Calcium 10.1 mg/dL (2.5 mmol/L)

Creatinine 2.2 mg/dL (194 mcmol/L)

Blood urea nitrogen (BUN) 42 mg/dL (15.0 mmol/L)

10. What is the possible significance of W.V.’s BP? 11. Interpret W.V.’s lab results.

Case study progress The provider suspects W.V. is experiencing acute rejection and orders a renal biopsy, which confirms a diagnosis of acute rejection.

12. What other signs and symptoms may be present with acute rejection?

13. Explain the pathophysiology of acute rejection. 14. What are the collaborative care options to save the kidney

when rejection is present? 15. The provider decides to add mycophenolate (CellCept) 1 gram

orally twice daily to W.V.’s immunosuppressive regimen. How does mycophenolate help protect W.V.’s kidney?

16. What do you need to teach W.V. about mycophenolate therapy?

17. Glipizide is prescribed for W.V.’s hyperglycemia. W.V. asks if this means he is now a diabetic. How would you answer him?

18. W.V. asks you if this means he is going to lose the kidney and go back on dialysis. How would you respond?

19. How can you best support W.V. and his family during this time?

Case study outcome W.V. does not experience any further episodes of acute rejection and within 6 months is able to be on lower doses of immunosuppressive therapy. With the lower level of immunosuppression, his elevated glucose resolves and he does not develop diabetes. His kidney continues to function well, and he says that despite a few challenges that come with being a posttransplant patient, “I feel better than I have in years.”

6

Intracranial Regulation

Case Study 76 Name _________________________________ Class/Group _____________________________ Date _____________

Scenario L.C. is a 78-year-old man with a 3-year history of Parkinson disease (PD). He is a retired engineer, married, and living with his wife in a small farming community. He has 4 adult children who live close by. Since his last visit to the clinic 6 months ago, L.C. reports that his tremors are “about the same” as they were. However, further questioning reveals that he feels his gait is a little more unsteady and his fatigue is slightly more noticeable. L.C. is also concerned about increased drooling. Among the medications L.C. takes are carbidopa- levodopa 25/100 mg (Sinemet) and pramipexole (Mirapex) 0.5 mg, each three times daily. On the previous visit the Sinemet was increased from 2 to 3 times daily. He reports that he has become very somnolent with this regimen and that his dyskinetic movements appear to be worse just after taking his carbidopa-levodopa (Sinemet).

1. What is PD? 2. What is parkinsonism? 3. What are the clinical manifestations of PD? Underline the

symptoms L.C. has mentioned. 4. L.C.’s wife asks you, “How do the doctors know he really has

Parkinson disease? They never did a lot of tests on him.” How

is PD diagnosed? 5. L.C. asks, “Why don’t they give me a dopamine pill? Wouldn’t

that just fix everything?” Why is oral dopamine not a replacement therapy?

6. Why is levodopa given in combination with carbidopa? 7. Why did L.C.’s dyskinetic movements appear to be worse just

after taking carbidopa-levodopa? What changes to his medication therapy may be needed?

8. Because L.C. takes Sinemet, what serious adverse effect should you assess for in him?

a. Suicidal thoughts b. Permanent hearing loss c. Steven-Johnson syndrome d. Spontaneous tendon rupture

9. L.C.’s wife asks, “They can do surgery for everything else. Why can’t they do some kind of surgery to fix Parkinson disease?” How would you describe the surgical treatments available for patients with PD?

Case study progress After examining L.C., the provider decides not to hospitalize him but to decrease the dosage of Sinemet. He tells L.C. and his wife that he thinks L.C. is likely experiencing some advancement in his disease and says that it is time for some changes in L.C.’s care. The provider looks at you and asks you to coordinate the “usual referrals.”

10. What interprofessional team members would be involved in L.C.’s care and how?

11. What factors do you need to take into consideration when helping L.C. with these referrals?

12. L.C. is reporting an increase in drooling, and you are

concerned about his ability to swallow. What further assessment could you perform to determine whether L.C. is at immediate risk for aspirating?

13. What are 3 nutrition interventions that should be implemented for L.C.?

14. Because L.C. is reporting that his gait is more unsteady,

there is an increased risk for falls. Which suggestion could you offer to diminish this risk?

a. Only use a wheelchair to get around b. Use a bag or backpack to carry objects c. Stand as upright as possible and use a walker d. Keep the feet close together while ambulating

15. What are 3 suggestions you can make to L.C. to help manage fatigue?

16. You are giving instructions to L.C. and his wife about

ambulating safely. You determine that they understand the directions if they say that L.C. will:

a. Schedule his PT appointments in the evening b. Sit on a large, soft sofa with supportive pillows c. Use a step stool to obtain difficult-to-reach items d. When rising from a seat, rock back and forth to

start moving 17. As L.C.’s case manager, identify 5 things that you would need

to assess to determine whether L.C. could be cared for in his home.

Case study outcome L.C. starts a multifaceted speech, OT, and PT program for persons with Parkinson disease. After 8 weeks, his gait is steadier and he is using several strategies to manage his fatigue and drooling.

Case Study 77 Name _________________________________ Class/Group _____________________________ Date _____________

Scenario N.T., a 79-year-old woman, arrives at the emergency department with expressive aphasia, left facial droop, left-sided hemiparesis, and mild dysphagia. Her husband states that when she awoke that morning at 0700, she stayed in bed, saying she had a mild headache over the right temple and was feeling weak. He went and got coffee, then thinking it was unusual for her to stay in bed, went back to check on her. He found she was having trouble saying words and had a left-sided facial droop. When he helped her up from the bedside, he noticed weakness in her left hand and leg and brought her to the emergency department. Her medical history includes atrial fibrillation, hypertension, and hyperlipidemia. A recent cardiac stress test was normal, and her blood pressure is under good control. N.T. is currently taking amiodarone, amlodipine (Norvasc), aspirin, simvastatin (Zocor), and lisinopril (Zestril). The provider suspects N.T. is having an acute cerebrovascular accident (CVA).

1. What role do diagnostic tests play in evaluating N.T. for a suspected CVA?

2. Explain how knowing the type of CVA is an important factor in planning care.

3. Which factor in N.T.’s history is the most likely contributor to her having a CVA and why?

4. The primary factor influencing the manifestations of a CVA is the:

a. Area of the brain affected b. Speed of onset of the CVA c. Amount of brain tissue affected d. Type of CVA the patient experienced

5. What are the common manifestations of a CVA? 6. How should you position N.T.? 7. Outline the focused assessment you need to obtain.

Case study progress

Your assessment findings are as follows: VS are 164/98, 94, 24, 97.2° F (36.2° C), Sao2 94% on room air. Her lungs are clear, and she is alert and oriented. She is able to follow simple commands, has PERRL with intact extraocular movements, and no vison loss. Her facial movements are asymmetrical, with left-sided drooping. Speech is slightly slurred, although it remains intelligible. She is unable to move her left arm and leg; sensation is intact. There is no ataxia; however, she is experiencing some visual and tactile neglect of the left side.

8. Complete the National Institutes of Health Stroke Scale (NIHSS) scores for each of N.T.’s symptoms.

Symptom Score

Alert

Knows month and age

Able to follow commands

Extraocular movements (EOMs) intact

No visual loss

Partial left facial paralysis

Left leg no movement

Left arm no movement

No ataxia

Sensation intact

Moderate aphasia

Neglect of left side

TOTAL SCORE

9. Based on your scoring, what level of CVA did N.T. experience? 10. There are a number of manifestations of a CVA. Match the

description of various losses with the term describing the loss:

_____A. Alexia 1. Total inability to communicate

_____B. Wernicke aphasia

2. Difficulty articulating words

_____C. Dysarthria

3. Inability to perform purposeful movements in the absence of motor problems

_____D. Apraxia 4. Sentences contain words that are irrelevant or non- existent

_____E. Agraphia 5. Loss of the ability to read

_____F. Agnosia 6. Inability to recognize familiar objects

_____G. Global aphasia

7. Loss of the ability to write

Case study progress A noncontrast CT scan confirms the diagnosis of a thrombolytic CVA. The provider writes the orders shown in the chart.

Chart View

Physician’s Orders

IV 0.9% NaCl at 75 mL/hr Activase (tPA) per protocol Stat CBC, PT/INR, CPK isoenzymes Neurologic assessment every hour Obtain patient weight VS every hour O2 at 2 L per nasal cannula (NC) NPO until swallowing evaluation

11. Outline a plan of care for implementing these orders. 12. Which interventions can you delegate to the UAP? Select all

that apply. a. Obtaining N.T.’s weight b. Obtaining a manual BP per protocol c. Initiating O2 therapy by nasal cannula d. Assisting N.T. in repositioning every 2 hours e. Performing N.T.’s neurologic checks every hour

13. What is the purpose of monitoring the CK isoenzyme levels? 14. The instructions on the tPA vials read to reconstitute with

50 mL of sterile water to make a total of 50 mg/50 mL (1 mg/mL). The hospital protocol is to infuse 0.9 mg/kg over 60 minutes with 10% of the dose given as a bolus over 1 minute. N.T. weighs 143 pounds. What is the amount of the bolus dose, in both milligrams and milliliters, you will give in the first minute? What is the amount of the remaining dose you will need to give?

15. Contraindications for beginning fibrinolytic therapy include which of the following? Select all that apply.

a. Systolic BP of 150 b. Worsening neurologic status c. Major surgery in the last 14 days d. Platelet count of less than 100,000 (100 x 109/L) e. Blood glucose of less than 50 mg/dL (2.8 mmol/L) f. Currently on warfarin with an INR of 1.4 g. History of myocardial infarction 3 months ago

16. What are your responsibilities during the administration of Activase (tPA)?

17. What signs and symptoms would alert you to the possible presence of an intracerebral hemorrhage during the tPA infusion?

Case study progress N.T. is admitted to the neurology unit. A second CT scan 24 hours

later reveals a small CVA in the right hemisphere. She is placed on aspirin, amiodarone, amlodipine (Norvasc), clopidogrel (Plavix), simvastatin (Zocor), and lisinopril (Zestril).

18. During the first 24 hours after Activase (tPA), the primary concern is controlling N.T.’s:

a. Glucose level b. Blood pressure c. Cardiac rhythm d. Oxygen saturation

19. Why was N.T. placed on clopidogrel (Plavix) post-CVA? 20. Because N.T. had a thrombolytic infusion, how many hours

had to pass before starting any anticoagulant or antiplatelet drugs?

21. Is there any benefit from continuing simvastatin? 22. While assessing N.T., you note the following findings. Which

one is unrelated to the CVA? a. Lethargy b. Headache c. Lumbar pain d. Blurred vision

23. As you walk into the nurses’ station, the charge nurse is coordinating the swallowing evaluation, including a modified barium swallow study and referral for a speech-language pathologist (SLP). Give the rationale for these orders.

24. If N.T.’s deficits are temporary, how long might it take before they completely reverse?

Case study outcome After spending 1 week on the neurology unit, N.T. is discharged to a rehabilitation center for continued therapy. Though she is still experiencing some mild deficits, she is able to go home after 6 weeks.

Case Study 78

Name _________________________________ Class/Group _____________________________ Date _____________

Scenario T.H. is a 55-year-old man with an 8-month history of progressive muscle weakness. Initially, he tripped over things and seemed to drop everything. He lost interest in activities because he was always exhausted. He sought medical care when his speech became slurred and he started to drool. During the initial evaluation, the provider noted frequent, severe muscle cramps, muscle twitching, and inappropriate, uncontrollable periods of laughter. After undergoing a series of tests, T.H. was diagnosed with amyotrophic lateral sclerosis (ALS). He is upset and bewildered about a disease that he has “never even heard of.” You are a home health nurse who is seeing T.H. for the first time.

1. How would you explain ALS to T.H.? 2. Who gets ALS? 3. How common is ALS? 4. What are the early manifestations of ALS? 5. What do patients with multiple sclerosis, myasthenia gravis,

and ALS have in common? a. Each is caused by a deficiency of essential

neurotransmitters b. Patients will experience a complete recovery after

several months c. The diseases place the patient at higher risk for

respiratory complications d. Manifestations include hyperactive deep tendon

reflexes and muscle twitching 6. T.H. has many questions. He asks you, “How long can I expect

to live?” How should you respond? 7. T.H. asks, “Will I lose my mind?” 8. T.H. then asks, “Are there any treatments for this?”

9. T.H. thinks a moment, then says, “How is the doctor even sure this is what I have?” What is your response?

10. As part of this initial visit, you will begin to coordinate care with speech, occupational, respiratory, and physical therapists, as well as a dietitian and a psychologist. Describe the role that each of these professionals will play in T.H.’s treatment.

11. Which actions will support communication among T.H.’s

care providers? Select all that apply. a. Maintaining one central medical record b. Designating the physician as the team leader c. Having open communication among team members d. Holding periodic team conferences to communicate

goals e. Inviting T.H. and his caregiver to take part in team

conferences 12. You hold a family meeting to recruit adequate help for the

caregiver—in this case, T.H.’s wife. Why is this important? 13. What are some suggestions you can give T.H.’s wife to help

her reduce caregiver strain? 14. How would you best determine whether T.H.’s wife was

experiencing caregiver strain? a. Ask how well T.H. thinks his spouse is caring for

him b. Assess the caregiving situation and health of T.H.

and his wife c. Evaluate his wife for any new symptoms of anxiety

and depression d. Determine whether his wife feels overwhelmed by

her responsibilities 15. T.H. asks you, “How will the end probably come for me?”

What should you tell him? 16. T.H. wants to know whether he “has to be put on a breathing

machine.” What factors will you take into consideration when deciding what to tell him?

17. Which legal document should T.H. formulate to describe his wishes about being placed on a “breathing machine”? Give your rationale.

a. Living will b. Living trust c. Standard will d. Health care power of attorney

Case study outcome T.H. stays at home and you continue to visit him weekly for the next 20 months. After aspirating, he develops pneumonia and after deciding not to be placed on mechanical ventilation, passes away surrounded by his family.

Case Study 79 Name _________________________________ Class/Group _____________________________ Date _____________

Scenario J.G. is a 34-year-old woman who underwent an emergency cesarean delivery after a prolonged labor, during which she had a sudden change in neurologic functioning and had a tonic-clonic (grand mal) seizure. After delivery, J.G. had 2 more seizures and demonstrated dyskinesia, resulting in frequent falls when ambulating. She was diagnosed with a basal ganglion hematoma with infarct and started on phenytoin (Dilantin). Once the seizure disorder appeared to be under control, she was transferred to a rehabilitation facility for evaluation and 2 weeks of intensive physical therapy (PT). She is now home. She still has occasional falls but has had no seizures. She is receiving PT 3 times a week in her home. As case manager for J.G.’s health maintenance organization, you make a home visit with her and her family for evaluation of long-term follow-up care.

1. A seizure is not a disease in itself but a symptom of a disease. What is the term for chronically recurring seizures?

2. Does J.G. have epilepsy? 3. In addition to the brain injury, what are some other possible

conditions that could be contributing to J.G.’s lowered seizure threshold?

4. What is the pathophysiology of a seizure? 5. J.G. had tonic-clonic, or grand mal, seizures. Describe this type

of seizure. 6. They ask how phenytoin (Dilantin) works in preventing

seizures. How would you respond? 7. What factors are considered when determining which seizure

medication a patient should take? 8. J.G. tells you she is having trouble remembering to take

her medication. Why does this concern you? 9. What are some strategies you could suggest to J.G. and

her husband to help her with remembering to take the phenytoin?

10. You check the chart and note that J.G.’s last phenytoin level was 12.7 mcg/mL (50.3 mcmol/L). What action do you expect based on this level?

a. Because this level is on the border of therapeutic, notify the neurologist.

b. This level is dangerously high, and an immediate reduction in dose is necessary.

c. J.G. is at immediate risk for a seizure so she should go to the emergency department.

d. Because this level is within normal limits, J.G. would continue therapy as prescribed.

11. J.G. asks, “Will my blood levels stay under control as long as I take my medicine?” How would you answer her question?

12. J.G.’s husband asks if the phenytoin could harm his wife in any way. What general information would you review with them about phenytoin?

13. J.G. says that because she has not had a seizure since she was in the hospital, she questions how long she will have to continue taking the phenytoin. Which is your best response?

a. “Your seizures are cured only as long as you take the medication.”

b. “This medication might need to be continued for the rest of your life.”

c. “This medication can be stopped after you are seizure free for 6 months.”

d. “This medication will have to be taken only when you are experiencing stress.”

14. J.G.’s husband asks you what he should do if she has a

seizure at home. What should you teach him? 15. Her husband states that he is afraid for J.G. to take care of the

baby. What would you say to him? 16. What aspects of the home environment do you need to

inspect and why? 17. Describe safety measures you can teach J.G. that will

minimize her risk of injury should she have a seizure. 18. You would determine further teaching is needed

regarding modifying their home environment to reduce J.G.’s risk of falling if J.G. or her husband states:

a. “The decorative rugs are all going to be put into storage.”

b. “We will put some nonskid strips in the shower in the master bath.”

c. “We will keep the stairway free of clutter and turn the light on as needed.”

d. “J.G. will need some new socks to wear so she is not going barefoot indoors.”

Case study outcome

J.G. continued on the phenytoin for 14 months, and then she chose to stop taking it so she could have another child. She had no further seizures until after she delivered her next child, at which time she experienced a grand mal seizure. She resumed the phenytoin and has remained seizure-free.

Case Study 80 Name _________________________________ Class/Group _____________________________ Date _____________

Scenario J.B. is a 58-year-old retired postal worker who has been on your floor for several days receiving plasmapheresis every other day for myasthenia gravis. About a year ago, J.B. started having difficulty chewing and swallowing, diplopia, and slurred speech, at which time he was placed on pyridostigmine (Mestinon). Before this admission he had been relatively stable. His other medical history includes hypertension controlled with metoprolol (Lopressor) and glaucoma treated with timolol (ophthalmic preparation). Recently J.B. had a sinus infection and was treated with ciprofloxacin (Cipro). On admission, J.B. was unable to bear any weight or drink fluids through a straw. There have been periods of exacerbation and remission since admission.

Chart View

Vital Signs

Blood pressure 170/68

Heart rate 118

Respiratory rate 32

Temperature 101.8° F (38.8° C)

1. You note that the UAP has just entered these vital signs into J.B.’s record. What is your immediate concern and why?

2. What action do you need to take based on this concern? 3. What other assessment findings would support this

complication being present? 4. What is the physiologic difference between a cholinergic crisis

and myasthenic crisis? 5. What medical treatment do you anticipate for J.B.? 6. What is your nursing priority at this time? 7. Based on this priority, what nursing interventions do you need

to perform? 8. Which actions do you need to implement to give

edrophonium safely? Select all that apply. a. Have IV atropine sulfate readily available. b. Place J.B. on continuous cardiac monitoring. c. Initiate precautions to prevent excessive bleeding. d. Give an as-needed antiemetic drug before injection. e. Monitor for any changes in his level of

consciousness. 9. J.B.’s wife asks you, “What may have caused my husband to

get worse, and why does he keep having these episodes?” What explanation should you give her?

Case study progress J.B.’s condition improves after receiving edrophonium and IV gamma globulin. Two days later, after he is stable, you sit down to discuss discharge plans with J.B. and his wife.

10. J.B.’s wife tells you she does not have a lot of information about myasthenia gravis (MG) and she would like to know more about it. What should you tell her?

11. They ask you to explain what to expect in terms of symptoms

as his illness progresses. What should you tell them? 12. J.B.’s wife asks, “How do they know my husband has

myasthenia gravis?” What should you tell her about how MG is diagnosed?

13. J.B.’s wife asks why he received plasmapheresis. Which statement best describes the purpose of this procedure? Plasmapheresis:

a. replaces affected blood with unaffected blood. b. decreases the production of antireceptor antibodies. c. reduces inflammation by infusing

immunoglobulins. d. removes circulating abnormal antibodies from the

blood. 14. J.B. wants to know when he will be able to go home. How will

you respond? 15. J.B.’s wife asks you what information they will need

before he goes home. What do you need to teach J.B. about taking pyridostigmine?

16. Outline other points you need to teach J.B. and his wife about managing myasthenia gravis.

17. You teach J.B. and his wife that the most effective means of preventing myasthenic and cholinergic crises is:

a. Doing all errands early in the day b. Eating three large, well-balanced meals c. Taking medications at the same time each day d. Doing muscle-strengthening exercises twice a day

18. How will you know that your teaching has been effective? 19. What community resources might J.B. and his wife find

helpful?

Case study outcome J.B. and his wife thank you for taking the time to sit down with them. They state that although they know they have a lot to deal with, and it won’t be easy, they feel that they are better prepared to cope with

myasthenia gravis having the information you shared.

Case Study 81 Name _________________________________ Class/Group _____________________________ Date _____________

Scenario Z.O. is a 3-year-old boy with no significant medical history. He is brought into the emergency department by the emergency medical technicians after experiencing a seizure that lasted for 3 minutes. His parents report no previous history that might contribute to the seizure. On questioning, they state that they have noticed that he has been irritable, has had a poor appetite, and has been clumsier than usual over the past 2 to 3 weeks. Z.O. is admitted for diagnosis and treatment for a suspected brain tumor. A magnetic resonance imaging (MRI) scan of the brain shows a 1-cm mass in the posterior fossa region of the brain, and Z.O. is tentatively diagnosed with a cerebellar astrocytoma. The tumor appears to be contained, and the treatment plan will consist of a surgical resection, with a definitive diagnosis determined by histologic examination of tissue obtained during surgery. The type of tumor and grading will determine further treatment.

1. What are common presenting symptoms of a brain tumor? Select all that apply.

a. Pallor b. Ataxia c. Diarrhea d. Seizures e. Vomiting with eating f. Headaches, especially on awakening

2. Explain the reason that a brain tumor can cause the signs and

symptoms listed in Question #1. 3. Outline a plan of care for Z.O., describing at least 2 nursing

interventions that would be appropriate for managing fluid status, providing preoperative teaching, facilitating family coping, and preparing Z.O. and his family for surgery.

Case study progress Z.O. returns to the pediatric intensive care unit after surgery. He is arousable but cannot answer questions. His pupils are equal and reactive to light. He has a head dressing covering the entire scalp with small amount of serosanguinous drainage. His IV is intact and infusing as ordered through a new central venous line. His breath sounds are equal and clear, and SpO2 is 98% on room air. You get him settled in his bed and leave the room.

Chart View

Postoperative Orders

Vital signs every 15 minutes × 4, and then every 15 to 30 minutes until stable

Contact surgeon for temperature less than 36° C or over 38.5° C (96.8° F to 101.3° F)

Maintain NPO until fully awake; may offer clear liquids as tolerated

Maintain Trendelenburg position Reinforce bandage as needed Neuro checks every 8 hours Elbow restraints if needed

4. You check the postoperative orders written by the resident, which are listed in the chart. Which orders are appropriate, and which would you question? State your rationale.

5. You return to the room later in the shift to check on Z.O. Which

assessment findings would cause concern? Select all that apply and explain your answers.

a. Facial edema b. Heart rate 120 c. Decreased responsiveness d. Blood pressure 90/55 e. Increased clear drainage on dressing

6. Discuss some of the emotional issues Z.O.’s parents will experience during the immediate postoperative period.

7. Which actions are appropriate ways to assist the family during this time? Select all that apply.

a. Reassure them that everything will be fine. b. Tell them you understand how they are feeling. c. Ensure that they have as much privacy as possible. d. Encourage them to talk about their feelings, if they

can. e. Remind them that they need to care of themselves

to be able to care for their child. f. Ask them if they would like to talk with the hospital

chaplain and/or social worker.

Case study progress The health care team will create a treatment plan for Z.O. This plan will outline what types of treatments will be used, how often they will be administered, and the expected length of treatment. It will be customized based on Z.O.’s overall health, age, cancer type, and stage of cancer. The team will meet daily to weekly to update the ongoing treatment plan.

8. Which health care team members would you expect to be included in this interdisciplinary team? Select all that are appropriate and explain why you think they should be included.

a. Pharmacist b. Oncologist

c. Neurologist d. Hematologist e. Neurosurgeon f. General surgeon g. Oncology social worker h. Charge nurse and/or nurse caring for Z.O. i. Oncology nurse practitioner/clinical nurse specialist

Case study progress Z.O.’s wound and neurologic status are monitored, and he continues to improve. He is transferred to the oncology service on postoperative day 7 for initiation of chemotherapy.

9. Outline a plan of care that addresses common risks secondary to chemotherapy, describing at least 2 nursing interventions that would be appropriate for managing risks for infection, bleeding, dehydration, altered growth and nutrition, altered skin integrity, and body image.

10. The unlicensed assistive personnel (UAP) is in the room

caring for Z.O. Which safety observations would you need to address? Explain your answer.

a. UAP assists Z.O. out of bed to prevent a fall. b. UAP encourages Z.O. to use a soft toothbrush for

oral care. c. UAP applies the disposable probe cover to the

rectal thermometer. d. UAP applies hand gel before and after assisting

Z.O. to the restroom.

Case study progress On day 10, after initiation of chemotherapy, you receive the lab results shown in the chart.

Chart View

Laboratory Test Results

Hemoglobin (Hgb) 12.5 g/dL (125 g/L)

Hematocrit (Hct) 36%

White blood cells (WBCs) 7.5 × 103 cells/mm3

Red blood cells (RBCs) 4.0 million/mm3 (4 x 1012/L)

Platelets 80,000 × 103/mm3 (80 x 109/L)

Albumin 2.5 g/dL (3.6 mcmol/L)

Absolute neutrophil count (ANC) 75/mm3 (0.0075 x 109/L)

11. Which lab results are you concerned about, and why? 12. Z.O. has a 5-year-old sister. She has been afraid of visiting at

the hospital because her “brother might die.” Discuss a preschooler’s concept of death and strategies to help cope with the illness of a sibling.

Case study progress Postoperatively, Z.O. completed his initial course of chemotherapy in the hospital. He completed the cancer protocol he was treated under and was in remission with follow-up monitoring every 3 months. Four months later, he is experiencing new symptoms, including behavior changes and regression in speech and mobility. After extensive diagnostic testing, it is determined that his tumor has recurred.

13. Before the health care team decides on the new treatment goals and how to achieve them, what are some of the questions/issues you think should be addressed with the parents?

14. After a long discussion with Z.O.’s parents, the health care team suggests hospice care. List at least 4 of the goals of hospice care for this patient and family.

15. Pain control, supplemental nutrition and hydration, and resuscitation are common ethical dilemmas nurses face when caring for terminally ill children. Complete the following chart, describing the common reasons for providing and withholding care for Pain Control, Supplemental Nutrition and Hydration, and Resuscitation.

Case study outcome Z.O. dies at home just before his fourth birthday. The hospice nurse and chaplain help the family by providing support and comfort for all family members and assistance in dealing with funeral arrangements. In addition, they offer the family ongoing bereavement resources and services.

Case Study 82 Name _________________________________ Class/Group _____________________________ Date _____________

Scenario J.H. is a 5-week-old infant brought to the emergency department (ED) by his mother, who speaks little English. Her husband is at work. The mother is young and appears frightened and anxious. Through a translator, Mrs. H. reports that J.H. has not been eating, sleeps all of the time, and is “not normal.”

1. What are some of the obstacles you need to consider, recognizing that Mrs. H. does not speak or understand English

well? 2. You perform your primary assessment and question Mrs. H.

with a translator. Which findings are abnormal and need to be reported? Select all that apply and state rationale.

a. High-pitched cry b. Pupils equal and + 3 c. Heart rate: 85 d. Positive Babinski reflex e. Refusal of PO intake per mother f. Anterior fontanel palpable and tense g. Temperature 36° C (96.8° F) rectally

3. Place an X where you would assess the Babinski reflex on an infant.

Case study progress J.H. is admitted to the medical unit with the diagnoses of meningitis and rule out sepsis. The ED physician gives the orders shown in the chart.

Chart View

Emergency Department Orders

CBC with differential Blood culture CMP UA Cerebrospinal fluid (CSF) for culture, glucose, protein, cell

count (after lumbar puncture) Ceftriaxone (Rocephin) 260 mg IV now (loading dose) Ampicillin 400 mg IV now and then every 6 hrs Acetaminophen 50 mg per rectum for irritability X 1 dose D5W 0.45% NS to infuse at 15 mL/hr

4. Prioritize the order of your interventions, with 1 being your first action and 7 being your last action.

_______1. Administer ceftriaxone and Ampicillin _______2. Place IV _______3. Straight catheterization for urine specimen _______4. Place on contact isolation and droplet

precautions _______5. Assist with lumbar puncture _______6. Administer acetaminophen _______7. Obtain blood culture, CMP

5. You have a difficult time placing the IV line, and the physician writes an order to give the ceftriaxone IM while you wait for the vascular access team to place the IV. Name the appropriate

site for an IM injection for an infant. 6. Before administering the ceftriaxone and Ampicillin, you

verify the dose with another RN. The therapeutic range is for Rocephin 100 mg/kg loading dose and then 80 to 100 mg/kg daily. The therapeutic range for Ampicillin is 200 to 400 mg/kg/day in 4 divided doses with a maximum daily dosage of 12 g/day. J.H. weighs 3.5 kg. Is the loading dose ordered for Rocephin safe? Is it therapeutic? Is the ordered dose for Ampicillin, if given every 6 hours, safe and therapeutic? Show your work.

7. Interpret J.H.’s lab findings, and explain the rationale for abnormal results.

Chart View

Laboratory Test Results Urinalysis

pH 7.2

Color Clear

Leukocytes Negative

Blood Tests

Hct 32%

HgB 10.5 g/dL (105 g/L)

WBC 22,000 cells/mm3 (22 x 109/L)

Sodium 136 mEq/L (136 mmol/L)

8. Interpret the CSF findings. Would you suspect bacterial or viral meningitis? Why?

Chart View

Cerebrospinal Fluid Analysis

CSF Clear

Gram stain Pending

Protein 300 mg/dL (elevated) (3.0 g/L)

Leukocytes (cell count) 1030 (elevated)

Glucose 40 mg/dL (decreased) (2.2 mmol/L)

9. What are the most common bacterial pathogens in this age group?

Case study progress J.H. is diagnosed with Escherichia coli meningitis. His medical care plan will include 21 days of antibiotic therapy with ceftriaxone. You are developing his nursing plan of care.

10. Outline a plan of care for J.H., describing at least 2 nursing interventions that would be appropriate for managing pain and infection, maintaining hydration, assisting with increased intracranial pressure (ICP), and teaching to review with his parents.

Case study progress Mrs. H., through her translator, asks you what could have caused her baby to be sick, given that he had an immunization when he was born. She asks whether he should get “more shots” so this won’t happen again. You reinforce to Mrs. H. that infants have immature immune systems, and they are vulnerable to infections until they have been fully immunized. Mrs. H. asks when J.H. will get more shots and what will they be.

11. According to the CDC immunization schedule, which immunizations will J.H. receive at 2 months? You can refer to the current immunization schedules posted at www.cdc.gov/vaccines/schedules/downloads/child/0-18yrs- child-combined-schedule.pdf

a. Hib b. IPV c. OPV d. MMR e. DTaP f. Hep B g. Varicella h. Rotavirus i. Pneumococcal conjugate (PCV13)

12. What is the effect of hospitalization on J.H.’s growth and development?

13. J.H. is being discharged after 3 weeks of IV antibiotic therapy. What educational topics will be important to discuss with J.H.’s parents when he is discharged?

14. You are providing developmental teaching to Mrs. H. with a translator. Which milestones would be appropriate to anticipate at 2 months? Select all that apply.

a. Coos and gurgles b. Develops a social smile c. Able to purposely reach for toys d. Able to roll from stomach to back e. Able to see an object 4 to 5 feet away

Case study outcome J.H. is discharged to home with his parents. He will continue antibiotics by mouth for 1 week and receive a home health visit for infant care follow-up. The parents are to return him to his primary care provider in 1 week or call with any concerns.

Case Study 83

http://www.cdc.gov/vaccines/schedules/downloads/child/0-18yrs-child-combined-schedule.pdf
Name _________________________________ Class/Group _____________________________ Date _____________

Scenario You admit L.M., a 2-month-old girl with a history of hydrocephalus and ventriculoperitoneal (VP) shunt placement 1 month earlier. Her parents report that she has been more irritable than usual, and for the past 3 days she has fed poorly and has had emesis 5 or 6 times every day.

1. Explain the pathophysiology of hydrocephalus and cerebrospinal fluid (CSF) imbalance.

2. Explain how the placement of a VP shunt helps the patient.

Case study progress You get L.M. settled on the unit and promptly perform her admission assessment.

3. A nursing student is assisting you with L.M.’s admission. Assessment of growth and development is an important part of patient assessment. Which of the nursing student’s statements is correct?

a. “We should not see any head lag.” b. “L. should be able to focus on objects that are near.” c. “We will assess her anterior fontanel because it

should be closed.” d. “I do not need to do a frontal occipital

circumference measurement because her sutures have fused.”

4. Your assessment includes the following findings. Indicate the abnormal findings with a * and state the possible rationale for each.

System Assessment and Vital Signs If Abnormal,

State Rationale

Weight 4.5 kg

Neurologic Irritable, awake, and fussy; difficult to console FOC: 44 cm, “increased 2 cm from measurement yesterday” per mother Anterior fontanel slightly bulging Unable to palpate posterior fontanel Pupils equal and reactive Shunt observed and palpated lightly behind (L) ear, no warmth, tenderness or drainage

Respiratory Bilateral breath sounds equal and clear Spo2 95% on room air Respiratory rate: 40

Cardiovascular Rectal temperature: 38.8° C Heart rate (HR): 182 Blood pressure (BP): 111/70 Pulses 2 + and equal bilaterally

Gastrointestinal Positive bowel sounds Emesis during examination Last feeding 6 hours ago

Genitourinary Last urine output 2 hours ago

Musculoskeletal Moves all extremities well Head lag noted

Skin Diaper rash noted

Abdominal incision with well approximated edges, no warmth, tenderness, or drainage

5. The doctors order a CT scan and lumbar puncture with a cell count, culture, Gram stain, glucose, and protein run on the CSF. What is the rationale for each procedure?

Case study progress It is determined that the VP shunt is infected and must be temporarily removed. L.M. is taken to surgery to have a left extraventricular drain (EVD) placed. She returns to your unit in stable condition. You get her settled back into her room and perform an assessment. You note that her EVD is intact and draining CSF. The dressing is clean and dry and intact under a sterile dressing.

6. True or False: The position of the EVD should be maintained at the level of the external auditory meatus (tragus). Explain your answer.

Chart View

Medication Administration Record (MAR)

Acetaminophen 15 mg/kg PO q4-6hr prn Morphine sulfate 0.05 mg/kg IV q4hr prn Enalapril (Vasotec) 5 mcg/kg PO q24hr Cefotaxime 150 mg/kg/day IV in divided doses q8hr Baclofen (Lioresal) 10 mg/kg/day PO q8hr Ondansetron (Zofran) 0.1 mg/kg IV now

7. Which medications are appropriate for L.M.’s diagnosis? Select all that apply and state the rationale for the ones you chose. Then, give a reason for those you feel are not appropriate.

8. You are preparing to give the first dose of antibiotic that is

ordered. Referring to L.M.’s medication administration record,

calculate the amount of the antibiotic that you will administer per dose. Show your work.

9. L.M. is very fussy, and you decide to medicate her for

pain. Calculate the amount of morphine L.M. will receive for pain per dose (do not round). Then calculate the amount you will draw up for the required dose, and mark the syringe appropriately (round to hundredths/two decimal places). The morphine is available in an injection solution of 2 mg/mL.

10. Which task can be appropriately delegated to the UAP? a. Changing the dressing on the surgical site b. Obtaining and charting a complete set of vital signs c. Performing the every-2-hour neurologic check d. Instructing the parents on changes in neurologic

status 11. In which position should L.M. be placed immediately

postoperatively unless ordered otherwise by the surgeon? a. Flat, left side-lying b. Flat, right side-lying c. Supine, Trendelenburg d. Supine, head of bed (HOB) 45 degrees

12. What points will you address while teaching the parents about the EVD system?

Case study progress Several days later, L.M.’s mother is changing L.M.’s diaper and she tells you that she is worried because L.M. has started having diarrhea

recently, and it is getting worse.

13. Based on the medications that L.M. is receiving, what is the most likely cause of the diarrhea? What is a possible concern you should consider, and what should your care plan include?

Case study progress L.M. responds well to the antibiotics, and her shunt is internalized 2 weeks later. She is released from the hospital after observation for 2 days.

14. While you are giving your discharge instructions, L.M.’s

mother states that she normally gives L.M. 1 mL of acetaminophen (Tylenol Elixir), 160 mg/5 mL, and asks whether this is the correct dose. L.M.’s current weight is 4.5 kg and the therapeutic range of acetaminophen dosage is 10 to 15 mg/kg PO q4-6hr. Which statement is most accurate?

a. “Tylenol should not be given to a child her age.” b. “This is a safe amount; you should continue to give

that dose every 4 hours.” c. “You should give 1.4 to 2.1 mL every 4 to 6 hours

based on her current weight.” d. “You can continue to give her that amount; you can

give her a dose every 2 hours.” 15. When giving discharge instructions for this 2 month old,

which guidelines/practices would help decrease the risk for a medication error occurring at home? Select all that apply. Then, give your rationale for why each answer is either appropriate or not.

a. L.M. may receive up to 6 doses of acetaminophen per day as needed.

b. Ask the pharmacist to give you an oral syringe to give the medication to L.M.

c. Give L.M. 1.4 to 2.1 mL every 4 to 6 hours as

needed, which is based on her weight at this time d. You should give L.M. 2 mL every 4 to 6 hours as

needed, which is based on her weight at this time. e. Give parents detailed medication instructions, such

as demonstration and return demonstrations, and picture-based-dosing instructions.

Case study outcome L.M. returns for her postoperative checkup 2 weeks later and is playful and alert. The neurologist will continue to monitor her closely with follow-up visits.

Case Study 84 Name _________________________________ Class/Group _____________________________ Date _____________

Scenario The charge nurse tells you that you will be admitting a 1-hour-old infant, Baby Girl R., to the neonatal intensive care unit (NICU) with a myelomeningocele that was discovered in utero. Her aunt and father arrive shortly after her admission while the mother remains at the local medical center recovering from a cesarean delivery.

1. What is the rationale for doing a cesarean delivery for babies with myelomeningocele?

Case study progress While you are getting vital signs, the father tells you that he has been trying to research myelomeningocele on the Internet, but he is still confused, especially about the difference between myelomeningocele and meningocele.

2. Using lay terms, what would you tell the father about the pathophysiology of myelomeningocele? What is the difference between myelomeningocele and meningocele?

3. After your discussion with the family, which of the father’s statements would indicate a need for more teaching?

a. “My baby will need to lie on her stomach in her incubator.”

b. “I need to wash my hands carefully to prevent spread of germs.”

c. “My baby will probably not require surgery until she is a year old.”

d. “My baby’s malformation can also be referred to as spina bifida cystica.”

4. The father asks questions about the infant’s condition but does not look at his newborn. Which statements are correct? Select all that apply.

a. People grieve for the loss of a “normal newborn” differently.

b. Most new fathers are not interested in looking at their newborns.

c. It is apparent that he does not care about his newborn’s condition.

d. This is an abnormal reaction for a parent of a child born with a very visible physical defect.

e. Even though the myelomeningocele was diagnosed in utero, seeing the congenital anomaly or physical defect on his infant is very difficult.

Case study progress R. is in an open warmer. You document the information shown in the chart.

Chart View

Admission Data

Blood pressure 67/33

Pulse 173

Respirations 52

Axillary temperature 37.1° C (98.8° F)

Spo2 95%

Weight 3.5 kg

Frontal occipital circumference (FOC) 35 cm

5. Which assessment and monitoring data are abnormal for a 1- hour-old infant? Select all that apply.

a. Acrocyanosis b. Bilateral clubfeet c. Breath sounds clear d. Fontanelles soft and flat e. Pupils 2 cm, brisk reaction f. Sleepy, squirms and fusses during pupil check g. No reaction when pulse oximeter is placed on right

foot h. Pulses 2 + and capillary refill time less than 2 to 3

seconds 6. You are carefully assessing the sutures and fontanelles. True or

False: The posterior fontanel is joined by the temporal and parietal bones.

7. Explain the rationale for each order in the table.

Case study progress The next day Baby Girl R. is taken to the operating room. The anesthesiologist orders cefazolin (Ancef) 140 mg IV to be given 30 minutes before the surgery begins.

8. You add 10 mL of sterile water to the 1-g vial for a

concentration of 100 mg/1 mL. Calculate how many milliliters you will draw up for this dose. Shade in the dose on the syringe.

Case study progress Postoperatively, the postanesthesia care unit (PACU) nurse tells you Baby Girl R. did well during surgery and is ready to return to your unit. When she arrives, you and the unlicensed assistive personnel start putting on the monitors. R.’s father is present, and he asks you to give the baby some pain medication. The open warmer starts alarming

because the infant’s skin temperature is reading 35° C (95° F). You look to see whether the temperature probe has fallen off. You see that it is still on, but you notice that the suture from surgery is no longer intact. The oxygen monitor reads 71% saturation with an accurate waveform, and the pulse oximeter probe is correctly placed. The respiratory rate is 25 and heart rate is 102.

9. Which of the issues should you address first? Give rationale.

Case study progress Baby Girl R.’s condition stabilizes. Her temperature is 36.7° C (98.1° F) per skin probe. Respiratory rate and heart rate improve and her Spo2 is 98% on ¼ L of oxygen per minute via nasal cannula. The surgeon is at the bedside and opts to return her to the OR for revision of the incision.

Two nights later, you are caring for Baby Girl R. In report, you hear that the parents really want to hold their baby, but they have not yet done so because they are afraid of causing the suture to open again. They are currently at the bedside, and the infant is due for a feeding.

10. How can you help the parents become comfortable with holding their baby?

11. When you take the bottle into the room, you notice a growth chart next to the bed tracking the FOC measurement at least once per shift. Baby Girl R.’s FOC has increased to 36 cm. Using the appropriate WHO (World Health Organization) growth chart (www.who.int/childgrowth/standards/second_set/cht_hcfa_girls_p_0_13.pdf? ua=1) is the following statement True or False? The FOC is close to the 95th percentile and can be monitored less frequently because this is a normal finding. (Explain your answer.)

Case study progress Discharge teaching is an essential part of Baby Girl R.’s care. Teaching

http://www.who.int/childgrowth/standards/second_set/cht_hcfa_girls_p_0_13.pdf?ua=1
and preparation of the family have been done since the diagnosis in utero and are ongoing issues.

12. Which topics would be important to include in discharge teaching for Baby Girl R.? Select all that apply.

a. Positioning b. Skin care and wound care c. Specialized feeding technique d. Maintenance of the Foley catheter e. Comfort measures and pain control f. Importance of multidisciplinary follow-up g. Signs and symptoms of when to call the physician h. Range-of-motion (ROM) exercises as appropriate

per PT i. Appropriate stimulation such as sitting in an infant

seat or swing

Case study outcome Baby Girl R. is followed closely in the Level 2 nursery/NICU. She stabilizes and is able to be discharged to home in 2 weeks with intensive discharge teaching and close multidisciplinary follow-up.

Case Study 85 Name _________________________________ Class/Group _____________________________ Date _____________

Scenario You are the nurse on a medical unit taking care of a 50-year-old man, A.A., who was admitted 18 hours ago with peptic ulcer disease secondary to suspected chronic alcoholism. You enter A.A.’s room and find him having a generalized convulsive (tonic-clonic) seizure.

1. What is your immediate concern for A.A.? 2. List 5 things you would do in order of priority. 3. Given A.A.’s history, state 3 possible causes for his tonic-clonic

seizure.

Case study progress The rapid response team is called, and the provider gives the orders shown in the chart.

Chart View

Medication Administration Record

Thiamine (vitamin B1) 100 mg IM now 50% glucose, 1 50-mL IV bolus now Lorazepam (Ativan) 4 mg IV now over 2 to 5 minutes

4. Indicate the expected outcome for A.A. associated with each medication.

5. In what order would you give A.A.’s medications? Give your reason.

_____ Thiamine (Vitamin B1) _____ Glucose _____ Lorazepam (Ativan)

6. List your primary concern when giving lorazepam intravenously.

7. The lorazepam is supplied in a single-use vial. How many

milliliters will A.A. receive? Shade in the dose on the syringe.

8. What assessments do you need to make during his ongoing seizure activity?

Case study progress A.A.’s seizure activity does not subside. The provider orders an additional 4 mg of IV lorazepam without effect. Twenty minutes has now elapsed since you initially found A.A. having seizure activity.

9. What is the significance of this time lapse? 10. Define status epilepticus. 11. The provider decides to administer propofol (Diprivan) and

intubate A.A. to support his airway. What is propofol? Why is it being given to A.A.?

12. The provider also orders a phenytoin 15 mg/kg IV loading

dose at a rate of 50 mg/min. What is the reason for giving A.A. phenytoin?

13. A.A. weighs 143 pounds. How much phenytoin will you

administer? 14. As you prepare to administer the phenytoin, you see that

A.A. has D5W infusing at 75 mL/hr. Why does this concern you, and what are your options?

15. You accompany A.A. as the rapid response team transfers him to the ICU. During the transport, his seizure activity ceases. Using SBAR, what information will you provide to the ICU nurse?

16. What are the main complications of status epilepticus that the nurse will monitor for?

17. Describe the assessment A.A. needs over the next few hours. 18. Identify nursing interventions that are appropriate for A.A.

since the seizure activity has subsided.

Case study outcome A.A.’s seizure is successfully treated with lorazepam and phenytoin, and he has no further seizure activity. After his acute care needs are resolved, A.A. decides to enter a detoxification program on discharge. He successfully completes the program and remains free of drug and alcohol use.

Case Study 86 Name _________________________________ Class/Group _____________________________ Date _____________

Scenario T.W. is a 22-year-old man who fell 50 feet (15 meters) from a chairlift

while skiing and landed on hard-packed snow. He is now at the emergency department (ED) with spinal cord injury (SCI) with paraplegia from a suspected T5-T6 fracture.

Chart View

Physician’s Orders

Insert indwelling urinary catheter ECG monitoring Immobilize the cervical spine Oxygen at 4 L per nasal cannula Neurologic assessment every hour Apply warming blankets as needed

1. Describe a plan for implementing these orders. 2. What are the nursing priorities at this time? 3. Which assessment would you complete first?

a. Auscultating breath sounds b. Testing the peripheral reflexes c. Determining pupil response to light d. Assessing ability to move the extremities

4. What other interventions would likely be done by the ED nurse?

5. Awareness of the prehospital management of a SCI is critical to T.W.’s ultimate neurologic outcome. What actions will the nurse take to ensure this goal is met?

6. T.W. anxiously asks, “With this broken back, am I going to be paralyzed for life or can it be reversed?” How would you respond?

Case study progress The diagnosis of the fracture is confirmed, and T.W. is transferred from the ED to the surgical intensive care unit (SICU). Although

T.W.’s injury is at a level at which independent respiratory function is expected, he experiences low oxygen saturation levels and is intubated and placed on mechanical ventilation. The provider states that this is because of spinal shock. He has a central venous catheter (CVC) inserted for medication administration. T.W.’s medication list includes pantoprazole 40 mg IV bid, propofol (Diprivan) 10 mcg/kg/min continuous IV infusion, and enoxaparin 30 mg SubQ every 12 hours.

7. How would you explain spinal shock to T.W.’s family and why T.W. needs mechanical ventilation?

8. Indicate the reason T.W. is receiving each medication. 9. T.W. weighs 158 pounds. The pharmacy-supplied

infusion bottle reads “propofol 500 mg/ 50 mL.” At how many milliliters per hour would you set the infusion pump? (Round to the nearest hundredth.)

10. After the CVC is inserted, T.W. has a STAT portable chest

x-ray examination. Why?

Case study progress T.W. is taken to surgery 48 hours after the accident for spinal stabilization. He spends 2 additional days in the SICU and 5 days in the neurology unit and now is in the rehabilitation unit. He continues to have paralysis of his lower extremities. Shortly after the transfer, T.W. turns on his call light and asks for medication for headache. As you walk into the room, you immediately note that T.W.’s face is flushed and he is profusely sweating.

11. What complication do you suspect T.W. is experiencing and why?

12. What further assessment data do you need to collect? 13. What interventions do you need to perform for T.W.? 14. What could happen if autonomic dysreflexia (AD) is left

untreated?

15. After your prompt intervention, T.W.’s AD resolves and you need to document what happened. Write an example of a documentation entry describing this event.

Case study progress After spending 6 weeks in acute care, T.W. is transferred to the rehabilitation unit for interprofessional, intensive therapy. The interprofessional team will address the complex effects of SCI, including paraplegia, respiratory disorders, bowel and bladder function, as well as emotional and psychological issues related to T.W.’s adjusting to a new way of life.

16. What members of the interprofessional team would likely be involved in his care?

17. What are realistic functional goals for T.W.? 18. Part of rehabilitation care includes teaching T.W. how to

manage his continuous urinary drainage system. What would this teaching include?

19. What outcome parameters would you use to determine whether efforts to promote urinary excretion have been effective?

20. T.W. is experiencing some spasticity of the lower extremities. What should be included in the plan of care to prevent developing contractures of the lower extremities?

21. T.W. has special dietary needs, and the registered dietitian is part of developing an optimal diet for T.W. Describe the components of this diet.

22. T.W. will be taught bowel-training techniques. What will this teaching include?

23. T.W. is concerned whether he will ever be able to have sex again. What would you tell him, and what are some possible referrals?

24. While assisting T.W. with his morning hygienic care, he states, “Why would anyone want to live like this? No woman will ever want me. I just wish you would have let me die.” How

would you initially respond? a. “You wish you would have died?” b. “Tell me why you are talking like this.” c. “Let’s finish your bath and then we can talk.” d. “I know this is hard now, but things will work

out.”

Case study outcome Although his rehabilitation was slow the first few years, he eventually progressed to the point that he was independent in all activities of daily living. He lives in his childhood home, which required some accessibility modification, and is periodically struggling with depression and anger.

Case Study 87 Name _________________________________ Class/Group _____________________________ Date _____________

Scenario J.R. is a 28-year-old man who was doing home repairs. He fell from the top of a 6-foot stepladder, striking his head on a rock. He experienced a momentary loss of consciousness. By the time his neighbor got to him, he was conscious but bleeding profusely from a laceration over the right temporal area. The neighbor drove him to the emergency department of your hospital. As the nurse, you immediately apply a cervical collar, lay him on a stretcher, and take J.R. to a treatment room.

1. What steps will you take to assess J.R.? 2. List at least 6 components of a neurologic examination. 3. What types of injuries may J.R. have sustained? 4. Differentiate between primary and secondary head injury.

5. What complication common to each of these diagnoses concerns you most?

6. Why is this complication clinically important? 7. Name at least 6 findings that would indicate this complication

is occurring. 8. Which one is the most sensitive indicator of neurologic change?

Case study progress You complete your neurologic examination and find the following: Glasgow Coma Scale (GCS) score of 15; pupils equal, round, reactive to light; and full sensation intact. J.R. complains of a headache and is somewhat drowsy. His vital signs are 120/72, 114, 30, 98.7° F (37.1° C), and Spo2 94%. As the radiology technician performs a portable cross- table lateral cervical spine x-ray examination, J.R. begins to speak incoherently and appears to drift off to sleep.

9. What are the next actions you will take?

Case study progress While waiting for the provider to arrive, J.R. becomes unresponsive to verbal stimuli. The right pupil is larger than the left and does not respond to light. J.R. responds to painful stimuli in the manner shown in the illustration.

10. What is this response called and what does it signify? 11. Calculate J.R.’s GCS score. Describe the clinical implications of

this score. 12. What is the most likely cause of the change in J.R.’s neurologic

status? 13. What are your immediate nursing priorities? 14. What immediate actions will you take? 15. His vital signs are now 160/72, 64, 10, 98.7° F (37.1° C), and

Spo2 94%. What is your concern and why?

Case study progress The provider arrives and gives the orders shown in the chart.

Chart View

Physician’s Orders

Insert indwelling urinary catheter Insert nasogastric tube to continuous low wall suction Intubate: Vent settings assist-control 16, VT 900 mL, Fio2 0.5,

PEEP (positive end-expiratory pressure) 3 cm IV fluid 0.9% normal saline at 100 mL/hr 160 grams Mannitol IV STAT over 30 minutes Phenytoin 1360 mg IV STAT over 30 minutes, then 100 mg IV

every 8 hours STAT CT scan STAT labs: CBC, CMP, UA, type and cross, PT/INR and PTT,

ABGs, toxicology screen

16. Outline a plan for implementing these orders. 17. What is mannitol, and why is it being given to J.R.? 18. What is the expected outcome associated with administering

phenytoin to J.R.?

Case study progress J.R. goes to radiology for a CT scan, which shows a large epidural hematoma on the right with a hemispheric shift to the left. He is taken straight to the operating room for evacuation of the hematoma. While he is in surgery, J.R.’s family arrives with their chaplain. They ask if they can anoint J.R. and pray over him.

19. How should you respond?

Case study progress Postoperatively J.R. is admitted to the neurologic ICU.

20. What assessment indicators will be closely monitored in J.R.? 21. An appropriate nursing intervention for J.R. to promote

cerebral tissue perfusion is: a. Avoiding passive range-of-motion exercises b. Repositioning him every 4 to 6 hours, using

logrolling c. Limiting endotracheal suctioning to no longer than

15 seconds d. Clustering nursing activities so he has periods of

uninterrupted rest 22. The best way to position J.R. is to:

a. Elevate the head of the bed to 30 degrees b. Keep him flat with his hips slightly flexed c. Place him on the right side with his head flexed d. Turn his head from side-to-side to decrease

aspiration risk 23. Name 4 independent nursing interventions that would be used

to control environmental stimuli during the first 48 postoperative hours.

24. What measures will be used to prevent J.R. from developing hyperthermia? Why are these important?

25. What outcome criteria would determine whether the independent nursing measures for J.R. were effective?

Case study outcome J.R. has lingering effects several months after his accident. He has frequent headaches, tires easily, and is sensitive to light. Neurologically, he is “normal,” though he has yet to return to work. Still, he feels lucky. “I’m still alive. If I would not have had that terrible cut, I may have died.”

Case Study 88 Name _________________________________ Class/Group _____________________________ Date _____________

Scenario C.J. is a 48-year-old violinist in the local symphony. Before the performance this evening, she told a friend that she was experiencing what she called “the worst headache I’ve ever had” and that she had taken two extra-strength acetaminophens, but they “didn’t touch my headache.” During the performance, she stopped playing, reached up, grasped her head, and then fell unconscious. When the paramedics arrived, she was intubated and an IV line was started with normal saline.

On arrival at the emergency department, she has a Glasgow Coma Scale (GCS) score of 3. Her husband reports a history of hypertension and states she recently quit taking her medication because it made her feel tired. She is trying to quit smoking and is down to a half pack of cigarettes per day. She drinks alcohol socially on weekends and has a remote history of cocaine use. He says that she was complaining of worsening, intermittent headaches for the past few weeks.

1. Describe C.J.’s neurologic presentation that equates with a GCS score of 3.

2. The provider immediately suspects a subarachnoid hemorrhage (SAH). Why?

Case study progress After CT angiography is done, C.J. is diagnosed with a massive grade V SAH. She is transported to the intensive care unit for close monitoring. She is ventilator dependent, is unresponsive to verbal or painful stimuli, and has no physical movement. Her husband, mother, and children are at her bedside; several relatives and friends are in the waiting area.

3. What is a SAH? 4. What are common causes of an SAH? 5. What are C.J.’s risk factors for a SAH? 6. Describe a patient with a grade V SAH. 7. What common complications of SAH would you anticipate? 8. What is the likely goal of treatment for C.J.? 9. Treatment of SAH can include surgery, embolization,

medications, and watchful waiting. What factors are considered in determining treatment?

Chart View

Physician’s Orders

Nimodipine 60 mg every 4 hours per NG tube for systolic BP > 140

0.9% normal saline at 100 mL/hr Labetalol 10 mg IV over 2 minutes q1hr for systolic BP greater

than 140 Acetaminophen 650 mg q6hr for temperature greater than

101.0° F (38.3° C) Insulin aspart (NovoLog) SubQ per sliding scale every 6 hours

10. Identify the expected outcome associated with each treatment C.J. is receiving.

11. Name 4 independent nursing interventions that would be used

to control environmental stimuli during the next 24 hours. 12. How would you position C.J.? 13. What assessment indicators will be closely monitored in C.J.? 14. Describe how you would support C.J.’s family during this

time.

Case study progress After C.J. has been in the ICU for 12 hours, the provider decides to begin testing C.J. to determine whether she is clinically brain dead.

15. What is brain death? 16. What are the general criteria for declaring a patient clinically

brain dead? 17. It is determined that C.J.’s condition meets the criteria, and she

is declared legally brain dead. She had previously indicated her willingness to be an organ donor, and her husband agrees to honor her wishes. C.J.’s husband asks you to explain the donation process. How will you explain it to him?

18. While you are waiting for the transplant team to arrive, you are working to maintain C.J.’s hemodynamic stability. Which parameter would indicate your efforts are successful? Give your rationale.

a. Urine output of 40 mL/hr b. Cardiac index less than 2.4 L/min c. Mean arterial BP is greater than 50 d. Left ventricular ejection fraction greater than 30%

Case study outcome C.J.’s family received several letters from donors. Each acknowledges that they will not forget their kindness, nor C.J.’s memory. C.J.’s family appreciated receiving these letters, which help them with their grief over her death.

7

Metabolism and Glucose Regulation

Case Study 89 Name _________________________________ Class/Group _____________________________ Date _____________

Scenario L.L., a 28-year-old house painter, has been too ill to work for the past 3 days. When he arrives at the emergency department with his girlfriend, he seems alert but acutely ill, with an average build and a deep tan over the exposed areas of skin. He reports headaches, joint pain, a low-grade fever, cough, anorexia, and nausea and vomiting, especially after eating any fatty food. L.L. describes vague abdominal pain that started about the same time as the other problems. He says he has been using “a lot of Tylenol” for his pain. His past medical history reveals he has no health problems, is a nonsmoker, and drinks “a few” beers each evening to relax. Vital signs are 128/84, 88, 26, 100.6° F (38.1° C); awake, alert, and oriented × 3; moves all extremities × 4 with complaints of aching pain in his muscles; very slight scleral jaundice present; heart and breath sounds clear and without adventitious sounds; bowel sounds positive × 4 quadrants; abdomen soft and palpable without distinct masses. L.L. mentions that his urine has been getting darker over the past 2 days and he has been feeling a little “itchy.”

1. Underline the assessment findings that concern you.

2. Your institution uses electronic charting. Based on the health history and assessment described in the scenario, document your findings.

□ Neurologic: □ Respiratory: □ Cardiovascular: □ Gastrointestinal: □ Genitourinary: □ Musculoskeletal: □ Skin: □ Pain:

Case study progress L.L. has key signs of hepatitis. The provider admits him to the medical unit and orders lab work to help identify the precise problem.

3. Which key diagnostic tests will determine exactly the type of hepatitis present?

Chart View

Laboratory Test Results

Indirect bilirubin 1.6 mg/dL (27.4 mcmol/L)

Total bilirubin 2.3 mg/dL (39.3 mcmol/L)

Albumin 3.8 g/dL (5.5 mcmol/L)

Total protein 6.5 g/dL (65 g/L)

ALT 66 units/L

AST 52 units/L

LDH 245 units/L

ALP 176 units/L (3.0 mckat/L)

PT/INR 12 seconds/1.06

aPTT 32 seconds

Urine urobilinogen 1.6 IU/L

Anti-HAV Negative

Anti-HCV Negative

HBsAg Positive

Anti-HBc Positive

IgM anti-HBc Positive

Anti-HBs Negative

4. Interpret L.L.’s lab results. 5. What is the difference between the hepatitis B surface antigen

(HBsAg) and the hepatitis B surface antibody (HBsAb)? 6. For each characteristic below, name whether it describes

hepatitis A (A) or hepatitis B (B). _____a. Fecal-oral transmission _____b. Transmitted by sharing needles _____c. Transmitted by blood transfusions _____d. Vaccination is a three-shot series _____e. Illness is usually mild, similar to a flulike

infection _____f. Symptoms include anorexia, nausea, vomiting,

fever, fatigue, and jaundice 7. What factors in his history could have compounded the

increased ALT levels? 8. How will you explain to L.L. the likely progression of his

disease? 9. Is L.L. contagious? What precautions would you take

while he is in the hospital? 10. What will you do to help ease L.L.’s itching? State 5

interventions.

11. What type of diet will you teach L.L. to follow? 12. Which new assessment finding, if present, would concern you

most? a. Irritable and disoriented b. Mild abdominal tenderness c. Guaiac-negative diarrheal stools d. Easily fatigued when ambulating

Case study progress L.L. is ready for discharge in a few days, and he confides to you that he feels so “guilty” about having hepatitis and endangering his girlfriend and family. L.L. lives at home with his parents and four younger siblings. The youngest is 4 years old. He tells you he was at a party and did not think the one-time needle use could hurt him. He hopes his family is not too afraid to have him return home.

13. What action will you take? 14. He asks how to prevent giving hepatitis to his family.

What specific instructions will you give? 15. Given L.L.’s lifestyle, what specific patient teaching points

must you emphasize?

Case study outcome L.L. remained ill for about 2 months after discharge. The provider did a liver biopsy, which showed some damage to the liver. He did not give his girlfriend hepatitis; her titers were adequate from prior immunization. L.L. continued to refrain from alcohol and substance use and followed a healthy lifestyle. Three years later, he is doing well.

Case Study 90 Name _________________________________ Class/Group _____________________________ Date _____________

Scenario D.G., 57 years old, is admitted to your unit for observation from the emergency department (ED) with the diagnosis of cirrhosis with possible hepatic encephalopathy. He is lethargic, appears cachectic, and is mildly combative when aroused. He is jaundiced, with a notably distended abdomen. He is receiving an IV infusion of D5 ½ NS.

1. Briefly discuss the pathophysiology of cirrhosis. 2. What are the common causes of cirrhosis? 3. How is hepatic encephalopathy related to cirrhosis?

Case study progress

Chart View

Admission Orders

IV D5 ½ NS with 20 mEq KCl at 75 mL/hr Insert indwelling urinary catheter to gravity drainage Bed rest Lactulose 30 mL orally daily Rifaximin 550 mg orally twice daily Spironolactone 100 mg orally twice daily Abdominal ultrasound in a.m. Vitamin K 10 mg/day IV × 3 doses; change to PO when alert

and able to swallow CBC with differential, BMP, liver function tests, PT/INR and

aPTT, serum ammonia (NH3) now and in a.m. 2-Gram sodium diet when alert and able to swallow Call for any signs of bleeding; systolic BP < 100; diastolic BP < 50; or pulse > 120

4. What do you need to do for D.G., and what can you delegate to the UAP?

5. What is the reason for routinely assessing D.G. for bleeding? 6. What assessment findings would indicate the presence of

bleeding? 7. Indicate the expected outcome associated with each of the

medications he is receiving. 8. What is the rationale for placing him on a low-sodium diet?

Case study progress

While you are getting D.G. settled, you continue your assessment.

Neurologic: PERRL; moves all extremities, but is sluggish, pulling away during assessment; follows commands sporadically.

CV: Pulse is regular without adventitious sounds. All peripheral pulses are palpable at 3 + bilaterally; 3 + pitting edema in lower extremities.

Respiratory: Breath sounds clear but diminished in all lobes; musty breath.

GI: Tongue and gums are beefy red and swollen. Abdomen moderately distended, firm, and slightly tender. Bowel sounds positive × 4.

GU: Foley to gravity drainage, with 75 mL dark amber urine past 2 hours.

Skin: Mild jaundice. Skin appears thin and dry. Numerous spider angiomas on upper abdomen with several dilated veins across abdomen. Several ecchymoses on lower extremities.

VS: 120/60, 104, 32, 99.1° F (37.3° C), Spo2 90%. Ht. 74 in (188 cm); wt. 145 lb (65.8 kg).

9. What is the significance of the spider angiomas, dilated abdominal veins, peripheral edema, and distended abdomen?

10. How would you further assess the distended abdomen? What is the clinical name for and the significance of your findings?

11. In what position should you place D.G? Write a brief rationale for your response.

a. Supine b. Fowler’s c. Right side-lying d. Left lateral recumbent

12. What objective findings concern you about his nutritional assessment and why?

13. Why is D.G.’s breath musty? 14. Which of D.G.’s assessment findings are consistent with

hepatic encephalopathy? 15. Which assessment findings, if present, would indicate a

deterioration of D.G.’s condition? a. Frequent diarrhea b. Nausea and vomiting c. Increased urine output d. Development of asterixis

16. What is asterixis?

Case study progress

Chart View

Laboratory Results

Potassium 3.4 mEq/L (3.4 mmol/L)

Alanine transaminase (ALT) 146 units/L

Aspartate transaminase (AST) 207 units/L

Alkaline phosphatase (ALP) 154 units/L

Total bilirubin 3.6 mg/dL (61.6 mcmol/L)

Albumin 2.1 g/dL (3.0 mcmol/L)

Total protein 5.3 g/dL (53 g/L)

Ammonia 155 mcg/dL (111.0 mcmol/L)

PT/INR 16 seconds/1.6

17. Interpret D.G.’s lab results. 18. You begin to develop D.G.’s care plan. It is imperative to

prevent bleeding when caring for a person with cirrhosis. List 4 specific interventions you will take to reduce D.G.’s bleeding risk.

19. Falls are particularly dangerous for someone in his

situation. Why? 20. What measures will you implement to promote optimal

skin integrity? 21. You identify the nursing problem “excess fluid volume.”

Which is the best short-term goal for D.G.? D.G. will: a. Have a decrease in abdominal girth b. Maintain his ordered fluid restriction c. Select low-sodium foods from the diet menu d. Have no further weight gain before discharge

22. Describe 5 interventions that will help improve D.G.’s nutritional status.

23. What interprofessional referrals might you initiate and why?

Case study outcome D.G. develops an upper GI bleed during his third hospital day. After 24 days, including a week in the intensive care unit, he is discharged to a rehabilitation facility. He had been employed as a loading dock worker; unfortunately, his health never recovers to the point where he can return to work.

Case Study 91 Name _________________________________ Class/Group

_____________________________ Date _____________

Scenario Y.L., a 34-year-old Southern Asian woman, comes to the clinic with chronic fatigue, increased thirst, constant hunger, and frequent urination. She denies any pain, burning, or low-back pain on urination. She tells you she has a vaginal yeast infection that she has treated many times with over-the-counter medication. She works full time as a clerk in a loan company and states she has difficulty reading numbers and reports, resulting in her making frequent mistakes. She says, “By the time I get home and make supper for my family, then put my child to bed, I am too tired to exercise.” She reports her feet hurt; they often “burn or feel like there are pins in them.” She has a history of gestational diabetes and reports following a traditional eating pattern, which is high in carbohydrates.

In reviewing Y.L.’s chart, you note she last saw the provider 6 years ago after the delivery of her last child. She has gained considerable weight; her current weight is 173 pounds (78.5 kg). Today her BP is 152/97, and a random plasma glucose level is 291 mg/dL (16.2 mmol/L). The provider suspects she has developed type 2 diabetes mellitus (DM) and orders the lab studies shown in the chart.

Chart View

Laboratory Test Results

Fasting glucose 184 mg/dL (10.2 mmol/L)

Hemoglobin A1c (A1C) 8.8%

Total cholesterol 256 mg/dL (6.6 mmol/L)

Triglycerides 346 mg/dL (3.91 mmol/L)

Low-density lipoprotein (LDL) 155 mg/dL (4.01 mmol/L)

High-density lipoprotein (HDL) 32 mg/dL (0.83 mmol/L)

Urinalysis (UA) + glucose, − ketones

1. Interpret Y.L.’s lab results. 2. Identify 3 methods used to diagnose DM. 3. Describe the major pathophysiologic difference between type 1

and type 2 DM. 4. Name 6 risk factors for type 2 DM. Underline those that Y.L.

has.

Case study progress Y.L. is diagnosed with type 2 DM. The provider starts her on metformin (Glucophage) 500 mg, glipizide (Glucotrol) 5 mg, orally each day at breakfast and atorvastatin 20 mg orally at bedtime. She is referred to the dietitian for instructions on starting a 1200-calorie diet using an exchange system to facilitate weight loss and lower blood glucose, cholesterol, and triglyceride levels. You are to provide teaching about pharmacotherapy and exercise.

5. How can you incorporate Y.L.’s cultural preferences as you develop her teaching plan?

6. What is the reason for starting Y.L. on metformin and glipizide?

7. Outline the teaching you need to provide to Y.L. about oral hypoglycemic therapy.

8. What do you teach Y.L. to do if she becomes ill with the flu or viral illness?

9. You determine she understands your teaching about treating hypoglycemia if she states, “If my blood sugar is low, I should first have:

a. an apple with milk.” b. peanut butter sandwich.” c. fruit juice or regular soda.” d. crackers with cheese slices.”

10. What benefits should Y.L. receive from exercising?

11. What do you need to teach Y.L. about exercise? 12. Besides the dietitian, what interprofessional and community

referrals may be appropriate for Y.L.?

Case study progress Y.L. comments, “I’ve heard many people with diabetes lose their toes or even their feet.” You take this opportunity to teach her about neuropathy and foot care.

13. Which symptoms that Y.L. reported today led you to believe she has some form of neuropathy?

14. What other findings in Y.L.’s history increased her risk for developing neuropathy?

15. What would you teach Y.L. about neuropathy? 16. Because Y.L. has symptoms of neuropathy, placing her at

risk for foot complications, you realize you need to instruct her on proper foot care. Outline what you will include when teaching her about proper diabetic foot care.

17. What monitoring will Y.L. need for nephropathy and retinopathy?

Case study outcome Y.L. returns to the clinic 6 weeks later. Her BP is 130/78 and fasting glucose level is 153 mg/dL (8.5 mmol/L). She says she has not had any episodes of tingling in her toes or blurred vision lately. She did meet with the diabetic educator. She is making changes to her eating, has started walking, and is happy to have lost 6 pounds (2.7 kg).

Case Study 92 Name _________________________________ Class/Group _____________________________ Date _____________

Scenario You are working as a registered nurse (RN) in a large women’s clinic. Y.L., a 28-year-old Asian woman, arrives for her regularly scheduled obstetric appointment. She is in her 26th week of pregnancy and is a primigravida. After examining the patient, the nurse-midwife tells you to schedule Y.L. for an oral glucose tolerance test (OGGT). You review Y.L.’s chart and note she is 5 feet, 3 inches (160 cm) tall and weighs 143 pounds (65 kg). Her prepregnancy body mass index (BMI) was 25. Her father has type 2 diabetes mellitus (DM), and both paternal grandparents had type 2 DM. You enter the room to talk to Y.L.

1. What is the purpose of an oral glucose tolerance test? 2. When is an OGGT performed? 3. What instructions would you provide Y.L. about the test?

Case study progress Y.L.’s 1 hour OGTT test is positive, and she returns to the clinic the next day for a 3-hour (100 g) glucose tolerance test. The results are listed next.

Chart View

Laboratory Test Results: 3 hour (100 g) OGTT

Time of Test Value Normal Range

0730 109 mg/dL (6.0 mmol/L) Less than 95 mg/dL (5.3 mmol/L)

0830 213 mg/dL (11.8 mmol/L)

Less than 180 mg/dL (10.0 mmol/L)

0930 162 mg/dL (9.0 mmol/L) Less than 155 mg/dL (8.6 mmol/L)

1030 156 mg/dL (8.7 mmol/L) Less than 140 mg/dL (7.8 mmol/L)

4. Interpret the results of Y.L.’s test. 5. Y.L. is diagnosed with gestational diabetes mellitus (GDM).

What is GDM? 6. List 5 risk factors for GDM. Place a star next to those risk

factors that Y.L. has.

Case study progress Medical nutrition therapy is the primary treatment for the management of GDM. Because treatment must begin immediately, you call the dietitian to come see Y.L. You also schedule Y.L. to meet with other members of the DM management team later in the week.

7. What is the goal of medical nutrition therapy? 8. Describe the usual diet used in treating GDM. 9. Why is medical nutrition therapy for a woman with GDM

higher in fat and protein than for a woman who is not pregnant?

10. Women with GDM cannot metabolize concentrated simple sugars without a sharp rise in blood glucose. Name 5 examples of simple sugars you would teach Y.L. to limit.

11. Complex carbohydrates (CHOs) do not cause a rapid rise in blood glucose when eaten in small amounts. Identify 5 foods from this group.

Case study progress During the meeting with the dietitian, Y.L. gives a diet history that is high in noodles and white rice with little protein. She informs the dietitian she is lactose intolerant but can have dairy products occasionally in small portions.

12. Is it important that Y.L. take a calcium supplement along with her prenatal vitamins?

Case study progress You provide teaching about monitoring her blood glucose levels and

watch as Y.L. checks her own blood glucose with the glucometer.

13. Y.L. is instructed to check her fasting blood glucose first thing in the morning and 2 hours after every meal. What are the purposes of this request?

14. Which blood glucose levels are considered abnormal for Y.L.? Select all that apply.

a. Fasting glucose 60 mg/dL (3.3 mmol/L) b. Fasting glucose 88 mg/dL (4.9 mmol/L) c. Fasting glucose 100 mg/dL (5.6 mmol/L) d. 2 hour post-meal glucose 119 mg/dL (6.6 mmol/L) e. 2 hour post-meal glucose 150 mg/dL (8.3 mmol/L)

15. Y.L. is instructed to complete ketone testing using the first- voided urine in the morning. What is the reason for this request?

16. You review what to do if she develops hypoglycemia, and then ask her to “teach back” to you what she has learned about what to do. Which statement by Y.L. reflects a correct understanding of treatment of a blood glucose below 60 mg/dL (3.3 mmol/L)?

a. “I will call my doctor immediately.” b. “I will eat a meal and check it again in an hour.” c. “I will drink a cup of orange juice mixed with

sugar.” d. “I will drink half a cup of regular soda, and recheck

it in 15 minutes. 17. Y.L. asks whether having gestational diabetes will hurt her

baby. How would you respond? 18. At the end of the visit, you need to evaluate your teaching.

Which statement made by Y.L. indicates that clarification is necessary?

a. “I should immediately report any ketones in my urine.”

b. “I will stay on the diabetic diet described by the dietitian.”

c. “I will monitor my glucose levels at least four times each day.”

d. “I need to stop exercising because I will need more carbohydrates.”

19. Y.L. states that she plans to have another child soon and asks you if she will develop GDM with that pregnancy. Select the best response:

a. “No, there is no further risk for development of GDM if you get pregnant again.”

b. “Yes, once you develop GDM during a pregnancy, you will develop it with any future pregnancies.”

c. “If you lose weight and do not eat any sweets before your next pregnancy, you will not develop GDM again.”

d. “There is a risk for recurrence of GDM in the next pregnancy. Let your health care provider know that you had GDM with this pregnancy.”

Case study outcome Y.L. safely delivers an 8 pound 9-ounce (3884 gram) baby boy at 39 weeks without complications. She is monitored closely for her GDM during the postpartum period.

Case Study 93 Name _________________________________ Class/Group _____________________________ Date _____________

Scenario You are working with a home care agency. One patient in your caseload is J.S., a 66-year-old man with chronic obstructive pulmonary disease (COPD) related to cigarette smoking. He has been on home oxygen, 2 L oxygen by nasal cannula, for a few years. About 5 months ago, he started on chronic oral steroid therapy. His current

medications include albuterol (Proair HFA) inhaler, fluticasone/salmeterol (Advair), prednisone, digoxin (Lanoxin), atenolol, aspirin, and furosemide.

1. On the way to J.S.’s home, you make a mental note to check him for signs and symptoms of Cushing syndrome. Why?

2. Differentiate between Cushing syndrome and Cushing disease. 3. Review J.S.’s list of medications. Based on what you know

about the side effects of loop diuretics and steroids, discuss the potential problem of using these with digoxin.

4. Differentiate between the glucocorticoid and mineralocorticoid effects of prednisone.

5. How would your assessment change if J.S. were taking a glucocorticoid that had significant mineralocorticoid activity?

6. You need to assess J.S. for signs and symptoms of an

infection. Why? 7. What signs and symptoms of infection do you assess for

in J.S.? Select all that apply. a. Pain b. Fever c. Palpitations d. Loss of function e. Localized edema f. Unusual drainage

8. You plan to ask him about the presence of gastric discomfort, vision, and joint pain. Why?

Case study progress Your assessment includes the following findings.

9. Determine whether each finding is attributable to J.S.’s COPD or possible Cushing syndrome. Place an L beside the symptoms consistent with COPD and a C next to those

consistent with Cushing syndrome. a. ____ Barrel chest b. ____ Full-looking face (moon face) c. ____ BP 180/94 d. ____ Pursed-lip breathing, especially when patient

is stressed e. ____ Striae over trunk and thighs f. ____ Bruising on both arms g. ____ Acne h. ____ Diminished breath sounds throughout lungs i. ____ Truncal obesity with thin extremities j. ____ Supraclavicular and posterior upper back fat

Case study progress You call the provider with J.S.’s assessment. The provider believes J.S. has developed Cushing syndrome and decides to change the prescription for prednisone from 10 mg daily to 10 mg given on alternate days.

10. Explain the reason for this change. 11. Discuss the possible consequences of suddenly stopping

prednisone therapy. 12. You reinforce with J.S. the importance of taking prednisone at

breakfast. Why? 13. Cushing syndrome can affect memory. Patients can easily

forget what medications have been taken, especially when there are several different medications and some are taken on alternating days. List 3 ways you can help J.S. remember to take his medications as prescribed.

14. J.S. states his appetite has increased but he is losing weight. He reports trying to eat, but he gets short of breath and cannot eat any more. How would you address this problem?

15. Realizing J.S. is susceptible to infection, you review with

him ways to reduce the risk for infection. Discuss 4 points to include.

16. In addition to ways to reduce the risk for infection, what other information do you want to stress to J.S. at your visit? Select all that apply.

a. Increase intake of foods high in sodium. b. Weigh yourself first thing in the morning. c. Take vitamin and electrolyte supplements as

prescribed. d. Notify the provider if your pulse is lower than 60. e. Take the furosemide first thing in the morning and

again at bedtime. f. Call the provider if your weight increases more than

2 to 3 pounds (1 to 1.5 kg) in 1 day.

Case study outcome At your next visit, J.S.’s respiratory status is unchanged. Although most of the symptoms of Cushing syndrome remain despite the decrease in the dose of the prednisone, you are pleased that his blood pressure is lower, measuring 150/82.

Case Study 94 Name _________________________________ Class/Group _____________________________ Date _____________

Scenario You are working in a community outpatient clinic where you perform the intake assessment on R.M., a 38-year-old woman who is attending graduate school and is very sedentary. Her chief complaint is overwhelming fatigue that is not relieved by rest. She is so exhausted that she has difficulty walking to classes and trouble concentrating when studying. She reports a recent weight gain of 15 pounds (6.8 kg) over 2 months without clear changes in her dietary habits. Her face

looks puffy, she has experienced excessive hair loss, and her skin is dry and pale. She says she has general body aches and pains with frequent muscle cramps and constipation. You note she is dressed inappropriately warmly for the weather.

Chart View

Vital Signs (VS)

Blood pressure (BP) 142/84

Heart rate 52

Respiratory rate 12

Temperature 96.8° F (36° C)

1. Compare her VS with those of a healthy person her same age. 2. List 8 general questions you might ask R.M. to help determine

what is going on. 3. What are some potential causes for R.M.’s symptoms? 4. As part of your screening, describe how you would begin to

determine whether any of these conditions explain R.M.’s symptoms.

5. What diagnostic tests are appropriate for R.M., and why?

Chart View

Laboratory Test Results

Thyroid-stimulating hormone (TSH) 20.9 mU/L (2–10 mU/L)

Thyrotropin-releasing hormone (TRH) 18.8 ng/dL (2–10 ng/dL)

T3 24 mU/L (70–205 ng/dL)

Free T4 0.2 ng/dL (0.8–2.4 ng/dL)

6. Interpret R.M.’s lab results. 7. The practitioner affirms a diagnosis of hypothyroidism. With

this diagnosis, what other signs and symptoms would you want to assess for in R.M.?

8. What is the most common cause of primary hypothyroidism? a. Thyroidectomy b. Acute thyrotoxicosis c. Hashimoto’s thyroiditis d. Radioactive iodine exposure

9. The practitioner prescribes levothyroxine (Synthroid) 1.7

mcg/kg body weight per day. Now, R.M. weighs 130 pounds. What should be her daily dose of levothyroxine in milligrams? How would her prescription read?

10. What teaching will you provide R.M. about levothyroxine? 11. What should you teach R.M. about preventing myxedema

coma? 12. What other general teaching issues will you address with R.M.

about hypothyroidism? 13. Which statements show R.M. understands your teaching about

hypothyroidism and taking levothyroxine? Select all that apply.

a. “It may take several weeks before I feel better.” b. “The best time to take my medicine is 1⁄2 hour

before breakfast.” c. “If my heart rate is over 100, I will hold my

medication until it is back below 100.” d. “I will be able to discontinue my medication after

the symptoms are under control.” e. “I will come in when needed so my blood levels can

be checked to make sure the medicine is working.” 14. Before R.M. leaves the clinic, she asks how she will know

whether the medication is “doing its job.” How long does it take to see effects of levothyroxine therapy?

15. Outline simple expected outcomes for R.M. 16. A few weeks later, R.M. calls the clinic saying she cannot

remember whether she took her thyroid medication. What other data should you obtain, and what would you tell her?

17. Discuss 2 ways you can help R.M. remember to take her

medication.

Case study outcome R.M. comes in 2 months later for a follow-up visit. You cannot believe she is the same person. She looks and walks as if she were 10 years younger. Her skin appears more radiant, and her hair looks much healthier. “You can’t believe how different I’m feeling,” she says. “I didn’t know how bad off I was; I’m starting to live again.”

Case Study 95 Name _________________________________ Class/Group _____________________________ Date _____________

Scenario You are working on the surgical floor and will be receiving a patient from the postanesthesia care unit. The nurse calls and gives the following report: C.P. is a 55-year-old woman who underwent a subtotal right thyroidectomy for papillary carcinoma. Estimated blood loss was 25 mL. Vital signs (VS) are 130/82, 80 to 90, 20, and SaO2 94% on room air. She is receiving an IV infusion of D5.45NS at 100 mL/hr. She has received a total of 3 mg morphine sulfate IV push, and she is awake, but drowsy, and fully oriented. C.P.’s past medical history includes a total abdominal hysterectomy for fibroids and low-level radiation treatments to the neck 30 years ago for eczema. Both parents are living; her father had a myocardial infarction at 70 years of age; her mother has hypothyroidism but never had thyroid tumors.

1. Name the major risk factor that likely contributed to C.P. developing thyroid cancer.

2. What are the common manifestations of thyroid cancer? 3. What specific preparations will you make before C.P. arrives? 4. You receive C.P. from the recovery room. How will you focus

your initial assessment, and why? 5. During your initial assessment, you document negative

Chvostek and Trousseau signs. Describe data that would support this conclusion.

6. Which assessment findings may indicate C.P. has laryngeal nerve damage? Select all that apply.

a. Stridor b. Hoarseness c. Breathy voice d. Circumoral numbness e. Difficulty swallowing

7. Name 3 interventions to use to reduce the risk for postoperative swelling.

8. Besides laryngeal nerve damage, C.P. is at risk for other complications postoperatively. List 3 complications, and describe actions you would include in her plan of care related to each.

9. What interventions will you use to meet the outcome of controlling C.P.’s surgical pain?

10. After surgery, C.P.’s thyroid hormone levels are elevated. The provider orders propranolol 80 mg ER orally twice daily for “surgically induced thyrotoxicosis.” Is this reaction expected after thyroid surgery, or did something go wrong during surgery? Explain.

11. What assessment findings would indicate that C.P. was experiencing thyrotoxicosis?

Case study progress Eighteen hours after surgery, C.P. calls you into her room complaining of numbness around her mouth, tingling at the tips of her fingers, and jitteriness. She appears restless but denies any pain at the operative site. She can swallow and speak without difficulty.

12. What is your immediate concern and why? 13. What actions do you need to take based on this concern? 14. The provider orders you to give C.P. 1 gram of IV

calcium gluconate over 15 minutes now, and then start an infusion of 2 grams of calcium gluconate in 500 mL D5W over 12 hours. After the bolus is complete, at what rate will you set the IV pump?

15. What precautions do you need to take while

administering the calcium gluconate infusion?

Case study progress Six hours after initiation of the calcium gluconate infusion, C.P. is no longer complaining of numbness around her mouth or tingling in her fingertips and toes. Chvostek and Trousseau signs remain negative.

Chart View

Laboratory Test Results

Serum calcium 7.4 mg/dL (1.85 mmol/L)

Ionized calcium 3.4 mg/dL (0.85 mmol/L)

Parathyroid hormone (PTH) < 3.0 pg/mL (< 3.0 ng/L) 16. Has C.P.’s status improved or not? Defend your response. Case study progress C.P. is started on oral calcium gluconate. Her calcium levels stabilize within 24 hours and she recovers without further complications. Two days postoperatively, you are preparing her for discharge. 17. As part of your discharge instructions, what would you teach C.P.? Select all that apply. a. Keep her head raised while sleeping for 3 to 4 days b. The importance of keeping follow-up medical appointments c. Not to return to work until her thyroid hormone level is normal d. Cover her incision with clothing or sunscreen when she is in the sun e. Proper care of the incision and signs of infection to report to the provider f. To avoid foods containing iodine because they increase her risk for recurrent cancer Case study outcome Two years later, C.P. began having dysphagia with pain on the left side of her throat. The remaining thyroid tissue was surgically removed after finding the cancer had reoccurred. Postoperatively she underwent 12 weeks of radiation treatment, which left her with some permanent voice changes. She is still cancer free 1 year later. Case Study 96 Name _________________________________ Class/Group _____________________________ Date _____________ Scenario You work in the diabetes mellitus (DM) center at a large teaching hospital. The first patient you meet is K.W., a 35-year-old Hispanic woman, who was just released from the hospital 2 days ago after being diagnosed with type 1 DM. Nine days ago, K.W. went to see the provider after a 1-month history of frequent urination, thirst, severe fatigue, blurred vision, and some burning and tingling in her feet. She attributed those symptoms to working long hours at the computer. Her random glucose level was 410 mg/dL (22.8 mmol/L). The next day her lab values were as follows: fasting glucose 335 mg/dL (18.6 mmol/L), A1C 8.8%, cholesterol 310 mg/dL (8.03 mmol/L), triglycerides 300 mg/dL (3.39 mmol/L), HDL 25 mg/dL (0.65 mmol/L), LDL 160 mg/dL (4.14 mmol/L), ratio 12.4, and creatinine 0.9 mg/dL (80.0 mcmol/L). Her body mass index was 29.6 with a BP 160/96. She was admitted to the hospital for control of her glucose levels and the initiation of insulin therapy with carbohydrate counting. After discharge, K.W. has been referred to you for diabetes education. You are to cover four areas: pharmacotherapy, glucose monitoring, basic nutrition therapy, and exercise. 1. What is the overall teaching goal with a patient newly diagnosed with type 1 DM? 2. What assessments do you need to make before starting your session? 3. K.W. was started on sliding scale lispro (Humalog) four times daily and glargine (Lantus) insulin at bedtime. What are the significant differences between the two therapies? 4. K.W. says she knows people who “only take 2 shots” because they are on NPH and regular insulin and wants to know why she cannot do that. Explain the advantages of using the glargine (Lantus) and lispro (Humalog) insulin regimen. 5. Outline important content to include about pharmacologic therapy. 6. What specific points would you include about managing insulin therapy? Select all that apply. a. Store unused insulin in the freezer. b. The insulin can be used if it is yellow but not expired. c. Administer the lispro (Humalog) within 15 minutes of eating. d. Ideally, the glargine (Lantus) should be administered at bedtime. e. Always administer the injections in the same, easy- to-reach location. f. The current vial of lispro (Humalog) can be kept at room temperature for 1 month. g. Two injections will be needed to administer lispro (Humalog) and glargine (Lantus). 7. What is the best way to determine whether K.W. can safely self-administer insulin? a. Having her describe the process step-by-step b. Evaluating her A1C levels and daily glucose logs c. Observing her draw up and administer an insulin dose d. Asking her to rate her confidence in her ability to give a self-injection 8. Outline important content to review about self blood glucose monitoring. 9. K.W. states her diet is mostly fast foods, and the foods cooked at home are high in starch and fat. Her mealtimes often vary from day to day because of her work schedule. What is CHO (carbohydrate) counting, and why would this method work well for K.W.? 10. Outline important points to cover about a basic nutrition plan with CHO counting. 11. K.W. states that she currently does not exercise at all. What benefits will K.W. receive from taking part in an exercise program? 12. What do you need to teach K.W. about safe exercise? 13. What can you and other members of the health care team do to help her deal with her condition and follow the treatment plan? 14. What evaluative parameters could you use to determine whether your teaching with K.W. was effective? Case study progress K.W. calls the clinic several days later says she has “the flu.” She says she has been nauseated and vomited once during the night. She has had two loose stools. On questioning, she states that she does have a few chills and might have a low-grade fever but does not have a thermometer to check her temperature. She did not check her glucose level this morning or take her insulin because she has “not eaten.” 15. Describe the instructions that you need to give K.W. about managing illness and DM. 16. Outline how you will document the phone call with K.W. Case study outcome In her mid-forties, K.W. began to show signs of diabetic nephropathy. At age 55, she was becoming increasingly fatigued upon mild physical exertion, felt nauseated most of the time, and had increased swelling in her ankles. She began hemodialysis; however, she died 9 months later from a myocardial infarction. Case Study 97 Name _________________________________ Class/Group _____________________________ Date _____________ Scenario K.B. is a 65-year-old man admitted to the hospital after a 5-day episode of “the flu” with symptoms of dyspnea on exertion, palpitations, chest pain, insomnia, and fatigue. K.B. was diagnosed with Graves’ disease 6 months ago and placed on methimazole (Tapazole) 15 mg/day. His other past medical history includes heart failure and hypertension requiring antihypertensive medications; however, he says that he has not been taking these medications on a regular basis. Vital signs (VS) are: 150/90, 124 irregular, 20, 100.2° F (37.9° C). Admission assessment findings are: height 5 ft, 8 in (173 cm); weight 132 lb (60 kg); appears anxious and restless; loud heart sounds; 1 + pitting edema noted in bilateral lower extremities; diminished breath sounds with fine crackles in the posterior bases. K.B. begins to cry when he tells you he recently lost his wife; you notice someone has punched several more holes in his belt so he could tighten it. Chart View Laboratory Test Results Hemoglobin (Hgb) 11.8 g/dL (118 g/L) Hematocrit (Hct) 36% Erythrocyte sedimentation rate (ESR) 48 mm/hr Sodium 141 mEq/L (141 mmol/L) Potassium 4.7 mEq/L (4.7 mmol/L) Chloride 101 mEq/L (101 mmol/L) Blood urea nitrogen (BUN) 33 mg/dL (11.78 mmol/L) Creatinine 1.9 mg/dL (168 mcmol/L) Free thyroxine (T4) 14.0 ng/dL (180 pmol/L) Triiodothyronine (T3) 230 ng/dL (353 nmol/L) 1. Which of K.B.’s assessment findings represent manifestations of hypermetabolism? 2. Interpret K.B.’s lab results. 3. You go to assess K.B. What other data do you need to obtain because he has Graves’ disease? Chart View Physician’s Orders Propranolol (Inderal) 20 mg PO q6hr Dexamethasone 10 mg IV q6hr Methimazole (Tapazole) 15 mg/day twice daily Verapamil (Calan SR) 120 mg/day PO Furosemide 80 mg IV push now, then 40 mg/day IV push Diet as tolerated Stat ECG and echocardiogram Up ad lib IV of D5W at 125 mL/hr Daily weights with intake and output (I&O) 4. The resident on call writes admission orders. Which will you question, and why? 5. What role does methimazole have in treating Graves’ disease? 6. Describe 4 priority problems that will guide K.B.’s nursing care. 7. What measures can you use to promote K.B. receiving adequate rest? Case study progress Later in your shift, you note that K.B. is extremely restless and disoriented to person, place, and time. VS are 174/82, 180 and irregular, 32 and labored, 104° F (40° C). His ECG shows atrial fibrillation. 8. What is likely happening with K.B.? Give your rationale. 9. What actions do you need to take first? 10. You need to call the resident regarding K.B.’s status. Using SBAR, what will you report? Case study progress The resident evaluates K.B. and determines he is in thyroid crisis. New orders are shown in the chart. Chart View Physician's Orders Oxygen at 2 L per nasal cannula Stat ABGs, BNP, and cardiac enzymes Digoxin 0.25 mg IV push now, then 0.125 mg IVP q8hr × 2 doses Diltiazem bolus dose of 0.25 mg/kg IV; after 15 minutes, give a second dose of 0.35 mg/kg IV for heart rate greater than 140 Increase methimazole to 15 mg PO q6hr Hydrocortisone 50 mg IV push q6hr Absolute bed rest Acetaminophen 650 mg PO q6hr prn for temperature over 100° F (37.8° C) 11. Describe how you would care for K.B. in the next hour. 12. How many total milliliters of diltiazem will you give for the first dose? How many for the second, if needed? 13. What is your primary nursing goal at this time? 14. Describe 6 interventions you will perform over the next few hours based on this priority. 15. Why was K.B. at risk for developing thyroid storm? 16. Identify 3 outcomes that you expect for K.B. as a result of your interventions. Case study progress After several hours of treatment, K.B.’s condition stabilizes. The resident discusses two treatment options with K.B. and his family: radioactive iodine (RAI) therapy and subtotal thyroidectomy. 17. K.B. is fearful of radiation treatment and asks you for your opinion. How would you respond? 18. K.B. decides to receive RAI. During pretreatment instructions, the family asks whether he will be radioactive and what precautions they should take. Outline important guidelines for teaching K.B. and his family about home precautions. 19. Amid all this, you remain concerned about K.B.’s bereavement after the loss of his wife. How would you address this issue? 20. K.B. does have some exophthalmos and is experiencing periodic photophobia and dry eyes. What should you include in teaching him how to manage these problems? Select all that apply. a. Always wear sunglasses when outside. b. Report any changes in vision to the provider. c. Use artificial tears to provide moisture as needed. d. Tape the eyes closed at night with nonallergenic tape. e. Apply warm compresses to the eyes if they are irritated. 21. Which statement shows K.B. understands the discharge instructions? a. “I will take this medication on a full stomach.” b. “If I get a sore throat, ice chips should help me feel better.” c. “I should see an improvement in my symptoms by tomorrow.” d. “I will follow the precautions for 2 weeks to keep my family safe.” Case study outcome Six months later, K.B.’s heart rate, blood pressure, and thyroid hormone levels are within normal limits. He has gained 14 lbs (6.4 kg) and has started walking in the mornings without any dyspnea. He says he has started to do woodworking and has been doing some volunteer work at the senior center. Case Study 98 Name _________________________________ Class/Group _____________________________ Date _____________ Scenario You are working in an outpatient clinic when a mother brings in her 20-year-old daughter, C.J., who has type 1 diabetes mellitus (DM) and has just returned from a trip to Mexico. She has had a 3-day fever and diarrhea with nausea and vomiting. She has been unable to eat and has tolerated only sips of fluid. Because she was unable to eat, she did not take her insulin as directed. You note C.J. is unsteady, so you take her to the examining room in a wheelchair. While helping her onto the examination table, you note her skin is warm and flushed. Her respirations are deep and rapid, and her breath is fruity and sweet smelling. C.J. is drowsy and unable to answer your questions. Her mother states, “She kept telling me she’s so thirsty, but she can’t keep anything down.” 1. List 5 pieces of additional information you need to obtain from C.J.’s mother. 2. Describe the pathophysiology of diabetic ketoacidosis (DKA). Case study progress C.J.’s mother tells you the following: “Blood glucose monitor has been reading ‘high.’” “C.J. has had sips of ginger ale, but that’s all.” “She has been vomiting about every other time she drinks.” “When she first got home, she went [voided] a lot, but yesterday she hardly went at all, and I don’t think she has gone today.” “She went to bed early last night, and I could hardly wake her up this morning. That’s why I brought her in.” Chart View Vital Signs Blood pressure 90/50 Heart rate 124 Respiratory rate 36 and deep Temperature 101.3° F (38.5° C) (tympanic) Laboratory Test Values Glucose 677 mg/dL (37.6 mmol/L) Potassium 6.3 mEq/L (6.3 mmol/L) 3. Interpret C.J.’s VS and lab results, relating them to the pathophysiology. 4. Explain the reason for C.J.’s other presenting signs and symptoms. 5. The decision is made to transport C.J. by ambulance to the local emergency department (ED). C.J.’s mother realizes that C.J. is more acutely ill than she thought. She leaves the room and begins to cry. How would you handle this situation? 6. After assessing C.J., the ED resident on call writes the following orders. Review each order. Mark an A if the order is appropriate; mark an I if inappropriate. For each order you mark as I, explain why it is inappropriate, and correct the order. 1. ____ 1000 mL lactated Ringer’s IV stat 2. ____ 36 units NPH (Humulin N) and 20 units regular (Humulin R) insulin subQ now 3. ____ CBC with differential; CMP; blood cultures × 2 sites; clean-catch urine for UA and C stool for ova and parasites, Clostridium difficile toxin, and C serum lactate, ketone, and osmolality; ABGs on room air 4. ____ 1800-calorie, carbohydrate-controlled diet 5. ____ Bed rest 6. ____ Acetaminophen 650 mg rectal suppository q4hr as needed 7. ____ Furosemide 60 mg IV push now 8. ____ Urinary output every hour 9. ____ VS every shift 7. List 5 other collaborative interventions that C.J. needs and the reason for using each. 8. Which ABG results would you expect to see in C.J.? a. pH 7.40, Pao2 88, Paco2 34, HCO3 23 b. pH 7.48, Pao2 90, Paco2 30, HCO3 28 c. pH 7.27, Pao2 90, Paco2 50, HCO3 20 d. pH 7.26, Pao2 94, Paco2 23, HCO3 18 9. When C.J. is attached to the cardiac monitor, what would you expect to see on the ECG tracing? a. Peaked P waves and a shortened PR interval b. Presence of a U wave and ST segment depression c. Tall, peaked T waves and widened QRS complexes d. Narrow QRS complexes and shortening of the QT interval Case study progress All orders have been corrected and therapies started. C.J. receives fluid resuscitation and sliding-scale insulin drip via infusion pump. After several hours, her latest lab findings are as shown in the chart. Chart View Laboratory Test Results Na 149 mEq/L (149 mmol/L) K 3.0 mEq/L (3.0 mmol/L) Cl 119 mEq/L (119 mmol/L) Total CO2 21 mEq/L (21 mmol/L) BUN 12 mg/dL (4.28 mmol/L) Creatinine 1.2 mg/dL (106 mcmol/L) Glucose 307 mg/dL (17 mmol/L) 10. Based on C.J.’s lab results, what changes in her IV fluids would you anticipate, and why? 11. The attending changes the insulin drip infusion, decreasing it from 6 units to 4 units per hour. The label on the bag infusing reads, “100 units regular (Humulin R) insulin in 250 mL of normal saline.” At how many milliliters per hour would you set the infusion pump? 12. What is the rationale behind using an infusion pump for the insulin drip? 13. True or False? A second registered nurse or physician must verify the IV infusion rate for the insulin dose change. State your rationale. Case study outcome C.J. continues to improve and is discharged from the hospital 3 days later. Her mother and she agree to attend an outpatient class offered by the diabetes education department to assist C.J. with better managing her diabetes. Case Study 99 Name _________________________________ Class/Group _____________________________ Date _____________ Scenario T.R. is a 19-year-old college freshman who lives in the honors dormitory. His friend finds him wandering aimlessly about the campus appearing pale and sweaty. He engages T.R. in conversation and walks him to the campus medical clinic, where you are on duty. The friend explains to you how he found T.R. and says T.R. is “diabetic” and takes insulin. T.R. is not wearing a medical warning tag. It is 1020. 1. What do you think is going on with T.R.? 2. What is the first action you would take? 3. Which assessment findings would support the premise that T.R. is experiencing a hypoglycemic reaction? a. Extreme thirst and nausea b. Nervousness and tachycardia c. Hypertension with bounding pulses d. Fruity breath with deep, rapid respirations 4. If no glucose meter were available, would you treat T.R. on the assumption he is hyperglycemic or hypoglycemic? Explain your reasoning. 5. It is 1025. T.R.’s glucose reading is 50 mg/dL (2.8 mmol/L). What should your next action be? 6. When you enter the room to give the juice, T.R. is not responsive enough to drink the juice safely. What should you do? 7. T.R.’s breathing rate is 16 and he has a pulse of 112 and regular. Because outpatient resources vary, describe your next actions if (1) your clinic is well equipped for emergencies or (2) your clinic has no emergency supplies. Case study progress A few minutes after administering 2 mg subcutaneous glucagon, T.R. begins to awaken. He becomes alert and asks where he is and what happened to him. You orient him and then explain what has transpired. 8. What questions would you ask to find out what precipitated this event? 9. What further action do you need to take at this time? 10. At 1045, you recheck T.R.’s glucose and the reading is 64 mg/dL (3.6 mmol/L). His vital signs are 120/72, 18, 92. Has his status improved or not? Defend your response. 11. What would your next action be? 12. At 1110, you recheck T.R.’s glucose and the reading is 104 mg/dL (5.8 mmol/L). What should you do now? Case study progress T.R. tells you he took 35 units glargine (Lantus) insulin and 12 units of regular (Humulin R) insulin at 0745. He says he was late to class, so he just grabbed an apple on the way. 13. Based on this information, why did T.R. experience this episode of hypoglycemia? 14. Based on your knowledge of the types of insulin T.R. is receiving, when would you expect T.R. to experience a hypoglycemic reaction? 15. T.R. says he had a few similar episodes recently. He treated them by eating a candy bar. He says he is on a 2000-calorie, carbohydrate-controlled diet but has been checking his blood glucose levels every “couple of days” only. What common mistake in previously treated episodes of hypoglycemia did T.R. make? 16. He goes on to say he has had “a little bit much to drink at a few of the parties he has been to” on the weekends. What effect does alcohol have on blood glucose? 17. What should you teach T.R. about alcohol consumption and managing his diabetes? 18. List 4 additional points you would stress in a teaching plan with T.R. 19. You tell T.R. to check his blood glucose at 1230 then eat lunch at the normal time. You determine that he understands your teaching regarding averting hypoglycemia if he states: a. “I need to eat within 30 minutes of taking the regular insulin.” b. “If I am too sick to eat, I will not take any insulin until I feel better.” c. “Only certain kinds of alcoholic drinks will affect my blood glucose levels.” d. “I will exercise just before eating and taking insulin so I do not get cramps.” 20. Write a sample documentation note for the encounter with T.R. Case study outcome T.R. follows up with you in 1 week to discuss how he has been managing his meals and insulin dosing. He states he feels that he has been doing a better job with eating meals at regular times and shows you the log from his new diabetes tracking app. You congratulate him on his progress and decide together that meeting weekly will help T.R. manage his diabetes while adjusting to college life. Case Study 100 Name _________________________________ Class/Group _____________________________ Date _____________ Scenario B.K. is a 63-year-old woman who is admitted to the step-down unit from the emergency department (ED) with nausea, vomiting, and epigastric and left upper quadrant abdominal pain that is severe, sharp, and boring and radiates through to her mid back. The pain started 24 hours ago and awoke her in the middle of the night. B.K. is a divorced, retired sales manager who smokes a half-pack of cigarettes daily. The ED nurse reports that B.K. is anxious and demanding. B.K. denies using alcohol. Her vital signs are as follows: 100/70, 97, 30, 100.2° F (37.9° C) (tympanic), SpO2 88% on room air and 92% on 2 L of oxygen by nasal cannula (NC). She is in normal sinus rhythm. She is under the care of the hospitalist service. She has no primary care provider and has not seen a physician “in years.” The ED nurse giving you the report states that the admitting diagnosis is acute pancreatitis of unknown etiology. An abdominal ultrasound showed “no cholelithiasis, gallbladder wall thickening, or choledocholithiasis. The pancreas was not well visualized due to overlying bowel gas.” An abdominal CT is scheduled for the morning. Admission labs have been drawn; a clean-catch urine specimen was sent to the lab, and the urine was dark in color. 1. What are the usual causes of pancreatitis? 2. What other information do you need from the ED nurse before you assume responsibility for B.K.’s care? Case study progress Chart View Medication Administration Record Esomeprazole 40 mg IV push daily Metoclopramide 10 mg IV push every 6 hrs Metronidazole 500 mg IV piggyback every 8 hrs Morphine sulfate 5 mg IV push every 4 hrs as needed Ondansetron 4 mg IV push every 6 hrs as needed 3. Indicate the expected outcome associated with each medication she is receiving. 4. Which admission order would you question? a. Clear liquid diet b. IV 0.9% NS at 150 mL/hr c. Bed rest with bathroom privileges. d. CBC with differential, BMP, amylase, and lipase now 5. What preparation is needed for B.K.’s CT scan? Case study progress You complete your admission assessment and note the following abnormalities: B.K. is restless and alert, lying on her right side in a semi-fetal position. Assessment findings are as follows: Skin is cool, diaphoretic, and pale with poor skin turgor; mucous membranes are dry. ECG shows sinus tachycardia, rate 106, heart sounds without murmurs or rubs. Peripheral pulses are palpable at 1 + in four extremities. Respiration rate 24, but unlabored on 2 L O2/NC with Spo2 90%. Breath sounds are extremely diminished in lower left lobe (LLL) posteriorly—otherwise, clear to auscultation throughout. She complains of nausea and is having dry heaves. Bowel sounds are hypoactive throughout. Abdomen is distended, firm, and tender in a diffuse fashion to light palpation, with guarding noted. The admission chest x-ray report reads, “moderate pleural effusion in the left lower lobe.” 6. Your institution uses electronic charting. Based on the assessment given, document your findings. □ Neurologic □ Respiratory □ Cardiovascular □ Gastrointestinal □ Genitourinary □ Musculoskeletal □ Skin □ Psychosocial □ Pain 7. Based on your assessment, what is your nursing priority right now? 8. Name 3 interventions you would initiate based on this priority. 9. Besides giving as-needed medications, what other interventions would help with B.K.’s pain management? 10. B.K. turns on her call light. Despite the nausea, she complains of thirst and demands something to drink. Her orders indicate “NPO, except sips and chips.” What is your response to her request? What might help her? Chart View Admission Laboratory Test Results Lipase 3000 units/L Amylase 2000 units/L ALP 350 units/L (6 µkat/L) ALT 90 units/L (1.53 µkat/L) AST 150 units/L (2.55 µkat/L) Total bilirubin 2.0 mg/dL (34.2 µmol/L) Albumin 3.0 g/dL (4.35 µmol/L) BUN 24 mg/dL (8.57 mmol/L) Creatinine 1.4 mg/dL (124 mcmol/L) WBC count 17,500/mm3 (17.5 x 109/L) 11. Which lab results support a diagnosis of pancreatitis? 12. Which lab results are the most important to monitor in acute pancreatitis? Why are they significant? 13. What do the BUN and creatinine tell you about her renal function and volume status? 14. Why are the WBCs elevated? Case study progress B.K. eventually falls asleep and seems to be sleeping peacefully. Several hours later, you hear an alarm on her pulse oximeter and enter her room to investigate. You find B.K. moaning softly; her oximeter reads 87%. 15. What will you do next? 16. B.K.’s respirations become increasingly labored. You perform a quick assessment, with findings as follows: Lung sounds absent in the LLL and very diminished in the right lower lobe. You percuss a dull thud over the left lung up to the scapula tip. On percussion, you hear resonance over the entire right lung. What is the significance of your findings? Case study progress The provider orders a stat chest x-ray examination, which shows a significant pleural effusion developing in the LLL, with extension into the lower portion of the upper lobe. 17. Based on the evolving pleural effusion with evidence of decompensation, the provider decides to perform a thoracentesis to remove fluid. What are your responsibilities regarding this procedure? Case study progress The provider removed 200 mL of slightly cloudy serous fluid. Antibiotics were adjusted to provide broad-spectrum coverage for an upper respiratory tract infection until culture and sensitivity results are returned. B.K. is resting quietly with oxygen at 3 L per NC, and her respirations are unlabored and regular. Her SpO2 reads 96%. It is now 72 hours after B.K.’s admission, and her lab test results show improvement. An abdominal CT scan is completed and shows “a moderately severe pancreatitis, but no local fluid collection or pseudocysts. No ileus or evidence of neoplasia was noted.” BUN is 9.0 mg/dL, and creatinine is 1.0 mg/dL. She has adequate urinary output. Her IV fluids are decreased to 75 mL/hr. Her amylase and lipase levels are decreasing toward normal levels. The provider writes an order to advance B.K.’s diet from clear to full liquids. 18. How would you know if B.K. was not tolerating the advancement in her diet? 19. If B.K. does not tolerate the advancement in diet, what physiologic need should be addressed at 72 hours? Case study outcome Three days later, B.K.’s pain is under control with oral pain medications. The provider advances her diet to “low-fat/low- cholesterol” and writes orders to discharge that evening if she tolerates the advancement in diet, which she does. 20. What will you include in your discharge teaching with B.K.? 21. Which statement shows that your teaching about dietary instructions was effective? a. “I must eat six, small meals a day.” b. “I need to eliminate all protein from my diet.” c. “Avoiding greasy and fried foods is important.” d. “It is okay for me to drink coffee, tea, and soda.” Case study outcome At her one-week follow-up appointment, B.K. relates that finding foods that she can eat has been the greatest challenge but that overall, she feels better. She says she is irritated with her friends, who assume this happened because she is secretly drinking. She has not resumed smoking. She says she does not want this to happen again because it has been a “really, really bad experience.” 8 Immunity Case Study 101 Name _________________________________ Class/Group _____________________________ Date _____________ Scenario C.Z. is a 37-year-old woman who was diagnosed with rheumatoid arthritis 3 years ago. Her current medications include naproxen 500 mg twice daily, methotrexate 20 mg once weekly, and folic acid 1 mg daily. She says she has improved on this plan but has persistent joint pain and swelling in her hands and feet, with some lingering fatigue. She came to the clinic today because her family recently moved to the area and she needs to establish a relationship with a new primary care provider. She is married with 2 school-aged children and was just hired part-time as a cashier at a local department store. 1. Describe the pathophysiology of rheumatoid arthritis (RA). 2. RA and osteoarthritis have some common symptoms but have different causes, progression, and treatment. Place an “R” next to the statements that describe RA and an “O” next to those that describe osteoarthritis. 1. _____ Develops over weeks to months 2. _____ Localized disease with variable, progressive course 3. _____ Morning stiffness lasts 1 hour to all day 4. _____ Pain improves with joint use 5. _____ Age of onset is generally over 40 years 6. _____ Joint effusions are common 7. _____ Typically affects small joints first 8. _____ Fever, fatigue, and loss of energy may occur 9. _____ Pain worsens with joint use 3. What are the long-term systemic complications of RA? 4. What are the overall goals of collaborative care for a patient with RA? 5. Outline the history information you need to obtain from C.Z. Case study progress C.Z. says her symptoms started suddenly. She stated, "she woke up one morning and was so weak she could barely get out of bed or walk.” Over the next few days she started to hurt and feel stiff all over, making it hard to sit, stand, or lay down without pain. She lost her appetite and noticed some hair loss. After having lab testing, the previous provider told her she had RA and started her on her current medications. Recent x-rays showed some bone erosion in her right wrist and 3 metacarpophalangeal (MCP) joints of her right hand. She denies any family history of RA or other autoimmune disorders and has had no hospitalizations except for childbirth. 6. What other signs of RA do you need to assess for when performing her assessment? Case study progress C.Z.’s physical assessment was unremarkable except that her left and right index and middle PIP joints were slightly tender and swollen with limited range of motion. Her vital signs were 120/78, 74, 16, 98.2° F (36.8° C). Her height is 5 ft, 5 in (165 cm), and she weighs 170 lbs (77 kg). Chart View Laboratory Results C-reactive protein (CRP) 2.7 mg/dL (25.7 nmol/L) (normal: < 1.0 mg/dL [<9.5 nmol/L]) Erythrocyte sedimentation rate (ESR) 38 mm/hr (normal: 0–20 mm/hr) Rheumatoid factor (RF) 344 (negative or titer < 1:17) ANA 26 mmol/L (negative at 1:40 dilution) 7. Explain the significance of C.Z.’s lab results. 8. The provider decides to start C.Z. on the immunomodulator etanercept 50 mg SC weekly via autoinjector and orders a tuberculin (TB) test and chest x-ray. What is the reason for performing a TB test and chest x-ray? 9. What teaching do you need to provide about etanercept therapy? Case study progress You ask C.Z. how having RA has affected her life. She states that she used to be a very active person, but now she must be careful about how much she does each day because she has noted that too much activity triggers increased fatigue and pain. 10. Name 3 priority nursing problems you need to address with C.Z. 11. Describe the teaching you would provide C.Z. about physical activity. 12. What would you review with C.Z. to meet the expected outcome of protecting joints from stress? 13. What suggestions can you make to C.Z. about managing fatigue? 14. After teaching C.Z. to use heat and cold therapy to relieve symptoms, you determine teaching has been effective when she states: a. “Taking a warm shower can help ease morning stiffness.” b. “Cold therapy can be applied for 30 minutes to relieve joint stiffness.” c. “I can use heat therapy for 20 minutes to relieve the symptoms of an acute flare.” d. “When my joints are swollen, a heating pad for 10 to 15 minutes can relieve the pain.” 15. You are concerned with C.Z.’s psychological status. Write a nursing outcome addressing this issue and identify independent nursing actions you would implement. 16. What interprofessional referrals may benefit C.Z.? 17. What assessment data would you use to determine whether C.Z.’s condition is improving? 18. Using a SOAP format, write a sample documentation entry for this encounter. Case study outcome After switching to etanercept, C.Z. noticed a substantial improvement in joint pain and swelling. She decided to continue with this regimen and began an exercise program under the supervision of PT. By implementing the recommendations made by the provider, PT, OT, and you, she says she is has less fatigue and finding it easier to “make it through each day.” Case Study 102 Name _________________________________ Class/Group _____________________________ Date _____________ Scenario You are working in a community health clinic and you have just taken C.Q., a 38-year-old woman, into the consultation room. C.Q. has been divorced for 5 years, has two daughters (ages 14 and 16), and works full time as a legal secretary. She is here for her yearly routine physical examination. C.Q. states she is in a serious relationship, is contemplating marriage, and just wants to make certain she is “okay.” No abnormalities were noted during C.Q.’s physical examination. Blood was drawn for routine blood chemistries and hematology studies; since she has never been tested, C.Q. agrees to a human immunodeficiency virus (HIV) test. The provider requests you perform a rapid HIV test, which is an antibody test. Within 20 minutes, the results are available and are positive. 1. Does a positive rapid HIV test mean that C.Q. has HIV? If it is negative, does it mean she does not have HIV? 2. What counseling do you need to offer to C.Q.? Case study progress C.Q. returns to the clinic 2 days later. The provider informs you that C.Q.’s Western blot test results confirm that she has HIV infection; he requests you be present when he talks to her. Before leaving C.Q.’s room, the provider requests that you give C.Q. verbal and written information about local support groups and help her call a friend to accompany her home this evening. She looks at you through her tears and states, “I can't believe it. J. is the only man I’ve had sex with since my divorce. He told me I had nothing to worry about. I can’t believe he would do this to me.” 3. C.Q.’s statement is based on 3 assumptions: (1) J. is HIV positive; (2) he intentionally withheld the information from her; and (3) he intentionally transmitted the HIV to her through unprotected sex. Based on your knowledge of HIV infection, how would you counsel C.Q.? 4. In addition to offering alternative explanations and exploring options, what is your most important role right now? 5. C.Q. asks you whether she has AIDS. What do you tell her? 6. Why is it a good idea for C.Q. to have someone she trusts take her home this evening? 7. C.Q. gives you the name and phone number of someone she wants you to call. You stay with her until she leaves with her friend. Has C.Q.’s right to privacy been violated? Explain why or why not. Case study progress C.Q. returns to the clinic 4 days later to discuss her diagnosis. 8. What are your goals for C.Q. at this time? 9. What additional lab tests would you anticipate for C.Q. and why? 10. C.Q. asks whether there is any treatment available. How would you respond? 11. The provider starts C.Q. on a regimen of tenofovir- emtricitabine (Truvada), darunavir (Prezista), and ritonavir (Norvir). What general information will you give C.Q. about antiretroviral therapy (ART) therapy? 12. C.Q. asks why she has to take so many drugs instead of a “big dose” of one drug. What would you tell her? 13. What other issues will you discuss with C.Q. at this visit? 14. Review the general measures you will discuss with C.Q. to promote her overall health. 15. C.Q. asks if she must tell J. of her HIV status. Does she have a legal responsibility to inform him? 16. What reporting obligations does the clinic have? 17. Before C.Q. leaves the clinic, you recognize the need for further teaching when she says: a. “Joining a support group can help me deal with my HIV diagnosis.” b. “I will not use any other medications without checking with my health care provider.” c. “If my viral load becomes undetectable, I will not have to worry about transmitting HIV to someone else.” d. “If my skin turns yellow, I have unusual muscle pain or feel dizzy or weak, I will call the provider immediately.” Case study outcome Two weeks later, C.Q. visits the office and asks to speak to you in private. She thanks you for talking to her the day she received the news of her diagnosis. She tells you that J. confessed to her that he has hemophilia and tested positive for HIV after having been infected through contaminated recombinant factor VIII products. He was afraid to tell her about his diagnosis because she might leave him. C.Q. tells you that she is angry with J. They are going through counseling and the wedding is “off” at the moment. Case Study 103 Name _________________________________ Class/Group _____________________________ Date _____________ Scenario K.D. is a 56-year-old man who has been living with human immunodeficiency virus (HIV) infection for 6 years. He had been on antiretroviral therapy (ART) with a regimen of tenofovir and emtricitabine (Truvada), with darunavir and cobicistat (Prezcobix). He stopped taking his medications 4 months ago because of depression. The appearance of purplish spots on his neck and arms persuaded him to make an appointment with his provider. At the provider’s office, K.D. stated he was feeling fatigued and having occasional night sweats. He said he had been working long hours and skipping meals. Other than the purplish spots, the remainder of K.D.’s physical examination findings was within normal limits. The doctor took 3 skin biopsy specimens and obtained a chest x-ray examination, tuberculin test, and lab studies, including a CBC, CD4 T-cell count, and viral load. Over the next week, K.D. developed a nonproductive cough and increasing dyspnea. Last night, he developed a fever of 102° F (38.9°C) and was acutely short of breath, so his partner brought him to the emergency department. He was admitted with probable Pneumocystis jiroveci pneumonia (PJP), which was confirmed with bronchoalveolar lavage examination under light microscopy. K.D.’s CD4 T-cell count is 175 cells/µL and viral load 35,230 copies/µL. K.D. is on nasal oxygen, IV fluids, and IV trimethoprim-sulfamethoxazole. His current VS are 138/86, 100, 30, 100.8° F (38.2° C) and SpO2 92%. 1. What is the importance of CD4 T-cell and viral load counts? 2. What is PJP? 3. The skin biopsies return a diagnosis of Kaposi sarcoma (KS). What is KS? 4. What is the significance of K.D. developing KS and PJP in light of the CD4 count? 5. K.D. has been seropositive for several years. What factors might have influenced his development of PJP and KS? 6. Name 4 problems you must manage at this time with K.D. 7. What type of isolation precautions do you need to use when caring for K.D.? a. Droplet b. Contact c. Standard d. Airborne 8. What immediate complication is K.D. at risk for experiencing? 9. To detect this complication, what will be the focus of your ongoing assessment? 10. Why was K.D. placed on trimethoprim-sulfamethoxazole? What major side effects do you need to monitor for in K.D.? 11. What aspects of K.D.’s care can you delegate to the licensed practical nurse (LPN)? Select all that apply. a. Providing instructions about a high-calorie, high- protein diet b. Administering first dose of IV trimethoprim- sulfamethoxazole c. Repositioning K.D. and having him deep breathe every 2 hours d. Developing a plan of care to improve K.D.’s oxygenation status e. Reinforcing teaching with K.D. about good hand washing techniques f. Monitoring K.D.’s pulse oximetry readings and reporting values under 95% 12. K.D. has 20 KS lesions on his neck, upper chest, and both upper arms, all of which are closed and painless. How will you care for these lesions? a. Keep each lesion covered with a clear, transparent dressing. b. Place sterile, saline-soaked gauze over each lesion twice daily. c. Keep the lesions dry, cleaning the affected areas gently as needed. d. Apply topical antibiotic ointment twice daily to the affected areas. 13. Because of compromised immune function, K.D. is at risk for developing other opportunistic infections. List 5 infections. 14. Outline the assessment you need to perform to determine whether these problems are present. 15. What interventions can you use to help K.D. in managing his depression? 16. Recognizing that K.D. has multiple posthospital needs, you begin discharge planning. What type of assessment do you need to complete as part of K.D.’s discharge planning? 17. What other health care team members might you involve in K.D.’s discharge planning? Case study progress K.D. is responding well to treatment for PJP and plans are being made for discharge. He will receive follow-up care at the outpatient clinic and soon begin radiation treatments for the KS. His ART regimen is changed to a fixed-dose combination regimen with emtricitabine- tenofovir-efavirenz (Atripla). 18. K.D. is kept on trimethoprim-sulfamethoxazole (Bactrim) 2 tablets once daily. He asks why he has to keep taking Bactrim “since the pneumonia is gone.” How would you respond? 19. What is the reason that K.D. receives combination ART therapy with emtricitabine-tenofovir-efavirenz (Atripla)? a. Combination ART is only effective when CD4- positive T-cell counts are below 200/µL b. Giving the drugs in a single pill reduces the incidence of cross-resistance between drugs c. Using smaller doses of three different drugs reduces the incidence of side effects of each drug d. Combination ART inhibits viral replication in different ways and decreases the chance of drug resistance 20. How can you help K.D. take Atripla as prescribed? 21. What ongoing lab monitoring will K.D. need? 22. K.D. was taught about disease transmission and safer sex and encouraged to maintain moderate exercise, rest, and dietary habits when he was first diagnosed with HIV infection. Give at least 3 additional topics to stress with K.D. before he goes home. Case study outcome K.D. took the Atripla as prescribed and was happy that after 3 months of therapy, his CD4 T-cell count was up to 403 cells/µL. The KS lesions have declined in number and size with the ART and radiation therapy. He says his partner and he are dealing with his fatigue., Overall, he says he feels better and taking the diagnosis of HIV “more seriously” by doing more to stay healthy. Case Study 104 Name _________________________________ Class/Group _____________________________ Date _____________ Scenario M.M., a 46-year-old male, presents to the emergency department (ED) after being stung 3 times by bees while doing yard work. He said he initially had localized pain and swelling but was not concerned because he has no prior history of an allergic reaction with previous stings. About 15 minutes after the stings, he began to feel dizzy and short of breath. He went inside and his girlfriend, seeing he had developed hives and was having some trouble breathing, drove him the 15 minutes to the ED. 1. What other information do you need to obtain? 2. What should be included as part of M.M.’s primary survey? Case study progress Chart View Physical Assessment VS: 100/60, 112, 32, SpO2 93% on RA, 98.8° F (37.1° C) HEENT: No periorbital edema. Mild edema of lip and tongue. PERRLA. States he has metallic taste in mouth. CV: Moderate wheezing with minimal retractions in all fields. AP S1, S2; regular. Neuro: GCS 15. Drowsy. Full movement in all extremities. Reports dizziness. GI: Soft, nontender, + BS × 4. Denies nausea and abdominal pain. Skin: Generalized urticarial rash with flushing. Reports pruritus. Bee sting sites present on left hand and forearm; foreign body seen in left-hand sting site. M.M. denies any history of allergies. He has mild hypertension that is controlled with furosemide 40 mg orally every morning. 3. What is most likely happening with M.M.? 4. What assessment findings led you to making this conclusion? 5. What is the immediate priority with M.M.? 6. How would you remove the stinger? Case study progress Chart View Provider’s Orders

PLACE THIS ORDER OR A SIMILAR ORDER WITH ONLINE NURSING PAPERS TODAY AND GET AN AMAZING DISCOUNT  ordernowcc-blue